You are on page 1of 177

CBSE Term II

2022

Physics
Class XII
Complete Theory Covering NCERT

Cased Based Questions

Short/Long Answer Questions

3 Practice Papers with Explanations

Author
Manish Dangwal

ARIHANT PRAKASHAN (School Division Series)


ARIHANT PRAKASHAN (School Division Series)

© Publisher
No part of this publication may be re-produced, stored in a retrieval system or by any means,
electronic, mechanical, photocopying, recording, scanning, web or otherwise without the written
permission of the publisher. Arihant has obtained all the information in this book from the sources
believed to be reliable and true. However, Arihant or its editors or authors or illustrators don’t take any
responsibility for the absolute accuracy of any information published and the damage or loss suffered
thereupon.
All disputes subject to Meerut (UP) jurisdiction only.

Administrative & Production Offices


Regd. Office
‘Ramchhaya’ 4577/15, Agarwal Road, Darya Ganj, New Delhi -110002
Tele: 011- 47630600, 43518550

Head Office
Kalindi, TP Nagar, Meerut (UP) - 250002, Tel: 0121-7156203, 7156204
Sales & Support Offices
Agra, Ahmedabad, Bengaluru, Bareilly, Chennai, Delhi, Guwahati,
Hyderabad, Jaipur, Jhansi, Kolkata, Lucknow, Nagpur & Pune.

ISBN : 978-93-25796-89-8
PO No : TXT-XX-XXXXXXX-X-XX
Published by Arihant Publications (India) Ltd.
For further information about the books published by Arihant, log on to
www.arihantbooks.com or e-mail at info@arihantbooks.com

Follow us on

CBSE Term II
2022
Contents
CHAPTER
Electromagnetic Waves -

CHAPTER
Ray Optics and Optical Instruments -

CHAPTER
Wave Optics -

CHAPTER
Dual Nature of Radiation and Matter -

CHAPTER
Atoms -

CHAPTER
Nuclei -

CHAPTER
Semiconductor Electronics: Materials, Devices -
and Simple Circuits

Practice Papers - -

Watch Free Learning Videos


Subscribe arihant Channel

þ Video Solutions of CBSE Sample Papers


þ Chapterwise Important MCQs
þ CBSE Updates
Syllabus
CBSE Term II Class 12
Time: Hours Max Marks:
Units Periods Marks
Unit V Electromagnetic Waves
Chapter : Electromagnetic Waves

Unit VI Optics
Chapter : Ray Optics and Optical Instruments
Chapter : Wave Optics

Unit VII Dual Nature of Radiation and Matter


Chapter : Dual Nature of Radiation and Matter
Unit VIII Atoms and Nuclei
Chapter : Atoms
Chapter : Nuclei

Unit IX Electronic Devices


Chapter : Semiconductor Electronics: Materials,
Devices and Simple Circuits

Total

UNIT-V Electromagnetic waves Periods


Chapter- Electromagnetic waves
Electromagnetic waves, their characteristics, their Transverse nature qualitative ideas
only . Electromagnetic spectrum radio waves, microwaves, infrared, visible, ultraviolet,
X-rays, gamma rays including elementary facts about their uses.

UNIT-VI Optics Periods


Chapter- Ray Optics and Optical Instruments
Ray Optics: Refraction of light, total internal reflection and its applications, optical
fibers, refraction at spherical surfaces, lenses, thin lens formula, lensmaker s formula,
magnification, power of a lens, combination of thin lenses in contact, refraction of light
through a prism.
Optical instruments: Microscopes and astronomical telescopes reflecting and
refracting and their magnifying powers.

CBSE Term II
2022
Chapter- Wave Optics
Wave optics: Wave front and Huygen s principle, reflection and refraction of plane wave
at a plane surface using wave fronts. Proof of laws of reflection and refraction using
Huygen s principle. Interference, Young s double slit experiment and expression for
fringe width, coherent sources and sustained interference of light, diffraction due to a
single slit, width of central maximum.

UNIT-VII Dual Nature of Radiation and Matter Periods


Chapter- Dual Nature of Radiation and Matter
Dual nature of radiation, Photoelectric effect, Hertz and Lenard s observations;
Einstein s photoelectric equation-particle nature of light.
Experimental study of photoelectric effect, Matter waves-wave nature of particles,
de-Broglie relation.

UNIT-VIII Atoms and Nuclei Periods


Chapter- Atoms
Alpha-particle scattering experiment; Rutherford s model of atom; Bohr model, energy
levels, hydrogen spectrum.
Chapter- Nuclei
Composition and size of nucleus, Nuclear force, Mass-energy relation, mass defect,
nuclear fission, nuclear fusion.

UNIT-IX Electronic Devices Periods


Chapter- Semiconductor Electronics: Materials, Devices and Simple Circuits Energy bands in
conductors, semiconductors and insulators qualitative ideas only , Semiconductor
diode - I-V characteristics in forward and reverse bias, diode as a rectifier;
Special purpose p-n junction diodes: LED, photodiode, solar cell.

CBSE Term II
2022
CBSE Circular
Acad - 51/2021, 05 July 2021

Exam Scheme Term I & II

dsUnh;
z ek/;fed f'k{kk cksMZ
(f'k{kk ea=ky;] Hkkjr ljdkj ds v/khu ,d Lok;r laxBu)

Hkkjr CENTRAL BOARD OF SECONDARY EDUCATION


(An Autonomous Organisation under the Ministryof Education, Govt. of India)
dsUnh;
z ek/;fed f'k{kk cksMZ
(f'k{kk ea=ky;] Hkkjr ljdkj ds v/khu ,d Lok;r laxBu)
Hkkjr
CENTRAL BOARD OF SECONDARY EDUCATION
(An Autonomous Organisation under the Ministryof Education, Govt. of India)
dsUnh;
z ek/;fed f'k{kk cksMZ
(f'k{kk ea=ky;] Hkkjr ljdkj ds v/khu ,d Lok;r laxBu)

Hkkjr CENTRAL BOARD OF SECONDARY EDUCATION


(An Autonomous Organisation under the Ministryof Education, Govt. of India)

To cover this
situation, we
have given
both MCQs and
Subjective
Questions in
each Chapter.
dsUnh;
z ek/;fed f'k{kk cksMZ
(f'k{kk ea=ky;] Hkkjr ljdkj ds v/khu ,d Lok;r laxBu)

Hkkjr CENTRAL BOARD OF SECONDARY EDUCATION


(An Autonomous Organisation under the Ministryof Education, Govt. of India)
CBSE Term II Physics XII 1

CHAPTER 01

Electromagnetic
Waves
In this Chapter...
l Maxwell's Equations

l Electromagnetic Waves

l Electromagnetic Spectrum

These charges produce an oscillating electric field in space,


Maxwell’s Equations which produces an oscillating magnetic field, which in turn is
These are the basic equations of electricity and magnetism. a source of oscillating electric fields and so on.
These equations give complete description of all The oscillating electric and magnetic fields regenerate each
electromagnetic interactions. There are four Maxwell’s other as a continuous wave which propagates through space.
equations (for free space), which are given below The frequency of EM wave is equal to the frequency of
q oscillation of charge.
(i) ò E × dS = (Gauss’s law of electrostatics)
e0 1
i.e. n=
(ii) ò B × dS = 0 (Gauss’s law of magnetostatics) 2p LC
dfB Nature of Electromagnetic Waves
(iii) ò E× dl = - dt In an electromagnetic wave, electric and magnetic fields are
(Faraday’s law of electromagnetic induction) perpendicular to each other and to the direction of wave
æ dfE ö propagation. A plane electromagnetic wave propagating
(iv) ò B × dl = m0 ç Ic + e0
è
÷
dt ø
(Ampere-Maxwell law)
along the z-direction is shown below
X
Maxwell on the basis of his equations predicted the existence Direction of wave propagation
of electromagnetic waves. B
O E
Z
B E
Electromagnetic Waves
Y
These waves are produced due to the change in electric field
E and magnetic field B sinusoidally and propagating through A plane EM wave travelling along Z-axis
space such that, the two fields are perpendicular to each
The electric field E x is along x-direction and dotted curve
other and perpendicular to the direction of wave
propagation. shows magnetic field B which is along y-direction. Both E
and B vary sinusoidally and become maximum at same
Source of Electromagnetic Waves position and time. As E and B are mutually perpendicular to
Maxwell found that, the accelerated or oscillating charge each other, so they are transverse in nature.
radiate electromagnetic waves.
2 CBSE Term II Physics XII

The EM wave propagating in the positive z-direction is represented as


E = E x = E 0 sin ( kx - wt ), B = By = B0 sin( ky - wt )
where, k is propagation vector or wave vector = 2p/ l and wis angular frequency = 2pn.

Important Characteristics of Electromagnetic Waves


Some features of EM waves are given below
(i) EM waves do not require any material medium for propagation.
(ii) These waves travel in free space with the speed of light (3 ´ 10 8 ms -1 ), given by c = 1 / m 0 e0 , which shows that light
waves are electromagnetic in nature.
(iii) Speed of electromagnetic wave in a medium is v = 1 / me, where e and m are the permittivity and magnetic
permeability of a material medium, respectively. This means, the speed of EM wave in a medium depends on electric
and magnetic properties of a medium.
(iv) The direction of variations of electric and magnetic fields are perpendicular to each other and also perpendicular to the
direction of wave propagation.
Thus, electromagnetic waves are transverse in nature.
(v) In free space, the magnitudes of electric and magnetic fields in electromagnetic waves are related by E 0 / B0 = c.
(vi) The energy in electromagnetic waves is divided, on an average, equally between electric and magnetic fields.
Ue = Um
where, U e = energy of electric field and U m = energy of magnetic field.
1 B2
(vii) The energy density (energy per unit volume) in an electric field E in vacuum is e 0 E 2 and that in magnetic field B is .
2 2m 0
(viii) Electromagnetic waves, being uncharged, are not deflected by electric and magnetic fields.
(ix) An electromagnetic wave carries energy and momentum. An electromagnetic wave also exerts pressure called
radiation pressure. If wave is incident on a completely absorbing surface, then momentum delivered is given by
U
p=
c

Electromagnetic Spectrum
The orderly arrangement of EM waves in increasing or decreasing order of wavelength l and frequency n is called
electromagnetic spectrum. The range varies from 10 -12 m to 10 4 m, i.e. from g-rays to radio waves.
Electromagnetic wave spectrum is shown below
Frequency (Hz) Wavelength (m)
1023 10–14
400 nm
1022 10–13
1021 Gamma rays 10–12 Violet
1020 10–11
10–10 450 nm
1019 X-rays
10–9 Blue
1018
1017 10–8 500 nm
1016 Ultraviolet 10–7
1015 Visible 10–6 Green
1014 10–5
Infrared 10–4 550 nm
1013
1012 10–3 Yellow
1011 Microwaves 10–2
1010 10–1 600 nm
Short radio waves
109 1 Orange
108 Television and FM radio 101 650 nm
107 AM radio
102
106 103 Red
105 104
104 Long radio waves 105 700 nm
103 106
102 107
101
Electromagnetic spectrum with common names for various parts of it
CBSE Term II Physics XII 3

Various Electromagnetic Waves of Electromagnetic Spectrum with their Features and Uses
Name of Frequency Wavelength Production Detection Uses
Wave Range Range
Radio waves 500 kHz to > 0.1 m Rapid acceleration Receiver’s aerials l
These are used in AM (Amplitude
1000 MHz and decelerations of Modulation) from 530 kHz to 1710 kHz and
electrons in aerials. ground wave propagation.
l
These are used in TV waves ranging from
54 MHz to 890 MHz.
l
These are used in FM (Frequency
Modulation) ranging from 88 MHz to
108 MHz.
Microwaves 1 GHz to 0.1 m to Klystron valve or Point contact diodes l
These are used in RADAR systems for
300 GHz 1 mm magnetron valve. aircraft navigation.
l
These are used in microwave oven for
cooking purpose.
l
These are used in study of atomic and
molecular structures.
Infrared 3 ´ 1011 Hz to 1 mm to Vibration of atoms Thermopiles l
These are used in physical therapy.
waves 4 ´ 1014 Hz 700 nm and molecules. bolometer and l
These are used in satellites for army
(heat waves) infrared purpose.
photographic film l
These are used in weather forecasting.
14
Light rays 4 ´ 10 Hz to 700 nm to Electrons in atoms Eye, l
These are used by the optical organs of
(visible rays) 7 ´ 1014 Hz 400 nm emit light, when they photocells and humans and animals for three primary
move from a higher photographic film purposes given below
energy level to a lower (i) To see things, avoid bumping into
energy level. them and escape danger.
(ii) To look for food.
(iii) To find other living things with which
to copulate, so as to prolong the
species.
Ultraviolet 1014 Hz to 400 nm Inner shell electrons Photocells and l
These are used in burglar alarm.
rays 1016 Hz to 1 nm in atoms moving from photographic film l
These are used in checking mineral sample.
higher energy level to l
These are used to study molecular
a lower energy level. structure.
l
To kill germs in water purifiers.
l
Used in LASER eye surgery.
X-rays 3 ´ 1016 Hz to 1 nm to X-ray tubes or inner Photographic film, l
These are used in medicine to detect the
3 ´ 1021 Hz 10-3 nm shell electrons, Geiger tubes and fracture, diseased organs, stones in the
bombarding metals by ionisation chamber body, etc.
high energy electrons. l
These are used in engineering to detect
fault, cracks in bridges and testing of welds.
l
These are used at metro-stations to detect
metals or explosive material.
Gamma ( g ) 3 ´ 1018 Hz < 10-3 nm Radioactive decay of Photographic film l
These are used to produce nuclear
rays to 5 ´ 1022 Hz the nucleus. and ionisation reactions.
chamber l
These are used in radio therapy for the
treatment of tumour and cancer.
l
These are used in food industry to kill
pathogenic micro-organism.
4 CBSE Term II Physics XII

Solved Examples
Example 1. An electromagnetic wave of frequency Example 4. About 5% of the power of a 100 W light
40 MHz travels in free space in the x-direction. bulb is converted to visible radiation. What is the
(i) Determine the wavelength of the wave. average intensity of visible radiation at a distance of
(ii) At some point and at some instant, the electric (a) 1m from the bulb and (b) 10 m?
field has its maximum value of 750 NC -1 and is Assume that, the radiation is emitted isotropically
along the Y-axis. Calculate the magnitude and and neglect reflection.
direction of the magnetic field at this position Sol. Total power = 100 W
and time. 8 Visible radiation power = 5% of total power
c 3.0 ´ 10 5
Sol. (i) Wavelength of the wave, l = = = 7.5 m = ´ 100 = 5 W
n 40 ´ 106 100
(ii) Given, maximum value of electric field, E0 = 750 NC -1 (a) At a distance of 1m, the energy distributed in the form of
E 750 sphere.
\ Magnetic field, B0 = 0 = = 2.5 ´ 10– 6 T Area of sphere = 4p (radius) 2
c 3 ´ 108
Power 5
Since, E and B are mutually perpendicular and they both Intensity of visible radiation = =
are perpendicular to the propagation of wave. Thus, we Area 4 ´ 3.14 ´ (1) 2
concluded that, magnetic field is in negative z-direction. = 0.4 W/m 2
Example 2. Find the amplitude of electric and (b) Intensity of visible radiation at a distance of 10 m
5
magnetic fields in a parallel beam of light of =
4 ´ 3.14 (10) 2
intensity 4.0 Wm -2 .
1 = 4 ´ 10- 3 W/m 2
Sol. The intensity of plane electromagnetic wave, I = e0 E02 c
2 Example 5. Evaluate the amplitude of electric and
2I magnetic fields produced by the radiation coming
\Amplitude of electric field, E0 =
e0 c from a 20pW bulb at distance of 2m. Assume that, the
2´4 efficiency of the bulb 20% and it is a point source.
=
8.86 ´ 10–12 ´ 3 ´ 108 Sol. Consider the situation shown below
= 54.87 NC -1 X

Further, amplitude of magnetic field, r


E 54.87 S
B0 = 0 = T P
c 3.0 ´ 108
= 1.83 ´ 10-7 T P
Intensity at distance from a point source (bulb), I =
Example 3. Light with an energy flux of 18 Wcm -2 falls 4pr 2
on a non-reflecting surface at normal incidence. If Output P
Efficiency, h = =
the surface has an area of 20 cm 2 , then find the Input P¢
average force exerted on the surface during a span
æ 20 ö
of 30 min. Þ P = hP ¢ = ç ÷ ( 20p ) = 4pW
è 100 ø
Sol. Total energy falling on the surface,
P 4p 1 W
U = Energy flux ´ area ´ time I= = =
4pr 2 4p( 2) 2 4 m 2
= (18 Wcm -2 )(30 ´ 60 s ) (20 cm 2 )
Also, intensity of EM wave is given by
= 6.48 ´ 105 J
1 1 1
Therefore, the total momentum delivered is, I = e0 E02 c Þ = e0 E02 ´ 3 ´ 108
2 4 2
U 6.48 ´ 105
p= = kg- ms -1 10-8 10-8 104
c 3.0 ´ 108 Þ E02 = = =
= 2.16 ´ 10–3 kg- ms -1 6e0 6 ´ 8.85 ´ 10-12 6 ´ 8.85
100
The average force exerted on the surface, \Amplitude of electric field, E0 = = 13.73 V/m
p 2.16 ´ 10–3 6 ´ 8.85
F= = (Q F × t = change in momentum)
t 30 ´ 60 E0 13.73
\Amplitude of magnetic field, B0 = = . ´10-8 T
= 457
= 1.2 ´ 10–6 N c 3 ´ 108
CBSE Term II Physics XII 5

Chapter
Practice
PART 1 5. The amplitude of the magnetic field part of a
harmonic electromagnetic wave in vacuum is
Objective Questions B 0 = 510 nT. What is the amplitude of the electric
field part of the wave?
(a) 130 NC -1 (b) 153 NC -1
l
Multiple Choice Questions -1
(c) 170 NC (d) 190 NC -1
1. Which of the following statement is correct in 6. An electromagnetic wave travelling along Z-axis is
regards to the Maxwell’s equations?
given as E = E 0 cos( kz - wt ). Choose the incorrect
(a) The most important prediction to emerge from Maxwell’s
equations is the existence of magnetic waves. statement from the following.
(b) Maxwell’s equations involves only electric and magnetic (a) The associated magnetic field is given as
fields. 1 1
B = k$ ´ E = ( k$ ´ E )
(c) The total current has the same value of current i for all c w
surfaces. (b) The electromagnetic field can be written in terms of the
(d) We can rephrase Faraday’s law of electromagnetic associated magnetic field as E = c ( B ´ k$ )
induction that there is an induced emf equal to the rate
of change of magnetic flux. (c) k$ ´E = 0, k$ ´ B = 0
(d) None of the above
2. The electric field of an electromagnetic wave
travelling through vacuum is given by the equation 7. A plane electromagnetic wave of frequency
E = E 0 sin ( kx - wt ). The quantity that is 25 GHz is propagating in vacuum along the
independent of wavelength is z-direction. At a particular point in space and time,
(a)
k
(b) kw
the magnetic field is given by B = 5 ´ 10 - 8 $j T. The
w corresponding electric field E is (Take, speed of
(c) w (d) k light, c = 3 ´ 10 8 ms - 1 )
3. The electric and the magnetic fields, associated (a) - 1 .66 ´ 10- 16 $i Vm -1
with an electromagnetic wave, propagating along (b) 1 .66 ´ 10- 16 $i V m -1
the + Y-axis, can be represented by
(c) - 15 $i Vm - 1
(a) ( E = E0 k$ , B = B0 i$ )
(d) 15 $i Vm -1
(b) ( E = E $j, B = B i$ )
0 0

(c) ( E = E0 $j, B = B0 k$ ) 8. Suppose that, the amplitude of electric field of an


(d) ( E = E0 i$, B = B0 $j ) electromagnetic wave is E 0 = 120 NC -1 and its
frequency is n = 50.0 MHz. The expressions for E
4. Light wave is travelling along y-direction. If the will be (if wave travels along X-axis)
corresponding E vector at any time is along the (a) [(120 NC -1 ) sin {(1.05 rad m -1 ) x
X-axis, the direction of B vector at that time is along - (3.14 ´ 108 rad s -1 ) t}] $i
Y (b) [(120 NC -1 ) sin {(1.05 rad m -1 ) x
- (3.14 ´ 108 rad s -1 ) t}] k$
(c) [(120 NC -1 ) sin {(1.05 rad m -1 ) x
X
O - (3.14 ´ 108 rad s -1 ) t}] $j
Z (d) [(120 NC -1 ) cos {(1.05 rad m -1 ) x
(a) Y -axis (b) X-axis - (3.14 ´ 108 rad s -1 ) t}] $j
(c) + Z-axis (d) -Z-axis
6 CBSE Term II Physics XII

9. The electric field part of an electromagnetic wave 15. An electromagnetic wave given as
in a medium is represented by E x = 0; E = E 0 $i cos ( kz - wt) is incident normally on a
N éæ rad ö æ - 2 rad ö ù perfectly reflecting infinite wall at z = a. Assuming
E y = 2.5 cos êç 2p ´10 6 ÷ t - ç p ´10 ÷x ;
C ëè m ø è s ø úû that, the material of the wall is optically inactive,
E z = 0. the reflected wave will be given as
(a) E r = E0 $i( kz - wt ) (b) E r = E0 $i cos( kz + wt )
The wave is
(a) moving along y-direction with frequency 106 Hz and (c) E r = - E0 $i cos( kz + wt ) (d) E = E $i sin( kz - wt )
r 0
wavelength 200 m -2
16. Radiations of intensity 0.5 Wm are striking a
(b) moving along x-direction with frequency 106 Hz and
metal plate. The pressure on the plate is
wavelength 100 m
(c) moving along x-direction with frequency 106 Hz and (a) 0.166 ´10-8 Nm -2 (b) 0.332 ´ 10-8 Nm -2
wavelength 200 m (b) 0111
. ´ 10-8 Nm -2 (c) 0.083 ´ 10-8 Nm -2
(d) moving along – x-direction with frequency 106 Hz and
wavelength 200 m 17. A pulse of light of duration 100 ns is absorbed
completely by a small object initially at rest. Power
10. In a plane electromagnetic wave, the directions of
of the pulse is 30 mW and the speed of light is
electric field and magnetic field are represented by
k$ and 2$i - 2$j, respectively. What is the unit vector 3 ´ 10 8 ms -1 . The final momenum of the object is
(a) 0.3 ´ 10-17 kg-ms -1 (b) 1 . 0 ´ 10-17 kg-ms -1
along direction of propagation of the wave?
1 $ $ 1 $ $ (c) 3.0 ´ 10-17 kg-ms -1 (d) 9.0 ´ 10-17 kg-ms -1
(a) ( i + j) (b) ( j + k)
2 2
18. One requires 11 eV of energy to dissociate a carbon
1 $ 1
(c) ( i + 2$j) (d) ( 2i$ + $j) monoxide molecule into carbon and oxygen atoms.
5 5
The minimum frequency of the appropriate
11. Electromagnetic waves travel in a medium with a electromagnetic radiation to achieve the
speed of 2 ´ 10 8 ms -1 . The relative magnetic dissociation lies in [NCERT Exemplar]
permeability of the medium is 1. The relative (a) visible region (b) infrared region
electrical permittivity is (c) ultraviolet region (d) microwave region
(a) 1.25 (b) 1
(c) 1.8 (d) 2.25
l
Assertion-Reasoning MCQs
12. Which of the following statement is incorrect? Direction (Q. Nos. 19-23) Each of these questions
(a) Electromagnetic waves cannot be deflected by any field. contains two statements, Assertion (A) and Reason (R).
1 Each of these questions also has four alternative
(b) The velocity of light in a medium is v = , thus it
me choices, any one of which is the correct answer. You
depends on both the electric and magnetic properties of have to select one of the codes (a), (b), (c) and (d) given
the medium. below.
(c) The constant velocity of electromagnetic waves in (a) Both A and R are true and R is the correct
vacuum is used to define a standard of time. explanation of A
(d) The direction of propagation vector k describes the (b) Both A and R are true, but R is not the correct
direction of propagation of the wave. explanation of A
13. An electromagnetic wave of frequency n = 3.0 MHz (c) A is true, but R is false
passes from vacuum into a dielectric medium with (d) A is false, but R is true
permittivity e = 4.0, then 19. Assertion When the sun shines on our hand, we
(a) wavelength is doubled and the frequency remains feel the energy being absorbed from the
unchanged electromagnetic waves (our hands get warm).
(b) wavelength is halved and frequency becomes half
(c) wavelength is halved and frequency remains unchanged Reason Electromagnetic waves transfer
(d) wavelength and frequency both remain unchanged momentum to our hand but because c is very large,
the amount of momentum transferred is extremely
14. The speed of electromagnetic wave in vacuum small and we do not feel the pressure.
depends upon the source of radiation
(a) increases as we move from g-rays to radio waves 20. Assertion Like light radiations, thermal radiations
(b) decreases as we move from g-rays to radio waves are also an electromagnetic radiations.
(c) is same for all of them Reason The thermal radiations require no medium
(d) None of the above for propagation.
CBSE Term II Physics XII 7

21. Assertion An electromagnetic wave is a (iii) A plane electromagnetic wave of frequency 25 MHz
self-sustaining oscillating wave. travels in free space along the x-direction. At a
Reason An oscillating charge produces an electric particular point in space and time, E = 6.3 $j V / m.
field in space, which produces an oscillating What is B at this point?
magnetic field, which in turn, is a source of electric (a) 2.1 ´ 10-8 k$ T (b) 2.1 ´ 108 k$ T
field and so on. (c) 3.5 ´ 106 k$ T (d) 3.0 ´ 105 k$ T
22. Assertion The frequency of the electromagnetic 1
(iv) The correct dimension of e 0 E 2 (e 0 is the
wave is naturally equal to the frequency of 2
oscillation of the charge. permittivity of free space and E is electric field), is
Reason The energy associated with the propagating (a) [ML2 T -1 ] (b) [ML-1T -2 ]
wave comes at the expense of the energy of the (c) [ML2 T -2 ] (d) [MLT -1 ]
source.
(v) A charged particle oscillates about its mean
23. Assertion Ultraviolet radiations of higher equilibrium position with a frequency of 10 9 Hz.
frequency wave are dangerous to human being. The electromagnetic waves produced
Reason Ultraviolet radiations are absorbed by the (a) will have frequency of 107 Hz
atmosphere. (b) will have frequency of 2 ´ 107 Hz
(c) will have wavelength of 05 . m
l
Case Based MCQs (d) falls in the region of radio waves
Direction Read the following passage and answer the
questions that follows
24. Oscillating Charge PART 2
An oscillating charge is an example of accelerating
charge. It produces an oscillating electric field in
Subjective Questions
space, which produces an oscillating magnetic
field, which in turn produces an oscillating electric
l
Short Answer (SA) Type Questions
field and so on. The oscillating electric and 1. How are electromagnetic waves produced by
magnetic fields regenerate each other as a wave oscillating charges? What is the source of the energy
which propagates through space. associated with the EM waves? [All India 2020]
X
2. (i) An electromagnetic wave is travelling in a
Direction of wave propagation
medium with a velocity v = v $i. Draw a sketch
E B showing the propagation of the electromagnetic
O Z
B E wave indicating the direction of the oscillating
electric and magnetic fields.
Y
(ii) How are the magnitudes of the electric and
(i) Total energy density of electromagnetic waves in magnetic fields related to velocity of the
vacuum is given by the relation electromagnetic wave? [Delhi 2013, All India 2008 C]
1 E2 B2 1 1 Or Depict the fields diagram of an electromagnetic
(a) × + (b) e0 E 2 + m 0 B 2 wave propagating along positive X-axis with its
2 e0 2m 0 2 2
electric field along Y-axis. [Delhi 2020]
E2 + B2 1 B2
(c) (d) e0 E 2 + 3. A plane electromagnetic wave travels in vacuum
c 2 2m 0
along z-direction. What can you say about the
(ii) The magnetic field of plane electromagnetic wave directions of its electric and magnetic field vectors?
is given by If the frequency of the wave is 30 MHz, what is its
B y = 2 ´ 10 -7 sin (0.5 ´10 3 x + 1.5 ´ 1011 t ). wavelength?
This electromagnetic wave is 4. The electric field of an electromagnetic wave is
(a) a visible light given by E = (50 NC -1 )sin w( t - x / c). Find the
(b) an infrared wave energy contained in a cylinder of cross-section
(c) a microwave 10 cm 2 and length 50 cm along the X-axis.
(d) a radio wave
8 CBSE Term II Physics XII

5. An electromagnetic wave is travelling in vacuum 14. Name the electromagnetic waves with their
8
with a speed of 3 ´ 10 m/s. Find its velocity in a frequency range, produced in
medium having relative electric and magnetic (a) some radioactive decay,
permeabilities 2 and 1, respectively. [Delhi 2019] (b) sparks during electric welding and
6. Even though an electric field E exerts a force qE on (c) TV remote. [All India 2020]
a charged particle yet electric field of an
electromagnetic wave does not contribute to the
l
Long Answer (LA) Type Questions
radiation pressure (but transfers energy). Explain. 15. Suppose that, the electric field amplitude of an
[NCERT Exemplar] electromagnetic wave is E 0 = 120 N/C and that its
7. Show that the radiation pressure exerted by an EM frequency is n = 50.0 MHz.
I
wave of intensity I on a surface kept in vacuum is . (i) Determine, B 0 , w, k and l.
c (ii) Find expressions for E and B.
[NCERT Exemplar]
16. Suppose that, the electric field part of an
8. (i) Why are infrared waves often called heat waves? electromagnetic wave in vacuum is
Explain. E = [3.1 cos {1.8 y + (5.4 ´ 10 6 t )}] $i
(ii) What do you understand by the statement, (i) What is the direction of propagation?
“electromagnetic waves transport
momentum”? [CBSE 2018]
(ii) What is the wavelength l?
(iii) What is the frequency n?
9. Identify the electromagnetic wave, whose
wavelengths vary as (iv) What is the amplitude of the magnetic field part
of the wave?
(i) 10 -12 m < l < 10 -8 m and (v) Write an expression for the magnetic field part of
(ii) 10 -3 m < l < 10 -1 m. the wave. [NCERT]
Write one use for each. [All India 2017] 17. In a plane electromagnetic wave, the electric field
10. (i) Arrange the following electromagnetic waves in oscillates sinusoidally at a frequency of
the descending order of their wavelengths. 2.0 × 10 10 Hz and amplitude 48 V/m.
(a) Microwaves (i) What is the wavelength of the wave?
(b) Infrared rays (ii) What is the amplitude of the oscillating magnetic
(c) Ultraviolet radiation field?
(d) g-rays (iii) Show that, the average energy density of the E
(ii) Write one use each of any two of them. field equals the average energy density of the B
[Delhi 2013 C] field. (Take, c = 3 × 10 8 m/s)
11. Name the constituent radiation of electromagnetic 18. (i) Which segment of electromagnetic waves has
spectrum which is used for highest frequency? How are these waves
produced? Give one use of these waves.
(i) aircraft navigation and
(ii) Which EM waves lie near the high frequency
(ii) studying the crystal structure.
end of visible part of EM spectrum? Give its one
Write the frequency range for each. [Delhi 2011C] use. In what way, this component of light has
12. Use the formula l m T = 0.29 cm-K to obtain the harmful effects on humans? [Foreign 2016]
characteristic temperature ranges for different 19. Answer the following questions.
parts of the electromagnetic spectrum. What do the (i) Show, by giving a simple example, how EM waves
numbers that you obtain tell you? carry energy and momentum.
13. Answer the following questions. (ii) How are microwaves produced? Why is it
(i) Find the energy stored in a 90 cm length of a necessary in microwaves ovens to select the
laser beam operating at 6 mW. frequency of microwaves to match the resonant
(ii) Find the amplitude of electric field in a parallel frequency of water molecules?
beam of light of intensity 17.7 W/ m 2 . (iii) Write two important uses of infrared waves.
[Delhi 2014 C]
CBSE Term II Physics XII 9

20. Answer the following questions. (iii) 2.7 K (temperature associated with the isotropic
(i) Name the EM waves which are used for the radiation filling all space thought to be a relic of
treatment of certain forms of cancer. Write the big-bang origin of the universe).
their frequency range. (iv) 5890 Å-5896 Å (double lines of sodium).
(ii) Thin ozone layer on top of stratosphere is
(v) 14.4 keV (energy of a particular transition in 57 Fe
crucial for human survival. Why?
nucleus associated with a famous high resolution
(iii) Why is the amount of the momentum spectroscopic method (Mössbauer spectroscopy).
transferred by the EM waves incident on the [NCERT]
surface so small? [Delhi 2014] l
Case Based Questions
21. State clearly how a microwave oven works to
heat up a food item containing water molecules. Direction Read the following passage and answer the
questions that follows
Why are microwaves found useful for the raw
systems in aircraft navigation? [Foreign 2011] 24. X-ray
22. Name the parts of the electromagnetic spectrum X-ray is a type of radiation known as electromagnetic
which is waves. It helps in creating pictures of the inside of
(i) suitable for RADAR systems in aircraft human body. These images shows the different parts
navigations. of the body in various shades of black and white. It is
due to the difference in amount of absorption by
(ii) used to treat muscular strain.
various tissues in the body.
(iii) used as a diagnostic tool in medicine.
As calcium in bones absorbs most of the X-rays, so
Write in brief, how these waves can be bones look white in colour. Fat and other soft tissues
produced? [All India 2015]
absorbs less and depicts grey colour.
23. Given below are some famous numbers (i) To which part of the electromagnetic spectrum
associated with electromagnetic radiations in does a wave of frequency 2 ´ 1018 Hz belong?
different contexts in Physics. State the part of the
electromagnetic spectrum to which each belongs. (ii) What is the range of wavelength for X-rays?
(i) 21 cm (wavelength emitted by atomic hydrogen (iii) How are the X-rays produced?
in interstellar space). (iv) What are the techniques by which X-rays can be
(ii) 1057 MHz ( frequency of radiation arising from detected?
two close energy levels in hydrogen, known as (v) Mention any two uses of X-rays.
Lamb shift ).
Chapter Test
Multiple Choice Questions Short Answer Type Questions
1. If e0 and m 0 are the electric permittivity and magnetic 6. A radio can tune into any station from 5.5 MHz to
permeability of free space and e and m are the 16 MHz band. What is the corresponding wavelength
corresponding quantities in the medium, the index of band? (Ans. 54.5 m, 18.75 m)
refraction of the medium in terms of above parameter is 7. When a plane electromagnetic wave travels in
1/ 2
em æ em ö vacuum along y-direction. Write the
(a) (b) çç ÷
÷
e0 m 0 è e0 m 0 ø (i) ratio of the magnitudes and
1/ 2
æe m ö æe m ö (ii) the direction of its electric and magnetic field
(c) çç 0 0 ÷÷ (d) çç 0 0 ÷÷ vectors.
è em ø è em ø
2. The ratio of contributions made by the electric field and 8. Green light of mercury has a wavelength
5 ´ 10 -5 cm.
magnetic field components to the intensity of an EM
wave is (i) What is the frequency (in Hz) and period (in s) in
(a) c : 1 (b) c 2 : 1 vacuum? (Ans. 6 ´ 10 14 and 0.16 ´ 10 - 14 )
(c) 1 : 1 (d) c : 1 (ii) What is the wavelength in glass, if refractive index
of glass is 1.5? (Ans. 3.3 ´ 10 - 3 m)
3. An electromagnetic wave going through vacuum is
described by E = E0 sin (kx - wt ) and B = B0 sin ( kx - wt ) . 9. Name the electromagnetic radiation to which waves
Which of the following equations is true? of wavelength in the range of 10 -2 m belong. Give
one use of this part of electromagnetic spectrum.
(a) E 0k = B0w (b) E 0w = B0k
(c) E 0B0 = wk (d) None of these
10. Find the wavelength of electromagnetic wave of
frequency 5 ´ 10 10 Hz in free space. Give its two
4. An electromagnetic wave travelling in the x-direction is applications. (Ans. 6 ´ 10 - 12 m)
described by the electric field
11. How are X-rays different from g-rays? Give a detailed
æVö æ xö
E y = 300 ç ÷ sin wçt - ÷ description.
è ø
m è cø
Long Answer Type Questions
An electron is constrained to move along the y-direction
with a speed of 2 ´ 10 7 ms -1 . The maximum electric force 12. Answer the following questions.
on the electron is (i) Which part of electromagnetic spectrum is
(a) 9. 4 ´ 10 -7
N (b) 4.8 ´ 10 -17
N absorbed from sunlight by ozone layer?
(c) 4.8 ´ 10 -7 N (d) 9. 4 ´ 10 -17 N (ii) Welders wear special glass goggles while working.
Explain, why.
5. A plane electromagnetic wave, has frequency of
2.0 ´ 10 10 Hz and its energy density is 1.02 ´ 10 -8 J / m 3 in (iii) Why are infrared waves often called as heat
vacuum. The amplitude of the magnetic field of the wave waves? Give their one application.
1 N - m2 13. (i) Identify the part of the electromagnetic spectrum
is close to (Take, = 9 ´ 10 9 and speed of light
4 pe0 C2 used in (a) radar and (b) eye surgery. Write their
frequency range.
= 3 ´ 10 8 ms -1 )
(a) 190 nT (ii) Prove that the average energy density of the
(b) 160 nT oscillating electric field is equal to that of the
(c) 180 nT oscillating magnetic field.
(d) 150 nT

Answers
Multiple Choice Questions For Detailed Solutions
1. (b) 2. (c) 3. (a) 4. (b) 5. (b) Scan the code
CBSE Term II Physics XII 11

EXPLANATIONS
E0
PART 1 As, B0 =
c
1. (c) We can rephrase Faraday’s law of electromagnetic 1 $ 1
induction by saying that a magnetic field, changing with \ B = ( k ´E ) = ( k$ ´ E )
c w
time, give rise to an electric field.
Thus, the statement given in option (c) is correct, rest are The associated electric field can be written in terms of
incorrect. magnetic field as
2. (a) Given, E = E0 sin ( kx - wt ) E = c ( B ´ k$ )
Comparing this equation with standard equation, we get Angle between k$ and E is 90° and between k$ and B is 90°.
k Therefore, k$ × E = k E cos 90° = 0 and k$ × B = k B cos 90°= 0 .
Wavelength, l =
w 7. (d) Given, B = 5 ´ 10-8 $j T and v = 3 ´ 108 k$
1 1 Y
or l= =
nl c B
k
Therefore, is independent of wavelength.
w
X
3. (a) In electromagnetic waves, electric vector, magnetic E
vector and velocity of wave are perpendicular to one
another. Z v
Hence, as v = v $j 0 Using E = B ´ v, we have
E = E0 k$ ü E = ( 5 ´ 10-8 $j) ´ ( 3 ´ 108 k$ ) = 15 $i Vm -1
Þ ýÞ k ´ i = j
$ $ $
B = B0 i$ þ 8. (c) Given, E0 = 120 NC-1, n = 50.0 MHz =50 ´ 106 Hz
4. (d) The given wave is an electromagnetic wave. Electric and \ Angular frequency, w = 2pn
magnetic components oscillate at right angles to each other = (2 ´ 3.14 rad) (50 ´ 106 Hz)
and to the direction of propagation, i.e. wave is travelling
= 3.14 ´ 108 rads -1
along E ´ B.
w 3.14 ´ 108 rads -1
Y Wave constant, k = = = 1.05 rad m -1
c 3 ´ 108 ms -1
According to the condition given in the question, the wave
X is propagating along X-axis, this means E should be along
E´B Y-axis and B should be along Z-axis.
Clearly, E = E0 sin ( kx - wt ) $j
Z
= [120 NC -1 sin { (1.05 rad m -1 ) x
Hence, B is along the –Z-axis at that time.
- (3.14 ´ 108 rad s -1 ) t}] $j
5. (b) Given, magnetic field part of harmonic electromagnetic
9. (c) Comparing the given equation,
wave, B0 = 510 nT
E N éæ rad ö æ -2 rad ö ù
Speed of light in vacuum, c = 0 Ey = 2.5 cos ê ç 2p ´ 106 ÷ t - ç p ´ 10 ÷x
B0 C ëè mø è s ø úû
where, E0 is the electric part of the wave. With the standard equation,
E0 Ey = E0 cos ( wt - kx )
Þ 3 ´ 108 =
510 ´ 10- 9 we get w = 2pn = 2p ´ 106
\ n = 106 Hz
or E0 = 153 NC -1
Moreover, we know that,
Thus, the amplitude of the electric field part of wave is
2p
153 NC -1. = k = p ´ 10-2 m -1
l
6. (c) Suppose an electromagnetic wave is travelling along Þ l = 200m
negative z-direction. Its electric field is given by As direction of field E of electromagnetic wave is in
E = E0 cos( kz - wt ) y-direction, so the wave is moving along positive x-direction
which is perpendicular to Z-axis. It acts along negative with frequency 106 Hz and wavelength 200 m.
y-direction. 10. (a) Direction of propagation of an electromagnetic
The associated magnetic field B in electromagnetic wave is wave is given by E ´ B.
along X-axis, i.e. along k$ ´ E. \A unit vector in the direction of propagation.
12 CBSE Term II Physics XII

E ´ B k$ ´ ( 2$i - 2$j) Thus, for the reflected wave z$ = - z$ , $i = - $i and additional


= =
|E ´B| |E ´ B | phase of p in the incident wave.
æQ E ´ B = k$ ´ ( 2$i - 2$j) ö Given, E = E0 i$ cos( kz - wt )
ç ÷
ç The reflected electromagnetic wave is given by
= 2( k ´ i ) - 2( k ´ j) ÷
$ $ $ $
ç ÷ E r = E0 ( - i$ ) cos[ k( - z ) - wt + p ]
ç = 2( $j) - 2( - $i ) ÷
ç ÷ = - E i$ cos[ -( kz + wt ) + p ]
0
ç = 2 j + 2i
$ $
÷ = E0 $i cos[ -( kz + wt ) = E0 i$ cos( kz + wt )]
ç 2 2 ÷
èQ|E ´ B| = 2 + 2 = 2 2 ø 16. (a) Intensity or power per unit area of the radiations,
2$j + 2$i I
= I = pc Þ p =
2 2 c
$i + $j 0.5
= = = 0.166 ´ 10-8 Nm -2
2 3 ´ 108
Energy
11. (d) Given, v = 2 ´ 108 ms -1 and m r = 1 17. (b) As we know, momentum, p =
c
The speed of electromagnetic waves in a medium is given by
Power ´ Time
1 =
v= …(i) c
me
Given, P = 30mW = 30 ´ 10-3 W
where, m and e are absolute permeability and absolute
t = 100ns = 100 ´ 10-9 s
permittivity of the medium, respectively.
Now, m = m 0 m r and e = e0 er c = 3 ´ 108 ms -1
1 1 1 30 ´ 10-3 ´ 100 ´ 10-9
\ Eq. (i) becomes, v = = ´ Þ p=
m 0m r e0 er m 0 e0 m r er 3 ´ 108
= 1.0 ´ 10-17 kg-ms -1
æ ö
Þ v=
c çQ c = 1 ÷ 18. (c) Given, energy required to dissociate a carbon monoxide
m r er ç m 0 e0 ÷
è ø molecule into carbon and oxygen atoms E = 11 eV
On squaring both sides, we get We know that, E = hn, where h = 6.62 ´ 10-34 J-s
c2 (3 ´ 108 ) 2 n = frequency Þ 11 eV = hn
er = 2 = = 2.25 and
v m r (2 ´ 108 ) 2 ´ 1 11 ´ 1.6 ´ 10-19
Þ n= J
12. (c) The statement in option (c) is incorrect and it can be h
corrected as, 11 ´ 1 . 6 ´ 10-19
The constancy of the velocity of electromagnetic waves in = J
6.62 ´ 10-34
vacuum is used to define a standard of length. The meter is
defined as the distance travelled by light in vacuum in a time = 2.65 ´ 1015 Hz
( 1 / c ) second. This frequency radiation belongs to ultraviolet region.
13. (c) In vacuum, e0 = 1, 19. (a) When the sun shines on our hand, we feel the energy
In medium, e = 4 being absorbed from the electromagnetic waves (our hands
get warm).
So, refractive index, hn 1
n = e/ e0 = 4 / 1 = 2 Since, momentum, p = Þ pµ (Q hn = constant)
c c
l l Electromagnetic waves also transfer momentum to our hand
Wavelength, l ¢ = =
n 2 but because c is very large, the amount of momentum
c c æ cö transferred is extremely small and we do not feel the
and wave velocity, v = = çQ n = ÷
n 2 è vø pressure.
Hence, it is clear that wavelength and velocity will become Therefore, both A and R are true and R is the correct
half but frequency remains unchanged when the wave is explanation of A.
passing through any medium. 20. (b) Light radiations and thermal radiations both belong to
1 electromagnetic spectrum. Light radiation belongs to visible
14. (c) Speed of electromagnetic waves in vacuum = region while thermal radiation belongs to infrared region of
m 0 e0
EM spectrum.
Therefore, speed of EM wave in vacuum is same for all of Also, EM radiations require no medium for propagation.
them.
Therefore, both A and R are true but R is not the correct
15. (b) When a wave is reflected from denser medium, then the explanation of A.
type of wave does not change but only its phase changes by
21. (a) An oscillating charge produces an electric field in space,
180° or p rad. which produces an oscillating magnetic field, which in turn,
is a source of electric field.
CBSE Term II Physics XII 13

Thus, EM wave is a self-sustaining oscillating wave. PART 2


Therefore, both A and R are true and R is the correct
explanation of A. 1. An oscillating charge is an example of accelerating charge. It
produces an oscillating electric field, which produces an
22. (b) The oscillating electric and magnetic fields, thus
oscillating magnetic field, which in turn produces an
regenerate each other, as the wave propagates through the
oscillating electric field and so on. The oscillating electric
space and the frequency of the electromagnetic wave
and magnetic fields regenerate each other as a wave which
naturally equals the frequency of oscillation of the charge.
propagates through space.
The energy associated with the propagating wave comes at
Electric and magnetic fieds are the source of energy
the expense of the energy of the source, the accelerated
associated with EM waves.
charge.
2. (i) Given that, velocity v = v $i, i.e. the wave is propagating
Therefore, both A and R are true but R is not the correct
explanation of A. along X-axis , so electric field E is along Y-axis and
magnetic field B is along Z-axis. The propagation of
23. (b) The wavelength of these waves ranges between 4000Å to
100 Å, i.e. smaller wavelength and higher frequency. They electromagnetic wave is shown in the figure
are absorbed by atmosphere and convert oxygen into ozone. Y E
They cause skin diseases and they are harmful to eye and E B E B
cause permanent blindness. v = v $i
O X
Therefore, both A and R are true but R is not the correct
explanation of A. B E B E
B Z
24. (i) (d) The energy in EM waves is divided equally between Direction of propagation
the electric and magnetic fields.
The total energy per unit volume, U = U e + U m (ii) Speed of electromagnetic wave can be given as
1 1 B2 E E
= e0 E2 + c= 0 =
2 2m0 B0 B
(ii) (c) We have, By = 2 ´ 10-7 sin ( 0.5 ´ 103 x + 1 .5 ´ 1011 t ) where, E0 and B0 are peak values of E and B or
instantaneous values of E and B.
Comparing with the standard equation, we get
3. As we know that, the direction of electromagnetic wave is
By = B0 sin ( kx + wt ) perpendicular to both electric and magnetic fields. Here,
Þ . ´ 103
k = 05 electromagnetic wave is travelling in z-direction, then
2p electric and magnetic fields are in xy-direction and are
Þ l= = 0.01256 perpendicular to each other.
. ´ 103
05
Frequency of waves, n = 30 MHz = 30 ´ 106 Hz
The wavelength range of microwaves is 10-3 to 0.3. The
wavelength of this wave lies between 10-3 to 0.3, so the Speed, c = 3 ´ 108 m/s
equation represents a microwave. Using the formula, c = nl
(iii) (a) According to Maxwell equation, the magnitude of the Wavelength of electromagnetic waves,
electric and magnetic fields in an electromagnetic wave c 3 ´ 108 300
are related as l= = = = 10 m
n 30 ´ 106 30
E 6.3$j Vm-1
B= = Thus, the wavelength of electromagnetic waves is 10 m.
c 3 ´ 108 $i ms-1
= 21. ´ 10-8 kT $ 4. The average value of energy density (energy / volume) is
given by
1
(iv) (b) The quantity enE2 represents energy per unit 1
2 U av = e0 E02
volume. 2
Thus, it has dimensions of Total volume of the cylinder, V = A × l
Energy [ML2 T -2 ] Total energy contained in the cylinder,
= æ1 ö
Volume [L3 ] U = ( U av )( V ) = ç e0 E02 ÷( Al )
è2 ø
= [ML-1T -2 ]
Substituting the values, we have
(v) (d) Given, frequency by which the charged particles 1
oscillates about its mean equilibrium position = 109 Hz. U = ´ ( 8.86 ´ 10-12 )( 50) 2 ( 10 ´ 10-4 )( 50 ´ 10-2 )J
2
So, frequency of electromagnetic waves produced by the
= 5.5 ´ 10-12 J
charged particle, n = 109 Hz.
5. Given, velocity of electromagnetic wave in vacuum,
c 3 ´ 108
Wavelength, l = = = 0.3 m c = 3 ´ 108m/s
n 109
The range of radiowaves is 10-1 to 104 m, so frequency of Relative electric permeability, er = 2
109 Hz falls in the region of radiowaves. and magnetic permeability, m r = 1
14 CBSE Term II Physics XII

Since, velocity of electromagnetic wave in a medium can be 12. Given, l mT = 0.29 cm-K
calculated by 0.29
1 1 lm = m
v= = T ´ 100
e0 erm 0m r e0m 0 ´ m r er
Let we take, l m = 10- 6 m
1 0.29
where, =c Required absolute temperature, T = = 2900 K
e0m 0 100 ´ 10- 6
Þ v=
c
…(i) Let we take, l m = 5 ´ 10- 5 m
m r er Required absolute temperature,
3 ´ 108 3 0.29
Therefore, v= Þ v= ´ 108 m/s T= = 6000 K
2´1 2 100 ´ 5 ´ 10- 5
6. Electric field of an electromagnetic wave is an oscillating We can find the temperature for other parts of the
field which causes force on the charged particle. This electromagnetic spectrum. These number tell us about the
electric force averaged over an integral number of cycles is temperature ranges for particular part of EM waves.
zero, because its direction changes with every half cycle. So, 13. (i) The time taken by wave to move a distance 90 cm,
electric field is not responsible for radiation pressure. 90 ´ 10-2
Force F t= = 3 ´ 10-9 s
7. Pressure = = 3 ´ 108
Area A Energy contained in 90 cm length,
Force is the rate of change of momentum. U = Pt
dp
i.e. F= = 6 ´ 10-3 ´ 3 ´ 10-9
dt
Energy in time dt, = 18 ´ 10-12 J
U 1
U = dp × c Þ dp = (ii) Intensity of light, I = e0 E02 c
c 2
1 U 1
\Pressure = × Þ 17.7 = ( 8.85 ´ 10-12 ) E02 ´ 3 ´ 108
A c × dt 2
I æ U ö 4
= çQ intensity, I = ÷ Þ E0 = ´ 104
2
c è A × dt ø 3
8. (i) Infrared waves have frequencies lower than those of 2
Þ E0 = ´ 102 V/m
visible light, they can vibrate not only the electrons, but 3
also the entire atoms or molecules in the structure of the
Therefore, the amplitude of electric field in parallel beam,
surface.
2
This vibration increases the internal energy and hence E0 = ´ 102 V/m
the temperature of the structure, which is why infrared 3
waves are often called heat waves. 14. (a) Gamma rays-3 ´ 1018 Hz to 5 ´ 1022 Hz
(ii) Electromagnetic wave transports linear momentum as it
(b) Ultraviolet rays-1014 Hz to 1016 Hz
travels through space. If an electromagnetic wave
transfer a total energy U to a surface in time t, then total (c) Radio waves-54 MHz to 890 MHz
linear momentum delivered to the surface is given as 15. Given, amplitude of an electromagnetic wave, E0 = 120 N/C
U
p= Frequency of wave, n = 50 MHz = 50 ´ 106 Hz
c
E
where, c is the speed of electromagnetic wave. (i) Speed of light in vacuum, c = 0
B0
9. (i) 10-12 m-10-8 m = 01 . Å - 100Å ® X-ray
E0 120
It is used in crystallography. B0 = = = 40 ´ 10- 8
c 3 ´ 108
(ii) 10-3 m - 10-1 m = 01. cm - 10 cm ® Microwaves
or B0 = 400 ´ 10- 9 T = 400 nT
It is used in microwave over for cooking purpose.
10. (i) The decreasing order of wavelengths of electromagnetic Angular frequency of wave,
waves is w = 2pn = 2 ´ 3.14 ´ 50 ´ 106
Microwaves > Infrared > Ultraviolet radiation > g-rays w = 3.14 ´ 108 rad/s
(ii) Microwaves They are used in RADAR devices. Wave number of electromagnetic waves,
g - rays It is used in radio therapy. w 3.14 ´ 108
k= = = 1.05 rad/m
11. (i) Microwaves are used for aircraft navigation, their c 3 ´ 108
frequency range is 109 Hz to 1012 Hz. Wavelength of electromagnetic wave,
(ii) X - rays are used to study crystal structure, their c 3 ´ 108
frequency range is 1016 Hz to 1020 Hz. l= = = 6.00 m
n 50 ´ 106
CBSE Term II Physics XII 15

(ii) Expression of electric field, 1


Speed of electromagnetic waves, c =
E = E0 sin ( kx - wt ) m 0 e0
E = 120 sin (1.05 x - 3.14 ´ 108 t ) Putting in Eq. (ii), we get
1 1
Expression of magnetic field, uE = e0 B02 ×
4 m 0 e0
B = B0 sin ( kx - wt )
B = 4 ´ 10- 7 sin (1.05 x - 3.14 ´ 108 t ) 1 B02
uE = ×
16. (i) The given equation signifies that, the electromagnetic 4 m0
wave is moving along Y-axis and also in negative We may express the average energy density in EM waves
direction, so it moves in - $j -direction. 1 B2
u = e0 E02 = 0
(ii) The electric part of electromagnetic wave in vacuum, 2 2m 0
E = [3.1 cos {1.8 y + (5.4 ´ 106 t )}] $i Thus, the average energy density of the E field equals the
Comparing with standard equation, average energy density of B field.
E = E0 cos ( ky + wt ), we get 18. (i) Gamma rays has the highest frequency in the
Angular frequency, w = 5.4 ´ 106 rad/s electromagnetic waves. These rays are of the nuclear
Wave number, k = 1.8 rad/m origin and are produced in the disintegration of
The amplitude of the electric field part of the wave, radioactive atomic nuclei and in the decay of certain
sub-atomic particles. They are used in the treatment of
E0 = 3.1 N/C cancer and tumours.
2p 2p
l= = = 3.491 m (ii) Ultraviolet rays lie near the high frequency end of
k 1.8 visible part of EM spectrum. These rays are used to
Þ l = 3.5 m preserve food stuff. The harmful effect from exposure
(iii) Angular frequency, w = 2pn to ultraviolet (UV) radiation can be life threatening and
include premature aging of the skin, suppression of the
w 5.4 ´ 106 ´ 7
n= = immune systems, damage to the eyes and skin cancer.
2p 2 ´ 22
19. (i) Consider a plane perpendicular to the direction of
= 0.86 ´ 106 Hz propagation of the wave. An electric charge, on the
E0 plane will be set in motion by the electric and
(iv) As, c =
B0 magnetic fields of EM wave, incident on this plane.
Amplitude of magnetic field, This is only possible, if EM wave constitutes
E 3.1 momentum and energy. Thus, this illustrates that EM
B0 = 0 = waves carry energy and momentum.
c 3 ´ 108 (ii) Microwaves are produced by special vacuum tube like the
= 1.03 ´ 10– 8 T klystron, magnetron and Gunn diode. The frequency of
(v) Expression for the magnetic field part of wave, microwaves is selected to match the resonant frequency
B = B cos ( ky + wt ) k$
0
of water molecules, so that energy is transformed
efficiently to increase the kinetic energy of the
B = 1.03 ´ 10– 8 cos ( 1.8 y + 5.4 ´106 t ) k$ molecules. Thus, facilitating the food to cook properly.
17. Given, frequency of oscillation = 2 ´ 1010 Hz, (iii) Uses of infrared rays
Speed of wave, c = 3 ´ 108 m/s (a) In knowing the molecular structure and therapy to
and electric field amplitude, E0 = 48 V/m heal muscular pain.
c 3 ´ 108 (b) In remote control of TV, VCR, etc.
(i) Wavelength of waves, l = = = 1.5 ´ 10- 2 m
f 2 ´ 1010 20. (i) g-rays are used for the treatment of certain forms of
E cancer. Its frequency range is
(ii) Using the formula, c = 0 3 ´ 1019 Hz to 5 ´ 1022 Hz.
B0
The amplitude of the oscillating magnetic field, (ii) The thin ozone layer on top of stratosphere absorbs
E 48 most of the harmful ultraviolet rays coming from the
B0 = 0 = = 1.6 ´ 10- 7 T sun towards the earth. They include UVA, UVB and
c 3 ´ 108 UVC radiations, which can destroy the life system on
(iii) The average energy density of electric field, the earth.
1 Hence, this layer is crucial for human survival.
uE = e0 E02 …(i)
4 (iii) An electromagnetic wave transports linear momentum as
E0 it travels through space. If an electromagnetic wave
We know that, =c transfers a total energy U to a totally absorbing surface
B0
Putting in Eq. (i), we get in time t, then total linear momentum delivered to the at
surface,
1
\ uE = e0 . c 2 B02 …(ii) U hn
4 p= Þ p=
c c
16 CBSE Term II Physics XII

This means, the momentum range of EM waves is Using the formula, l m T = b = 0.29 cm-K
10-19 to 10- 41. Thus, the amount of momentum 0.29
lm = cm = 0.11 cm
transferred by the EM waves incident on the surface 2.7
is very small. This wavelength corresponds to the microwaves region
21. In microwave oven, the frequency of the microwaves is of the electromagnetic spectrum.
selected to match the resonant frequency of water (iv) This wavelength lies in the visible region of the
molecules. This leads to the vibrations of these molecules. electromagnetic spectrum.
As these vibrations increase with time, the temperature (v) Energy, E =14.4 keV = 14.4 ´ 103 ´ 1.6 ´ 10-19 J
increases leading to production of heat and this is the heat
which is responsible for the cooking of food in the oven. Frequency of wave,
As, microwaves are short wavelength radio waves, with E 14.4 ´ 1.6 ´ 10-16
n= =
frequency of order of GHz. Due to short wavelength, they h 6.6 ´ 10-34
have high penetrating power with respect to atmosphere 18
= 3.5 ´ 10 Hz
and less diffraction in the atmospheric layers. So, these
This frequency lies in the X-ray region of the electromagnetic
waves are suitable for the radar systems used in aircraft
spectrum.
navigation.
24. (i) A wave of frequency 2 ´ 1018 belong to X-rays of
22. (i) Microwaves are suitable for RADAR systems that are
electromagnetic spectrum.
used in aircraft navigation. These rays are produced by
special vacuum tubes, namely klystrons, magnetrons (ii) The range of wavelength for X-rays is around 1 nm to
and gunn diodes. 10-3 nm.
(ii) Infrared rays are used to treat muscular strain. These (iii) X-rays are produced due to change in speed of fast
rays are produced by hot bodies and molecules. moving electrons, when they collide and interact with
(iii) X-rays are used as a diagnostic tool in medicine. the target anode.
These rays are produced, when high energy electrons (iv) X-rays can be detected with the help of Geiger-Muller
are stopped suddenly on a metal of high atomic tube or GM counter and also with ionisation chamber.
number.. (v) Two applications or uses of X-rays
23. (i) This wavelength (21 cm) corresponds to the radio (a) X-rays are used in medical diagnosis and to cure
waves. malignant growths.
(ii) This frequency (1057 MHz) also corresponds to the (b) These rays are used in detecting faults, cracks, etc., in
radio waves (short wavelength). metal products.
(iii) As, T = 2.7 K
CBSE Term II Physics XII 17

CHAPTER 02

Ray Optics and


Optical Instruments
In this Chapter...
l Ray Optics l Lens
l Refraction l Prism
l Total Internal Reflection (TIR) l Optical Instruments
l Refraction at a Spherical Surface

sin i 1
Ray Optics i.e.
sin r
= m 2 or m 21
A light wave can be considered to travel from one point to
another, along a straight line joining them is called a ray of where, m 21 is constant, called refractive index of
light. A bundle of such rays constitutes a beam of light and second medium with respect to first medium.
the branch of study of light is called optics.
This is called also as Snell’s law of refraction.

Refraction Refractive Index


It is the phenomenon of bending of ray of light, when they The refractive index or index of refraction m of a material is
pass from one transparent medium to another depending on the ratio of the speed of light ( c) in vacuum to the speed of
their optical densities. light in the medium (v).
Incident ray
Normal Mathematically, refractive index is given by the relation
Reflected Speed of light in the vacuum c
ray m= =
i i Speed of light in the material v
Following are few important points related to refractive
Reflecting index
surface r (i) If n 21 > 1, r < i, then the refracted ray bends towards
the normal. In such a case, medium 2 is said to be
optically denser than medium 1.
Refracted ray
(ii) If n 21 < 1, r > i, the refracted ray bends away from the
Laws of Refraction normal. This is the case, when incident ray in a denser
medium refracts into a rarer medium.
There are two laws of refraction which are given below
(i) The incident ray, the refracted ray and the normal to Rarer Denser
the refracting surface at the point of incidence, all lie i 90° i 90°
in the same plane.
(ii) The ratio of the sine of angle of incidence to the sine r
r
of angle of refraction is constant. Rarer
Denser
18 CBSE Term II Physics XII

Principle of Reversibility of Light mg sin r2 a


= = mg ...(iii)
When a light ray, after suffering any number of reflections ma sin i2
and refractions, has its final path reversed, it travels back
Multiplying Eqs. (ii) and (iii), we get
along its entire initial path. This is called principle of
sin i2 sin r2 g
reversibility of light. In the figure given below, OA is an ´ = m a ´ am g
incident ray in medium 1 and AB is the refracted ray in sin r2 sin i2
medium 2. 1 = g m a ´ am g ,
O
Medium 1 a 1
mg = g
i ma
A
Medium 2 From Eqs. (i) and (iii), we get
sin i1 sin r2
r = ...(iv)
B sin r1 sin i2
As, i2 = r1 (alternate angles)
1 1 \ sin i2 = sin r1
m2 = 2
m1 From Eq. (iv), we get
Thus, the refractive index of medium 2 relative to medium 1 sin r2 = sin i1 or r2 = i1
is equal to the reciprocal of the refractive index of medium 1 Hence, the emergent ray KL is parallel to the incident ray
relative to medium 2. MN as shown in the figure. We observe that the incident ray
MN is displaced laterally, on suffering two refractions
Refraction of Light Through a Rectangular through a glass slab.
Glass Slab
Expression for Lateral Displacement
Let ABCD be a rectangular glass slab. A ray of light is
incident along MN on the face AB of the rectangular slab at Now, from K, draw KL¢ ^ MN produced.
Ði1 . It is refracted along NK with Ðr1 . \ Lateral displacement of the ray on passing through the
parallel slab = KL¢.
The refracted ray NK falls on face CD with Ði2 and emerges
Let ÐKNL ¢ = d = deviation on first refraction.
out along KL with Ðr2 . KL¢
In D NKL¢, sin d =
M i1 NK
ma
A B \ KL¢ = NK sin d ...(v)
N NN ¢
r1 d In D NN ¢K, cos r1 =
L¢ t NK
mg i2
NN ¢ t
\ NK = =
D C
cos r1 cos r1
ma N¢ K r L
2
P where, t = NN ¢ = thickness of glass slab.
t
From Eq. (v), we get KL¢ = sin d
Applying Snell’s law at N, cos r1
m a ´ sin i1 = m g ´ sin r1 t sin( i1 - r1 )
sin i1 m g a KL¢ =
or = = mg ....(i) cos r1
sin r1 m a
This is the required expression for lateral displacement (or
Again, applying Snell’s law at K,
shift), which is obviously proportional to thickness (t) of glass
m g ´ sin i2 = m a ´ sin r2
slab. Further, lateral displacement (or shift) will increase with
m a sin i2 g
Þ = = ma .... (ii) increasing angle of incidence ( i1 ).
m g sin r2
According to the principle of reversibility of light, when final
Apparent Depth and Normal Shift
path of a light ray after suffering a number of reflections and When an object is in denser medium and observer is in rarer
refractions is reversed, then the ray retraces its entire path. medium, then object appears to be at lesser depth than its
actual depth.
Now, imagine a plane mirror P held normal to KL so that on
reflection from mirror, path KL is reversed. The ray would Real and apparent depth are related as follows
retrace its entire path. For the reversed ray, the application Real depth ( h )
n 21 =
of Snell’s law at K gives Apparent depth ( h ¢ )
m a ´ sin r2 = m g ´ sin i2
CBSE Term II Physics XII 19

Effect of Atmospheric Refraction at Optical Fibres These fibres are fabricated with high quality
composite glass/quartz fibres. Each fibre consists of a core
Sunrise and Sunset
and cladding such that refractive index of core is higher than
The refraction of light through the atmosphere is responsible that of the cladding. When a signal in the form of light is
for many natural phenomena. e.g. directed at one end of the fibre at a suitable angle, it
(i) Due to refraction, the sun is visible a little before undergoes repeated total internal reflection along the length
the actual sunrise and until a little after the actual of the fibre and finally, comes out from other end.
sunset. Thus, these are extensively used for transmitting audio and
(ii) The apparent flattening (oval shape) of the sun at video signals through long distances.
sunset and sunrise is also due to the same Prisms Prisms are designed to bend ray by 90° and 180° or to
phenomenon. invert image without changing its size by the use of total
internal reflection.
Critical Angle
The angle of incidence is in denser medium for which the
angle of refraction in rarer medium becomes 90° is called
Refraction at a Spherical Surface
critical angle. If an object is placed in a medium of refractive index n1 at a
m2 distance u from the pole of a spherical surface of radius of
ic Denser curvature R and after refraction, its image is formed in a
medium of refractive index n 2 at a distance v, then
90 Rarer n 2 n1 n 2 - n1
- =
m1 v u R
1 This equation holds for any curved spherical surface.
1
Þ m2 =
sin ic
n1 N n2
where, ic is critical angle. i
r

Total Internal Reflection (TIR) O C I


M
A ray of light travelling from denser medium to rarer medium R
is incident at the interface of two media at an angle greater u v
than the critical angle for the two media, then the ray is Refraction at a spherical surface
totally reflected back to denser medium and this
phenomenon is called total internal reflection. Cartesian Sign Convention for Spherical Surfaces
B
l
The principal axis of the spherical surface is taken as X-axis
Rarer and the optical centre as origin, here the principal axis is
medium
(air) the diameter extended.
r r¢ Water-air
O1 O2 O3 D O4 interface l
The direction of the incident light is taken as the positive
ic i > ic
direction of X-axis and opposite to it is taken as negative.
i i¢ N Totally
N N
reflected ray
l
The upward direction is taken as positive and the
Denser
medium
Partially
downward direction as negative.
(water)
A C reflected rays
Lens
There are some applications of total internal reflection which A lens is a transparent medium bounded by two surfaces of
are given below which one or both surfaces are spherical.
Mirage It is the phenomenon, in which an inverted image of Lenses are of two types as given below
distant tall objects cause an optical illusion of water. This (i) Convex or Converging Lens A lens which is thicker
type of mirage is especially common in hot deserts. at the centre and thinner at its ends is called convex
Looming The optical illusion of an object floating in air is lens. Convex lenses are of three types as shown below
called superior mirage. It is also known as looming.
This occurs in very cold regions due to total internal
reflection.
Diamond The critical angle for diamond-air interface is very
small, therefore once light enters a diamond, it is very likely
to undergo total internal reflection inside it. Due to this,
(a) Double convex (b) Plano-convex (c) Concavo-convex
diamond shines brilliantly. lens lens lens
20 CBSE Term II Physics XII

(ii) Concave or Diverging Lens A lens which is thinner (b) Second Principal Focus It is a point on the
at the centre and thicker at its ends is called a principal axis at which the rays coming parallel to
concave lens. the principal axis converge (convex lens) or
passing through it appear to diverge (concave
lens) at this point after refraction from the lens.

(a) Double concave (b) Plano-concave (c) Convexo-concave O F2 F2 O


lens lens lens

Converging and Diverging Action of Lenses f2 f2


Convex lens or converging lens are the lens which converges
Both the foci of convex lens are real, while that of
all the light rays, coming parallel to its principal axis.
concave lens are virtual.
Concave lens or diverging lens diverges all the light rays
coming parallel to its principal axis. (vi) Aperture The effective diameter of the circular
outline of a spherical lens is called its aperture.
(vii) Refractive Axis It is an imaginary axis at the optical
centre perpendicular to the principal axis which
represents the lens.

Converging lens Diverging lens

Some Definitions Related to Lens


(a) Real path of ray (b) Path of ray as shown with
Important terminologies related to lenses are given below
reference to refractive axis
(i) Optical Centre The optical centre is a point lying on
the principal axis of the lens, directed to which Image Formation in Lenses
incident rays pass without any deviation in the path.
Using Ray Diagrams
P P We can represent image formation in lenses using ray
diagrams. For drawing ray diagrams in lenses like spherical
C mirrors, we consider any two of the following rays
O O
Optical S S (i) Rays which are parallel to the principal axis after
Optical
centre centre refraction, will pass through principal focus in case of
convex lens and will appear to be coming from
(ii) Centre of Curvature The centres of the two principal focus in case of concave lens.
imaginary spheres of which the lens is a part, are
called centres of curvature of the lens.
F1 F2 F1 F2
(iii) Radii of Curvature The radii of the two imaginary
spheres of which the lens is a part are called radii of
curvature of the lens.
(iv) Principal Axis The imaginary line joining the two (ii) Rays passing through or directed to the focus will
centres of curvature is called principal axis of lens. emerge parallel to the principal axis.
(v) Principal Focus Lens has two principal foci
(a) First Principal Focus It is a point on the F2
F1
principal axis of lens, the rays starting from this
O 2F1 F1 O F2 2F2
point in convex lens or rays directed to this point
in concave lens become parallel to principal axis
after refraction.
(iii) Rays directed to optical centre will emerge out
undeviated.

F1 O F1 F2
F1 O F2 F1 O
f1 f1
CBSE Term II Physics XII 21

Refraction by a Lens : Lens Maker’s Formula If f 1 = f 2 , then f is infinite, i.e. combination will behave
1 æ 1 1 ö æ n2 ö like plane glass sheet.
= ( 1 n 2 - 1) ç - ÷ çQ 1 n 2 = ÷
f è R1 R 2 ø è n1 ø If the lenses are placed d distance apart, then
This is known as lens Maker's formula. 1 1 1 d
= + -
If the lens is thin, then lens Maker’s formula is given as f f1 f 2 f1 f 2
1 1 1
- =
v u f Prism
This is the thin lens formula. A prism is a portion of a transparent medium bounded by two
where, f is focal length of lens, v is image distance and u is plane faces inclined to each other at a suitable angle.
object distance.
Refraction of Light Through a Prism
Linear Magnification The figure below shows the passage of light through a
Produced by a Lens (m) triangular prism ABC.
It is defined as the ratio of the height of the image to height A
of the object.
h¢ v A K
Linear magnification, m = =
h u
N T d
For erect (and virtual) image, m is positive and for an Q e
i
inverted (and real) image, m is negative. r1 r2
R
S
Power of a Lens P O
B C
It is the ability to converge or diverge the rays of light
incident on it. The SI unit of power of lens is dioptre (D). The angles of incidence and refraction at first face AB are i
The power of a lens is measured as the reciprocal of its focal and r1 . The angle of incidence at the second face AC is r2 and
length (in metre). the angle of emergence is e.
1
P= The angle between the emergent ray RSand incident ray PQ
f (in m) is called angle of deviation (d).
d = ( i + e) - ( r1 + r2 )
Combination of Thin Lenses in Contact
Q r1 + r2 = A
If several thin lenses of focal lengths f 1 , f 2 , f 3 , ¼ are in
contact, then the effective focal length and power of their \ d = ( i + e) - A
combination is given by If m is the refractive index of material of the prism, then
1 1 1 1 d = (m - 1) A
= + + +.. .
f f1 f2 f3 This is the angle through which a ray deviates on passing
and P = P1 + P2 + P3 +.. . through a thin prism of small refracting angle A.

Magnification by Combination of Lenses Prism Formula


Combination of lenses helps to obtain diverging or If the angle of incidence is increased gradually, then the
converging lens of desired magnification. It also enhances angle of deviation first decreases, attains a minimum value
sharpness of the image. Thus, the net magnification of such a ( d m ) and then again starts increasing.
combination (m) is given as
Angle of deviation

m = m1 ´ m 2 ´ m 3 ´ … d
l
If combination of lenses consists of one convex lens ( f 1 )
and one concave lens ( - f 2 ), then dm
i =e
f f
f = 1 2
f 2 - f1 i e
Angle of incidence
l
If f 1 > f 2 , then f is negative, i.e. combination will behave
When angle of deviation is minimum, the prism is said to be
like concave lens, when focal length of convex lens is larger. placed in the minimum deviation position.There is only one
If f 1 < f 2 , then f is positive, i.e. combination will behave angle of incidence for which the angle of deviation is
like convex lens, when focal length of convex lens is smaller. minimum.
22 CBSE Term II Physics XII

When d = d m (prism in minimum deviation position) Case I When the image is formed at the near point
e=i A¢
and r2 = r1 …(i) A¢¢ A
Q r1 + r2 = A a b
Þ r+r=A B¢ FB C
A
or r=
2 D

Also, we have b
Magnifying power, m =
A+ d = i+ e ...(ii) a
D
Putting d = d m and e = i in Eq. (ii), we get m=1+
f
A + dm = i + i
(Q v = - D, because image is formed at near point)
æ A + dm ö
Þ i= ç ÷ When the eye is placed behind the lens at a distance a, then
è 2 ø
D-a
m=1+
sin i f
From Snell’s law, m =
sin r Case II When the image is formed at infinity
D
æ A + dm ö m=
sin ç ÷ f
è 2 ø
\ m=
A Compound Microscope
sin
2 It consists of two convex lenses coaxially separated by some
This relation is called a prism formula. distance. The lens nearer to the object is called the objective.
The lens through which the final image is viewed is called
For thin prisms (i.e. A is very small), the value of d m is also
the eyepiece.
very small. Objective lens Eyepiece
æ A + dm ö
sin ç ÷ B A¢¢
è 2 ø A + dm
So, m= » Q
a
A 2 A Fo C Fo Fe A¢ b C¢
sin
2 A/ 2 uo fo
Þ d m = (m - 1) A B¢
ue
B¢¢
Optical Instruments vo
fe
Using the reflecting and refracting properties of mirrors, D
lenses and prisms, many optical instruments have been Angular magnification or magnifying power of a compound
designed like microscopes and telescopes. Our eye is a microscope is defined as the ratio of the angle b subtended by
natural optical device. the final image at the eye to the angle a subtended by the
object seen directly, when both are placed at least distance of
The Eye distinct vision.
The eye lens is a convex lens whose focal length can be b
\ Angular magnification, m =
modified by the ciliary muscles. This property of eye is called a
tan b
accommodation. The image is formed on a film of nerve For small angles, m =
fibres called retina. tan a
Near point is the closest distance for which the lens can form The magnification produced by the compound microscope is
the product of the magnification produced by the eyepiece
image and its value is 25 cm for a normal eye. The far point of
and objective.
a normal eye is infinity.
m = me ´ mo
Simple Microscope where, m e and m o are the magnifying powers of the eyepiece
and objective, respectively.
It is an optical instrument which forms large image of close
and minute objects. It is a converging lens of small focal Case I When the final image is formed at near point
Linear magnification is given by
length. When an object is at a distance less than the focal
length of the lens, the image obtained is virtual, erect and D
me = 1 +
magnified. fe
CBSE Term II Physics XII 23

where, f e is focal length of the eyepiece. Case II When final image is formed at near point
æ v öæ Dö fo ue
m = ç1 - o ÷ ç1 + ÷
è fo ø è fe ø Parallel rays from
object at infinity
where, v o , f o and f e are image distance, focal length Fe Fo b
a B¢¢
of objective lens and focal length of eyepiece, C1 B¢ C2
Eye
respectively. A¢
Case II When the final image is at infinity fe

If u o is the distance of the object from the objective A¢¢


and v o is the distance of the image from the objective, D
v
then the magnifying power of the objective, m o = o b
uo Angular magnification, m =
a
When the final image is at infinity, then angular f
magnification is given by m= o
D - ue
me =
fe fo æ fe ö
m=- ç1 + ÷
The total magnification, when image is at infinity is fe è Dø
given by
æv Dö Reflecting (Cassegrain) Telescope
m = mo ´ m e = ç o ´ ÷
è uo fe ø It consists of concave mirror of large aperture and large focal
If the object is very close to the principal focus of the length (objective). A convex mirror is placed between the
objective and the image formed by the objective is concave mirror and its focus. A small convex lens works as
very close to the eyepiece, then eyepiece.
-L D Objective
m= × mirror
fo fe Secondary
mirror
where, L = length of the tube of microscope.
Eyepiece
Astronomical (Refracting) Telescope
An optical instrument which is used for observing distinct
images of heavenly bodies like stars, planets, etc., when the One popular configuration of mirror and eyepiece is called
final image is formed at infinity. the Newtonian reflecting type telescope, named after its
designer Newton.
Case I When the final image is formed at infinity
Objective lens Eyepiece M1= Parabolic mirror
fo fe
Parall
from oel rays Eyepiece
infinity bject at Rays arrive
Fo, Fe parallel from
a B
C1 a I b C2 very distant
Eye
object
age M2= Plane mirror
im
in al inity The plane mirror reflects the beam and a real image is
F in f
at formed infront of eyepiece. The eyepiece acts as a magnifier
Angular magnification is given by and the final magnified image of the distant object can be
b observed by the eye.
m=
a Advantages of Reflecting Telescope
Since, b and a are very small. over Refracting Telescope
I I For astronomical telescope, the mirror affords several
tan a = and tan b =
fo - fe advantages over the objective lens. A mirror is easier to
where, I is the image formed by the objective, f o and produce with a larger diameter, so that it can intercept rays
f e are the focal lengths of objective and eyepiece, crossing a larger area and direct them to the eyepiece.
respectively. The mirror can be made parabolic to reduce spherical
f aberration. Aberration is further reduced because passage
m=- o through one layer of glass (the objective lens) is eliminated.
fe
24 CBSE Term II Physics XII

Solved Examples
Example 1. (i) Find the speed of light of wavelength From Fig. (c), i = angle of incidence = 45°.
l = 780 nm (in air) in a medium of refractive index \ Using the relation,
sin i sin 45°
m = 1 . 55. wm g = or 1 .28 =
sin r sin r
(ii) What is the wavelength of this light in the given
sin 45° 1 / 2
medium? or sin r = =
8 1.28 1 .28
c 3.0 ´ 10
Sol. (i) Speed of light, v = = 0.707
m 1 .55 = = 0552
. 5 = sin 33 .54¢
1 .28
= 1.94 ´ 108 m/s \ Angle of refraction, r = 33 .54°
(ii) Wavelength of light in the given medium,
l 780 Example 4. Locate the image of the point object O.
l medium = air = = 503 nm The point C is centre of curvature of the
m 1 .55
spherical surface.
Example 2. A printed page is kept pressed by a glass cube m = 1.0 m = 4/3
(m = 1 .5 ) of edge 9.0 cm. By what amount will the O
printed letters appear to be shifted When viewed from P C
the top?
Sol. The thickness of the cube, t = 9.0 cm. 10 cm 30 cm

The shift in the position of the printed letters, Sol. Given, m1 = 1, m 2 = 4 / 3, u = - 10 cm and R = 30 cm
æ 1ö æ 1 ö m m m - m1
Dd = ç1 - ÷ d = ç1 - ÷ ´ 9.0 cm = 3.0 cm Using the relation, 2 - 1 = 2
è mø è 1 .5 ø v u R
Substituting the values, we get
Example 3. Figs. (a) and (b) show refraction of an incident 4/ 3 1 4/ 3 - 1 4 1 1
ray in air at 60° with the normal to a glass-air and Þ - = Þ + =
v -10 30 3v 10 90
water-air interface, respectively. Predict the angle of 4 1 1 -8
refraction in glass, when the angle of incidence in Þ = - = Þ
3v 90 10 90
water is 45° with the normal to a water-glass interface v = - 15 cm
[Fig. (c)]. The image is formed 15 cm left of spherical surface and
is virtual.
35° Example 5. Double-convex lenses are to be
60°
a

Glass Air Glass manufactured from a glass of refractive index


Air Water Water 1.55, with both faces of the same radius of
60° 45°
47° curvature. What is the radius of curvature
(a) (b)
required, if the focal length is to be 20 cm?
(c)
Sol. Using the relation,
Sol. From Fig. (a),
1 æ 1 1 ö
i = angle of incidence = 60° = ( m - 1 ) çç - ÷ , we get
÷
f R
è 1 R 2ø
r = angle of refraction = 35°.
\ Refractive index of glass with respect to water, 1 é 1 æ 1 öù
= ( 1 .55 - 1 ) ê - ç ÷ú
sin i sin 60° 0.8660 20 ë R è -R ø û
am g = = = = 1 .51
sin r sin 35° 05736
. é1 1ù 2
= 055. ê + ú = 055 . ´
From Fig. (b), here, i = 60° and r = 47° ë R R û R
\Refractive index of water with respect to air, or R = 1.10 ´ 20 = 22 cm
sin i sin 60° 0.8660 Example 6. The distance between two point
am w = = = = 1 .18
sin r sin 47° 0.7314 sources of light is 24 cm. Find out where would
\ We have, am g = am w ´ wm g you place a converging lens of focal length 9 cm,
am g 1 .51 so that the images of both the sources are
or wm g = = = 1 .28 formed at the same point.
am w 1 .18
CBSE Term II Physics XII 25

f = 9 cm
Example 9. One face of a prism with a refractive angle
S1
of 30° is coated with silver. A ray incident on
S2
another face at an angle of 45° is refracted and
reflected from the silver coated face and retraces its
path. What is the refractive index of the prism?
x 24-x
1 1 1 Sol.
Sol. Using the relation, = = 90 °
v u f 45°
1 1 1 1 1 1
For S1 , - = Þ \ = - …(i)
v1 - x 9 v1 9 x
1 1 1
For S2 , – =
v2 - ( 24 - x ) 9
1 1 1 Given, A = 30°, i1 = 45° and r2 = 0
\ = - …(ii)
v2 9 24 - x Since, r1 + r2 = A
Since, sign convention for S1 and S 2 is just opposite. Hence, \ r1 = A = 30°
1 1 Now, refractive index of the prism,
v1 = - v2 or =- sin i1 sin 45° 1 / 2
v1 v2 m= = = = 2
1 1 1 1 sin r1 sin 30° 1/ 2
\ – = -
9 x 24 - x 9 Example 10. A compound microscope has a
Solving this equation we get, x = 6 cm. Therefore, the lens magnifying power of 100, when the image is formed
should be kept at a distance of 6 cm from either of the object. at infinity. The objective has a focal length of 0.5 cm
Example 7. An object of size 3.0 cm is placed at 14 cm and the tube length is 6.5 cm. Find the focal length
from concave lens of focal length 21 cm. Describe of the eyepiece.
the image produced by the lens. What happens, if Sol. When the final image is at infinity,
the object is moved further away from the lens? ue = f e = tube length - vo
1 1 1 \ f e = 6.5 – vo …(i)
Sol. Using the relation, = - , we get
f v u v D
Since, M¥ = o ×
1 1 1 1 1 uo f e
= + = +
v f u -21 ( -14) v 25 vo
\ 100 = o × or =-4 …(ii)
\ v = - 8.4 cm uo f e uo f e
\ The image is virtual, erect and located at 8.4 cm from the For the objective,
lens on the same side as the object. 1 1 1 1 1 1
- = = Þ - =2 …(iii)
Also, we know that, vo uo f o 0.5 vo uo
I v
m= = We have three unknowns vo, uo and f e solving Eqs. (i), (ii)
O u and (iii), we get
v -8.4
\ I = ´ O= ´ 3 = 1.8 cm f e = 2 cm
u -14
i.e. The image is of diminished size. Example 11. A small telescope has an objective lens of
If the object is moved away from the lens, the virtual image focal length 140 cm and an eyepiece of focal length
moves towards the focus of the lens (but never beyond 5.0 cm. What is the magnifying power of the
focus). telescope for viewing distance objects, when
Example 8. A converging lens of focal length 5.0 cm is (i) the telescope is in normal adjustment (i.e. when
placed in contact with a diverging lens of focal the final image is at infinity) and
length 10.0 cm. Find the combined focal length of (ii) the final image is formed at the least distance of
the system. distinct vision 25 cm?
Sol. Given, f 1 = + 5.0 cm and f 2 = - 10.0 cm Sol. (i) When the telescope is in normal adjustment, the
magnifying power is given by
Therefore, the combined focal length F is given by
f 140
1 1 1 1 1 1 M= o =+ = 28
= + = – =+ | f e| 5
F f1 f 2 5.0 10.0 10.0
(ii) When the final image is formed at the least distance of
\ F = + 10.0 cm distinct vision, then M is given by
i.e. The combination behaves as a converging lens of focal f æ f ö 140 æ 5.0 ö
M = o ç1 + e ÷ = ç1 + ÷ = 33.6
length 10.0 cm. | f e| è Dø 5 è 25 ø
26 CBSE Term II Physics XII

Chapter
Practice
PART 1 6. For the refraction shown below, the correct relation
is
Objective Questions n1 i
r
n2

l
Multiple Choice Questions O P C I
u R
1. A ray of light strikes an air-glass interface at an v
angle of incidence ( i = 60° ) and gets refracted at an
angle of refraction r. On increasing the angle of n 2 n1 n 2 - n1 n1 n 2 n 2 - n1
(a) - = (b) - =
incidence ( i > 60° ), the angle of refraction r v u R v u R
(a) decreases (b) remains same n1 n 2 n1 - n 2 n 2 n1 n1 - n 2
(c) is equal to 60° (d) increases (c) - = (d) - =
v u R v u R
2. The refractive indices of water and glass with respect 7. First and second focal lengths of spherical surface
to air are 4/3 and 5/3, respectively. The refractive
of refractive index n are f1 and f 2 , respectively. The
index of glass with respect to water will be
relation between them, is
(a) 1 / 3 (b) 4 / 3
(a) f 2 = f1
(c) 5 / 4 (d) 20 / 9
(b) f 2 = - f1
3. A ray of light is incident at the glass-water interface (c) f 2 = nf1
at an angle (as shown below) and it emerges finally
(d) f 2 = - nf1
parallel to the surface of water, then the value of n g
would be 8. Which of the following is true for rays coming
Air from infinity?
n w = 4/ 2 m1
Water
Glass m2
i
(a) ( 4 / 3) sin i (b) 1 /sin i
(a) Two images are formed at two different points
(c) 4 / 3 (d) 1 (b) Continuous image is formed between focal points of
upper and lower lens
4. If the critical angle for light going from medium A (c) One image is formed by the lens
to B is q. Then, find the speed of light in medium B, (d) None of the above
if speed of light is v in medium A.
v v 9. The radius of curvature of the curved surface of a
(a) v( 1 - cos q) (b) (c) (d) v( 1 - sin q) plano-convex lens is 20 cm. If the refractive index
cos q sin q
of the material of the lens be 1.5, it will
5. The phenomena involved in the reflection of [NCERT Exemplar]
radiowaves by ionosphere is similar to (a) act as a convex lens only for the objects that lie on its
[NCERT Exemplar] curved side
(a) reflection of light by a plane mirror (b) act as a concave lens for the objects that lie on its curved
(b) total internal reflection of light in air during a mirage side
(c) dispersion of light by water molecules during the (c) act as a convex lens irrespective of the side on which the
object lies
formation of a rainbow
(d) act as a concave lens irrespective of side on which the
(d) scattering of light by the particles of air object lies
CBSE Term II Physics XII 27

10. A double convex lens whose refractive index is 1.33 16. A prism has refractive angle 60°. When a light ray
has both radii of curvature of magnitude 10 cm. If is incident at 50°, then minimum deviation is
an object is placed at a distance of 5 cm from this obtained. What is the value of minimum deviation?
lens, the position of the image formed (in cm) is (a) 40° (b) 45° (c) 50° (d) 60°
(a) 7.46 same side of the object
17. In order to increase the angular magnification of a
(b) 7.46 opposite side of the object
simple microscope, one should increase the
(c) 14.45 same side of the object
(a) object size (b) aperture of the lens
(d) 14.45 opposite side of the object
(c) focal length of the lens (d) power of the lens
11. A plano-convex lens fits exactly into a plano-concave 18. F1 and F2 are focal lengths of objective and
lens. Their plane surfaces are parallel to each other. If
lenses are made of different materials of refractive eyepiece respectively, of the telescope. The angular
indices n1 and n 2 and R is the radius of curvature of magnification of the given telescope is equal to
the curved surface of the lenses, then the focal length F1 F2
(a) (b)
of the combination is F2 F1
R R FF
1 2 F1 + F2
(a) (b) (c) (d)
2( n1 + n 2 ) 2( n1 - n 2 ) F1 + F2 FF
1 2
R 2R
(c) (d) 19. An astronomical telescope has an angular
n1 - n 2 n 2 - n1
magnification of magnitude 5 for distant objects.
12. Which of the following statement is correct? The separation between the objective and the
(a) Power of a lens is a measure of the convergence or eyepiece is 36 cm and the final image is formed at
divergence, which a lens introduces in the light falling on it. infinity. The focal length f o of the objective and the
(b) Power is defined as the cosine of angle by which it focal length f e of the eyepiece are
converges or diverges a beam of light falling at unit (a) 45 cm and - 9 cm (b) - 7.2 cm and 5 cm
distance from the optical centre.
(c) 50 cm and 10 cm (d) 30 cm and 6 cm
(c) Cutting of lenses helps to obtain diverging or converging
lenses of desired magnification.
(d) Total magnification m of the combination is the sum of
l
Assertion-Reasoning MCQs
magnification ( m1 , m 2 , m 3 .....) of individual lenses. Direction (Q. Nos. 20-25) Each of these questions
13. A biconcave lens of power P vertically splits into contains two statements Assertion (A) and Reason (R).
two identical plano-concave parts. The power of Each of these questions also has four alternative
each part will be choices, any one of which is the correct answer. You
(a) 2P (b)
P have to select one of the codes (a), (b), (c) and (d) given
2 below.
P
(c) P (d) (a) Both A and R are true and R is the correct
2 explanation of A.
14. A light ray incident normally on one of the face of a (b) Both A and R are true, but R is not the correct
triangular prism follow the path as shown below, explanation of A.
then the angle of refraction r2 at the second face is (c) A is true, but R is false.
30º (d) A is false and R is also false.
r2 20. Assertion Refractive index of glass with respect to
air is different for red light and violet light.
Reason Refractive index of a pair of media does
(a) 60° (b) 90°
not depends on the wavelength of light used.
(c) 30° (d) 45°
15. White light is incident on one of the refracting 21. Assertion The refractive index of diamond is 6
surfaces of a prism of angle 5°. If the refractive and that of liquid is 3. If the light travels from
indices for red and blue colours are 1.641 and 1.659 diamond to the liquid, it will be totally reflected
respectively, the angular separation between these when the angle of incidence is 30°.
two colours when they emerge out of the prism is Reason n = sin i c , where n is the refractive index of
(a) 0.9° (b) 0.09° diamond with respect to liquid and i c is the critical
(c) 1.8° (d) 1.2° angle.
28 CBSE Term II Physics XII

22. Assertion When monochromatic light is incident (ii) A ray of light will undergo total internal reflection
on a surface separating two media, the reflected inside the optical fibre, if it
and refracted light both have the same frequency (a) goes from rarer medium to denser medium
as the incident frequency. (b) incident at an angle less than the critical angle
(c) strikes the interface normally
Reason Speed of light and wavelength of light
(d) incident at an angle greater than the critical angle
does not changes in refraction and hence the ratio
n = c/ l is not a constant. (iii) If in core, incidence angle is equal to critical angle,
then refraction angle will be
23. Assertion Propagation of light through an optical
(a) 0° (b) 45°
fibre is due to total internal reflection taking place
(c) 90° (d) 180°
at the core-clade interface.
(iv) In an optical fibre (shown), correct relation of
Reason Refractive index of the material of the
refractive indices of core and cladding is
core of the optical fibre is greater than that of
n2
cladding.
24. Assertion The property of convergent lens of Core n1
converging rays remain same in all media. Cladding
Reason Property of lens, whether the rays are
(a) n1 = n 2 (b) n1 > n 2
diverging or converging does not depends on the
(c) n1 < n 2 (d) n1 + n 2 = 2
surrounding medium.
(v) If the value of critical angle is 30° for total internal
25. Assertion Secondary rainbow is fainter than reflection from given optical fibre, then speed of
primary rainbow. light in that fibre
Reason Secondary rainbow is formed by a four step (a) 3 ´ 108 m/s (b)1.5 ´108 m/s
process and hence, the intensity of light is reduced at (c) 6 ´108 m/s (d) 4.5 ´108 m/s
the second reflection inside the rain drop.
l
Case Based MCQs PART 2
Direction Read the following passage and answer the
questions that follows
Subjective Questions
26. Optical Fibre l
Short Answer (SA) Type Questions
An optical fibre is a thin tube of transparent
material that allows light to pass through, without 1. When monochromatic light travels from a rarer to a
being refracted into the air or another external denser medium, explain the following, giving
reasons.
medium. It makes use of total internal reflection.
(i) Is the frequency of reflected and refracted light
same as the frequency of incident light?
(ii) Does the decrease in speed imply a reduction in
the energy carried by light wave? [Delhi 2013]
Light Core
ray Cladding 2. Mention any two situations in which Snell’s law of
refraction fails.
These fibres are fabricated in such a way that, light 3. A ray of light is incident at an angle of 45° on one
reflected at one side of the inner surface strikes the face of a rectangular glass slab of thickness 10 cm
other at an angle larger than critical angle. Even, if and refractive index 1.5. Calculate the lateral shift
fibre is bent, light can easily travel along the length. produced.
(i) Which of the following is based on the 4. Why does the sun rising in the sky appear oval in
phenomenon of total internal reflection of light? shape?
(a) Sparkling of diamond
(b) Optical fibre communication
5. Show analytically from the lens equation that when
(c) Instrument used by doctors for endoscopy
the object is at the principal focus, the image is
(d) All of the above
formed at infinity.
CBSE Term II Physics XII 29

6. A student measures the focal length of a convex From which face will the ray emerge? Justify your
lens by putting an object pin at a distance u from answer. [All India 2016]
A
the lens and measuring the distance v of the P
image pin. What will be the graph drawn between
u and v ? Q

7. A magician during a show makes a glass lens


n = 1.47 disappear in a trough of liquid. What is the B
60°
C
refractive index of the liquid? Could the liquid be
water?
16. An equilateral glass prism has a refractive index 1.6
in air. Calculate the angle of minimum deviation of
8. A tank is filled with water to a height of 12.5 cm. the prism, when kept in a medium of refractive
The apparent depth of a needle lying at the bottom 4 2
of the tank is measured by a microscope to be index .
5 [Delhi 2019]
9.4 cm. What is the refractive index of water? If
water is replaced by a liquid of refractive index 1.63 17. Is it possible to increase the range of a telescope by
upto the same height, by what distance would the increasing the diameter of the objective lens?
microscope have to be moved to focus on the needle 18. Explain two advantages of a reflecting telescope
again? [NCERT] over a refracting telescope.
9. What should be the position of the object relative 19. A small telescope has an objective lens of focal
to the biconvex lens, so that this lens behaves like a length 144 cm and an eyepiece of focal length 6 cm.
magnifying glass? What is the magnifying power of the telescope?
What is the separation between the objective and
10. How does the magnification of a magnifying glass the eyepiece? [NCERT]
differ from its magnifying power?
20. The objective of an astronomical telescope has a
11. Calculate the radius of curvature of an diameter of 150 mm and a focal length of 4 m. The
equi-concave lens of refractive index 1.5, when it is eyepiece has a focal length of 25 mm. Calculate the
kept in a medium of refractive index 1.4, to have a magnifying power of telescope (l = 6000 Å for
power of -5D ? [Delhi 2019] yellow colour). [Delhi 2011C]
12. An equi-convex lens of focal length f is cut into 21. Define power of a lens. Write its units. Deduce the
two equal halves in thickness. What is the focal 1 1 1
relation = + for two thin lenses kept
length of each half ? f f1 f 2
13. What is the focal length of a convex lens of focal in contact co-axially. [Foreign 2012]
length 30 cm in contact with a concave lens of focal 22. A symmetric biconvex lens of radius of curvature R
length 20 cm? Is the system a converging or a and made of glass of refractive index 1.5, is placed
diverging lens? Ignore thickness of the lenses. on a layer of liquid placed on the top of a plane
[NCERT] mirror as shown in the figure below.
14. The figure shows a ray of light falling normally on An optical needle with its tip on the principal axis
the face AB of an equilateral glass prism having of the lens is moved along the axis until its real and
refractive index 3/2, placed in water of refractive inverted image coincides with the needle itself.
index 4/3. Will this ray suffer total internal The distance of the needle from the lens is
reflection on striking the face AC? Justify your measured to be x.
answer. [CBSE 2018]
A

B C

15. A ray PQ incident normally on the refracting face On removing the liquid layer and repeating the
BA is refracted in the prism BAC made of material experiment, the distance is found to be y. Obtain
of refractive index 1.5. Complete the path of ray the expression for the refractive index of the liquid
through the prism. in terms of x and y. [CBSE 2018]
30 CBSE Term II Physics XII

23. State the conditions of total internal reflection. view the moon, find the diameter of the image of the
Refractive indices of the given prism material for moon formed by the objective lens. The diameter of
red, blue and green colours are 1.39, 1.48 and 1.42, the moon is 3.48 ´ 10 6 m and the radius of lunar
respectively. Trace the path of rays through the . ´ 10 8 m.
orbit is 38 [Delhi 2019]
prism. [All India 2019]
A
l
Long Answer (LA) Type Questions
28. (i) A point object O is kept in a medium of refractive
G index n1 infront of a convex spherical surface of
B
radius of curvature R which separates the second
R
medium of refractive index n 2 from the first one,
45º as shown in the figure.
B C
`
Draw the ray diagram showing the image
24. A ray of light incident on the face AB of an isosceles formation and deduce the relationship between
triangular prism makes an angle of incidence i and the object distance and the image distance in
deviates by angle b as shown in the figure. Show terms of n1 , n 2 and R.
that in the position of minimum deviation Ðb = Ða. n1 n2
Also find out the condition, when the refracted ray C
QR suffer total internal reflection. [All India 2019] O u R
A

Q b (ii) When the image formed above acts as a virtual


i R object for a concave spherical surface separating
the medium n 2 from n1 ( n 2 > n1 ), draw this ray
P
diagram and write the similar [similar to (i)]
relation. Hence, obtain the expression for the
a a lens Maker’s formula. [All India 2015]
B C
29. (i) Define the term focal length of a mirror. With
25. (i) A ray of light incident of face AB of an equilateral the help of a ray diagram, obtain the relation
glass prism, shows minimum deviation of 30°. between its focal length and radius of curvature.
Calculate the speed of light through the prism.
(ii) Calculate the angle of emergence ( e) of the ray of
A light incident normally on the face AC of a glass
prism ABC of refractive index 3. How will the
angle of emergence change qualitatively, if the
ray of light emerges from the prism into a liquid
B C of refractive index 1.3 instead of air? [Delhi 2020]
(ii) Find the angle of incidence at face AB, so that 30. (i) Under what conditions is the phenomenon of
the emergent ray grazes along the face AC. total internal reflection of light observed? Obtain
[Delhi 2017] the relation between the critical angle of
26. An optical instrument uses an objective lens of incidence and the refractive index of the
power 100 D and an eyepiece of power 40 D. The medium.
final image is formed at infinity when the tube (ii) Three lenses of focal lengths +10 cm, -10 cm and
length of the instrument is kept at 20 cm. +30 cm are arranged co-axially as in the figure
(i) Identify the optical instrument. given below. Find the position of the final image
formed by the combination. [All India 2019]
(ii) Calculate the angular magnification produced by
the instrument. [Delhi 2020] +10 cm –10 cm +30 cm
O
27. Draw a labelled ray diagram of an astronomical
telescope in the near point adjustment position. 30 cm
A giant refracting telescope at an observatory has an
objective lens of focal length 15 m and an eyepiece 5 cm 10 cm
of focal length 1.0 cm. If this telescope is used to ]
CBSE Term II Physics XII 31

31. (i) Two thin lenses are placed co-axially in contact. The focal lengths of the objective and eyepiece of a
Obtain the expression for the focal length of this microscope are 1.25 cm and 5 cm, respectively.
combination in terms of the focal lengths of the Find the position of the object relative to the
two lenses. objective in order to obtain an angular
(ii) A converging lens of refractive index 1.5 has a magnification of 30 in normal adjustment.
[Delhi 2012]
power of 10 D. When it is completely immersed
in a liquid, it behaves as a diverging lens of focal 36. (i) Draw a labelled ray diagram showing the image
length 50 cm. Find the refractive index of the formation of a distant object by refracting
liquid. [All India 2020] telescope.
32. (i) A ray PQ of light is incident on the face AB of a Deduce the expression for its magnifying power
glass prism ABC (as shown in the figure) and when the final image is formed at infinity.
emerges out of the face AC. Trace the path of the (ii) The sum of focal lengths of the two lenses of a
ray. Show that, refracting telescope is 105 cm. The focal length
i + e= A +d of one lens is 20 times that of the other.
Determine the total magnification of the
A telescope when the final image is formed at
infinity. [All India 2014]
i Q
P
l
Case Based Questions
B C
34. Refraction of Light
where, d and e denote the angle of deviation and Refraction involves change in the path of light due
angle of emergence, respectively. to change in the medium.
Plot a graph showing the variation of the angle of Incident ray
Normal
Reflected
deviation as a function of angle of incidence. ray
i
State the condition under which Ðd is minimum.
i

(ii) Find out the relation between the refractive Reflecting


surface r
index m of the glass prism and ÐA for the case,
when the angle of prism A is equal to the angle Refracted ray
of minimum deviation d m . Hence, obtain the
value of the refractive index for angle of prism When a beam of light encounters another
A = 60°. [Delhi 2015] transparent medium, a part of light gets reflected
back into the first medium, while the rest enters
33. Define magnifying power of a telescope. Write its the other. The direction of propagation of an
expression. A small telescope has an objective lens obliquely incident ray of light, that enters the other
of focal length 150 cm and an eyepiece of focal medium, changes at the interface of two media.
length 5 cm. If this telescope is used to view a This phenomenon is called refraction of light.
100 m high tower 3 km away, find the height of the
(i) For the same value of angle of incidence, the
final image, when it is formed 25 cm away from the
angles of refraction in three media A, B and C are
eyepiece. [Delhi 2012]
15°, 25° and 35°, respectively. In which medium,
34. Draw a ray diagram to show the working of a would the velocity of light be minimum?
compound microscope. Deduce an expression for [All India 2012]
the total magnification, when the final image is (ii) Why does a crack in a glass window pane appear
formed at the near point. silvery?
In a compound microscope, an object is placed at a (iii) The refractive index of diamond is much higher
distance of 1.5 cm from the objective of focal length than that of glass. How does a diamond cutter
1.25 cm. If the eyepiece has a focal length of 5 cm make use of this fact? [All India 2011]
and the final image is formed at the near point.
(iv) What is the apparent position of an object below
Estimate the magnifying power of the microscope.
[Delhi 2010] a rectangular block of glass 6 cm thick, if a layer
of water 4 cm thick is on the top of the glass?
35. How is the working of a telescope different from (Take, n ga = 15
. and n wa = 1.33) [Delhi 2015 C]
that of a microscope?
Chapter Test of the surface of water through which light from the
bulb can emerge out? Refractive index of water is 1.33.
(Consider the bulb to be a point source) (Ans. 2.58 m 2 )
Multiple Choice Questions 8. A beam of light converges at a point P. Now, a lens is
1. Two convex and concave lens are in contact and placed in the path of the convergent beam 12 cm from
having focal lengths 12 cm and 18 cm, respectively. P. At what point does the beam converge, if the lens
Focal length of joint lens will be is (i) a convex lens of focal length 20 cm and
(a) 50 cm (b) 45 cm
(ii) a concave lens of focal length 16 cm?
[Ans. (i) 7.5 cm and (ii) 48 cm]
(c) 36 cm (d) 18 cm
9. A ray of light PQ enters an isosceles right angled prism
2. Two lenses are kept in contact with powers + 2 D and
ABC of refractive index 1.5 as shown in figure.
- 4 D. The focal length of this combination will be
A
(a) + 50 cm (b) - 50 cm
(c) - 25 cm (d) + 25 cm Q 90º
3. A thin lens of glass (m = 1 . 5) of focal length ± 10 cm is
immersed in water (m = 1 . 33 ). The new focal length is 45º
(a) 20 cm (b) 40 cm B C
(c) 48 cm (d) 12 cm P

4. A plot of angle of deviation D versus angle of incidence (i) Trace the path of the ray through the prism.
i for a triangular prism is shown below. (ii) What will be the effect on the path of the ray, if the
refractive index of the prism is 1.4?
The angle of incidence for which the light ray travels
parallel to the base is 10. Which two of the following lenses L1, L2 and L3 will you
select as objective and eyepiece for constructing best
Angle of deviation

possible (i) telescope and (ii) microscope? Give reason


50°
to support your answer.
Lens Power (P) Aperture (A)
40°
L1 6D 1 cm
0
30° 45° 60° L2 3D 8 cm
Angle of incidence
L3 10D 1 cm
(a) 30°
(b) 60° Long Answer Type Questions
(c) 45°
11. (i) When a convex lens of focal length 30 cm is in
(d) Data insufficient
contact with a concave lens of focal length 20 cm,
5. An equilateral prism is in condition of minimum find out if the system is converging or diverging.
deviation. If incidence angle is 4/5 times of prism
(ii) Obtain the expression for the angle of incidence of
angle, then minimum deviation angle is
a ray of light which is incident on the face of a
(a) 72° (b) 60 °
prism of refracting angle A, so that it suffers total
(c) 48 ° (d) 36 ° internal reflection at the other face. (Given, the
Short Answer Type Questions refractive index of the glass of the prism is m).
6. You are given two converging lenses of focal lengths 12. An angular magnification (magnifying power) of 30 is
1.25 cm and 5 cm to design to compound microscope. desired using an objective of focal length 1.25 cm and
If it is desired to have a magnification of 30, find out an eyepiece of focal length 5 cm. How will you
the separation between the objective and the eyepiece. set up the compound microscope? (Ans. 11.67 cm)
(Ans. 11.67 cm)
7. A small bulb is placed at the bottom of a tank
containing water to a depth of 80 cm. What is the area

Answers
Multiple Choice Questions For Detailed Solutions
1. (c) 2. (b) 3. (b) 4. (c) 5. (d) Scan the code
CBSE Term II Physics XII 33

EXPLANATIONS
PART 1 6. (a) The refraction formula for curved surface,
n 2 n1 n 2 - n1
1. (d) From Snell’s law of refraction, - =
v u R
sin i
a
mg = = constant …(i) 7. (b) When medium is equal on both sides of lens, then the
sin r
numerical value of both focal lengths is equal, hence
Since, angle of incidence increase, so the angle of refraction f 2 = - f1 .
æ sin i ö
has to increase. Hence, the ratio ç ÷ is a constant 8. (a) Since, lens is made of two layers of different refractive
è sin r ø indices, for a given wavelength of light it will have two
according to Eq. (i). different focal lengths or will have two images at two
4 5 1
2. (c) Given, a n w = , a n g = different points as µ ( m - 1 ) (from lens Maker’s formula).
3 3 f
\ a n w ´ wn g = a n g
a ng 5/ 3 5 3 5 9. (c) Given, R = 20 cm, and m = 1.5, on substituting the values
w ng = = = ´ = R 20
n
a w 4/ 3 3 4 4 in f = = = 40 cm, of converging nature as
m - 1 1.5 - 1
3. (b) The given ray of light is as shown below
f >0. Therefore, lens act as a convex lens irrespective of the
side on which the object lies.
90° Air
10. (a) Using lens Maker’s formula,
r Water 1 æ1 1 ö
r = ( n - 1) çç - ÷÷ …(i)
f è R1 R 2 ø
Glass
i Given, R1 = 10 cm, R 2 = - 10 cm, u = - 5 cm and n = 1.33
Substituting the given values in Eq. (i), we get
1 æ1 1ö
= ( 1.33 - 1 ) ç + ÷
For glass-water interface, applying Snell’s law, f è 10 10 ø
sin i n w 1 2 0.33
= = 0.33 ´ = Þ f = 15.15 cm
sin r n g f 10 5
n w sin r 1 1 1
Þ ng = …(i) Now, from thin lens formula, = -
sin i f v u
For water-air interface, angle of incidence in water = r uf - 5 ´ 15.15 - 75.75
sin r n 1 or image distance, v = = =
Again, = a = u + f - 5 + 15.15 1015
.
sin 90° n w n w = - 7.46 cm
1 Since, v is negative, hence image will be formed on the same
Þ sin r = …(ii)
nw side of the object.
From Eqs. (i) and (ii), we get 11. (c) Focal length of the combination of two given lenses,
æ1 ö 1 1 1
( n w ) ´ çç ÷÷ = + …(i)
è nw ø Þ n = 1 f f1 f2
ng = g
sin i sin i where, f1 and f 2 are the focal lengths of plano-convex lens
n2 and plano-concave lens, respectively.
4. (c) We know that, sin q = As for plano-convex lens, R1 = R and R 2 = ¥
n1
1 æ 1 1 ö n -1
c So, = ( n1 - 1 ) ç - ÷ = 1
As, refractive index of a medium, n = , where c and v are f1 èR ¥ø R
v
the speed of light in vacuum and medium, respectively. Similarly, for plano-concave lens,
v R1 = - R and R 2 = ¥
Þ sin q = 1 æ 1 1 ö -( n2 - 1 )
v¢ So, = ( n2 - 1 ) ç - ÷=
Here, v is speed in medium A and v¢ is speed in medium B. f2 è - R ¥ ø R
v 1 1
Þ v¢ = Putting these values of and in Eq. (i), we get
sin q f1 f2
5. (b) The phenomenon involved in the reflection of 1 ( n1 - 1 ) ( n 2 - 1 ) ( n1 - 1 - n 2 + 1 ) n1 - n 2
= - = =
radiowaves by ionosphere is similar to total internal f R R R R
reflection of light in air during a mirage, i.e. angle of R
incidence is greater than critical angle. f =
n1 - n 2
34 CBSE Term II Physics XII

12. (a) The statement given in option (a) is correct but rest are 17. (d) For least distance of distinct vision, the angular
incorrect and these can be corrected as, magnification of simple microscope,
Power is defined as the tangent of angle by which it D
m =1 +
converges or diverges a beam of light falling at unit distance f
from the optical centre.
æ 1ö
Combination of lenses helps to obtain diverging or Þ m = 1 + DP çç\ Power, P = ÷÷
è fø
converging lenses of desired magnification. It also enhances
sharpness of the image. D
and for normal adjustment, m =
Also, total magnification m of the combination of lenses is a f
product of magnification ( m1 , m 2 , m 3 ,.....) of individual Þ m = DP Þ m µ P
lenses, i.e. m = m1 m 2 m 3 … .
18. (a) Given, f o = F1 and f e = F2
13. (b) If a symmetrical biconcave lens of focal length f (say) is
We know that, angular magnification for telescope,
vertically splitted into two identical plano-concave parts (as
shown below), then focal length of each part will be 2 f . f F F
| m| = o = 1 = 1
f 2f 2f fe F2 F2
| fo |
19. (d) For telescope,| m | = =5 …(i)
| fe |
and length of the telescope,
L = | f o | + | f e | = 36 …(ii)
1 From Eqs. (i) and (ii), we get
As we know, power of a lens =
focal length Þ f e = 6 cm
1 and f o = 30 cm
So, power of the biconcave lens, P = … (i) 20. (c) Refractive index of any pair of media is inversely
f
proportional to wavelength of light.
Similarly, power of each part of plano-concave lens,
As, lv < lr
1 1
P¢ = = P [using Eq. (i)] Þ nv > nr
2f 2
where, l v and l r are the wavelengths of violet and red light,
14. (c) The given prism can be shown as below respectively and n r and n v are refractive index of violet and
A red light, respectively.
Hence, refractive index of glass with respect to air is different
30º
i=0º for red light and violet light.
Therefore, A is true but R is false.
r1=0º r2 21. (d) Refractive index of diamond w.r.t. liquid,
1 n
l
nd = = d
sin ic n l
B C
6 1
Given, A = 30°, i = 0° Þ =
3 sin ic
Þ r1 = 0°
1
As, the angle of prism, A = r1 + r2 Þ sin ic = = sin 45 °
2
Þ 30° = 0 + r2 Þ r2 = 30°
\ ic = 45 °
15. (b) Given, angle of prism = 5 °; n r = 1.641 , n b = 1.659
This means that, when the ray of light while travelling from
For this prism, the deviation, D = ( n - 1 ) A
diamond to liquid is incident at angle of 45°, then it will be
So, for blue colour light, Db = ( n b - 1 ) A totally reflected due total internal reflection.
Similarly, for red colour light, Dr = ( n r - 1 ) A Therefore, A is false and R is also false.
\Angle between the emergent blue and red rays 22. (c) Reflection and refraction arise through interaction of
= Db - Dr = ( n b - n r ) A incident light with constituents of matter. When
= (1.659 - 1.641) ´ 5 monochromatic light is incident on a surface separating two
= 0.018 ´ 5 ° media, then the frequency of scattered light equals the
= 0.09° frequency of incident light.
The speed of light and wavelength of light both changes in
16. (a) Given, incidence angle, i = 50° c
refraction, then the frequency of light n = remains
Refractive angle, A = 60° l
Minimum deviation, d = 2i - A = 2 ´ 50° - 60° = 40° constant.
Therefore, A is true but R is false.
CBSE Term II Physics XII 35

23. (b) In optical fibre communication, propagation of signal


through optical fibre takes place, which is based on the
PART 2
phenomenon of total internal reflection at core-clade 1. (i) The frequency of reflected and refracted light remains
interface. same as that of incident light because frequency only
The refractive index of the material of the cladding is less depends on the source of light.
than that of core, hence light striking at core-cladding (ii) Since, the frequency remains same, hence there is no
interface gets totally internally reflected. reduction in energy.
Therefore, both A and R are true but R is not the correct 2. Snell’s law of refraction fails in two situations
explanation of A. (i) When TIR (Total Internal Reflection) takes place at angle
24. (d) When a convergent lens is placed inside a transparent greater than the critical angle.
medium having refractive index greater than that of material (ii) When light is incident normally on a surface, as i = 0,
of lens, it behaves as a divergent lens. r = 0.
Hence, property of a lens, whether the rays are converging 3. Given, i1 = 45 °, t = 10 cm = 01 . m and m =1 .5
or diverging depends on the surrounding medium. sin i1
By Snell’s law, m =
Therefore, A is false and R is also false. sin r1
25. (a) Secondary rainbow is a result of four-step process, which sin i1 sin 45 °
Þ sin r1 = =
are as follows m 1 .5
(i) Refraction at the first surface of raindrop. 0.707 æ\ sin 45 ° = 1 / 2 ö
Þ sin r1 = ç ÷
(ii) Two total internal reflection from the second surface of 1 .5 ç here, 2 = 1.414÷
raindrop. è ø
(iii) Again refraction from the first surface of raindrop from Þ sin r1 = 0.4713
where the light finally emerges out. Þ r1 = sin -1 (0.4713)
Thus, the intensity of light is reduced at the second Þ r1 = 2812 . °
reflection and hence the secondary rainbow is fainter than t sin ( i1 - r1 ) 0.1 sin ( 45 ° - 2812 . °)
Lateral shift = =
the primary rainbow. cos r1 . °
cos 2812
Therefore, both A and R are true and R is the correct 0.1 sin 16.88° 01 . ´ 0.2904
= =
explanation of A. cos 2812. ° 0.8819
26. (i) (d) Total internal reflection is the basis for following = 0.033 m
phenomenon
4. It is due to the refraction of sunlight as it travels through
(a) Sparkling of diamond.
different layers of the earth’s atmosphere. Refraction of light
(b) Optical fibre communication. by these layers can make the sun appear flattened or
(c) Instrument used by doctors for endoscopy. distorted. The rays of light from the upper part and lower
(ii) (d) Total internal reflection (TIR) is the phenomenon that part of the periphery of the sun bend unequally on travelling
involves the reflection of all the incident light off the through earth’s atmosphere, making the sun appears oval in
boundary. TIR only takes place, when both of the shape.
following two conditions are met.
5. Given, u = - f
The light is in the more denser medium and approaching
the less denser medium. 1 1 1
\From lens formula, - =
The angle of incidence is greater than the so-called v u f
critical angle. 1 1 1 1
Þ + = Þ =0
(iii) (c) If incidence angle, i = critical angle C, then refraction v f f v
angle, r = 90°. 1
Þ v = = infinity
(iv) (b) In optical fibres, core is surrounded by cladding, 0
where the refractive index of the material of the core is 1 1 1
higher than that of cladding to bound the light rays inside 6. As we know that, - =
the core. i.e. n1 > n 2 . v u f
v \Graph between u and v is as shown below
(v) (b) From Snell’s law, sin C = 1 n 2 = 1
v2 v (in cm)
where, C = critical angle = 30°
and v1 and v2 are speed of light in medium and vacuum,
respectively. u (in cm)
O
We know that, v2 = 3 ´ 108 m/s
v1
\ sin 30° =
3 ´ 108
1 7. If m1 = m 2 , then f = ¥
Þ v1 = 3 ´ 108 ´
2 Hence, the lens in the liquid acts like a plane sheet, when
Þ v1 = 1 .5 ´ 108 m/s refractive index of the lens and the surrounding medium is
the same. Therefore, m1 = m 2 = 1.47.
36 CBSE Term II Physics XII

Hence, the liquid medium is not water, refractive index for 1 2 (m - 1 )


Þ =
water = 1.33. f¢ R
8. Case I When tank is filled with the water. \ f¢ = 2f
Given, the apparent depth = 9.4 cm Hence, focal length of each half becomes twice of the
Height of water, t = 12.5 cm original value.
So, real depth = 12 .5 cm 13. Given, focal length of convex lens, f1 = 30 cm
Refractive index of water, Focal length of concave lens, f 2 = - 20 cm
Real depth 12 .5 Using the formula of combination of lenses,
mw = = = 1.33
Apparent depth 9.4 1 1 1 1 1
= + = -
Case II When tank is filled with the liquid. f f1 f 2 30 20
Refractive index of liquid, m l = 1.63 2-3 1
= =-
Real depth 60 60
Again, ml =
Apparent depth Þ f = - 60 cm
12.5 Since, the focal length of combination is negative in nature.
Þ 1.63 =
Apparent depth So, the combination behaves like a diverging lens, i.e. as a
12.5 concave lens.
Apparent depth = = 7.67 cm
1.63 14. Given, refractive index of water, m w = 4 / 3
\ The microscope is shifted by 9.4 – 7.67 = 1.73 cm. 3
Refractive index of glass prism, m g =
9. Whenever object is placed within the focus of the biconvex 2
lens, we will obtain enlarged image, hence the biconvex lens A
behaves like a magnifying lens. 60°
10. The magnification of a magnifying glass depends upon,
where it is placed between the user’s eye and the object 30°
being viewed and the total distance between them, while
the magnifying power is equivalent to angular magnification. i=60°
11. Given, m1 = 1 .4, m 2 = 1.5, P = - 5 D B C
Using lens Maker’s formula,
1 æ m - m1 ö æ 1 1 ö For total internal reflection occurrence the incident angle
P = = çç 2 ÷ç - ÷ must be greater than critical angle.
F è m1 ÷ø çè R1 R 2 ÷ø
\ Let us calculate critical angle C.
æ 1.5 - 1.4 ö æ 1 1 ö 1
-5 = ç ÷ ç- - ÷ As we know that, sinC =
è 1.4 ø è R R ø m
(for equi-concave lens, R1 = - R and R 2 = R) refractive index of glass ( a m g )
0.1 æ 2 ö where, m =
-5 = ç- ÷ refractive index of water ( a m w )
1.4 è R ø
1 1 1
1 2 1 \ sin C = = =
Þ R= ´ = = 0.0286 m = 2.86 cm æ a m g ö æ 3/ 2 ö 9/ 8
14 5 35 çç ÷÷ çç ÷÷
12. Focal length can be given as, è a m w ø è 4/ 3 ø
1 æ1 1 ö or sin C =
8
= 0.88
= ( m - 1 ) çç - ÷÷
f è R1 R 2 ø 9
where, m is the refractive index of the lens medium and Þ C = 61 . 6° (as, sin 60° = 3 / 2 = 0.86)
R1 & R 2 are radii of curvature. As the critical angle, i.e. 61 . 6° is greater than the angle of
Focal length = f incidence, i.e. 60°, hence TIR will not occurs.
15. Given, refractive index of the material of the prism, m = 1 .5
A

P 30°
Q
60°
30°
Equi-convex lens have the same radius of curvature, i.e.
R1 = - R 2 m =1.5
1 é 1 æ 1 öù
\ = (m - 1 ) ê - ç - ÷ ú 60°
f¢ ë R è R øû B C
CBSE Term II Physics XII 37

\Critical angle for the material, 20. The diameter of objective of the telescope = 150 ´ 10-3 m
1 1 f o = 4 m, f e = 25 ´ 10-3 m
sin C = = = 2/ 3
m 1 .5 and D = 25 mm = 0.25 m
æ 2ö f æ Dö
Þ C = sin -1 ç ÷ ~- 42°. Magnifying power, m = - o çç1 + ÷
è 3ø fe è f e ÷ø

From the ray diagram, it is clear that angle of incidence 4 æ 0.25 ö


=- ç1 + ÷
i = 30° < C. 25 ´ 10-3 è 25 ´ 10-3 ø
Therefore, the ray incident at the face AC will not suffer = - 1760
total internal reflection and merges out through this face. 1.22l 1.22 ´ 6 ´ 10-7
Now, dq = =
16. Given, A = 60° (for equilateral prism) D 0.25
= 2.9 ´10- 6 rad
4 2
m1 = , m 2 = 1.6 21. The power of a lens is equal to the reciprocal of its focal
5 length, when it is measured in metre. Power of a lens,
P = 1 / f (in m) and its SI unit is dioptre (D).
The refractive index is given by A B
æ A + Dö
sin ç ÷
m2 è 2 ø
=
m1 æ Aö
sin ç ÷ O P I I1
è 2ø v
where, D = angle of minimum deviation.
u v1
æ 60° + D ö
sin ç ÷
1.6 ´ 5 è 2 ø Consider two lenses A and B of focal lengths, f1 and f 2 placed
=
4 2 æ 60° ö in contact with each other. An object is placed at a point O
sin ç ÷ beyond the focus of the first lens A.
è 2 ø
The first lens produces an image (real image) at I1 , which
æ 60° + D ö
2 ´ sin 30° = sin ç ÷ serves as a virtual object for the second lens B producing the
è 2 ø final image at I.
1 æ 60° + D ö Since, the lenses are thin, we assume the optical centres P of
Þ = sin ç ÷
2 è 2 ø the lenses to be coincident. For the image formed by the first
æ 60° + D ö lens A, we obtain
Þ sin 45 ° = sin ç ÷
è 2 ø 1 1 1
- = …(i)
60° + D v1 u f1
Þ 45 ° =
2 For the image formed by the second lens B, we get
D = 90° - 60° = 30° 1 1 1
- = …(ii)
17. By increasing the diameter of the objective lens, we can v v1 f2
increase the range of the telescope because as the diameter
of lens increases, the area covered by the lens also increases. Adding Eqs. (i) and (ii), we obtain
i.e. Lens is able to focus on a large area thereby helping us 1 1 1 1
- = + …(iii)
to view the object better. v u f1 f2
18. Advantages of reflecting telescope over refracting telescope If the two lenses system is regarded as equivalent to a single
are as follows lens of focal length f , then we have
(i) In reflecting telescope, image formed is free from 1 1 1
chromatic aberration defect. So, it is sharper than image - = …(iv)
formed by a refracting type telescope. v u f
(ii) A mirror is easier to produce with a large diameter, so From Eqs. (iii) and (iv), we get
that it can intercept rays crossing a large area and direct 1 1 1
them to the eyepiece. + =
f1 f2 f
19. Given, focal length of objective lens, f o = 144 cm
22. First measurement gives the focal length ( feq = x ) of
Focal length of eyepiece, f e = 6 cm
Magnifying power of the telescope in normal adjustment combination of the convex lens and the plano-convex liquid
(i.e. when the final image is formed at ¥), lens. Second measurement gives the focal length ( f1 = y) of
f 144 the convex lens.
m = - o =- = - 24 Focal length ( f 2 ) of plano-convex lens is given by
fe 6
\ Separation between lenses, 1 1 1 1 1
= - = -
L = f o + f e = 144 + 6 = 150 cm f2 feq f1 x y
38 CBSE Term II Physics XII

xy 24. Given, a = 60° (for isosceles triangle)


Þ f2 = …(i)
y-x r1 = 90° - b and r2 = b - 30°
For equi-convex glass lens using lens Maker’s formula, we get For minimum deviation, r1 = r2
1 æ1 1 ö Þ 90° - b = b - 30°
= ( n g - 1 ) çç - ÷÷
f1 è R1 R 2 ø Þ 2 b = 120° or b = 60° = a
1
1 æ 2ö For total internal reflection, £m
= (1.5 - 1 ) ç ÷ (as, R1 = R and R 2 = - R) sin iC
y èRø
1 1 2 1
Þ = ´ Þ R=y £m (Q r2 = iC = 30° )
y 2 R sin 30°
Þ m2 ³ 2
P P¢
Q Q¢ 25. (i) Given, angle of minimum deviation, dm = 30°
\ Angle of prism, A = 60°
By prism formula, reflected index
d + A 30° + 60°
sin m sin
2 2 sin 45 °
m = = =
sin A / 2 sin 30° sin 30°
1
Now, we apply lens Maker’s formula for plano-convex lens. = ´2 = 2
2
Here, R1 = R, R 2 = ¥ and let n l = refractive index of liquid.
speed of light in vacuum (c)
1 æ1 1ö Also, m =
= ( nl - 1 ) ç - ÷ speed of light in prism (v)
f2 èR ¥ø
1 æ1ö Þ v = c / m = ( 3 ´ 108 / 2 ) m/s
Þ = ( nl - 1 ) ç ÷
f2 èRø Hence, speed of light through prism is ( 3 ´ 108 / 2 ) m / s.
R y (ii) As the emergent ray grazes along the face AC, e = 90°. At
Þ nl = 1 + =1 + the interface AC, using Snell’s law,
f2 æ xy ö
çç ÷÷ sin i
è y - xø =m
sin e
y-x y
=1 + = sin i/sin e = 2
x x
Þ sin i = 2 sin e = 2 ´ sin 90°
23. There are two conditions for total internal reflection as
follows i = sin -1 ( 2 )
(i) Light must travel from denser to rarer medium. 26. Given, power of objective lens, Po = 100 D
(ii) Angle i > ic Power of eyepiece, Pe = 40 D
Given, m red = 1.39, m blue = 1.48 and m green = 1.42 As we know, power of a lens
A 1 100
= =
focal length of lens (in m) f (in cm)
G
i=45º Focal length of objective lens,
B
i=45º 1 100
R fo = = = 1 cm
45º Po 100
45º Similarly, focal length of eyepiece,
B C 1 100
fe = = = 2.5 cm
Pe 40
(i) Since, the focal length of eyepiece is more than the focal
1
Q sin iC = length of objective. So, the optical instrument is
m compound microscope.
æ 1 ö (ii) Since, the final image is formed at infinity, so the angular
\ ( iC ) red = sin -1 ç ÷ = 46°
è 1.39 ø magnification is given as
æ 1 ö L D
( iC ) green = sin -1 ç ÷ = 44.8° m=-
è 1.42 ø fo fe
æ 1 ö where, L is the tube length of the instrument = 20 cm
( iC ) blue = sin -1 ç ÷ = 43° (given) and D is the least distance of distinct vision
è 1.48 ø
= 25 cm.
Q Angle of incidence at face AC is 45° which is more than
Substituting the values in the above equation, we get
the critical angle for blue and green colours, therefore blue
- 20 ´ 25
and green colours will undergo total internal reflection but m= = - 200
red colour will refract to other medium. 1 ´ 25
.
CBSE Term II Physics XII 39

27. The ray diagram of an astronomical telescope in the near For DNOC, Ði is the exterior angle.
point adjustment position is as shown below
\ Ði = ÐNOM + ÐNCM
fo fe
MN MN
Parallel rays from For small angles, i = + ...(i)
object at infinity OM NC
fe fo b
a B¢¢ Similarly, r = ÐNCM - ÐNIM
C1 B¢ C2
Eye MN MN
A¢ Þ r= - ...(ii)
NC NI
By Snell's law, we get
A¢¢
D n1 sin i = n 2 sin r

Given, diameter of the moon, do = 3.48 ´ 10 m 6 For small angles, n1 i = n 2 r


Radius of lunar orbit, r = 3.8 ´ 108 m Putting the values of iand r from Eqs. (i) and (ii), we get
Focal length of objective lens, f o = 15 m æ MN MN ö æ MN MN ö
n1 ç + ÷ = n2 ç - ÷
The diameter of the image of moon formed by the objective è OM MC ø è MC MI ø
lens is given by n1 n n - n1
d Þ + 2 = 2 ...(iii)
dI = o ´ f o OM MI MC
r
Applying new cartesian sign conventions, we get
3.48 ´ 106
= ´15 OM = - u, MI = + v and MC = + R
3.8 ´ 108
= 01373
. m Substituting this in Eq. (iii), we get
= 13.74 cm n 2 n 2 n 2 - n1
- = ...(iv)
28. (i) Let a spherical surface separate a rarer medium of v u R
refractive index n1 from the second medium of refractive (ii) Now, the image I¢ acts as a virtual object for the second
index n 2 . Let C be the centre of curvature and R = MC be surface that will form a real at I. As, refraction takes place
the radius of the surface. from denser to rarer medium,
Consider a point object O lying on the principal axis of the
surface. Let a ray starting from O incident normally on the r¢
n2 n1
surface along OM and pass straight. Let another ray of light
incident on NM along ON and refract along NI. From M , c¢
draw MN perpendicular to OI. M¢ I¢ I


A v
Rarer n1 N
i Denser n2
r - n 2 n1 n 2 - n1
\ + ¢ = ...(v)
O v v R1
P M C I
R On adding Eqs. (iv) and (v), we get
1 1 æ1 1 ö
- = ( n 2 - n1 ) ç - ÷
B
v¢ u è R R¢ø
u v 1 1 æ n2 ö æ1 1 ö
- = ç - 1 ÷÷ ç - ÷
The above figure shows the geometry of the formation of v u çè n1 ø è R R ¢ø
image I of an object O and the principal axis of a spherical 1 æ1 1 ö
surface with centre of curvature C and radius of curvature R. = ( n 21 - 1 ) ç - ¢ ÷
f èR R ø
Here, we have to make following assumptions
æ n 1 1 1ö
(a) the aperture of the surface is small as compared to the ççQ n 21 = 2 , = - ÷÷
other distance involved. è n1 f v u ø
(b) NM will be taken as nearly equal to the length of the 29. (i) Focal length The distance of the principal focus from
perpendicular from the point N on the principal axis. the pole of the mirror is called the focal length of the
MN mirror.
tan ÐNOM =
OM Relation between focal length and radius of curvature
MN of mirror
tan ÐNCM =
MC Consider a ray parallel to the principal axis striking the
MN mirror at point M, then CM will be perpendicular to the
tan ÐNIM = mirror at point M.
MI
40 CBSE Term II Physics XII

Let q be the angle of incidence and MD be perpendicular 30. (i) Following are the criteria for total internal reflection
to the principal axis. (a) Light must pass from a optically denser to a optically
M rarer medium.
q
q (b) Angle of incidence in denser medium is must be greater
than critical angle for two media.
q 2q From Snell’s law, m 2 sin iC = m1 sin 90°
C F D P m2 1
=
m1 sin iC
1
1m 2 =
(a) (where, iC is the critical angle)
sin iC
q
q M iC
Denser (m2)

2q q
P D F C
90°
Rarer (m1)
(b)

Then, ÐMCP = q and ÐMFP = 2q 1 1 1


MD MD (ii) - =
Now, tan q = and tan 2q = …(i) v u f
CD FD
For small q (condition true for paraxial rays),
tan q » q and tan 2q » 2q O
+10 cm –10 cm +30 cm
Therefore, from Eq.(i), we get
MD MD CD
=2 or FD = …(ii)
FD CD 2
Final
Again, for small q, we can observe that the point D is very
close to the point P. Therefore, FD = f and CD = R.
From Eq.(ii), we have
R
f = 30 cm 5 cm 10 cm 30 cm
2
(ii) Given, refractive index of the prism ABC, m = 3 Here, for first lens,
A u = - 30 cm and f = + 10 cm
1 1 1 1 1
= + = -
30° v1 f u 10 30
1 2
60° =
30° v1 30
e
Þ v1 = 15 cm
For second lens, u = + 10 cm and f = - 10 cm
60° 1 1 1 1 1
= + = - =¥
B C v2 f u 10 10
Applying Snell’s law at interface AB. Thus, for last lens the object is at infinity, hence the image
sin i 1 1 formed at the focus of the lens, which is at a distance
= =
sin e m 3 of 30 cm.
Þ 3 sin 30° = sin e 1
31. (i) Power of first lens, P1 =
1 f1
Þ 3 ´ = sin e or e = 60° 1
2 Power of second lens, P2 =
Now, if the ray of light emerges from prism in a liquid of f2
refractive index 1.3, then Net power of lens combination,
3 sin 30° = 1.3sin e Pnet = P1 + P2
3 1 1 1
Þ = 1.3sin e Þ = +
2 f net f1 f2
Þ sin e = 0.666 f1 f 2
Þ f net =
or e = sin -1 ( 0.666) = 41.76° f1 + f 2
CBSE Term II Physics XII 41

(ii) Given, m g = 1 .5 and P = 10 D i - d graph is shown in the figure,


As we know, focal length of the lens
1

Deviation angle (d)


=
power (in dioptre)
1
Þ fa = = 01. m = 10 cm
10
Using the lens Maker’s formula,
dm
Focal length of the lens when it is in air,
Angle of incidence (i)
1 æ1 1 ö
= ( m g - 1 ) çç - ÷÷ … (i) The conditions for the angle of minimum deviation are
fa è R1 R 2 ø given as below
and focal length of the lens when it is immersed in liquid, (a) Angle of incidence i and angle of emergence e are
1 æm g öæ1 1 ö equal.
=ç - 1 ÷÷ çç - ÷÷ … (ii)
f l çè m l øè 1R R 2ø
i.e. Ði = Ðe
(b) In equilateral prism, the refracted ray is parallel to
Dividing Eq. (i) by Eq. (ii), we get base of prism.
fl (m g - 1 ) (c) The incident and emergent rays are bent on same
= angle from refracting surfaces of the prism.
fa æ m g ö
çç - 1 ÷÷ i.e. Ðr1 = Ðr2
m
è l ø For minimum deviation position,
Substituting the given values, we get Putting r = r1 = r2 and i = e in Eq. (iv)
- 50 ( 1.5 - 1 ) A
Þ = (Q f l = - 50 cm) 2r = A Þ r = …(vi)
10 æ 1.5 ö 2
çç - 1 ÷÷ From Eq. (i), dm = 2 i - A
èml ø A + dm
i= …(vii)
1.5 - 05. 2
-1 = = - 01
.
ml 5 \Refractive index of material of prism,
sin i
Þ ml =
1.5 5
= = 1.67 m =
0.9 3 sin r
From Eqs. (vi) and (vii), we get
\Refractive index of liquid is 1.67.
æ A + dm ö
32. (i) Let PQ and RS are incident and emergent rays and sin ç ÷
incident ray get deviated by d by the prism. Þ m = è 2 ø
sin ( A/ 2)
i.e. ÐTMS = d
Let d1 and d2 are deviation produced at refractions taking (ii) As per the question,
place at AB and AC, respectively. Angle of prism, A = Angle of minimum deviation, dm
A i.e. ÐA = Ðdm …(viii)
T
D Substituting the value of Ðdm from Eq. (viii) we get
M æ A + Aö
d sin ç ÷
Þ m = è 2 ø
Q R sin ( A / 2 )
i e
r1 r2 sin A
Þ m=
S sin ( A / 2 )
N
P 2sin ( A / 2 ) × cos ( A / 2 )
B C Þ m =
sin ( A / 2 )
\ d = d1 + d2 = ( i - r1 ) + ( e - r2 )
Þ m = 2 cos( A / 2 )
= ( i + e ) - ( r1 + r2 ) …(i)
This is the required relation between refractive index of
But in DFNR,
the glass prism and angle of prism.
ÐQNR + ÐRQN + ÐQRN = 180°
Since, ÐA = 60° (given)
or ÐQNR = 180° - ( r1 + r2 ) …(ii)
æ 60° ö
In ~QARN, ÐAQN and ÐARN are right angles. Þ m = 2 cos ç ÷
So, ÐQNR = 180° - A …(iii) è 2 ø
where, A is angle of prism. Þ m = 2 cos 30°
From Eqs. (ii) and (iii), we have 3
Þ m =2´
A = r1 + r2 …(iv) 2
From Eqs. (i) and (iv), we have Þ m = 3
d = ( i + e) - A …(v) Þ m = 1 .732
or i+ e = A + d
42 CBSE Term II Physics XII

33. Magnifying power of a telescope is the ratio of the angle b The objective lens forms real and inverted magnified image
subtended at the eye by the image to the angle a subtended A ¢ B ¢ of object AB in such a way that AB¢ fall some, where
at the eye by the object. between pole and focus of eye lens.
Objective lens So, A ¢ B ¢ acts as an object for eyepiece and its virtual magnified
Parall
fo fe image A ¢¢ B ¢¢ formed by the lens.
from oel rays Eyepiece
infinity bject at The magnifying power of a compound microscope is defined
as the ratio of the visual angle subtended by final image at
a B fo, fe
a b
eye ( b ) and the visual angle subtended by object at naked
C1 I C2 Eye
eye, when both are at the least distance of distinct vision from
a ge the eye.
im
al nity Visual angle with instrument ( b )
Fin infi m Þ
at Visual angle when object is placed at least
The final image is magnified and inverted
distance of distinct vision ( a )
i.e. m = b/ a = f o / f e b tan b B ¢ A ¢ / ue
For telescope Þ m= = =
a tan a BA / D
Focal length of objective lens, f o = 150 cm
Focal length of eyepiece, f e = 5 cm æ B¢ A¢ ö D
=ç ÷ ´ = mo me
When final image forms at D = 25 cm, è BA ø ue
\ Magnification, M = - f o / f e ( 1 + f e / D) m = m o m e , where m o and m e are magnification produced by
objective and eyepiece, respectively.
150 æ 5 ö 150 6 B¢ A¢ v
=-
ç1 + ÷=- ´ Now, mo = = o
5 è 25 ø 5 5 BA - uo
M = - 36 D D
me = =1 + (by lens formula)
Let height of final image is h cm. ue fe
\ tan b = h / 25 æv ö æ Dö
b = visual angle formed by final image at eye \ m = - çç o ÷÷ çç1 + ÷÷
a = visual angle subtended by object at objective è uo ø è fe ø
100 m 1 This is the required expression.
tan a = =
3000 m 30 Also, uo = + 1.5 cm
tan b ( h / 25 )
But, M= Þ - 36 = Þ f o = 1.25 cm, f e = 5 cm
tan a ( 1 / 30 ) ve = - D = - 25 cm
h 6h
Þ -36 = ´ 30 = For objective lens,
25 5
36 ´ 5 1 1 1
h=- = - 30 cm = -
6 f o vo uo
Negative sign indicates inverted image. 1 1 1
34. A compound microscope consists of two convex lenses Þ = +
1 .25 vo 1 . 5
co-axially separated by some distance. The lens nearer to
the object is called the objective. The lens through which 1 1 1
Þ = -
the final image is viewed is called the eyepiece. The focal vo 1 .25 1 .5
length of objective lens is smaller than eyepiece. 1 .5 - 1 .25
Eyepiece =
E
1 .5 ´ 1 .25
Objective lens 0 .25 1
= =
A 1 .5 ´ 1 .25 750
.
vo = 7.5 cm
B¢¢ Fo B¢ 7.5 æ 25 ö
B a C2 M= ç1 + ÷ = - 5 ´ 6 = - 30
Fo C1 Fe b 2.5 è 5 ø
uo Fo 35. Differences between telescope and microscope are as given
below

u Characteristics Telescope Microscope
Position of object At infinity Near objective at a distance
lying between f o and 2 f o

A¢¢ Position of image Focal plane Beyond 2 f o , where f o is the


D of objective focal length of objective.
Compound microscope, final image at D
CBSE Term II Physics XII 43

For microscope When the final image is formed at infinity, angular


b
f o = 1.25 cm and f e = 5 cm magnification is given by M = .
When final image forms at infinity, then magnification a
produced by eyepiece is given by However, b and a are very small.
L D L 25 \ b » tan b or a » tan a
M=- × Þ - 30 = - ´ tan b
fo fe 1 .25 5 Þ M=
tan a
30 ´ 1 .25
L= Þ L = 7.50 cm I is the image formed by the objective, f o and f e are the
5 focal length of objective and eyepiece, respectively.
For objective lens I
tan a =
vo = L = 7.5 cm fo
and f o = 1.25 cm I
Applying lens formula, or tan b =
- fe
1 1 1
= - - I / fe
f o vo uo \ M=
I / fo
1 1 1
Þ = - or M = - fo / fe
1 .25 75. uo
1 1 1 (ii) Given, f o + f o = 105, f o = 20 f e
= - 105
uo 7 .5 1 .25 fe = =5
1 .25 - 75 . 6.25 21
= =-
7 .5 ´ 1 .25 7 .5 ´ 1 .25 Þ f o = 20 ´ 5 = 100 cm
7.5 ´ 1 .25 f o 100
Þ uo = - = - 1.5 cm M= = = 20
6 .25 fe 5
sin i c
The object must be at a distance of 1.5 cm from objective 37. (i) From Snell’s law, m = =
lens. sin r v
Þ v µ sin r, for given value of i.
\ Magnifying power,
Smaller the angle of refraction, smaller the velocity of
æv ö æ Dö light in medium.
m = - çç o ÷÷ çç1 + ÷÷
è uo ø è fe ø Velocity of light is minimum in medium A, as the angle of
refraction is minimum, i.e. 15°.
æ 75. öæ 25 ö
= - ç ÷ ç1 + ÷ = -5 ´6 (ii) Whenever rays of light travels through glass, they strike
è 1 .5 ø è 5 ø the glass-air interface at an angle greater than critical
m = - 30 angle of glass. They are totally reflected, hence crack
appears silvery.
36. It consists of an objective lens of a large focal length ( f o ) and
(iii) The refractive index of diamond is much higher than that
large aperture, also an eyepiece of small aperture and focal of glass. Due to high refractive index, the critical angle
length. for diamond-air interface is low. The diamond is cut
(i) Magnification when final image is formed at infinity, suitably, so that the light entering the diamond from any
f face suffers multiple total internal reflections at the
Magnification, m = - o and length of telescope,
fe various surfaces. This gives sparkling effect to the
L = | f o| + | f e| diamonds.
real depth / thickness of object
Objective lens (iv) Here, m =
fo apparent depth
Paralle fe
l Eyepiece Now, due to refraction at two different boundaries, the
from o rays
b
infinity ject at apparent depth of object
thickness of glass thickness of water
a Fo Fe = +
C1
a B¢ b C2 Eye
m glass m water
6 4
ag
e = +
im
al inity 1.5 1.3
i n f
F in
at = 4 + 3 = 7 cm
L
44 CBSE Term II Physics XII

CHAPTER 03

Wave Optics
In this Chapter...
l Wavefront l Superposition Principle

l Huygens' Principle l Young's Double Slit Experiment

l Doppler's Effect in Light l Diffraction of Light

Wavefront
It is the locus of points (wavelets) having the same phase portion of spherical or cylindrical wavefront appears to
(a surface of constant phase) of oscillations. A wavelet is the be plane. Such a wavefront is called a plane wavefront.
point of disturbance due to propagation of light.
Depending on the shape of source of light, wavefronts can
be of three types, which are explained as given below
Light
(i) Spherical wavefront When the source of light is a rays
point source, the wavefront is a sphere with centre Plane
as the source. wavefront
Plane wavefront
Ray
Huygens’ Principle
Wavefront
According to Huygens’ principle,
(i) Each point on the given wavefront (called primary
S
wavefront) is the source of a secondary disturbance
(called secondary wavelets) and the wavelets emanating
Spherical wavefront from these points spread out in all directions with the
speed of the wave.
(ii) Cylindrical wavefront When the source of light is
(ii) A surface touching these secondary wavelets,
linear, all the points equidistant from the source
tangentially in the forward direction at any instant gives
lie on a cylinder. Therefore, the wavefront is
the new wavefront at that instant. This is called
cylindrical in shape. secondary wavefront.
G1 F1 G1
F1
Wavefront A1 A2
S D1
B1 B2
A A¢ C2
O C1
D2
D1 D2

Cylindrical wavefront
F2 F2 G2
(iii) Plane wavefront When the point source or linear G2 t=0 t=t
source of light is at very large distance, a small (a) (b)
CBSE Term II Physics XII 45

In Fig.(a), F1 F2 is the section of the given spherical wavefront Laws of Refraction (Snell’s Law) at a Plane Surface
and G1 G 2 is the new wavefront in the forward direction. Let 1, 2, 3 be the incident rays and 1 ¢, 2¢, 3 ¢ be the
In Fig.(b), F1 F2 is the section of the given plane wavefront corresponding refracted rays.
and G1 G 2 is the new wavefront in the forward direction. N N¢
3

Refraction and Reflection of Plane Waves 2 Incident


wavefront B
Using Huygens’ Principle 1
E Rarer medium
Huygens’ principle can be used to explain the phenomena of i
Medium 1 v1 , m 1
reflection and refraction of light on the basis of wave theory of X
i F C
Y
light. A r
Medium 2 Denser medium
r r v2, m2
Laws of Reflection at a Plane Surface G

Let 1, 2, 3 be the incident rays and 1 ¢, 2¢, 3 ¢ be the D Refracted
wavefront 2¢
corresponding reflected rays.

N N¢
3 Incident 1¢ Laws of refraction by Huygens’ principle
wavefront If v 1 , v 2 are the speeds of light in the two media and t is the

2 D
B time taken by light to go from B to C or A to D or E to G
3¢ EF FG
1 E G Reflected through F, then t= +
i r wavefront v1 v2
i r EF
X Y
A F C In DAFE, sin i =
Laws of reflection by Huygens’ principle AF
FG
If c is the speed of light, t is the time taken by light to go from In DFGC, sin r =
FC
B to C or A to D or E to G through F, then AF sin i FC sin r
EF FG Þ t= + …(iii)
t= + …(i) v1 v2
c c
EF AC sin r æ sin i sin r ö
Þ t= + AF çç - ÷
In DAEF, sin i =
AF v2 è v1 v 2 ÷ø
FG For rays of light from different parts on the incident
In DFGC, sin r =
FC wavefront, the values of AF are different. But light from
AF sin i FC sin r different points of the incident wavefront should take the
or t= + same time to reach the corresponding points on the refracted
c c
wavefront. So, t should not depend upon AF. This is possible
AC sin r + AF (sin i - sin r )
Þ t= only, if
c sin i sin r sin i v 1
(Q FC = AC - AF) - =0 Þ = …(iv)
v1 v2 sin r v 2
For rays of light from different parts on the incident
Now, if c represents the speed of light in vacuum, then
wavefront, the values of AF are different. But light from c c
different points of the incident wavefront should take the m1 = and m 2 = are known as the refractive indices of
v1 v2
same time to reach the corresponding points on the reflected
wavefront. medium 1 and medium 2, respectively.
So, t should not depend upon AF. This is possible only, if In terms of refractive indices, Eq. (iv) can be written as
sin i
sin i - sin r = 0 m 1 sin i = m 2 sin r Þ m =
sin r
i.e. sin i = sin r
or Ði = Ðr …(ii) This is known as Snell’s law of refraction.
which is the first law of reflection. Further, if l1 and l 2 denote the wavelengths of light in
medium 1 and medium 2, respectively and if the distance BC
Further, the incident wavefront AB, the reflecting surface XY is equal to l1 , then the distance AD will be equal to l 2 , thus
and the reflected wavefront CD are all perpendicular to the
l1 BC v 1 v v
plane of the paper. = = or 1 = 2
Therefore, incident ray, normal to the mirror XY and l 2 AD v 2 l1 l 2
reflected ray all lie in the plane of the paper. This proves the
Þ n1 = n 2 æQ v = n ö
second law of reflection. ç ÷
è l ø
46 CBSE Term II Physics XII

Hence, the frequency does not change on refraction. where, a and b are the respective amplitudes of the two
Thus, frequency n being a characteristic of the source, remains waves and fis the constant phase angle by which second
the same as light travels from one medium to another. wave leads the first wave.
Also, wavelength is directly proportional to the (phase) speed Applying superposition principle, the resultant displacement
and inversely proportional to refractive index. of wave is
l l cl c y = A sin( wt + q)
\ l¢ = , m = = = b
m l¢ v l v
where, A = a 2 + b 2 + 2ab cos f or f
A
b sin f
Doppler’s Effect in Light and tan q =
a + b cos f f
According to this effect, whenever there is a relative motion q
a
between a source of light and observer, the apparent Intensity is directly Resultant of amplitudes a and b
frequency of light received by observer is different from the proportional to the square of
true frequency of light emitted from the source of light. the amplitude of the wave. i.e. I µ a 2 .
Astronomers call the increase in wavelength due to Doppler In general, resultant intensity,
effect as red shift, since a wavelength in the middle of the Þ I R = I1 + I 2 + 2 I1 I 2 cos f
visible region of spectrum moves towards the red end of the
spectrum. When waves are received from a source moving For constructive interference
towards the observer, there is an apparent decrease in cos f = maximum = +1
wavelength, this is referred to as blue shift.
\ Phase difference, f = 0, 2p, 4 p, ...
The fractional change in frequency is given by
i.e. f = 2np, where n = 1, 2 , ...
Dn v
= - radial Path difference, Dx = nl
n c
where, v radial is the component of the source velocity along I max µ ( a + b) 2
the line joining the observer to the source relative to the For destructive interference
observer. cos f = minimum = -1
v radial is considered positive, when the source moves away \ Phase difference, f = p, 3 p, 5 p, ...
from the observer. The above formula is valid only, when the
speed of the source is small compared to that of light. f = ( 2n - 1 ) p, where n = 1, 2, ...
l
Path difference, Dx = ( 2n - 1 )
Superposition Principle 2
According to this principle, at a particular point in the I min µ ( a - b) 2
medium, the resultant displacement (y) produced by a Comparison of Intensities of Maxima and Minima
number of waves is the vector sum of the displacements I max ( r + 1 ) 2
produced by each of the waves ( y1 , y2 , ... ). =
I min ( r - 1 ) 2
i.e. y = y1 + y 2 + y 3 + y4 + ...
a
This principle was stated first for mechanical waves but is where, r = (ratio of amplitudes).
equally applicable to the electromagnetic (light) waves. b

Interference of Light Waves Coherent and Incoherent Sources


The phenomenon of formation of maximum intensity at some Light sources are of two types, i.e. coherent and incoherent
points and minimum intensity at some other points by two light sources. The sources of light, which emit light waves of
identical light waves travelling in same direction is called the same wavelength, same frequency and are in same phase or
interference of light. having constant phase difference are known as coherent
sources. Two such sources of light, which do not emit light
At the points, where the resultant intensity of light is waves with constant phase difference are called incoherent
maximum, interference is said to be constructive. At the sources.
points, where the resultant intensity of light is minimum,
interference is said to be destructive.
Young’s Double Slit Experiment
Theory of Interference of Waves Suppose S1 and S2 are two fine slits, a small distance d apart.
Let the waves from two sources of light be represented as They are illuminated by a strong source Sof monochromatic
y1 = a sin wt light of wavelength l and MN is a screen at a distance D from
and y 2 = b sin ( wt + f) the slits.
CBSE Term II Physics XII 47

Consider a point P at a distance y from O, the centre of The separation between two consecutive dark fringes,
screen.
Dl Dl Dl
The path difference between two waves arriving at point P is b¢ = ( 2n - 1 ) - {2( n - 1 ) - 1} =
2d 2d d
equal to S2 P - S1 P.
M
P
Fringe Width
S1 y
The distance between two consecutive bright or dark fringes
d
is called fringe width.
S O Dl
i.e. b or w =
S2 d
D If YDSE apparatus is immersed in a liquid of refractive index
m, then wavelength of light and hence fringe width decreases
N m times.
é d ù é
2
d ù
2
Now, (S2 P ) 2 - (S1 P ) 2 = êD 2 + æç y + ö÷ ú - êD 2 + æç y - ö÷ ú Intensity of the Fringes
êë è 2 ø úû êë è 2 ø úû
For a bright fringe, f = 2np
= 2yd \ I R = I max = I1 + I 2 + 2 I1 I 2 = 4I
2yd For a dark fringe, f = ( 2n - 1 )p
Thus, S2 P - S1 P =
S2 P + S1 P
\ I R = I min = I1 + I 2 - 2 I1 I 2 = 0
But S2 P + S1 P » 2D
Distribution of Intensity
dy
\ S2 P - S1 P » If w1 and w 2 are widths of two slits from which intensities of
D light I1 and I 2 emanate, then
For constructive interference (Bright fringes) I 1 a 2 w1
= =
dy I2 b2 w2
Path difference = = nl, where n = 0, 1 , 2 , 3, ...
D where, a and b are the respective amplitudes of two waves.
nDl
\ y= (Q n = 0, 1, 2, 3, ... ) Fringe Shift
d
If refracting slab of thickness t is placed infront of one of the
Hence, for n = 0, y 0 = 0 at O for central bright fringe
two slits of YDSE, then fringe pattern gets shifted by n
Dl
for n =1, y1 = for 1st bright fringe fringes and is given by
d
(m - 1 ) t = nl
2Dl
for n = 2, y 2 = for 2nd bright fringe Fringe shift due to insertion of two slabs having thicknesses
d
nDl t 1 & t 2 and refractive indices m 1 & m 2 .
for n = n, y n = for nth bright fringe (m 2 - m 1 ) t = nl
d
The separation between two consecutive bright
nDl ( n - 1 )Dl Dl Diffraction of Light
fringes, b= - = The phenomenon of bending of light around the sharp
d d d
corners and the spreading of light within the geometrical
For destructive interference (Dark fringes) shadow of the opaque obstacles is called diffraction of light.
dy l The dimensions of the aperture or the obstacle are
Path difference = = ( 2n - 1 ) comparable to the wavelength of light.
D 2
Dl
or y = ( 2n - 1 ) , where n = 1, 2, 3, ... Diffracted
2d Incident
wave wave
Dl
Hence, for n = 1, y1¢ = for 1st dark fringe
2d a
3 Dl
for n = 2, y ¢2 = for 2nd dark fringe l
2d
Dl
for n = n, y ¢n = ( 2n - 1 ) for nth dark fringe Screen
2d a>l
48 CBSE Term II Physics XII

Diffraction of Light at a Single Slit Hence, the diffraction pattern can be graphically shown as
below
A parallel beam of light with a plane wavefront is made to fall Y
on a single slit LN. As width of the slit LN = a is of the order Intensity(I)
of wavelength of light, therefore diffraction occurs when I0
beam of light passes through the slit.
The wavelets from the single wavefront reach the centre C on
the screen in same phase. Hence, interfere constructively to
give central maximum (bright fringe). XN X
–3l –2l –l C l 2l 3l
The diffraction pattern obtained on the screen consists of a Path difference ( d sin q)
central bright band, having alternate dark and weak bright
bands of decreasing intensity on both sides. The point C corresponds to the position of central maxima
and the position -3 l, -2l, -l, l, 2l,3l… are secondary
minima. The above conditions for diffraction maxima and
P minima are exactly reverse of mathematical conditions for
interference maxima and minima.
q y
L Width of Central Maximum
M1
Beam of It is the distance between first secondary minimum on either
light from source a M q
M2 Q M2 C side of the central bright fringe C.
N q 2Dl
Width of central maximum = 2y =
a
D As, the slit width a increases, width of central maximum
decreases.
2l
Geometry of single slit diffraction \ Angular width of central maxima, 2q =
a
Consider a point P on the screen at which wavelets travelling Fresnel’s Distance
in a direction, make an angle q with MC. The wavelets from
points L and N will have a path difference equal to NQ. When a slit or hole of size a is illuminated by a parallel beam,
l
From the right angled D LNQ, we have it is diffracted with an angle q » .
a
NQ = LN sin q
or NQ = a sin q
To establish the condition for secondary minima, the slit is zl
a q = l/a a
divided into 2, 4, 6,… equal parts such that corresponding
wavelets from successive regions interfere with path
difference of l/ 2 or for nth secondary minima, the slit can be z
divided into 2n equal parts. Diffraction of a parallel beam
Hence, for nth secondary minima, In travelling a distance z, size of beam is z l / a.
a l zl a2
Path difference = sin q = So, taking ³ a or z ³
2 2 a l
nl Now, distance z F is called Fresnel’s distance ( z F = a 2 / l).
or sin q n = , where n = 1, 2, 3, ...
a
To establish the condition for secondary maxima, the slit is Difference between
divided into 3, 5, 7, … equal parts such that corresponding Interference and Diffraction
wavelets from alternate regions interfere with path difference (i) The interference pattern has a number of equally
of l/ 2.. spaced bright and dark bands whereas the diffraction
For nth secondary maxima, the slit can be divided into pattern has a central bright maximum, which is twice as
( 2n + 1 ) equal parts. wide as the other maxima. The intensity falls as we go to
successive maxima away from the centre on either side.
Hence, for nth secondary maxima,
(ii) We calculate the interference pattern by superposing
l two waves originating from the two narrow slits. The
a sin q n = ( 2n + 1 ) ( n = 1, 2, 3, ... )
2 diffraction pattern is a superposition of a continuous
l family of waves originating from each point on a
or sin q n = ( 2n + 1 ) single slit.
2a
CBSE Term II Physics XII 49

Solved Examples
Example 1 A plane wavefront incident on a reflecting superimpose with a phase difference of (i) zero, (ii)
surface at an angle of 30° with horizontal. Find the p/ 2, (iii) p and (iv) ratio of maximum and minimum
angle of reflected wavefront with horizontal. intensities.
Sol. Consider a plane wavefront OA incident at 30° with the Sol. Given, I1 = I and I2 = 4I
horizontal, ON is normal to the reflecting surface, Ði is angle Intensity, IR = I1 + I2 + 2 I1I2 cos f
of incidence and Ðr is angle of reflection. According to law
of reflection, Ði = Ðr. (i) At f = 0,
Intensity, IR = I + 4I + 2 I × 4I cos 0° = 9I = Imax
N
p
(ii) At f = ,
2
A B Intensity, IR = I + 4I + 2 I × 4I cos( p/ 2) = 5 I
r (iii) At f = p,
i Intensity, IR = I + 4I + 2 I × 4I cos( p ) = I = Imin
30°
O
(iv) Ratio of maximum and minimum intensities,
Imax 9I
= = 9:1
Now, from figure, we have Imin I
Ði + 30° = 90°
Þ Ði = 60° = Ðr Example 4 In a YDSE, green light of wavelength
Angle with horizontal = 90° - Ðr = 30° 500 nm is used. Where will be the second bright
fringe be formed for a set-up in which separation
Example 2 A plane wavefront is incident from air between slits is 4 mm and the screen is placed 1m
( m = 1) at an angle of 37° with a horizontal boundary from the slits?
of a refractive medium from air of refractive index 2l D
Sol. Position of second bright fringe, y2 =
3 d
m = . Find the angle of refracted wavefront with
2 2 ´ 500 ´ 10-9 ´ 1
= = 0.25 mm
the horizontal boundary. 4 ´ 10-3
Sol. It has been given that, incident wavefront makes 37° with
horizontal. Hence, incident ray makes 37° with normal as
Example 5 Fringe width in a particular YDSE is
the ray is perpendicular to the wavefront. measured to be b. What will be the fringe width, if
N Incident
wavelength of the light is doubled, separation
Incident ray wavefront between the slits is halved and separation between
the screen and slits is tripled?
37° lD
Sol. We know that, b =
37° d
r Refracted d
Now, the changed values are l ¢= 2 l, d ¢= and D¢= 3D.
r wavefront 2
Refracted The new fringe width will be
ray l¢ D¢
b ¢=

( 2l )( 3D) lD
sin 37° 3 Þ b ¢= = 12 = 12b
Now, by Snell’s law, = æ dö d
sin r 2 ç ÷
2 3 2 è 2ø
Þ sin r = ´ = The fringe width increases to 12 times of the original.
3 5 5
æ 2ö Example 6 In Young’s double slit experiment,
Þ r = sin -1 ç ÷ , which is same as angle of refractive
è ø
5 separation between slits is 1mm, distance of screen
wavefront with horizontal. from slits is 2m. If wavelength of incident light is
Example 3 Two sources of intensity I and 4I are used 500 nm. determine
in an interference experiment. Find the intensity at (i) fringe width,
a point, where the waves from two sources (ii) angular fringe width,
50 CBSE Term II Physics XII

(iii) distance between 4th bright fringe and 3rd dark 3l 3 ´ 6500 ´ 10- 10 m
\ d= =
fringe and 2sin q 2 ´ sin 30°
(iv) if whole arrangement is immersed in water = 1.95 ´ 10-6 m
(m w = 4/ 3), new angular fringe width. Example 8 A beam of light of wavelength 600 nm from
Sol. Given, d = 1 mm, D = 2m, l = 500 nm = 500 ´ 10-9 m a distant source falls on a single slit 1.0 mm wide
(i) Fringe width, and the resulting diffraction pattern is observed on
Dl 2 ´ 500 ´ 10-9 a screen 2m away. What is the distance between
b= = = 10-3 m = 1 mm
d 1 ´ 10-3 the second bright fringe on either side of the
(ii) Angular fringe width, central bright fringe?
l 500 ´ 10-9 Sol. For the diffraction at a single slit, the position of maxima is
q0 = = = 5 ´ 10-4 rad
d 1 ´ 10-3 given by
l
(iii) Distance of 4th bright fringe from centre, asin q = ( 2n + 1 )
nDl 4Dl 2
y4 = = y
d d For small value of q, sin q » q =
Distance of 3rd dark fringe from centre, D
( 2n + 1 ) Dl For second bright fringe, n = 2.
y3¢ = y 5l 5l
d \ a´ = or y = D
( 2 ´ 2 + 1 ) Dl D 2 2a
=
2d Substituting the values, we have
5 Dl 5 ´ 2 ´ 6 ´ 10-7
= y=
2d 2 ´ 1 ´ 10-3
Distance between 4th bright fringe and 3rd dark fringe, = 3.0 ´ 10-3 m = 3.0 mm
4Dl 5 Dl 3Dl
y4 - y3 ¢= - = Distance between second maxima on either side of central
d 2d 2d
3 3 3 maxima = 2y = 6.0 mm.
= b = ´ 1 = mm
2 2 2 Example 9 Estimate how large can be the aperture
l l 3 opening to work with laws of ray optics using a
(iv) Wavelength in water, l w = = = l
m w 4/ 3 4 monochromatic light of wavelength 450 nm to a
q0 5 ´ 10-4 distance of around 20 m.
( q0 ) w = =
mw 4/ 3 Sol. Now, here we are given Fresnel distance equal to 20 m and
15 l = 450nm, we have to estimate value of a.
= ´ 10-4 rad
4 a2
Putting z F = , a = z F l
l
Example 7 A slit of width d is illuminated by red light
of wavelength 6500Å. For what value of d, will the a = 20 ´ 450 ´ 10- 9
first maximum fall at angle of diffraction of 30°? = 9000 ´ 10- 9
Sol. For first secondary maximum of the diffraction pattern,
3l = 9 ´ 10- 6
d sin q =
2 = 3 ´ 10-3 m or 3 mm
CBSE Term II Physics XII 51

Chapter
Practice
When i > i C (critical angle of incidence), then
PART 1 wavefront EC is
Objective Questions (a) formed further deep in medium 2
(b) formed closer to surface line AC
(c) formed perpendicular to AC
l
Multiple Choice Questions (d) formed in medium 1 (on same side of AB)
7. Which of the following statement is incorrect
1. Rays diverging from a point source produce a conditions for producing sustained interference?
wavefront of which shape?
(a) Phase difference between interferring waves remains
(a) Cylindrical (b) Spherical constant with time.
(c) Plane (d) Cubical (b) Interferring waves have nearly same amplitude levels.
2. According to Huygens’ principle, each point of the (c) Interferring waves are of same frequency.
wavefront is the source of (d) Interferring waves are moving only in opposite
directions.
(a) secondary disturbance (b) primary disturbance
(c) third disturbance (d) fourth disturbance 8. In Young’s double slit experiment, if source S is
shifted by an angle f as shown in figure. Then,
3. For light diverging from a point source, the
central bright fringe will be shifted by angle f
wavefront is spherical and the intensity ……… in
towards [Delhi 2020]
proportion to the distance squared.
(a) increases (b) decreases A
(c) remains the same (d) None of these S¢ S1
f
4. The direction of wavefront of a wave with wave S
Q
O
motion is S2
(a) parallel (b) perpendicular
B
(c) opposite (d) at an angle of q
(a) end A of screen
5. Light waves travel in vacuum along the Y-axis. (b) end B of screen
Which of the following may represent the (c) does not shift at all
wavefront? (d) Either ends A and B depending on extra phase difference
(a) y = constant (b) x = constant caused by shifting of source
(c) z = constant (d) x + y + z = constant
9. The correct curve between fringe width b and
6. In given figure, light passes from denser medium 1 distance between the slits d in figure is
to rarer medium 2.
Incident wavefront
(a) (b)
Medium 1 B
v1t
v1 d d
i A
i C
Medium 2 r
r v2t
v2
(c) (d)
v2 > v1 Refracted wavefront
E d d
52 CBSE Term II Physics XII

10. A thin film of oil is spread on the surface of water. 17. A parallel beam of fast moving electrons is incident
The beautiful colours exhibited in the light of sun is normally on a narrow slit. A screen is placed at a
due to large distance from the slit. If the speed of the
(a) dispersion of light electrons is increased, which of the following
(b) polarisation of light statement is correct?
(c) interference of light (a) Diffraction pattern is not observed on the screen in the
(d) diffraction of light case of electrons.
(b) The angular width of the central maximum of the
11. The phase difference between the two light waves diffraction pattern will increase.
reaching at a point P is 100p. Their path difference (c) The angular width of the central maximum will decrease.
is equal to (d) The angular width of the central maximum will remains
(a) 10 l (b) 25 l the same.
(c) 50l (d) 100l 18. In diffraction from a single slit the angular width of
12. In the phenomenon of interference, energy is the central maxima does not depends on
(a) destroyed at destructive interference (a) l of light used
(b) created at constructive interference (b) width of slit
(c) conserved but it is redistributed (c) distance of slits from the screen D
(d) same at all points (d) ratio of l and slit width
13. Two sources S1 and S2 of intensity I1 and I 2 are 19. What should be the slit width to obtain 10 maxima
placed infront of a screen Fig (a). The pattern of of the double slit pattern within the central maxima
intensity distribution seen in the central portion is of the single slit pattern of slit width 0.4 mm?
given by Fig (b). [NCERT Examplar] (a) 0.4 mm (b) 0.2 mm (c) 0.6 mm (d) 0.8 mm

S1 20. In a single diffraction pattern observed on a screen


x placed at D m distance from the slit of width d m,
S1
x the ratio of the width of the central maxima to the
(a) (b) width of other secondary maxima is
(a) 2 : 1 (b) 1 : 2 (c) 1 : 1 (d) 3 : 1
In this case, which of the following is/are incorrect
for S1 and S2 ? l
Assertion-Reasoning MCQs
(a) Same intensities
(b) Constant phase difference
Direction (Q.Nos. 21-29) Each of these questions
(c) Same phase contains two statements Assertion (A) and Reason (R).
(d) Same wavelength Each of these questions also has four alternative choices,
any one of which is the correct answer. You have to select
14. Two identical and independent sodium lamps act as one of the codes (a), (b), (c) and (d) given below.
(a) coherent sources
(b) incoherent sources (a) Both A and R are true and R is the correct
(c) Either (a) and (b) explanation of A.
(d) None of the above (b) Both A and R are true, but R is not the correct
explanation of A.
15. In Young’s double slit experiment, distance
between slits is kept 1 mm and a screen is kept 1 m (c) A is true, but R is false.
apart from slits. If wavelength of light used is (d) A is false and R is also false.
500 nm, then fringe spacing is 21. Assertion Speed of light is independent of its
(a) 0.5 mm (b) 0.5 cm
colour only in vacuum.
(c) 0.25 mm (d) 0.25 cm
Reason Red colour light travels slower than violet
16. In a Young’s double slit experiment, the source is colour light in glass.
white light. One of the holes is covered by a red
filter and another by a blue filter. In this case, there 22. Assertion Increase in the wavelength of light due
shall be [NCERT Exemplar] to Doppler’s effect is red shift.
(a) alternate interference patterns of red and blue Reason When the wavelength increases, then
(b) an interference pattern for red distinct from that for blue wavelength in the middle of the visible region of
(c) no interference fringes
(d) an interference pattern for red mixing with one for blue
the spectrum will move towards the red end of the
spectrum.
CBSE Term II Physics XII 53

23. Assertion If we have a point source emitting waves wavefront. Light travels in the form of waves.
uniformly in all directions, the locus of point which A wavefront is the locus of points (wavelets) having
have the same amplitude and vibrate in the same the same phase (a surface of constant phase) of
phase are spheres. oscillations. A wavelet is the point of disturbance
Reason Each point of the wavefront is the source of due to propagation of light. Wavefront may also be
a secondary disturbance and the wavelets defined as the hypothetical surface on which the
emanating from these points spread out in all light waves are in the same phase.
directions with the speed of the wave. (i) Huygens’ original theory of light assumed that,
light propagates in the form of
24. Assertion Two coherent source transmit waves of (a) minute elastic particles
equal intensity I 0 . Resultant intensity at a point, (b) transverse electromagnetic wave
l (c) transverse mechanical wave
where path difference is is also I 0 .
3 (d) longitudinal mechanical wave
Reason In interference, resultant intensity at any (ii) A wave normal
point is the average intensity of two individual (a) is parallel to a surface at the point of incidence of a
intensities. wavefront
(b) is the line joining the source of light and an observer
25. Assertion In Young’s double slit experiment, ratio (c) gives the direction of propagation of a wave at a given
I max point
of is infinite.
I min (d) is the envelope that is tangential to the secondary
wavelets
Reason If width of any one of the slits is slightly
(iii) Rays diverging from a linear source form a
increased, then this ratio will decrease.
wavefront that is
26. Assertion No interference pattern is detected, (a) cylindrical
when two coherent sources are infinitely close to (b) spherical
each other. (c) plane
(d) cubical
Reason The fringe width is inversely proportional
to the distance between the two slits. (iv) According to Huygens’ principle, a wavefront
propagates through a medium by
27. Assertion In Young’s double slit experiment, the (a) pushing medium particles
fringe width for dark fringes is same as that for (b) propagating through medium with speed of light
white fringes. (c) carrying particles of same phase along with it
Reason In Young’s double slit experiment, when (d) creating secondary wavelets which forms a new
the fringes are formed with a source of white light, wavefront
then only dark and bright fringes are observed. (v) In case of reflection of a wavefront from a reflecting
28. Assertion Diffraction does not determines the surface,
limitations of the concepts of light rays.
Incident
Reason A beam of width a starts to spread out due to wavefront
diffraction after it has travelled a distance ( 2a 2 / l ). E
B
Reflected
wavefront
29. Assertion In interference, different maximas have i
same intensities. A i r C
M N
Reason In diffraction phenomenon, different
maximas have different intensities. I. points A and E are in same phase.
II. points A and C are in same phase.
l
Case Based MCQs III. points A and B are in same phase.
Direction Read the following passage and answer the IV. points C and E are in same phase.
questions that follows Which of the following is/are correct?
30. Wavefront (a) Both I and II
(b) Both II and III
In 1678, a Dutch scientist, Christian Huygens’
(c) Both III and IV
propounded the wave theory of light. According to
(d) Both I and IV
him, wave theory introduced the concepts of
54 CBSE Term II Physics XII

nl
PART 2 angles
d
. Justify this by suitably dividing the slit

Subjective Questions to bring out the cancellation. [NCERT]


11. Two wavelengths of sodium light of 590 nm and
l
Short Answer (SA) Type Questions 596 nm are used in turn to study the diffraction
taking place at a single slit of aperture 2 ´ 10 -6 m. If
1. What is the shape of the wavefront on earth for
sunlight? [NCERT Exemplar]
the distance between the slit and the screen is 1.5
m, calculate the separation between the positions of
2. Discuss Doppler’s effect in the electromagnetic the second maxima of diffraction pattern obtained
waves. in the two cases. [CBSE 2019]
3. Answer the following questions.
12. In the diffraction due to a single slit experiment,
(i) When a low flying aircraft passes overhead, we the aperture of the slit is 3 mm. If monochromatic
sometimes notice a slight shaking of the picture light of wavelength 620 nm is incident normally on
on our TV screen. Suggest a possible the slit, calculate the separation between the first
explanation. order minima and the third order maxima on one
(ii) As, you have learnt in the text, the principle of side of the screen. The distance between the slit and
linear superposition of wave displacement is the screen is 1.5 m. [CBSE 2019]
basic to understanding intensity distributions in
13. (i) In a single slit diffraction experiment, the width
diffraction and interference patterns.
of the slit is made double the original width.
What is the justification of this principle? [NCERT] How does this affect the size and intensity of the
4. The 6563 Å H a -line emitted by hydrogen in a star central diffraction band? Explain.
is found to be red shifted by 15 Å. Estimate the (ii) When a tiny circular obstacle is placed in the
speed with which the star is receding from the path of light from a distant source, a bright spot
earth. [NCERT] is seen at the centre of the obstacle. Explain,
5. In Young’s double slit experiment, the two slits are why. [CBSE 2011]
illuminated by two different lamps having same 14. For the same objective, find the ratio of the least
wavelength of light. Explain with reason, whether separation between two points to be distinguished
interference pattern will be observed on the screen by a microscope for light of 5000 Å and electrons
or not. [All India 2017] accelerated through 100V used as the illuminating
6. For a single slit of width a, the first minimum of the substance. [NCERT Exemplar]
interference pattern of a monochromatic light of 15. (i) Why are coherent sources necessary to produce
l a sustained interference pattern?
wavelength l occurs at an angle of . At the same
a (ii) In Young’s double slit experiment using
l
angle of , we get a maximum for two narrow slits monochromatic light of wavelength l , the
a intensity of light at a point on the screen, where
separated by a distance a . Explain. [Delhi 2014]
path difference is l , is K unit. Find out the
7. Draw the intensity pattern for single slit diffraction intensity of light at a point, where path
and double slit interference. Hence, state two difference is l / 3. [Delhi 2012, NCERT]
differences between interference and diffraction
16. Two coherent light waves of intensity 5 ´ 10 -2 Wm -2
patterns. [All India 2017]
each superimpose and produce the interference
8. State briefly two features which can distinguish the pattern on a screen. At a point where the path
characteristic features of an interference pattern l
from those observed in the diffraction pattern due difference between the waves is , l being
6
to a single slit.
wavelength of the wave, find the
9. Why is the diffraction of sound waves more evident (i) phase difference between the waves,
in daily experience than that of light wave?
[NCERT Exemplar] (ii) resultant intensity at the point and
10. In deriving the single slit diffraction (iii) resultant intensity in terms of the intensity at the
pattern, it was stated that the intensity is zero at maximum. [All India 2020]
CBSE Term II Physics XII 55

17. Consider a two slit interference arrangements (ii) Is the frequency of reflected and refracted light
(figure), such that the distance of the screen from same as the frequency of incident light?
[Delhi 2020, 13]
the slits is half the distance between the slits.
Obtain the value of D in terms of l, such that the 22. (i) Using Huygens’ construction of secondary
first minima on the screen falls at a distance D from wavelets explain how a diffraction pattern is
the centre O. [NCERT Exemplar] obtained on a screen due to a narrow slit on
which a monochromatic beam of light is incident
S1 T1 normally.
Source P
O (ii) Show that the angular width of the first diffraction
S C
fringe is half that of the central fringe.
S2 T2
Screen 23. (i) What is the effect on the interference fringes to a
Young’s double slit experiment, when
18. Answer the following questions:
(i) In a double slit experiment using light of (a) the width of the source slit is increased and
wavelength 600 nm, the angular width of the (b) the monochromatic source is replaced by a source
fringe formed on a distant screen is 0. 1°. Find the of white light? Justify your answer in each case.
spacing between the two slits.
(ii) The intensity at the central maxima in Young’s
(ii) Light of wavelength 500 Å propagating in air double slit experiment set-up is I 0 . Show that the
gets partly reflected from the surface of water. intensity at a point, where the path difference is
How will the wavelengths and frequencies of the l / 3 is I 0 / 4. [Foreign 2012]
reflected and refracted light be affected?
[Delhi 2020, 15] 24. Consider two coherent sources S1 and S2
19. A beam of light consisting of two wavelengths producing monochromatic waves to produce
650 nm and 520 nm is used to obtain interference interference pattern.
fringes in a Young’s double slit experiment. Let the displacement of the wave produced by S1
(i) Find the distance of the third bright fringe on be given by y1 = a cos wt and the displacement by
the screen from the central maximum for S2 be y 2 = a cos(wt + f ).
wavelength 650 nm. Find out the expression for the amplitude of the
(ii) What is the least distance from the central resultant displacement at a point and show that the
maximum, where the bright fringes due to both intensity at that point will be
f
the wavelengths coincide? [NCERT] I = 4a 2 cos 2
2
20. A small transparent slab containing material of
Hence, establish the conditions for constructive
m = 15
. is placed along AS2 (figure). What will be
and destructive interference. [Delhi 2015]
the distance from O of the principal maxima and of
the first minima on either side of the principal 25. (i) In Young’s double slit experiment, derive the
maxima obtained in the absence of the glass slab? condition for
[NCERT Exemplar] (a) constructive interference and
S1
P1 (b) destructive interference at a point on the screen.
A q (ii) A beam of light consisting of two wavelengths,
C O
800 nm and 600 nm is used to obtain the
L = d/4 S2 interference fringes on a screen placed 1.4 m
Screen
away in a Young’s double slit experiment. If the
(AC = CO = D, S1C = S2C = d << D) two slits are separated by 0.28 mm, calculate the
least distance from the central bright maximum
l
Long Answer (LA) Type Questions where the bright fringes of the two wavelengths
coincide. [All India 2012]
21. (i) State Huygens’ principle. Using this principle,
draw a diagram to show how a plane wavefront 26. Four identical monochromatic sources A, B, C, D as
incident at the interference of the two media gets shown in the (figure) produce waves of the same
refracted when it propagates from a rarer to a wavelength l and are coherent. Two receiver R1
denser medium. and R 2 are at great but equal distances from B.
Hence, verify Snell’s law of refraction.
56 CBSE Term II Physics XII

(i) Which of the two receivers picks up the larger beams overlap. Alternate bright and dark equally
signal? spaced vertical bands (interference fringes) can be
(ii) Which of the two receivers picks up the larger observed on a screen placed at same distance from
signal, when B is turned off? the slits. If either of S1 or S2 is covered, then the
fringes disappear.
(iii) Which of the two receivers picks up the larger
signal, when D is turned off?
(iv) Which of the two receivers can distinguish Bright
which of the sources B or D has been turned off?
(R1 B = d = R 2 B Dark
AB = BC = BD = l / 2) [NCERT Exemplar]
R2 Bright

Monochromatic
S1

light source
Dark

}
Interference
effects in
S0 Bright
region, where
S1 Dark beams overlap
l/2 l/2
Bright
R1 A B C Single
slit Double
l/2 Dark
slits

Bright
D
Viewing
l
Case Based Questions screen
Young’s experimental arrangement to produce interference pattern
Direction Read the following passage and answer the
questions that follows (i) Suppose while performing double slit
27. Young’s Experiment experiment, the space between the slits and the
In 1801, Thomas Young was the first who screen is filled with water. How does the
demonstrated the interference of light. interference pattern change?
In his experimental arrangement (as shown in (ii) In Young’s double slit experiment, if the distance
figure), monochromatic light (single wavelength) between the slits is halved, what changes in the
from a narrow vertical slit S falls on two other fringe width will take place?
narrow slits S1 and S2 which are very close together (iii) The ratio of the widths of two slits in Young’s
and equidistant to S. S1 and S2 act as two coherent double slit experiment is 4:1.
sources (both being derived from S ). The emerging Evaluate the ratio of intensities at maxima and
beams spread into the region beyond the slits. minima in the interference pattern. [Delhi 2015]
Superposition occurs in the shaded area, where the
Chapter Test
Multiple Choice Questions 9. Whose fringe width will be larger, the one for red
1. A monochromatic light refracts by the medium of light or the one for yellow light, all other things be
refractive index 1.5 in vacuum. The wavelength of the same?
refracted wave will be 10. What do you understand by optical path? Prove that
(a) equal optical path length d o = nd m, where d m is the
(b) increase distance travelled in medium having refractive
(c) decrease index n .
(d) depend upon the intensity of refracted light
Long Answer Type Questions
2. Two distinct light bulbs as sources
(a) can produce an interference pattern 11. Explain the following giving reasons:
(b) cannot produce a sustained interference pattern (i) When monochromatic light is incident on a
(c) can produce an interference pattern, if they produce light of surface separating two media, then both reflected
same frequency and refracted light have the same frequency as the
(d) can produce an interference pattern only when the light incident frequency.
produced by them is monochromatic in nature (ii) When light travels from a rarer to a denser
3. From a single slit, the first diffraction minima is obtained medium, then speed decreases. Does this decrease
at 30° for a light of 6500 Å wavelength. The width of the in speed imply a reduction in the energy carried
slit is by the wave?
(a) 3250 Å (b) 1.3 m (iii) In the photon picture of light, intensity of light is
(c) 5. 4 ´ 10 - 4 km . ´ 10 - 2 cm
(d) 12 determined by the square of the amplitude of the
wave. What determines the intensity in the photon
4. In Young’s double slit experiment, two disturbance
picture of light?
arriving at a point P have phase difference of p / 2. The
intensity of this point expressed as a fraction of 12. A light of 2000 - 8000 Å range is allowed to fall on
maximum intensity I0 is two slits having separation of 2 mm between them. A
3 1 screen is placed at 2.5 m away from the slit.
(a) I 0 (b) I 0
2 2 Find the wavelength in the visible region that will be
4 3 present on the screen at 10 -3 m from the central
(c) I 0 (d) I 0
3 4 maxima. Also, find the wavelength that will be
present at that point of screen in the infrared as well
5. If the width of slit is decreased in a single slit diffraction,
as in the ultraviolet region.
then the width of central maxima will
[Ans. 8000 Å (infrared), 4000 Å (visible),
(a) increase
2666 Å (ultraviolet), 2000 Å (ultraviolet)]
(b) decrease
(c) remain unchanged 13. (i) Explain two features to distinguish between the
(d) not depend on the width of slit. interference pattern in Young's double slit
experiment with the diffraction pattern obtained
Short Answer Type Questions due to a single slit.
6. Use Huygens’ geometrical construction to show the (ii) A monochromatic light of wavelength 500 nm is
behaviour of a plane wavefront, incident normally on a single slit of width 0.2 mm
(i) passing through a biconvex lens and to produce a diffraction pattern. Find the angular
(ii) reflected by a concave mirror. width of the central maximum obtained on the
screen.
7. The ratio of maximum and minimum intensities of two
sources is 4 : 1. Find the ratio of their amplitudes. Estimate the number of fringes obtained in
(Ans. 3 : 1) Young's double slit experiment with fringe width
0.5 mm, which can be accommodated within the
8. In a single slit diffraction experiment, the width of the slit region of total angular spread of the central
is decreased. How will the (a) size (b) and intensity of the
maximum due to single slit. [Ans. 5)
central bright band be affected? Justify your answer.

Answers
Multiple Choice Questions For Detailed Solutions
1. (c) 2. (b) 3. (b) 4. (b) 5. (a)
Scan the code
58 CBSE Term II Physics XII

EXPLANATIONS
PART 1 We define critical angle iC by the formula,
n
sin iC = 2 ...(ii)
1. (b) Wavefronts emitting from a point source are spherical n1
wavefronts.
Thus, if i = i C , then from Eqs. (i) and (ii), sin r =1 and r = 90°.
Obviously for i > i C , there cannot be any refracted wave. So,
for all angles of incidence greater than the critical angle, we
Rays are will not have any wavefront in medium 2. Hence, it is formed
perpendicular
in medium 1 (on same side of AB).
Point to wavefronts
source 7. (d) The statement given in option (d) is incorrect and it can
Wavefronts be corrected as,
are concentric
spheres The motion of interferring waves can be in same or in
opposite directions.
Light rays Rest statements are correct.
8. (b) Since, when the source S is on the perpendicular bisector,
2. (a) According to Huygens’ principle, each point of the then the central bright fringe occurs at O, which is also on
wavefront is the source of a secondary disturbance and the the perpendicular bisector.
wavelength emanating from these points spread out in all If S is shifted by an angle f to point S¢, then the central fringe
directions with the speed of the wave. appears at a point O¢ at an angle -f (shown in figure below),
3. (b) Consider the diagram in which light diverges from a which means that it is shifted by the same angle on the other
point source O. side of the bisector, i.e. towards end B of screen.
A
S'

f S1
O S O
Q f
r S2
Spherical
O'
Due to the point source, light propagates in all directions
symmetrically and hence, wavefront will be spherical as B
shown in the diagram.
lD
If power of the source is P, then intensity of the source will be 9. (b) As, b =
P d
I= 1
4pr 2 \ bµ
d
where, r is radius of the wavefront at any time.
So, the correct graph is represented by option (b).
Hence, intensity will decreases in proportion to the distance
squared distance. 10. (c) The light reflected from the oil film produced two
coherent waves and these waves are superposed
4. (b) The speed with which the wavefront moves outwards
(interference) and produce beautiful colours.
from the source is called the speed of the wave. The
direction of the wave travels in a direction perpendicular to 11. (c) Given, Df = 100 p
the wavefront (as shown below) We know that, change in phase difference,
2p
i.e. Df = ´ Dx
l
where, Dx = path difference.
Light rays
l l
Þ Dx = D f ´ = 100p ´ = 50l
2p 2p
Plane wavefront 12. (c) In the phenomenon of interference, energy is conserved
but it is redistributed.
5. (a) As, velocity of light is perpendicular to the wavefront and
light wave is travelling in vacuum along the Y-axis, therefore 13. (c) Consider the pattern of the intensity shown in the figure
given in question.
the wavefront is represented by y = constant.
(i) As intensities of all successive minima is zero, hence
6. (d) As we know, for given figure from Snell’s law we can say that two sources S1 and S2 are having same
sin i n 2 intensities.
= ...(i)
sin r n1 (ii) Regular pattern shows constant phase difference
where, n1 and n 2 = refractive indices of medium 1 and between sources S1 and S2 but these sources are not in
medium 2, respectively. same phase.
CBSE Term II Physics XII 59

(iii) We are using monochromatic light in YDSE to avoid 23. (a) According to Huygens’ principle, each point of the
overlapping and to have very clear pattern on the wavefront is the source of a secondary disturbance and the
screen, so wavelength of S1 and S2 will be same. wavelets emanating from these point spread out in all
Thus, the statement given option (c) is incorrect, rest are directions with the speed of wave.
correct. These wavelets emanating from the wavefront are usually
14. (b) Two identical and independent sodium lamps (i.e. two referred to as secondary wavelets and if we draw a common
independent sources of light) can never be coherent. tangent to all these spheres, we obtain the new position of the
So, sodium lamps will be incoherent. wavefront at a later time.
15. (a) Fringe spacing, So, for a point source emitting waves uniformly in all
directions, the locus of points which have the same amplitude
Dl 1 ´ 5 ´ 10-7
b= = m (1 nm = 10-9 m) and vibrate in the same phase are spheres and we have a
d 1 ´ 10-3 spherical wave.
= 5 ´ 10-4 m = 0.5 mm Therefore, both A and R are true and R is the correct
explanation of A.
16. (c) In this case due to filteration, only red and blue lights are
2p 2p l 2p
present. In YDSE, monochromatic light is used for the 24. (c) Phase difference, f = × Dx = ´ =
formation of fringes on the screen. Hence, in this case there l l 3 3
shall be no interference fringes. 2 æ fö 1
and I = 4I0 cos ç ÷ = 4I0 ´ = I0
17. (c) Angular fringe width is the ratio of fringe width to è 2ø 4
distance D of screen from the source. i.e. Therefore, A is true but R is false.
b
q= 25. (b) Imax > 4I0 and Imin = 0
D
As, D is taken large, hence angular fringe width of the Imax
= infinite
central maximum will decrease. Imin
18. (c) Angular width of central maxima, If width of one slit is slightly increased Imin > 0, then this
2q = 2l / e ratio will be less than infinite.
Thus, q does not depend on D, i.e. distance between the slit Therefore, both A and R are true but R is not the correct
and the screen. explanation of A.
19. (b) As, the path difference aq is l, then 26. (a) When distance between slits (d) is negligibly small,
l æ lD ö 1
q= fringe width b ç = ÷ which is proportional to may
a è d ø d
10l 2l become too large. Even a single fringe may occupy the
Þ =
d a whole screen, hence the pattern cannot be detected.
d 10 Therefore, both A and R are true and R is the correct
Þ a= = = 0.2 mm
5 5 explanation of A.
So, the width of each slit is 0.2 mm. 27. (c) In Young’s double slit experiment, fringe width for dark
and white fringes are same, while in the same experiment,
20. (a) Width of central maxima = 2lD/ e
when a white light (as source) is used, the central fringe is
Width of other secondary maxima = lD / e white around which few coloured fringes are observed on
\ Width of central maxima : Width of other secondary either side.
maxima = 2:1. Therefore, A is true but R is false.
21. (c) Speed of light is independent of its colour only in 28. (d) Diffraction determines the limitations of the concept of
vacuum. light rays.
As wavelength of red colour ( l1 ) is higher than wavelength a2
of blue colour (l 2 ). A beam of width a travels a distance , called the Fresnel
l
v l
For light travelling from medium 1 to medium 2, 1 = 1 distance, before it starts to spread out due to diffraction.
v2 l 2 Therefore, A is false and R is also false.
where, v1 and v2 are velocities. 29. (c) All bands of bright interference receive equal amount of
So, red colour light travels faster than violet colour light in energy collectively. Thus, the different maximas have same
glass. intensities.
Therefore, A is true but R is false. But in diffraction, only central maxima receives most of the
22. (a) Increase in wavelength of light, when the source move away energy from the slit and hence the remaining maxima’s
from the observer due to Doppler’s effect is called red shift. intensity falls down significantly.
The visible regions shifts towards red end of Therefore, A is true but R is false.
electromagnetic spectrum and hence called red shift. 30. (i) (d) Huygens’ original wave theory of light assumes that,
Therefore, both A and R are true and R is the correct light propagates in the form of longitudinal mechanical wave.
explanation of A. (ii) (c) A wave normal is a line perpendicular to the
wavefront. It gives the direction of a moving wave.
60 CBSE Term II Physics XII

(iii) (a) Wavefronts emanating from a linear source are 4. Given, l = 6563 Å, Dl = + 15 Å and c = 3 ´ 108 ms-1
cylindrical wavefronts.
Since, the star is receding away, hence its velocity v is
(iv) (d) According to Huygens’ principle, each point of the negative.
wavefront is the source of a secondary disturbance and
vl
the wavelets emanating from these points spread out in \ Dl = -
all directions with the speed of the wave. c
(v) (c) Figure shows AB as incident wavefront, so A and B are c Dl
or v=-
in same phase. l
3 ´ 108 ´ 15
B =-
6563
= - 6.86 ´ 105 ms -1
A
Negative sign shows that star is receding away from the
earth.
E B 5. No interference pattern will be observed on the screen. This
is because, the sources will serve as incoherent sources.
A C Here, two different lamps will act as different sources of
light which do not maintain constant phase difference.
By the time B reaches C, secondary wavelet from A
Dx
reaches E. 6. In diffraction, angular position, q =
So, points C and E are same time intervals apart as they a
For first minima, Dx = l
are in same phase.
l
Hence, option (c) is correct. \ q=
a
In interference, d = a (given) and angular position,
PART 2 q=
Dx
a
1. We know that, the sun is at very large distance from the
earth. Assuming sun as spherical, it can be considered as \ Angular position of first maxima ( Dx = l ), q = l / a
point source situated at infinity. 7. Intensity pattern for single slit diffraction is shown below
Due to the large distance, the radius of wavefront can be Intensity
considered as large (infinity) and hence wavefront is almost Imax
plane.

q
O Earth –l l Angular
a a
Sun position
Almost Intensity pattern for double slit interference pattern is as
plane shown below
wavefront Intensity
2. According to the Doppler’s effect, whenever there is a 4I0
relative motion between a source of light and observer, the
apparent frequency of light received by observer is different
from the true frequency of light emitted from the source of
light. The fractional change in frequency is given
by –l 0 l
Path
–l l difference
Dn v 2 2
= - radial
n c Difference between diffraction and interference pattern are
where, vradial is the component of the source velocity along (i) In interference pattern, all maxima and all minima are of
the line joining the observer to the source relative to the same width but in diffraction pattern, width of central
observer and vradial is considered positive, when the source maxima is maximum and for successive maxima, it goes
moves away from the observer. on decreasing.
3. (i) We notice a slight shaking of the picture on our TV (ii) In interference pattern, each maxima have same
screen because a low flying aircraft reflects the TV signal intensity while in diffraction pattern, intensity of central
and there may be an interference between the maxima is largest and it decreases rapidly for successive
direct signal and the reflected signal. maxima.
(ii) The superposition principle follows the linear character of 8. In case of single slit, the diffraction pattern obtained on the
the differential equation governing wave motion. If y1 and screen consists of a central bright band having alternate
y2 be the solutions of wave equation, so there can be any dark and weak bright band of decreasing intensity on both
linear combination of y1 and y2 . When the amplitudes are sides. The diffraction pattern can be graphically represented
large and non-linear effects are important, then the as
situation is more complicated.
CBSE Term II Physics XII 61

Y The distane of first order minima from central maxima,


Intensity(I) nlD lD
x1 = = ( n = 1)
b b
620 ´ 10-9 ´ 1.5
=
3 ´ 10-3
XN X = 310 ´ 10-6 m
–3l –2l –l C l 2l 3l
Path difference ( d sin q) The distance at third order maxima from central maxima,
( 2n + 1 ) l D 7l D
x3 ¢ = =
Points to compare the intensity distribution between 2b 2b
interference and diffraction
7 ´ 620 ´ 10-9 ´ 1.5
(i) In interference, all bright fringes have same intensity, =
but in diffraction all the bright fringes are not of same 2 ´ 3 ´ 10-3
intensity. = 1085 ´ 10-6 m
(ii) In interference, the widths of all the fringes are same but
in diffraction fringes are of different widths. Thus, distance between x1 and x 3 is
The point C corresponds to the position of central maxima x = x 3 - x1
and the position -3l, -2l, - l, l, 2l, 3l… are secondary = ( 1085 - 310) ´ 10-6
minima. The above conditions for diffraction maxima and
= 775 ´ 10-6 m
minima are exactly reverse of mathematical conditions for
interference maxima and minima. 13. (i) We know that, width of central maximum is given as
9. As we know that, the frequencies of sound waves lie 2Dl
2y =
between 20 Hz to 20 kHz, so their wavelength ranges a
between 15 m to 15 mm. The diffraction occurs, if the where, a = width of slit.
wavelength of waves is nearly equal to slit width. When a¢ = 2a, then width of central maximum
As, the wavelength of light waves is 7000 ´ 10-10 m to 2Dl lD
= =
4000 ´ 10-10 m, the slit width is very near to the wavelength 2a a
of sound waves as compared to light waves. Thus, the Thus, the width of central maximum became half. But
diffraction of sound waves is more evident in daily life than in case of diffraction, intensity of central maxima does
that of light waves. not changes with slit width. Thus, the intensity remains
10. Let the slit be divided into n smaller slits each of width, same in both cases.
d (ii) When a tiny circular obstacle is placed in the path of
d¢ = light from a distant source, a bright spot is seen at the
n
nl nl l centre of the obstacle because the waves diffracted
Angle, q= = = from the edge of circular obstacle interfere
d d ¢n d ¢ constructively at the centre of the shadow resulting in
Therefore, each of the smaller slit would send zero intensity the formation of a bright spot.
in the direction of q. Hence, for the entire single slit, intensity 14. We know that,
nl 1 2sin b
at angle would be zero. Resolving power = =
d d 1 .22 l
11. Given, l1 = 590 nm, l 2 = 596 nm, 1.22 l
Þ dmin =
d = 2 ´ 10-6 m , D = 1.5 m 2 sin b
Distance of secondary maxima from centre, where, l is the wavelength of light and b is the angle
3 Dl subtended by the objective at the object.
x=
2 d For the light of wavelength 5500 Å,
Spacing between the first two maxima of sodium light 1.22 ´ 5500 ´ 10-10
3D dmin = ... (i)
Þ x 2 - x1 = ( l 2 - l1 ) 2 sin b
2d
For electrons accelerated through 100 V, the de-Broglie
3 ´ 1.5
= ( 596 - 590) ´ 10-9 wavelength,
2 ´ 2 ´ 10-6 12. 27
l=
= 6.75 ´ 10-3 m = 6.75 mm V
12. Given, l = 620 nm = 620 ´ 10-9 m 12.27
= = 0.12 ´ 10-9 m
100
Aperture of slit, b = 3 mm = 3 ´ 10-3 m
. ´ 10-9
1.22 ´ 012
Distance between source and screen, ¢ =
dmin
D = 1.5 m 2 sin b
62 CBSE Term II Physics XII

Ratio of the least separation, 17. From the given figure of two slit interference arrangements,
¢
dmin . ´ 10-9
012 we can write
= = 0.2 ´ 10-3
dmin 5500 ´ 10-10 D
T1
S1
15. (i) In order to obtain a well defined observable Source PD
x
interference pattern, the intensity at points of C
O
S
constructive and destructive interference must be D
maintained maximum and almost zero, respectively. S2 T2
The two sources producing interference must be Screen
coherent in nature. T2 P = T2 O + OP = D + x
(ii) Intensity of light at a point on the screen is given by and T1P = T1O - OP = D - x
IR = I1 + I2 + 2 I1I2 cos f 2 2 2 2
1 = ( ST
SP 1 1 ) + ( PT1 ) = D + ( D - x )
For the path difference l, phase difference is 2p.
As, sources are coherent and taken out of the same source and S2 P = ( S2T2 ) 2 + ( T2 P ) 2 = D2 + ( D + x ) 2
in Young’s double slit experiment, l
I1 = I2 = I The minima will occur when S2 P - SP
1 = ( 2n - 1 )
2
Þ IR = 2I + 2I cos 2p l
i.e. [ D2 + ( D + x ) 2 ]1/ 2 - [ D2 + ( D - x ) 2 ]1/ 2 =
Þ IR = 4I 2
Þ 4I = K unit …(i) (for first minima, n = 1)
l If x = D, we can write
For the path difference, corresponding to phase
3 l
2p ( D2 + 4D2 )1/ 2 - ( D2 + 0)1/ 2 =
difference of . 2
3 l
2p Þ ( 5 D2 )1/ 2 - ( D2 )1/ 2 =
IR = 2I + 2I cos = 2I - I = I …(ii) 2
3 l
From Eqs. (i) and (ii), we conclude Þ 5D - D =
2
K
IR = unit Þ D ( 5 - 1) = l / 2
4
16. Given, intensity, I = 5 ´ 10-2 Wm -2 l
or D=
l 2 ( 5 - 1)
Path difference, Dx =
6 Putting 5 = 2 . 236
(i) As, path difference between the interfering waves is Þ 5 - 1 = 2 . 236 - 1 = 1 . 236
l
given as Dx = f l
2p \ D= = 0.404 l
where, f = phase difference. 2( 1.236)
l l p l l
Þ f= Þ f= 18. (i) Angular width, q = or d =
2p 6 3 d q
(ii) Resultant intensity, Given, l = 600 nm = 6 ´ 10-7 m
IR = I1 + I2 + 2 I1I2 cos f
0.1 p p
Here, I1 = I2 = I = 5 ´ 10-2 Wm -2 q= rad = rad
180 1800
p
Þ IR = 2I + 2I cos 6 ´10-7 ´1800
3 \ d= = 3. 44 ´10-4 m
p
= 3I = 3 ´ 5 ´ 10-2
(ii) Frequency of a light depends on its source only, so the
= 15 ´ 10-2 Wm -2
frequencies of reflected and refracted light will be same
(iii) As, IR = 3I …(i) as that of incident light.
Maximum intensity is obtained, when phase difference is Reflected light is in the same medium (air), so its
zero or even integral multiple of 2p. wavelength remains same as 500 Å.
Þ Imax = I + I + 2 I ´ I cos 2np Wavelength of refracted light, l r = l / m w
= 2I + 2I (Q cos 2np = 1 ) where, m w = refractive index of water.
= 4I …(ii) So, wavelength of refracted wave will be decreased.
Dividing Eq. (i) by Eq. (ii), we get 19. Given, wavelength, l1 = 650 nm = 650 ´ 10- 9 m
IR 3I
= and l 2 = 520 nm = 520 ´ 10- 9 m
Imax 4I
(i) For third bright fringe, n = 3
3
or IR = Imax The distance of third bright fringe from central
4
maximum.
CBSE Term II Physics XII 63

nlD D
x= = 3 ´ 650 ´ 10- 9 ´ m 21. (i) Huygens’ Principle
d d Each point on the given wavefront (called primary
3 ´ 650 ´ 10- 9 ´ 1.2 wavefront) is the source of a secondary disturbance
= = 1.17 ´ 10- 3 m
2 ´ 10-3 (called secondary wavelets) and the wavelets
emanating from these points spread out in all
(ii) Let nth bright fringe due to wavelength l 2 = 520 nm,
directions with the speed of the wave. Consider a
coincide with ( n + 1)th bright fringe due to wavelength plane wavefront AB incident on a plane surface XY,
l1 = 650 nm. separating two media 1 and 2, as shown in figure. Let v1
D D and v2 be the velocities of light in two media, with
i.e. nl 2 = ( n - 1 ) l1
d d v2 < v1.
n ´ 520 ´ 10- 9 = ( n - 1 ) 650 ´ 10- 9 N
Incident wavefront
or 4n = 5 n - 5 B
or n =5 S v1 t
i
Thus, the least distance, i Rarer-v1
D D X Y
r C Denser-v2
x = nl 2 = 5 ´ 520 ´ 10- 9 A
r v2 t
d d
Refracted wavefront
D -9
x = 2600 ´ 10 m D
d
1.2 ´ 10-9 The wavefront first strikes at point A and then at the
= 2600 ´ m
2 ´ 10- 3 successive points towards C. According to Huygens’
principle from each point on AC, the secondary
= 1.56 ´ 10- 3 m = 1.56 mm wavelets starts growing in the second medium with
20. In case of transparent glass slab of refractive index m, the speed v2 . Let the distrubance takes time t to travel
path difference will be calculated as from B to C, then BC = v1t.
Dx = 2d sin q + ( m - 1 ) L During the time the disturbance from B reaches the
point C, the secondary wavelets from point A must
For the principal maxima (path difference is zero), have spread over a hemisphere of radius AD = v2 t in
i.e. 2d sin q0 + ( m - 1 ) L = 0 the second medium. The tangent plane CD drawn from
L ( m - 1 ) - L ( 05
. ) point C over this hemisphere of radius v2 t will be the
or sin q0 = - = (Q L = d / 4 ) new refracted wavefront. Let the angles of incidence
2d 2d
and refraction be i and r, respectively.
-1
or sin q0 = From right DABC, we have, sin ÐBAC = sin i =
BC
16 AC
-D AD
\ OP = D tan q0 » D sin q0 = From right DADC , we have sin ÐDCA = sin r =
16 AC
l sin i BC v1t sin i v1
For the first minima, the path difference is ± . \ = = or = = m 21
2 sin r AD v2 t sin r v2
l This proves Snell’s law of refraction. The constant m 21
\ 2d sin q1 + 05. L=±
2 is called the refractive index of the second medium
± l / 2 - 05
. L ± l/ 2 - d / 8 with respect to first medium.
or sin q1 = = Further, since the incident ray SA, the normal AN and
2d 2d
the refracted ray AD are respectively perpendicular to
± l/ 2 - l/ 8 1 1 the incident wavefront AB, the dividing surfae XY and
= =± -
2l 4 16 the refracted wavefron CD (all perpendicular to the
[Q The diffraction occurs, if the wavelength of waves is nearly plane of the paper), therefore they all lie in the plane
equal to the side width (d)] of the paper, i.e. in the same plane. This proves
another law of refraction.
+ 1 1 3
On the positive side sin q¢1 = + - = (ii) The reflection and refraction phenomenon occur due to
4 16 16
interaction of corpuscles of incident light and the
1 1 5
On the negative side sin q¢¢1- = - - =- atoms of matter on receiving light energy, the atoms
4 16 16 are forced to oscillate about their mean positions with
The first principal maxima on the positive side is at distance the same frequency as incident light. According to
sin q¢1+ 3 3D Maxwell’s classical theory, the frequency of light
D tan q¢1+ = D =D = above emitted by a charged oscillator is same as its frequency
2 2 2 247
1 - sin q¢1 16 - 3 of oscillation. Thus, the frequency of reflected and
point O. refracted light is same as the incident frequency.
The first principal minima on the negative side is at distance 22. (i) Consider a parallel beam of light from a lens falling on a
5D 5D slit AB. As diffraction occurs, the pattern is focused on
D tan q¢¢1 = = below point O. the screen with the help of lens L 2 . We will obtain a
2 2 231
16 - 5 diffraction pattern that is central maximum at the centre
64 CBSE Term II Physics XII

O, flanked by a number of dark and bright fringes called (ii) Given, OP = yn


secondary maxima and minima. P
Slit L2
r yn
L1 a n dicula S1
Perpe P
q O
A y d
q q S2
S C O
B q
D

Plane
wavefront D The distance OP equals one-third of fringe width of the
pattern.
Each point on the plane wavefront AB sends out the b 1 æ Dl ö Dl
secondary wavelets in all directions. The waves from i.e. yn = = ç ÷=
3 3 è d ø 3d
points equidistant from the centre C , lying on the
upper and lower half, reach point O with zero path dyn l
Þ =
difference and hence reinforce each other producing D 3
maximum intensity at O. dyn l
Path difference = S2 P - SP 1 = =
(ii) Let l and a be the wavelength and slit width of D 3
diffracting system, respectively. Let O be the position 2p
\ Phase difference, f = ´ path difference
of central maximum. l
Condition for the first minimum is given by 2p l 2p
= ´ =
asin q = ml ... (i) l 3 3
Let q be the angle of diffraction. If intensity at central fringe is I0 , then intensity at a point
As, diffraction angle is small P, where phase difference f, is given by
\ sin q » q I = I0 cos 2 f
For first diffraction minimum, q = q1 (let) æ 2p ö
2
Þ I = I0 ç cos ÷
è 3ø
2
A æ pö
= I0 ç - cos ÷
q1 è 3ø
q B O 2
æ 1ö I
= I0 ç - ÷ = 0
è 2ø 4
C
I0
Hence, the intensity at point P would be .
4
For the first minimum, take m = 1 24. Given, the displacements of two coherent sources y1 = a cos
l
aq1 = l Þ q1 = w t and y2 = a cos( wt + f).
a
Now, angular width, AB = q1 By principle of superposition,
Angular width, BC = q1 y = y1 + y2 = a cos wt + a cos( wt + f)
y = a cos wt + a cos wt cos f - asin wt sin f
Angular width, AC = 2 q1
y = a( 1 + cos f) cos wt + ( - asin f)sin wt
Hence, the angular width of the first diffraction fringe
is half that of the central fringe. Let a( 1 + cos f) = A cos q …(i)
s l and asin f = A sin q …(ii)
23. (i) (a) For interference fringes to be seen, £ condition \ y = A cos q cos wt - A sin qsin wt
S d
should be satisfied, where s = size of the source and Þ y = A cos( wt + q) …(iii)
d = distance of the source from the plane of two slits. Squaring and adding Eqs. (i) and (ii), we get
As, the source slit width increases, fringe pattern gets
( A cos q) 2 + ( A sin q) 2 = a2 ( 1 + cos f) 2 + ( asin f)2
less and less sharp. When the source slit is too wide,
the above condition does not get satisfied and the A 2 (cos 2 q + sin 2 q)
interference pattern disappears. = a ( 1 + cos f + 2 cos f) + a2 sin 2 f
2 2

(b) The interference pattern due to the different colour


components of white light overlap. The central bright Þ A 2 ´ 1 = a2 + a2 + 2a2 cos f = 2a2 ( 1 + cos f)
fringes for different colours are at the same position. æ fö æ fö
Therefore, central fringe is white. And on the either Þ A 2 = 2a2 ç 2 cos 2 ÷ = 4a2 cos 2 ç ÷
è 2ø è 2ø
side of the central fringe (i.e. central maxima),
coloured bands will appear. The fringe close to either f
If I is the resultant intensity, then I = 4a2 cos 2 .
side of central white fringe is red and the farthest will 2
be blue.
CBSE Term II Physics XII 65

f For destructive interference


From constructive interference, cos = ± 1
2 l
f Path difference = ( 2n - 1 ) , where n =1, 2, 3,...
Þ = np Þ f = 2np 2
2 yn¢ d l
For destructive interference, Þ = ( 2n - 1 ) [from Eq. (i)]
D 2
f ( 2n - 1 ) D l
cos = 0 Þ yn¢ =
2 2d
f p where, yn¢ is the separation of nth order dark fringe from
Þ = ( 2n + 1 )
2 2 central fringe.
f = ( 2n + 1 ) p Dl
\ yn¢ + 1 = ( 2n + 1 )
25. (i) Let two coherent sources of light, S1 and S2 (narrow 2d
slits) are derived from a source S. The two slits S1 and S2 \ Fringe width of bright fringe = Separation between
are equidistant from source, S. ( n + 1 )th and nth order dark fringe from centred fringe,
Now, suppose S1 and S2 are separated by distance d. The Þ b = yn¢ + 1 - yn¢
slits and screen are distance D apart. ( 2n + 1 ) Dl ( 2n - 1 ) Dl
or b= -
2d 2d
P Dl
= ( 2n + 1 - 2n + 1 )
2d
Dl
S1 yn =
A 2d
q q
d/2 Fringe width of bright fringe,
S O d Dl
C M b= …(iii)
d/2 d
B From Eqs. (ii) and (iii), we can see that,
S2
D
Screen Fringe width of dark fringe = Fringe width of bright
fringe
Dl
b=
d
Considering any arbitrary point P on the screen at a
distance yn from the centre O. The path difference (ii) Given, l1 = 800 nm, l 2 = 600 nm
between interfering waves is given by S2 P - S1 P. D = 1.4 m and d = 0.28 mm = 2.8 ´ 10-4 m
i.e. Path difference = S2 P - S1 P = S2 M Let nth order bright fringe of l = 800 nm coincide with
S2 P - S1 P = d sin q ( n + 1 )th order 600 nm wavelength.
where, S1 M ^ S2 P Dn l1 D( n + 1 ) l 2
\ =
[Q ÐS2 S1 M = ÐOCP (by geometry) d d
Þ S1 P = PM Þ S2 P = S2 M ] Þ nl1 = ( n + 1 ) l 2
If q is small, then sin q » q » tan q n ´ 800 ´ 10- 9 = ( n + 1 ) ´ 600 ´ 10- 9
\ Path difference, n+1 4
S2 P - S1 P = S2 M = dsin q » d tan q =
n 3
æy ö 1 4 1
Path difference = d ç n ÷ …(i) Þ = -1 =
è Dø n 3 3
OP yn
(Q in DPCO, tan q = = ) n=3
CO D
Dn l 1
For constructive interference \ Least distance from central fringe, yn =
Path difference = nl, where n = 0, 1, 2, ... [from Eq. (i)] d
1.4 ´ 3 ´ 800 ´ 10- 9 -3
Dnl yn = = 12 ´ 10 m
Þ yn = 2.8 ´ 10- 4
d
D( n + 1 ) l yn = 12 mm
Þ yn+ 1 = 26. Consider the disturbances at the receiver R1, which is at a
d
Q Fringe width of dark fringe = yn + 1 - yn distance d from B.
(Q dark fringe exist between two bright fringes) Let the wave at R1 because of A be yA = a cos wt. The path
Dl Dnl difference of the signal from A with that from B is l / 2 and
b= (n + 1) - hence the phase difference is p.
d d
Dl Dl Thus, the wave at R1 because of B is
= ( n + 1 - n) =
d d yB = a cos ( wt - p) = - a cos wt
Dl The path difference of the signal from C with that from A is l
Fringe width of dark fringe, b = …(ii)
d and hence the phase difference is 2p.
66 CBSE Term II Physics XII

Thus, the wave at R1 because of C is YC = a cos ( w t - 2p) \ < I > = 2a12


= a cos wt Thus, R1 picks up the larger signal.
R2
(ii) If B is turned off,
R1 picks up y = a cos wt
1
\ IR1 = a2
2
R 2 picks up y = a cos wt
l/2 l/2 a2
R1 A B C
\ I R2 = a2 < cos 2 wt > =
2
l/2 Thus, R1 and R 2 pick up the same signal.
(iii) If D is turned off,
R1 picks up y = a cos wt
D 1
\ IR1 = a2
The path difference between the signal from D with that of A 2
is R 2 picks up y = 3 a cos wt
æ lö
2
æ l ö
1/ 2
l 9a 2
d + ç ÷ - ( d - l / 2) = d çç1 +
2 ÷ -d+ \ I R2 = 9 a2 < cos 2 wt > =
è ø
2 è 4 d 2 ÷ø 2 2
1/ 2
Thus, R 2 picks up larger signal compared to R1.
æ l2 ö l l (iv) Thus, a signal at R1 indicates B has been turned off and an
= d çç1 + ÷ -d+ »

(Q d >> l)
è 8 d ø 2 2 enhanced signal at R 2 indicates D has been turned off.
Therefore, phase difference is p. 27. (i) The pattern will remain the same, but it gets shifted
slightly.
\ yD = a cos ( wt - p) = - a cos wt Dl
Thus, the signal picked up at R1 from all the four sources is (ii) Fringe width, b =
d
yR1 = yA + yB + yC + yD d 2Dl
When d ¢= , then b¢ = = = 2b
= a cos wt - a cos wt + a cos wt - a cos wt = 0 2 d
(i) Let the signal picked up at R 2 from B be yB = a1 cos wt. So, fringe width is doubled.
The path difference between signal at D and that at B is (iii) The intensity of light due to slit is directly proportional to
l / 2. width of slit.
\ yD = - a1 cos wt I1 w1 4
\ = =
I2 w2 1
The path difference between signal at A and that at B is
1/ 2 a12 4
æ lö
2
æ l2 ö 1 l2 Þ =
( d ) + ç ÷ - d = d çç1 +
2 ÷ - d~
- a2 2 1
è 2ø è 4 d 2 ÷ø 8 d2
a1 2
Þ =
As d >> l, therefore this path difference ® 0 a2 1
2p æ 1 l2 ö Þ a1 = 2 a2
phase difference = ç ÷
and
l ç 8 d2 ÷ ® 0
è ø Imax ( a1 + a2 ) 2
=
Hence, yA = a1 cos ( wt - f) Imin ( a1 - a2 ) 2

Similarly, yC = a1 cos ( wt - f) ( 2a2 + a2 ) 2 9a2 2


= = 2 =9 : 1
\Signal picked up by R 2 is ( 2a2 - a2 ) 2 a2
yA + yB + yC + yD = y = 2a1 cos ( wt - f) Hence, the ratio of the intensities of maxima and minima
\ y
2
= 4a12 2
cos ( wt - f) is 9 : 1.
CBSE Term II Physics XII 67

CHAPTER 04

Dual Nature of
Radiation and
Matter
In this Chapter...
l Particle Nature of Light : The Photon
l Electron Emission
l Photocell
l Photoelectric Effect
l Wave Nature of Particles :
l Einstein's Photoelectric Equation de-Broglie Hypothesis

Electron Emission Work Function


The electrons in the outer shell (valence electrons), which are For electron emission to take place, a certain minimum
free to move easily within the metal surface but cannot leave amount of energy is required for an electron to pull it out
it, are called free electrons. from the surface of the metal. This minimum energy required
by an electron to escape from the metal surface is called the
These electrons can be emitted from the metals, if they have work function of the metal. It is generally denoted by f0 or
sufficient energy to overcome the attractive pull of metal W0 and measured in eV (electron volt).
surface. So, the phenomenon of emission of electrons from
the surface of a metal is called electron emission. One electron volt (1 eV) is the energy gained by an electron,
when it has been accelerated by a potential difference of one
The minimum energy required for the electron emission from
volt (1 V), so that 1 eV = 1.602 ´ 10 -19 J. It depends on the
the metal surface can be supplied to the free electrons by any
of the following physical processes properties of the metal and the nature of its surface. It
decreases with the increase in temperature.
(i) Thermionic Emission Sufficient thermal energy can
be imparted to the free electrons by suitable heating, so
that they can come out of the metal. Photoelectric Effect
(ii) Field Emission or Cold Cathode Emission It is the It is the phenomenon of emission of electrons from the
phenomenon of emission of electrons from the surface of surface of metal, when radiations of suitable frequency fall on
a metal by applying a very strong electric field it. The emitted electrons are called photoelectrons and the
current, so produced is called photoelectric current.
( ~ 10 8 Vm -1 ) to a metal. One of the examples of cold
Note Alkali metals like lithium, sodium, etc., show photoelectric effect
emission is spark plug. with visible light, whereas the metals like zinc, cadmium, etc.,
(iii) Photoelectric Emission It is the phenomenon of are sensitive only to ultraviolet light.
emission of electrons from the surface of metal, when Hertz’s Observations
light radiations of suitable frequency fall on it. Here,
He observed that high voltage sparks across the detector loop
the energy to the free electrons for their emission is
were enhanced, when the emitter plate was illuminated by
being supplied by light photons. ultraviolet light from an arc lamp.
68 CBSE Term II Physics XII

Hallwachs’ and Lenard’s Observations As, the photoelectric current is directly proportional to the
number of photoelectrons emitted per second. So, the
Lenard observed that, if a potential difference is applied number of photoelectrons emitted per second is directly
across the two metal plates enclosed in an evacuated tube, proportional to the intensity of the incident radiation.
then there is no flow of current in the circuit.
However, when one plate (called emitter plate) enclosed in Effect of Potential on Photoelectric Current
the evacuated tube, kept at negative potential is exposed with Variation of photoelectric current with potential for different
ultraviolet radiations, current begins to flow in the circuit. intensities but constant frequency is as shown in the graph
As soon as ultraviolet radiations falling on the emitter plate below
are stopped, the current flowing is also stopped.

Photoelectric
current
Hallwachs and Lenard also observed that when ultraviolet
I 3 > I2 > I1
light fall on the emitter plate, no electrons were emitted, I3
until the frequency of the incident light was smaller than a I2
certain minimum value, called the threshold frequency. I1
This minimum frequency depends on the nature of the Stopping
material of the emitter plate. potential

O Collector plate
Experimental Study of –V0
Retarding potential potential
Photoelectric Effect
From the above graph, we can observe that
On the basis of the experimental arrangement used for
(i) After a certain value of accelerating potential, when all
studying photoelectric effect, the variations of photocurrent
photoelectrons reach the plate A and the photocurrent
with intensity of radiation, frequency of radiation and the
ceases. On increasing the value of accelerating
potential difference between the plates A and C are as follows
potential, this maximum value of photoelectric current
Quartz Source is called saturation current.
window Evacuated
Photosensitive glass tube
(ii) When the potential is decreased, the current decreases
plate but does not become zero at zero potential. This shows
Electrons
that even in the absence of accelerating potential, few
C A photoelectrons manages to reach the collector plate on
Reversing key
their own due to their kinetic energy.
mA
(iii) For a particular frequency of incident radiation, when
minimum negative potential V0 is applied to the
collector plate w.r.t. emitter plate, photoelectric current
becomes zero at a particular value of negative potential
V V0 called stopping potential or cut-off potential.
In this condition, the stopping potential is sufficient to repel
even the most energetic photoelectron with maximum kinetic
K B energy K max , which is given as
Experimental arrangement for the study of 1 2
photoelectric effect K max = eV0 = mv max
2
Effect of Intensity of Light on where, m is the mass of photoelectron and v max is the
maximum velocity of emitted photoelectron.
Photoelectric Current
For the radiation of a given frequency and material of emitter
For a fixed frequency of incident radiation and accelerating
plate, the value of stopping potential V0 is independent of
potential, the photoelectric current increases linearly with
increase in intensity of incident light which is as shown the intensity of the incident radiation. It means that, the
below maximum kinetic energy of emitted photoelectron depends
on the light source and the emitter plate material but is
independent of intensity of incident radiation.
Photoelectric

Effect of Frequency of Incident Radiation on


current

Stopping Potential
Variation of photoelectric current with potential for different
frequencies but constant intensity of incident radiation is as
O Intensity of light
shown in the graph below
CBSE Term II Physics XII 69

photoelectrons ejected per second is directly


proportional to the intensity of the incident light.
Photoelectric current
(ii) For a given material and frequency of incident
n3 > n2 > n1
radiation, saturation current is found to be proportional
Saturation to the intensity of incident radiation, whereas the
n3 current stopping potential is independent of its intensity.
n2 (iii) For a given material, there exists a certain minimum
n1
frequency of the incident radiation below which no
–V03 –V02 –V01 0 Collector plate
emission of photoelectrons takes place. This frequency
potential is called threshold frequency.
Retarding potential
Above the threshold frequency, the maximum kinetic
From the above graph, we observe that energy of the emitted photoelectrons or equivalent
(i) The value of stopping potential is different for radiations stopping potential is independent of the intensity of the
of different frequencies but the value of saturation incident light but depends upon only the frequency (or
current (for a given intensity) remains constant. wavelength) of the incident light.
(ii) The value of stopping potential is more negative for (iv) The photoelectric emission is an instantaneous process.
incident radiation of higher frequency. This means The time lag between the incidence of radiations and
that, the energy of the emitted electrons depends on emission of photoelectrons is very small, less than even
the frequency of incident radiations. 10 -9 s.
So, greater the frequency of incident radiation, greater
is the maximum kinetic energy of photoelectrons. Photoelectric Effect and Wave
Consequently, greater retarding potential or stopping Theory of Light
potential is required to stop them completely. Huygens’ wave theory of light could not explain the
(iii) The value of saturation current depends upon the photoelectric emission due to the following main reasons
intensity of incident radiation but is independent of the (i) According to the wave nature of light, free electrons at
frequency of the incident radiation. the surface of the metal absorb the radiant energy
The graph between stopping potential and the frequency of continuously.
the incident radiation for two different metals A and B, is as The greater the intensity of radiation, the greater
shown below should be the energy absorbed by each electron. The
Metal A maximum kinetic energy of the photoelectrons on the
Stopping Metal B surface is then expected to increase with increase in
potential
n > n0
intensity.
(V0) n > n0N
0 n0 n0¢ But according to experimental facts, the maximum
kinetic energy of ejected photoelectrons is
Frequency of incident
radiation (n)
independent of intensity of incident radiation.
(ii) According to wave theory of light, no matter what the
From the graph, we observe that
frequency of radiation is, a sufficiently intense beam of
(i) The stopping potential V0 varies linearly with the radiation should be able to impart enough energy to
frequency of incident radiation for a given the electrons, so that they exceed the minimum energy
photosensitive material.
needed to escape from metal surface.
(ii) There exists a certain minimum cut-off frequency n 0 for
A threshold frequency, therefore should not exist
which the stopping potential is zero. This frequency is
which contradicts the experimental fact that, no
called threshold or cut-off frequency. For a frequency
lower than cut-off frequency, no photoelectric emission photoelectric emission takes place below that
is possible even, if the intensity is large. threshold frequency, no matter whatsoever may be its
intensity.
If frequency of incident radiation is more than
the threshold frequency, the photoelectric emission (iii) According to the wave theory of light, the absorption of
starts instantaneously without any apparent time lag energy by electron takes place continuously over the
( -10 -9 s or less) even, when the incident radiation is entire wavefront of the radiation. Since, a large number
very dim. of electrons absorb energy, the energy absorbed per
electron per unit time turns out to be small.
Laws of Photoelectric Emission Hence, it will take hours or more for a single electron
The laws of photoelectric emission are as follows to come out of the metal which contradicts the
(i) For a given material and a given frequency of incident experimental fact that photoelectron emission is
radiation, the photoelectric current or number of instantaneous.
70 CBSE Term II Physics XII

(ii) Frequency n and maximum kinetic energy K max


Einstein’s Photoelectric Equation
graph
According to Planck’s quantum theory, the energy of an
electromagnetic wave is not continuously distributed over As, K max = hn - f0
the entire wavefront of waves. Þ K max µ n
Instead of this, these waves travel in the form of discrete Kmax
packets or bundels of energy called quanta of energy of
radiation. Each quantum of energy radiate an energy, which

Maximum KE
is given by
q tan q = h
E = hn n0 n
where, h is Planck’s constant and n is the frequency of light Frequency
radiation. – f0
On the basis of this, Albert Einstein explained photoelectric
effect and the following photoelectric equation (iii) Frequency n and photoelectric current I graph
1 2 The graph given below shows that, the photoelectric
K max = mv max = hn - f0 current I is independent of frequency of the incident
2
light, till intensity remains constant.
where, f0 is work function and K max is the maximum kinetic I
energy of emitted electrons.

Photoelectric
Also, K max = eV0

current
Þ eV0 = h (n - n 0 )
In terms of threshold wavelength, photoelectric equation is
given as n0 Frequency n
æ1 1 ö (iv) Intensity and stopping potential V0 graph
eV0 = hc çç - ÷÷
è l l0 ø V0
l = wavelength of the incident radiation,
Stopping

where,
potential

l 0 = threshold wavelength of the metal surface


and c = velocity of light.
Intensity
Graphs Related to Photoelectric Effect (v) Photoelectric current I and time lag t graph
From Einstein Photoelectric Equation I
Photoelectric

The important graphs related to photoelectric effect are as


current

follows
(i) Frequency n and stopping potential V0 graph
t
We know that, eV0 = hn - f0 0 10 –9 s Time
hn f0
Þ V0 = -
e e Particle Nature of Light : The Photon
So, V0 µ n Photoelectric effect gave evidence that light consists of
packets of energy. These packets of energy were called light
V0 quantum that are associated with particles named as
Stopping potential

photons. So, photons confirm the particle nature of light.

q tan q =
h
= Slope Characteristic Properties of Photons
e
0 n0 n Different characteristic properties of photons which are as
Frequency
given below
– f0 (i) In interaction of radiation with matter, radiation
e
behaves as if it is made up of particles called photons.
It could be seen that, V0 versus n curve is a straight line (ii) A photon travels at a speed of light c in vacuum,
with slope = h/e and is independent of the nature of i. e. 3 ´ 10 8 m / s.
material. (iii) It has zero rest mass, i.e. the photon cannot exist at rest.
CBSE Term II Physics XII 71

(iv) The inertial mass of a photon is given by Applications of Photocell


E h hv
m= 2 = = (i) Used in television camera for telecasting scenes and in
c cl c 2 photo telegraphy.
(v) Photons travel in a straight line. (ii) Reproduction of sound in cinema film.
(vi) Irrespective of the intensity of radiation, all the photons (iii) Used in counting devices.
of a particular frequency n or wavelength l have the same (iv) Used in burglar alarm and fire alarm.
æ hc ö æ hn h ö
energy E ç = hn = ÷ and momentum, p ç = = ÷. (v) To measure the temperature of stars.
è lø è c lø (vi) Used for the determination of Planck’s constant.
(vii) Photons are not deflected by electric and magnetic
fields. This shows that, photons are electrically Wave Nature of Particles :
neutral.
(viii) In a photon-particle collision (such as photoelectron
de-Broglie Hypothesis
collision), the energy and momentum are conserved. Louis victor de-Broglie, put forward a hypothesis that, moving
However, the number of photons may not be material particles of matter should display wave like
conserved in a collision. properties under suitable condition. Thus, he proposed that
the wavelength l associated with moving particle is given as
(ix) Photons may show diffraction under given conditions.
h h
l= =
mv p
Photocell
It is a device which converts light energy into electrical where, m, v and p are the mass, velocity and momentum of the
energy. It is also called an electric eye. As, the photoelectric particle respectively and h is Planck’s constant.
current sets up in the photoelectric cell corresponding to Relation between de-Broglie Wavelength (l)
incident light, it provides the information about the objects and Temperature (T )
as seen by our eye in the presence of light.
For a thermal neutron at temperature T, de-Broglie
Incident wavelength is given as
light
h h
C l= =
p 3 mKT
Collector (Anode)
A
de-Broglie Wavelength of Charged Particles
Evacuated The wavelength associated with charged particles accelerated
Emitter glass bulb through a potential is given as
(Cathode) h h
l= =
p 2mqV
– B +
mA where, q is the charge on the particle.
1 . 227
Specifically, for electrons, l = nm.
Photocell V
72 CBSE Term II Physics XII

Solved Examples
Example 1. The photoelectric cut-off voltage in a \ Threshold frequency, n 0
certain experiment is 1.5 V. What is the maximum . ´ 10-31 ´ ( 6.0 ´ 105 ) 2
91
= 7.21 ´ 1014 -
kinetic energy of photoelectrons emitted? 2 ´ 6.62 ´ 10-34
Sol. Maximum kinetic energy of photoelectrons, K max = eV0 = 7.21 ´ 1014 - 2.47 ´ 1014
where, e = 1.6 ´10-19 C, V0 = 1.5 V = 4.74 ´ 1014 Hz
\ K max = 1 .6 ´ 10-19 ´ 1 .5 = 2.4 ´ 10-19 J Example 5. A student performs an experiment on
photoelectric effect, using two materials A and B. A
Example 2. In an experimental study of photoelectric plot of Vstop versus n is given in the figure.
effect, the stopping potential for a metallic surface
Vstop A B
is 5.68 V. What will be the maximum velocity of the
3
electrons?
2.5
Sol. Stopping potential, V0 = 5.68 V 2
. ´ 10-31 kg
m e = 91 1.5
1 1
2
Maximum KE, K max Þ mvmax = eV0
2
5 10 15
2eV0 2 ´ 1 .6 ´ 10-19 ´ 5 .68
Þ vmax = = (× 1014 Hz)
m . ´ 10-31
91 Frequency (Hz)
6 -1
= 1 .414 ´ 10 ms (i) Which material A or B has a higher work
Example 3. In an experiment on photoelectric effect, function?
the slope of the cut-off voltage versus frequency of (ii) Given the electric charge of an electron
incident light is found to be 4 .12 ´ 10 -15 V-s. = 1.6 ´ 10 -19 C, find the value of h obtained from
Calculate the value of Planck’s constant. the experiment for both A and B.
Sol. We know that, eV0 = hn - W Comment on whether it is consistent with the
æ hö W Einstein’s theory.
\ V0 = ç ÷ n -
è eø e Sol. (i) Given, threshold frequency of A is given by
h h n OA = 5 ´ 104 Hz
\ Slope = or 4.12 ´ 10-15 =
e e and for B, n OB = 10 ´ 1014 Hz
\ . ´ 10-15 ´ e
h = 412 We know that, work function,
. ´ 10-15 ´ 1 .6 ´ 10-19 Js-1
= 412 f = hn 0 or f0 µ n 0
= 6.592 ´ 10-34 Js -1 fOA 5 ´ 1014
So, = <1 Þ fOA < fOB
14 fOB 10 ´ 1014
Example 4. Light of frequency 7.21 ´ 10 Hz is
Thus, work function B is higher than A.
incident on a metal surface. Electrons with a h 2
maximum speed of 6.0 ´ 105 m/ s are ejected from (ii) For metal A, slope = =
e ( 10 - 5 ) 1014
the surface. What is the threshold frequency for 2 ´e 2 ´ 1 .6 ´ 10-19
photoemission of electrons? or h= 14
=
5 ´ 10 5 ´ 1014
Sol. Using Einstein’s photoelectric equation,
1 = 6.4 ´ 10-34 J-s
2
hn - hn 0 = mvmax h 2 .5
2 For metal B, slope = =
1 e ( 15 - 10) 1014
2
We get hn 0 = hn - mvmax . ´e
25
2 or h=
2
mvmax 5 ´ 1014
\ n0 = n - 2 .5 ´ 1 .6 ´ 10-19
2h = = 8 ´ 10-34 J-s
Given, n = 7.21 ´ 1014 Hz 5 ´ 1014
vmax = 6.0 ´ 105 m/s Since, the value of h from experiment for metals A and B is
different. Hence, experiment is not consistent with theory.
. ´ 10- 31 kg
m = mass of electron = 91
CBSE Term II Physics XII 73

Example 6. The momentum of photon of 6.63 ´ 10- 34 ´ 3 ´ 108 6.63 ´ 10- 34 ´ 3 ´ 108
= - 4.26 ´ 10-16
electromagnetic radiation is 3.3 ´ 10 -29 kg -ms -1 . l2 1.4 ´ 10- 9
Find out the frequency and wavelength of the wave 6.63 ´ 10- 34 ´ 3 ´ 108
= 14.21 ´ 10-16 - 4.26 ´ 10-16
associated with it. l2
Sol. (i) Given, h = 6.63 ´ 10-34 J/s, c = 3 ´ 108 ms -1 l 2 = 2 ´ 10-10 m
and p = 3.3 ´ 10-29 kg- ms -1 = 2.0 Å
hn
Momentum, p = Example 8. What is the
c
pc 3.3 ´ 10-29 ´ 3 ´ 108 (a) momentum,
Þ n= = = 1.5 ´ 1013 Hz (b) speed and
h 6.63 ´ 10-34
c 3 ´ 108 (c) de-Broglie wavelength of an electron with
(ii) l= = = 2 ´ 10-5 m kinetic energy of 120 eV?
n 1 .5 ´ 1013
p2
Example 7. The wavelength of a photon is 1.4 Å. It Sol. (a) Using the relation, E = , we get
2m
collides with an electron. The energy of the p = 2mE
scattered electron is 4.26 ´ 10 - 16 J. Find the
= . ´ 10-31 ´ 120 ´ 1 .6 ´ 10-19
2 ´ 91
wavelength of photon after collision.
(Take, h = 6.63 ´ 10 - 34 J-s) = 5.91 ´ 10-24 kg - ms -1
Sol. Initial wavelength of photon is l1 and final wavelength of p 5.91 ´ 10-24
(b) Speed, v = = = 6.5 ´ 106 ms -1
photon is l 2 . m 9.1 ´ 10-31
hc hc
\ 4.26 ´10-16 = - (c) de-Broglie wavelength is given by
l1 l 2
h 6.62 ´ 10-34
hc hc l= = = 1.12 ´ 10-10 m
= - 4.26 ´ 10-16 p 5 .91 ´ 10-24
l 2 l1
74 CBSE Term II Physics XII

Chapter
Practice
PART 1 4. In an experiment on photoelectric effect with the
increase in potential difference of emitter and
Objective Questions collector plate, keeping the frequency and intensity
fixed of the incident light, the photoelectric current
(a) increases
l
Multiple Choice Questions (b) decreases
(c) remains constant
1. Work-function f0 for a metal will change, if it is (d) increases initially and then becomes constant
(a) heated
(b) cooled
5. The cathode of a photoelectric cell is changed, such
(c) coated with some other metal that the work function changes from W1 to W2
(d) All of the above ( W2 > W1 ). If the current before and after are i1 and
i 2 , respectively and all other conditions remaining
2. Hallwach connected an uncharged zinc plate to an unchanged, then [Delhi 2020]
electroscope as shown below (a) i1 = i2 (b) i1 < i2
Uncharged W1 i
zinc plate (c) i1 > i2 (d) = 1
W2 i2

6. In photoelectric effect experiment, collector plate


is made negative with respect to emitter plate as
shown in figure below till it reach a certain
potential V 0 , when photocurrent is zero.
This plate is then irradiated by UV-light. The result
of this is correctly shown in Light
Collector
plate
+ P -
(a) (b)
Emitter plate
V0

If K indicates kinetic energy of an emitted


(c) (d) photoelectron, then at point P
(a) K > eV0 (b) K < eV0
(c) K = eV0 (d) 0 £ K £ eV0
7. Consider a beam of electrons (each electron with
3. In the phenomenon of electric discharge through energy E 0 ) incident on a metal surface kept in an
gases at low pressure, the coloured glow in the tube evacuated chamber, then [NCERT Exemplar]
appears as a result of (a) no electrons will be emitted as only photons can emit
(a) excitation of electrons in the atoms electrons
(b) collision between the atoms of the gas (b) electrons can be emitted but all with an energy, E0
(c) collision between the charged particles emitted from the (c) electrons can be emitted with any energy, with a
cathode and the atoms of the gas maximum of E0 - f (f is the work function)
(d) collision between different electrons of the atoms of the (d) electrons can be emitted with any energy, with a
gas maximum of E0
CBSE Term II Physics XII 75

8. The formula for kinetic mass of a moving photon is (a) Curves a and b represent incident radiations of different
frequencies and different intensities.
(where, h is Planck constant and n, l, c are
frequency, wavelength and speed of photon, (b) Curves a and b represent incident radiations of same
frequency but of different intensities.
respectively)
(c) Curves b and c represent incident radiations of different
(a) hn / l (b) hl / e (c) hn/ e (d) h / cl frequencies and different intensities.
(d) Curves b and c represent incident radiations of same
9. The wavelength of a photon needed to remove a frequency having same intensity.
proton from a nucleus which is bound to the nucleus
with 1 MeV energy is nearly [NCERT Exemplar] 13 The value of stopping potential V 0 from the given
(a) 1 . 2 nm (b) 1 . 2 ´ 10-3 nm graph is [All India 2020]
(c) 1 . 2 ´ 10-6 nm (d) 1 . 2 ´ 10 nm

Photoelectric
current
10. A point source of light is used in an experiment on
photoelectric effect. Which of the following curves
best represents the variation of photoelectric
current i with distance s of the source from the
emitter?
i
a – 0.54 V O 0.2 V Collector plate
Photoelectric

potential
b
current

c (a) - 0.54 V (b) 0.54 V


d (c) 0.2 V (d) - 0.2 V

Distance
s 14. A student plot a graph while performing an
experiment on photoelectric effect using an
(a) a (b) b
(c) c (d) d
evacuated glass tube, for light radiation of same
intensity at various frequencies as shown below
11. Variation of photoelectric current with collector
plate potential for different intensities I1 and I 2 Y
(such that I1 > I 2 ) at a fixed frequency is
Photoelectric
current
Photoelectric

I1 Z
n2 n1
current

I2
(a) (b) I1 I2 X
O

V0 Collector plate -V01 -V02 V0 Collector plate


potential potential Here,
(a) X ® Photoelectric current; Y ® Retarding potential; Z ®
Photoelectric
Photoelectric

current

I2 I1 Stopping potential
current

(c)
I1
(d) I2 (b) X ® Retarding plate potential; Y ® Photocurrent; Z ®
Stopping potential
(c) X ® Collector plate potential; Y ® Photocurrent; Z ®
-V0 O Collector plate -V0 O Collector plate
potential
Saturation current
potential
(d) X ® Retarding plate potential; Y ® Photocurrent; Z ®
12. The figure shows a plot of photocurrent versus Saturation current
anode potential for a photosensitive surface for
three different radiations. Which one of the
15. The photoelectric threshold wavelength for silver is
l 0 . The energy of the electron ejected from the
following is a correct statement?
Photocurrent
surface of silver by an incident wavelength
l ( l < l 0 ) will be [All India 2020]
b æ l - lö h æ l0 - l ö
(a) hc ç 0 ÷ (b) ç ÷
c a è ll 0 ø c è ll 0 ø
hc
(c) (d) hc ( l 0 - l )
Retarding potential Anode potential l0 - l
76 CBSE Term II Physics XII

16. If all the following particles are moving with the K max . When the ultraviolet light is replaced by
same velocity, then the particle of maximum X-rays, both V 0 and K max increase.
momentum will be Reason Photoelectrons are emitted with speeds
(a) b-particle (b) proton ranging from zero to a maximum value because of the
(c) a-particle (d) neutron range of frequencies present in the incident light.
17. An electron is moving with an initial velocity 23. Assertion The photocells inserted in the door
v = v 0 $i and is in a magnetic field B = B 0 $j. Then, its light electric circuit are used as automatic door
de-Broglie wavelength [NCERT Exemplar] opener.
(a) remains constant Reason Abrupt change in photocurrent, helps to
(b) increases with time
open the door.
(c) decreases with time
(d) increases and decreases periodically 24. Assertion A particle at rest breaks into two
18. The de-Broglie wavelength of a particle of kinetic particles of different masses. They fly off in
energy K is l. What will be the wavelength of the different directions and their de-Broglie
K wavelengths will be same.
particle, if its kinetic energy is ?
9 Reason Their speed will be same.
(a) l (b) 2l
(c) 3l (d) 4l
l
Case Based MCQs
19. A proton, a neutron, an electron and an a-particle Direction Read the following passage and answer the
have same energy. Then, their de-Broglie questions that follows
wavelengths compare as [NCERT Exemplar] 25. Photocell
(a) l p = l n > l e > l a (b) l a < l p = l n < l e
Photocell is a device which converts light energy
(c) l e < l p = l n > l a (d) l e = l p = l n = l a into electrical energy. It is also called an electric
eye. As, the photoelectric current sets up in the
l
Assertion-Reasoning MCQs photoelectric cell corresponding to incident light, it
Direction (Q. Nos. 20-24) Each of these questions provides the information about the objects as has
contains two statements Assertion (A) and Reason (R). been seen by our eye in the presence of light.
Each of these questions also has four alternative Incident
choices, any one of which is the correct answer. You light
have to select one of the codes (a), (b), (c) and (d) given C
below. Collector (Anode)
A
(a) Both A and R are true and R is the correct
explanation of A.
Evacuated
(b) Both A and R are true, but R is not the correct Emitter glass bulb
explanation of A. (Cathode)
(c) A is true, but R is false.
(d) A is false and R is also false. – B + mA
20. Assertion Heinrich Hertz observed that, high
voltage spark across detector loop were enhanced
A photocell consists of a semi-cylindrical
when the emitter plate was illuminated by
photosensitive metal plate C (emitter) and a wire
UV-light.
loop A (collector) supported in an evacuated glass
Reason Light shining on the metal surface or quartz bulb. When light of suitable wavelength
facilitate the escape of free electrons. falls on the emitter C, photoelectrons are emitted.
21. Assertion If distance of the point source is (i) It is observed that no electrons are emitted, when
increased from the photoelectric plate, then frequency of light is less than a certain minimum
stopping potential will remain unchanged. frequency.
Reason Saturation current will decrease. This minimum frequency depends on
(a) potential difference of emitter and collector plates
22. Assertion When ultraviolet light is incident on a (b) distance between collector and the emitter plate
photocell, its stopping potential is V 0 and the (c) size (area) of the emitter plate
maximum kinetic energy of the photoelectrons is (d) material of the emitter plate
CBSE Term II Physics XII 77

(ii) The work function of a metal used in photocell is 3. Two monochromatic radiations, blue and violet,
hc/ l 0 . If light of wavelength l is incident on its of the same intensity are incident on a
surface, then the essential condition for the electron photosensitive surface and cause photoelectric
to come out from the metal surface is emission. Would
(a) l ³ l 0 (b) l ³ 2l 0 (c) l £ l 0 (d) l £ l 0 / 2 (i) the number of electrons emitted per second and
(iii) Variation of photoelectric current with intensity of (ii) the maximum kinetic energy of the electrons be
light for a photocell is equal in the two cases? Justify your answer.
Photoelectric [Delhi 2010]
Photoelectric

4. Draw a graph to show the variation of stopping


current
current

(a) (b) potential with the frequency of radiation incident on


a metal plate. How can the value of Planck’s
constant be determined from this graph?
Intensity of light Intensity of light
5. If light of wavelength 412.5 nm is incident on each
Photoelectric

Photoelectric

of the metals given below, which ones will show


current

current

(c) (d) photoelectric emission and why? [CBSE 2018]

Metal Work Function (eV)


Intensity of light Intensity of light Na 1.92
K 2.15
(iv) A photon of energy 3.4 eV is incident on a metal
Ca 3.20
surface of a photocell whose work function is 2 eV.
Maximum kinetic energy of the photoelectron Mo 4.17
emitted by the metal surface will be
6. The work function of Cs is 2.14 eV. Find
(a) 1.4 eV
(i) threshold frequency for Cs and
(b) 1.7 eV
(c) 5.4 eV (ii) wavelength of incident light, if the photocurrent
(d) 6.8 eV is brought to zero by stopping potential of 0.6 V.
(v) Photocell is based on photoelectric effect that gave 7. The threshold frequency for a certain metal is
evidence that light in interaction with matter, 3.3 ´ 1014 Hz. If light of frequency 8.2 ´ 1014 Hz is
(a) is converted into particles of same size incident on the metal, predict the cut-off voltage
(b) is converted into particles of same energy for the photoelectric emission. [NCERT]
(c) is converted into mass following E = mc 2
8. Find the frequency of light which ejects electrons
(d) behaves as if it was made of packets of energy, each of from a metal surface, fully stopped by a retarding
energy hn
potential of 3.3 V. If photoelectric emission begins
in this metal at a frequency of 8 ´ 1014 Hz, calculate
PART 2 the work function (in eV) for this metal.
[All India 2018C]
Subjective Questions 9. Consider a metal exposed to light of wavelength
600 nm. The maximum energy of the electron
l
Short Answer (SA) Type Questions doubles, when light of wavelength 400 nm is used.
Find the work function in eV. [NCERT Exemplar]
1. There are materials which absorb photons of
shorter wavelength and emit photons of longer
10. Light of same wavelength is incident on three
photosensitive surfaces A, B and C. The following
wavelength. Can there be stable substances which
observations are recorded.
absorb photons of larger wavelength and emit light
of shorter wavelength? [NCERT Exemplar] (i) From surface A, photoelectrons are not emitted.
2. Two monochromatic beams A and B of equal (ii) From surface B, photoelectrons are just emitted.
intensity I, hit a screen. The number of photons (iii) From surface C, photoelectrons with some
hitting the screen by beam A is twice that by beam kinetic energy are emitted.
B, then what inference can you make about their Compare the threshold frequencies of the three
frequencies? [NCERT Exemplar] surfaces and justify your answer. [Delhi 2020]
78 CBSE Term II Physics XII

11. Figure shows the stopping potential V 0 for the 18. Show that the wavelength of electromagnetic
1 radiation is equal to the de-Broglie wavelength of
photoelectron versus graph, for two metals A and its quantum (photon). [NCERT]
l
B, l being the wavelength of incident light. 19. A proton and an a-particle are accelerated through
V0 the same potential. Which one of the two has
A B
(i) greater value of de-Broglie wavelength
associated with it and
O 1/l (ii) less kinetic energy?
Give reasons to justify your answer. [Delhi 2014]
20. The two lines marked A and B in the given figure,
(i) How is the value of Planck’s constant 1
determined from the graph? show a plot of de-Broglie wavelength l versus ,
V
(ii) If the distance between the light source and the where V is the accelerating potential for two nuclei
surface of metal A is increased, how will the 2 3
1 H and 1 H.
stopping potential for the electrons emitted from
it be effected? Justify your answer. [Delhi 2020] A

12. Explain with the help of Einstein’s photoelectric


l B
equation any two observed features in
photoelectric effect with cannot be explained by
wave theory. [Delhi 2019]
1
13. Why is wave theory of electromagnetic radiation ÖV
not able to explain photoelectric effect ? How does
(i) What does the slope of the lines represent?
photon picture resolve this problem? [Delhi 2019]
(ii) Identify, which of the lines corresponded to
14. In the wave picture of light, intensity of light is these nuclei. [All India 2010]
determined by the square of the amplitude of the
wave. What determines the intensity in the photon 21. Assuming an electron is confined to a 1 nm wide
picture of light? [All India 2016] region, find the uncertainty in momentum using
Heisenberg uncertainty principle (Dx ´ Dp » h). You
15. If the frequency of light incident on the cathode of can assume, the uncertainty in position Dx as 1 nm.
a photocell is increased, how will the following be
Assuming p » Dp, find the energy of the electron
affected? Justify your answer.
in eV. [NCERT Exemplar]
(i) Energy of the photoelectrons
(ii) Photocurrent [Delhi 2020]
22. A particle is moving three times as fast as an
electron. The ratio of the de-Broglie wavelength of
16. A 100 W sodium lamp radiates energy uniformly in the particle to that of the electron is 1.813 ´ 10 -4 .
all directions. The lamp is located at the centre of a Calculate the particle’s mass and identify the
large sphere that absorbs all the sodium light which particle. [All India 2011]
is incident on it.
The wavelength of the sodium light is 589 nm. 23. (i) For what kinetic energy of a neutron will associated
de-Broglie wavelength be 1.40 ´10 -10 m?
(i) What is the energy per photon associated with
the sodium light? (ii) Also, find the de-Broglie wavelength of a neutron,
(ii) At what rate are the photons delivered to the in thermal equilibrium with matter, having an
sphere? [NCERT] average kinetic energy of (3/2) kT and
17. The work functions for the following metals are temperature is 300 K. [NCERT]
given, Na = 2.75 eV, K = 2.30 eV, Mo = 4.17 eV, 24. Electrons are emitted from the cathode of a
Ni = 5.15 eV. Which of these metals will not give photocell of negligible work function, when
photoelectric emission for a radiation of photons of wavelength l are incident on it. Derive
wavelength 3300 Å from a He-Cd laser placed 1 m an expression for the de-Broglie wavelength of the
away from the photocell? What happens, if the electrons emitted in terms of the wavelength of the
laser is brought nearer and placed 50 cm away? incident light. [All India 2017 C]
[NCERT]
CBSE Term II Physics XII 79

l
Long Answer (LA) Type Questions (i) Which one of the two metals has higher
threshold frequency?
25. Sketch the graphs showing variation of stopping (ii) Determine the work function of the metal which
potential with frequencies of incident radiations for has greater value.
two photosensitive materials A and B having
threshold frequencies n A > n B . (iii) Find the maximum kinetic energy of electron
emitted by light of frequency 8 ´ 1014 Hz for this
(i) In which case is the stopping potential more and
metal. [Delhi 2017]
why?
(ii) Does the slope of the graph depend on the nature
31. (i) Describe briefly three experimentally observed
features in the phenomenon of photoelectric
of the material used? Explain. [All India 2016]
effect.
26. Define the term cut-off frequency in photoelectric (ii) Discuss briefly how wave theory of light cannot
emission. The threshold frequency of a metal is n. explain these features. [Delhi 2015, 16]
When the light of frequency 2n is incident on the
metal plate, the maximum velocity of photoelectron 32. What is the de-Broglie wavelength of
is v1 . When the frequency of the incident radiation (i) a bullet of mass 0.040 kg travelling at the speed
is increased to 5n, the maximum velocity of of 1.0 km/s,
photoelectrons is v 2 . Find the ratio v1 : v 2 . (ii) a ball of mass 0.060 kg moving at a speed of
[Foreign 2016]
1.0 m/s and
27. (i) State two important features of Einstein's (iii) a dust particle of mass 1.0 ´ 10 –9 kg drifting with
photoelectric equation. a speed of 2.2 m/s? [NCERT]
(ii) Radiation of frequency 1015 Hz is incident on
33. A mercury lamp is a convenient source for studying
two photosensitive surfaces P and Q. There is no
frequency dependence of photoelectric emission,
photoemission from surface P. Photoemission
since it gives a number of spectral lines ranging
occurs from surface Q but photoelectrons have
from the UV to the red end of the visible spectrum.
zero kinetic energy. Explain these observations
In our experiment with rubidium photocell, the
and find the value of work function for surface Q.
following lines from a mercury source were used
[Delhi 2017]
l1 = 3650 Å, l 2 = 4047 Å, l 3 = 4358 Å,
28. (i) Write the important properties of photons which
are used to establish Einstein’s photoelectric l 4 = 5461 Å , l5 = 6907 Å
equation. The stopping voltages respectively were measured
(ii) Use this equation to explain the concept of to be
(a) threshold frequency and V 01 = 1.28 V, V 02 = 0.95 V,
(b) stopping potential. [Delhi 2015] V 03 = 0.74 V, V 04 = 0.16 V, V 05 = 0
Determine the value of Planck’s constant h, the
29. Write Einstein’s photoelectric equation. The
threshold frequency and work function for the
maximum kinetic energy of the photoelectrons gets
material. [NCERT]
doubled when the wavelength of light incident on
the surface changes from l1 to l 2 . Derive an 34. Light of intensity 10 -5 Wm -2 falls on a sodium
expressions for the threshold wavelength l 0 and photocell of surface area 2 cm 2 . Assuming that, the
work function for the metal surface. [All India 2015] top 5 layers of sodium absorb the incident energy,
30. In the study of a photoelectric effect, the graph estimate the time required for photoelectric
between the stopping potential V and frequency n emission in the wave picture of radiation. The work
of the incident radiation on two different metals P function of the metal is given to be about 2 eV.
and Q is shown below What is the implication of your answer?
P Effective atomic area = 10 -20 m 2 . [NCERT]
4 Q 35. An electron, a-particle and a proton have the same
2 de-Broglie wavelengths. Which of these particle
V has
(volt) 0
4 6
–2 (i) minimum kinetic energy?
n (× 1014) Hz
(ii) maximum kinetic energy and why?
80 CBSE Term II Physics XII

In what way has the wave nature of electron beam When the emitter plate was illuminated by
exploited in electron microscope? ultraviolet light from an arc lamp. According to this
effect, there is emission of electrons from the
l
Case Based Questions surface of metal when a light beam of suitable
Direction Read the following passage and answer the frequency is incident on it.
questions that follows (i) Light of wavelength 2500 Å falls on a metal
surface of work function 3.5 eV. What is the
36. Observation of Photoelectric Effect
kinetic energy (in eV) of
The phenomenon of photoelectric emission was (a) the fastest and
discovered in 1887 by Heinrich Hertz during his
electromagnetic waves by means of spark across the (b) the slowest electron emitted from the surface?
detector loop were enhanced. If the same light falls on another surface of work
Light
function 5.5 eV, what will be the energy of
Metal emitted electrons?
Detector surface
(ii) An electron is accelerated through a potential
difference of 64 V. What is the de-Broglie
Electrons
wavelength associated with it? To which part of
+ – the electromagnetic spectrum this wavelength
Ammeter
Vacuum chamber
correspond? [Delhi 2010]

+ –
Battery
Chapter Test
Multiple Choice Questions 8. Ultraviolet light of wavelength 200 nm is incident on
polished surface of iron. Work function of the surface is
1. If photons of frequency n are incident on the surfaces 4.71 eV. Calculate its stopping potential. (Ans. 1.50 V)
n n
of metals A and B of threshold frequencies and ,
2 3 9. Light of wavelength 488 nm is produced by an argon
respectively, the ratio of the maximum kinetic energy laser, which is used in the photoelectric effect. When
of electrons emitted from A to that from B is light from this spectral line is incident on the emitter,
(a) 2 : 3 (b) 3 : 4 (c) 1 : 3 (d) 3 : 2 the stopping (cut-off) potential of photoelectrons is
2. The kinetic energy of a proton and that of an 0.38 V. Find the work function of the material from
a-particle are 4 eV and 1 eV, respectively. The ratio of which the emitter is made. (Ans. 2.17 eV)
the de-Broglie wavelengths associated with them, will 10. Find the
be (i) maximum frequency and
(a) 2 : 1 (b) 1 : 1 (ii) minimum wavelength of X-rays produced by 30 kV
(c) 1 : 2 (d) 4 : 1 electrons. [Ans. (i) 7.24 ´ 10 18 Hz and (ii) 0.0414 nm]
3. A photocell connected in an electrical circuit is placed 11. The energy flux of sunlight reaching the surface of the
at a distance d from a source of light. As a result, earth is 1.388 × 10 3 W/m 2 . How many photons (nearly)
current I flows in the circuit. What will be the current per square metre are incident on the earth per second?
in the circuit when the distance is reduced to d/2? Assume that the photons in the sunlight have an
(a) I (b) 2I average wavelength of 550 nm. (Ans. 3.838 ´ 10 2 )
(c) 4I (d) I / 2
Long Answer Type Questions
4. The de-Broglie wavelength of a neutron at 27°C is l.
12. What is the de-Broglie wavelength of a nitrogen
What will be its wavelength at 927°C?
l l molecule in air at 300 K? Assume that, the molecule is
(a) (b) moving with the root-mean-square speed of molecules
2 3
l l at this temperature.
(c) (d) (Take, atomic mass of nitrogen = 14.0076 u)
4 9
(Ans. 0.028 nm)
5. Which one of the following statements regarding
photoemission of electrons is correct? 13. The work function of caesium metal is 2.14 eV. When
light of frequency 6 ×10 14 Hz is incident on the metal
(a) Kinetic energy of electrons increases with the intensity of
surface, photoemission of electrons occurs. What is the
incident light.
(i) maximum kinetic energy of the emitted electrons,
(b) Electrons are emitted when the wavelength of the
(ii) stopping potential and
incident light is above a certain threshold wavelength.
(iii) maximum speed of the emitted photoelectrons?
(c) Photoelectric emission is instantaneous with the incidence
[Ans. (i) 0.35 eV, (ii) 0.35 V and (iii) 350.7 km/s]
of light.
(d) Photoelectrons are emitted whenever a gas is irradiated
14. Monochromatic light of wavelength 632.8 nm is
produced by a helium-neon laser. The power emitted is
with ultraviolet light.
9.42 mW.
Short Answer Type Questions (i) Find the energy and momentum of each photon in the light
6. What are the energies of photons at the (i) violet and beam.
(ii) red ends of the visible spectrum? The wavelength (ii) How many photons per second, on the average, arrive at a
of light is about 390 nm for violet and about 760 nm target irradiated by this beam? (Assume the beam to have
for red. [Ans. (i) 3.17 eV and (ii) 1.63 eV] uniform cross-section, which is less than the target area.)
7. The de-Broglie wavelength of a particle of kinetic (iii) How fast does a hydrogen atom have to travel in order to
have the same momentum as that of the photon?
energy K is l. What would be the wavelength of the
K [Ans. (i) 3.14 ´ 10 - 19 J, 1. 05 ´ 10 -27 kg-m/s
particle, if its kinetic energy were
?
4 (Ans. 1 : 2) (ii) 3 ´ 10 16 photons/s, (iii) 0.63 m/s]

Answers
Multiple Choice Questions
1. (b) 2. (b) 3. (c) 4. (a) 5. (c)
For Detailed Solutions
Scan the code
82 CBSE Term II Physics XII

EXPLANATIONS
PART 1 \ mc 2 = hn

1. (d) The work function f 0 depends on the properties of the Þ m = hn/ c 2


metal and the nature of its surface. ( hc / l ) h
Kinetic mass, m = =
So, its value will vary with the presence of surface c2 cl
impurities and with increase/decrease in temperature of the 9. (b) Given,
surface. Energy of a photon, E = 1 MeV = 106 eV
Thus, the value of f 0 for a metal will change, if it is cooled Now, hc = 1240 eV-nm
or heated or coated with some other metal. hc
2. (a) Hallwach observed that when the uncharged zinc plate is Now, E=
l
irradiated by UV-rays, then it becomes hc 1240 eV- nm
positively charged. Positive charge on this plate is further Þ l= =
enhanced when it was continuously illuminated by E 106 eV
UV-light.This will make the leaves to move apart from each = 1 . 2 ´ 10-3 nm
other, as same type of charge flows through the leaves of an 10. (d) As the distance of source from the surface increases,
electroscope, making it repel each other. intensity of radiation decreases.
Thus, it can be concluded that, negatively charged particles 1
are emitted from zinc plate under the action of UV-light, Q Intensity I µ
(distance d) 2
making the leaves to move apart from each other.
Hence, option (a) is correct. Since, photoelectric current (i) µ intensity (I)
3. (c) In the phenomenon of electric discharge through gases at Thus, the variation of i versus s is correctly depicted by the
low pressure, as the charged particles emitted from the curve d in the given figure.
cathode collides with the atoms of the gas, coloured glow 11. (d) Here for greater intensity I1, more photoelectrons are
appears in the tube. emitted and hence saturation current is more. Thus, graph
4. (d) Keeping the frequency n of the light and its intensity I corresponding to I1 will be above than that of I2 . Since, the
fixed, firstly with the increase in the positive potential of stopping potential is independent of intensity, hence the
collector plate, the photoelectric current increases. At some graphs converge at same value of stopping potential V0 for
stage, for a certain positive potential of collector plate, the both the intensities I1 and I2 .
photoelectric current becomes maximum or saturates. If we Therefore, for fixed frequency and intensity of incident
further increase the potential of collector plate, the light, photoelectric current increases with increase in
photocurrent does not increase. potential applied to the collector as shown in the graph
Photoelectric

5. (a) We know that, for a given photosensitive material and


current

frequency of incident radiation (above the threshold I1


frequency), the photoelectric current is directly I2
proportional to the intensity of incident light only. Since, in
the given case, only the work function has been changed
from W1 to W2 keeping the intensity of the incident light -V0 O Collector plate
same. This means, the value of photocurrent in both the potential
cases will also remain same. i.e. i1 = i2 . 12. (b) Since in the graph retarding potential is same in graphs a
6. (c) In the given condition, photocurrent is zero when the and b and photocurrent is different, so for curves they have
stopping potential is sufficient to repel even the most same frequency but different intensity of light.
energetic photoelectrons, with the maximum kinetic energy
Photocurrent
K, so that K = eV0 .
Thus, at point P (as shown in the given figure), it can be seen
clearly that the electron must have been repelled, so
maximum kinetic energy, K = eV0 .
7. (d) When a beam of electrons of energy E0 is incident on a b a
metal surface kept in an evacuated chamber electrons can
c
be emitted with maximum energy E0 (due to elastic
collision) and with any energy less than E0 , when part of
incident energy of electron is used in liberating the Retarding potential Anode potential
electrons from the surface of metal. 13. (a) For a particular frequency of incident radiation, the
8. (d) We know that, E = hn and minimum negative (retarding) potential V0 given to the
and E = mc 2 (Einstein mass-energy equation)
CBSE Term II Physics XII 83

collector plate for which the photoelectric current stops As, m e < m n, therefore l e > l n
or becomes zero is called the cut-off or stopping potential. Hence, l a < l p = l n < l e
From given the graph, the value of stopping potential, 20. (a) In a detector loop, when the light falls on the metal
V0 = - 054
. V. surface, i.e. the emitter plate, some electrons near the
14. (c) For incident radiation at same intensity and at varying surface absorb enough energy from incident radiations, i.e.
frequencies, the given graph represents the variation of from UV-light, to overcome the attraction of the positive
photocurrent with collector plate potential. This can be as ions in the material of the surface.
shown in graph below After gaining sufficient energy from UV-light, the electrons
escape from the emitter plate into the surrounding space,
Photocurrent Y thus enhancing the high voltage spark across detector loop.
Therefore, both A and R are true and R is the correct
explanation of A.
Saturation current Z 21. (b) Stopping potential is dependent on frequency, but
independent of the intensity. So, increasing the distance,
n2 n1
affects the intensity as.
–V02 –V01 1
O Intensity µ
Retarding Collector plate (Distance) 2
potential potential X
So, the stopping potential will not change. But value of
Thus, X ® collector plate potential, saturation current depends on the intensity of incident
radiation. So, more the distance, the intensity will decrease.
Y ® photocurrent and Z ® saturation current. Hence, the saturation current will also decrease.
15. (a) From the photoelectric equation, Therefore, both A and R are true but R is not the correct
E = W + KE explanation of A.
hc hc 22. (c) Since the frequency of ultraviolet light is less than the
Þ KE = E - W = -
l l0 frequency of X-rays, the energy of each incident photon will
æ1 1 ö æ l - lö be more for X-rays, KE photoelectron = hn - f
= hc ç - ÷ = hc ç 0 ÷ Stopping potential is used to stop the fastest photoelectron,
è l l0 ø è ll 0 ø
hn f
16. (c) Momentum, p = m ´ c Þ p µ m V0 = -
e e
Because mass of a-particle is more in comparison to other So, KE max and V0 both increase.
particles, so the momentum of a-particle will be highest.
But KE ranges from zero to KE max because of loss of energy
17. (a) Given, v = v0 $i Þ B = B0 $j due to subsequent collisions before getting ejected and not
Force on moving electron due to magnetic field, due to range of frequencies in the incident light.
F = - e( v ´ B ) Therefore, A is true but R is false.
= - e[ v0 $i ´ B0 $j] 23. A person approaching a doorway may intrupt a light beam
which is incident on photocell.
Þ = - ev B k$
0 0
This interuption will leads to abrupt change in the amount
As this force is perpendicular to v and B, so the magnitude of
of photocurrent. Thus, this change in photocurrent helps to
v will not change, i.e. momentum ( = mv) will remain
start a motor which is fitted in the doorway which opens the
constant in magnitude. Hence, de-Broglie wavelength
h door.
l= remains constant. Therefore, both A and R are true and R is the correct
mv
h explanation of A.
18. (c) de-Broglie wavelength, l = ...(i) h
2mK 24. (c) de-Broglie wavelength, l =
K p
When the kinetic energy is , then
9 Since, their momenta are same due to conservation of linear
h 3h momentum.
l¢ = = = 3l [using Eq. (i)]
æKö 2 mK Hence, their wavelengths are same but their speed will be
2m ç ÷ different.
è 9ø
Therefore, A is true but R is false.
19. (b) We know that, the relation between l and K is given by
h 25. (i) (d) According to photoelectric effect, no electrons were
l= emitted at all when the frequency of the incident light
2mK was smaller than a certain minimum value, called the
1 threshold frequency. This minimum frequency depends
or lµ
m on the nature of the material of the emitter plate.
Since, m p = m n, hence l p = l n (ii) (c) When the wavelength of incident light is l £ l 0 , then
As, m a > m p , therefore l a < l p the electrons will come out of the metal surface.
84 CBSE Term II Physics XII

(iii) (d) Photocurrent varies linearly with intensity. The Take, two points C and D on the graph.
photocurrent is directly proportional to the number of
photoelectrons emitted per second. This implies that, it B
is a straight line passing through origin. Stopping V2 D
potential (V)
Hence, option (d) is correct. V1 C
(iv) (a) Given, work function = 2eV
Energy of incident photon = 3.4 eV
A
From Einstein’s equation of photoelectric effect, n0 n1 n2
hn = hn 0 + K Frequency of radiation (n)
3.4 eV = 2 eV + K The corresponding frequency of radiation is n1, n 2 and
K = 3 .4 eV -2.0 eV = 1.4 eV stopping potential is V1, V2 .
(v) (d) According to photoelectric effect, light consists of Then, eV1 = hn1 - f 0 and eV2 = hn 2 - f 0
packets of energy. \ e ( V2 - V1 ) = h ( n 2 - n1 )
In interaction of radiation with matter, radiation behaves e ( V2 - V1 )
as, if it is made up of particles called photons. Thus, or h= = e ´ slope of ( V - n ) graph
photons confirms the particle nature of light. n 2 - n1
Each photon have energy, E = hn. Thus, Planck’s constant can be determined using graph.
5. Given, l = 4125 . nm = 412.5 ´ 10-9 m
PART 2 hc 6.63 ´ 10-34 ´ 3 ´ 108
\ E= = eV = 3.01 eV
1. According to first statement, when the materials which l 412.5 ´ 10-9 ´ 1 . 6 ´ 10-19
absorb photons of shorter wavelength has high energy of the
From the given question, work function f of the following
incident photon on the material and low energy of emitted
metals are given as
photon of longer wavelength.
Na ® 1.92 eV, K ® 2.15 eV
But in second statement, the energy of the incident photon
is low for the substances which has to absorb photons of Ca ® 3.20 eV, Mo ® 4.17 eV
larger wavelength and energy of emitted photon is high to As the given energy is greater than the work function of Na
emit light of shorter wavelength. This means in this and K only, hence these metals shows photoelectric
statement material has to supply the energy for the emission emission.
of photons. But this is not possible for a stable substances. 6. Given, f = 214
. eV = 214. ´ 1.6 ´ 10-19 J
2. The number of photons of beam A = n A and Vstopping = 0.6 V
The number of photons of beam B = n B (i) Work function, f = hn 0
According to the question, n A = 2n B . ´ 1.6 ´ 10-19
f 214
So, threshold frequency, n 0 = =
Let n A be the frequency of beam A and n B be the frequency h 6.62 ´ 10-34
of beam B. Þ n 0 = 5.17 ´ 1014 Hz
\ Intensity µ Energy of photons
(ii) As, K max = eV0 = 0.6 eV
Þ I µ hn ´ Number of photons Energy of photon, E = K max + f
IA n A n A
\ = = 0.6 eV + 2.14 eV
IB n Bn B
= 2.74 eV
According to the question, IA = IB hc
\ n A n A = n Bn B Hence, wavelength of photon, l =
E
n A nB 1 -34
or = = 6.62 ´ 10 ´ 3 ´ 10 8
n B nA 2 = (Qc = 3 ´ 108 m/s)
2.74 ´ 1.6 ´ 10-19
So, nB = 2 nA
= 4530 Å
3. The intensities for both the monochromatic radiations are
h( n - n0 )
same but their frequencies are different. It represents 7. Cut-off voltage , V0 =
(i) the number of electrons ejected in two cases are same e
because it depends on the number of incident photons. 6.63 ´ 10-34 ( 8.2 ´ 1014 - 3.3 ´ 1014 )
=
(ii) As, KE max = hn - f 0 = hc / l - f 0 1.6 ´ 10–19
(Einstein’s photoelectric equation) = 2.03 V
Since, wavelength l of violet colour is minimum. So, 8. Given, V = 3.3 V
the KE max of violet radiation will be more. and frequency of photons, n = 8 ´ 1014 Hz
\ The KE max of violet radiation will be more. As we know,
4. The variation of stopping potential with the frequency of eV0 = hn - f
radiation, incident on a metal plate is a straight line AB as
So, f = hn - eV0
shown in the figure.
CBSE Term II Physics XII 85

6.63 ´ 10-34 ´ 8 ´ 1014 æ 1 1ö


Þ f ( eV ) = eV - 3.3 eV Þ e( V2 - V1 ) = hc ç - ÷
1.6 ´ 10-19 è l 2 l1 ø
= ( 3.31 - 3.3) eV e( V2 - V1 ) e( V2 - V1 ) l1l 2
or h= =
Þ f = 0.01 eV æ 1 1ö c( l1 - l 2 )
cç - ÷
9. Given, for the first condition, l = 600nm è l 2 l1 ø
For the second condition, l ¢ = 400nm Thus, Planck’s constant can be determined from graph.
¢ = 2K max
K max Note Since, h is a constant, so it will be same for both
hc metals A and B.
Here, ¢ =
K max -f (ii) Stopping potential V0 for the electrons emitted will not
l
be affected by the increase in distance between light
hc
Þ 2K max = - f0 source and the metal surface A.
l¢ This is because, V0 is independent of the intensity of the
æ 1240 ö æ 1240 ö incident light but depends only upon the frequency (or
Þ 2ç - f÷ » ç - f÷ (Qhc » 1240 eV -nm )
è 600 ø è 400 ø wavelength) of incident light.
1240 So, increase in the given distance affects only the
Þ f= = 1 .03 eV intensity of the light but not the frequency. Thus, V0
1200
remains same.
10. From Einstein’s photoelectric equation,
12. Two salient features observed in photoelectric effect and
1 2
K max = mvmax = h( n - n 0 ) their explanation on the basis of Einstein’s photoelectric
2 equation is given as below
where, h = Planck’s constant, (i) Threshold Frequency For KE max ³ 0.
n = frequency of incident light Þ n ³ n0
and n 0 = threshold frequency of the photosensitive surface. i.e. The phenomenon of photoelectric effect takes place,
So, for photoemission to takes place, n > n 0 . when incident frequency is greater or equal to a
As the wavelength of light incident is same for all the three minimum frequency (threshold frequency) n 0 fixed for
surfaces, so given metal.
(i) threshold frequency of surface A is higher than the (ii) Effect of Intensity of Incident Light The number of
frequency of incident light, as no emission takes photons incident per unit time per unit area increases
place. with the increase of intensity of incident light. More
(ii) threshold frequency of surface B is equal to the number of photons facilitates ejection of more number
frequency of incident light, as photoelectrons are just of photoelectrons from metal surface leads to further
emitted. increase of photocurrent till its saturation value is
(iii) threshold frequency of surface C is lower than the reached.
frequency of incident light, as the emitted 13. The wave theory of light is not able to explain the observed
photoelectrons have some kinetic energy. features of photoelectric current because of following
\ ( n0 )A > ( n0 )B > ( n0 )C reasons
11. (i) The variation of stopping potential V0 for the (i) According to wave nature of light, the free electrons at
1 the surface of the metal absorb the radiant energy
photoelectron versus graph is as shown below continuously.
l
V0 B The greater the intensity of radiation, the greater should
be the energy obtained by each electrons. The maximum
D kinetic energy of the photoelectrons on the surface is
V2
C then expected to increase with increase in intensity. But
V1
according to experimental facts, the maximum kinetic
1/l energy of ejected photoelectrons is independent of
O 1/l1 1/l2 intensity of incident radiation.
(ii) Wave theory states that, energy carried by wave is
independent of frequency of light wave and hence wave of
Take, any two points C and D on the graph as shown high intensity and low frequency (less than threshold
above. frequency) should stimulate photoelectric emission but
According to Einstein’s photoelectric equation, we can practically, it does not happen.
hc Considering the following few properties of photon, the
write, eV1 = - f0 …(i)
l1 above problem was resolved
where, f 0 is the work function of metal A. (i) In interaction of radiation with radiation behaves as, if it
hc is made up of particle called photon.
and eV2 = - f0 …(ii) (ii) Energy of a photon is directly proportional to the
l2
frequency of the incident light.
Subtracting Eq. (i) from Eq. (ii), we get 14. For a given frequency, intensity of light in the photon
picture is determined by
86 CBSE Term II Physics XII

energy of photons n ´ hn Thus, wavelength of electromagnetic radiation is equal to the


I= =
area ´ time A´t de-Broglie wavelength.
where, n is the number of photons incident normally on 19. (i) The de-Broglie wavelength of a particle is given by
cross-sectional area A in time t. h
l=
15. (i) The energy of photoelectrons in a photocell is given by 2mV0 q
hc
E= = hn Þ E µ n Since, a-particle and proton both are accelerated
l
through the same potential V0 .
So, if the frequency of light incident on the cathode is
1
increased, the energy of photoelectrons increases \ lµ
linearly. mq
(ii) As, photoelectric current/photocurrent of the photocell is la mp qp
independent of frequency of the incident light, till or =
intensity remains constant. So, when the frequency of lp ma qa
light incident on the cathode of photocell is increased As, charge on a-particle = 2 ´ charge on proton
keeping other factors same, the photoelectric current qp 1
remains the same. q a = 2q p Þ =
qa 2
16. Given, power of lamp, P = 100 W
Mass of a-particle = 4 ´ mass of proton
Wavelength of the sodium light, l = 589 nm = 589 ´ 10- 9 m mp 1
ma = 4 ´ mp Þ =
Planck’s constant, h = 6.63 ´ 10- 34 J-s ma 4
(i) Energy of each photon, la 1 1 1
\ = × =
hc 6.63 ´ 10- 34 ´ 3 ´ 108 lp 4 2 2 2
E= =
l 589 ´ 10- 9 Þ lp = 2 2la
(Q c = 3 ´ 108 m / s ) i. e. Proton has greater de-Broglie wavelength than that of
= 3.38 ´ 10-19 J a-particle.
3.38 ´ 10-19 (ii) KE µ q (for same accelerating potential)
= eV = 211
. eV The charge of an a-particle is more as compared to a
1.6 ´ 10-19 proton, so it will have a greater value of KE . Hence,
(ii) Let n photons are delivered per second. proton will have lesser KE.
Power 20. de-Broglie wavelength of accelerating charged particle is
\ n= (from P = En )
Energy of each photon given by
100 h
= = 3 ´ 1020 photons/s l=
3.38 ´ 10-19 2mqV
= 3 ´ 1020 photons/s h
Þ l V = = constant
2mq
17. Energy of the incident radiation of wavelength l,
h
hc ( 6.63 ´ 10-34 ) ´ ( 3 ´ 108 ) (i) The slope of the lines represents
E= = 2mq
l 3300 ´ 10-10 ´ 1 .6 ´ 10-19
where, h = Planck’s constant, q = charge and
= 3.76 eV
m = mass of charged particle.
This energy of the incident radiation is greater than the
work function of Na and K but less than those of Mo and Ni. (ii) 1H2 and 1H3 carry same charge (as they have same atomic
So, photoelectric emission will occur only in Na and K number).
metals and not in Mo and Ni. 1
\ l V µ
If the laser is brought closer, the intensity of incident m
radiation increases. This does not affect the result regarding The lighter mass, i.e. 1H2 is represented by line of greater
Mo and Ni metals, while photoelectric current from Na and K slope, i.e. A and similarly, 1H3 by line B.
will increase in proportion to intensity.
21. Given, Dx = 1 nm = 10-9 m
18. The momentum of an electromagnetic wave of frequency n,
wavelength l is given by As, DxDp » h
hn h h h
p= = \ Dp = =
c l Dx 2pDx
h 6.6 ´ 10-34 J-s
or l= =
p 2 ´ ( 22 / 7) 10-9 m
de-Broglie wavelength of photon, = 1 .05 ´ 10- 25 kg -m / s
h
l= p2 ( Dp) 2
p \ Energy, E = = (Q p » Dp)
2m 2m
CBSE Term II Physics XII 87

( 1 .05 ´ 10-25 ) 2 le =
h
= J
2 ´ 9.1 ´ 10-31 mv
( 1.05 ´ 10-25 ) 2 h l hl
= eV le = =
2 ´ 9.1 ´ 10-31 ´ 1.6 ´ 10-19 2mhe 2me
= 3.8 ´ 10-2 eV 25. We know that, K max = eV = h( n - n 0 )
22. Given, vparticle = 3 velectron ...(i) h h
or V = n - n0
-4 e e
and l particle = 1.813 ´ 10 l electron
V
h
As, l= (de-Broglie equation)
mv
m particle l electron ´ velectron
Þ = Stopping B
m electron l particle ´ vparticle potential (V) A
l electron ´ velectron
= [from (Eq. (i)] A
n
1.813 ´ 10- 4 ´ l electron ´ 3velectron nB nA n
Frequency of radiation (n)
\ m particle = 1839 m electron
(i) From the graph for the same value of n , stopping
= 1839 ´ 9.1 ´ 10-31 potential is more for material B.
= 1.673 ´ 10-27 kg h
As, V = ( n - n 0 )
\ Particle is either a proton or a neutron. e
\ V is higher for lower value of n 0 . Here n B < n A , so
23. (i) de-Broglie wavelength, l = 1.40 ´ 10-10 m
VB > VA .
Mass of neutron, m n = 1.675 ´ 10- 27 kg h
(ii) Slope of the graph is given by which is constant for all
Using the formula, wavelength associated with kinetic e
energy, the materials. Hence, slope of the graph does not depend
h on the nature of the material used.
l=
2m KE 26. For a given material, there exists a certain minimum
frequency of the incident radiation below which no
h2
or KE = emissions of photoelectrons takes place. This frequency is
2l2 m n called threshold frequency or cut-off frequency of that
( 6.63 ´ 10- 34 ) 2 material. Above the threshold frequency, the maximum
= kinetic energy of the emitted photoelectrons or equivalent
2 ´ ( 1.40 ´ 10-10 ) 2 ´ 1.675 ´ 10- 27
stopping potential is independent of intensity of incident
= 6.686 ´ 10- 21 J light but depends only upon the frequency (or wavelength)
(ii) Kinetic energy associated with temperature, of the incident light.
3 3 Given that, threshold frequency of metal is n and frequency
KE = kT = (1.38 ´ 10-23 ) ´ 300
2 2 of light is 2n. Using Einstein’s equation for photoelectric
= 6.21 ´ 10-21 J effect, we can write
1
(Q absolute temperature, T = 300 K and h ( 2n - n ) = mv12 … (i)
2
Boltzmann’s constant, k = 1.38 ´ 10-23 J / K)
Similarly, for light having frequency 5 f , we have
de-Broglie wavelength associated with kinetic energy, 1
h h ( 5 n - n ) = mv22 … (ii)
l= 2
2m n KE Using Eqs. (i) and (ii), we get
6.63 ´ 10-34 n v2
= = 12
2 ´ 1.675 ´ 10-27 ´ 6.21 ´ 10-21 4n v2
= 1.45 ´ 10-10 m v1 1 v 1
Þ = Þ 1 =
= 1.45 Å v2 4 v2 2
24. We know that, Hence, the ratio is 1 : 2.
hc hc 1 27. (i) Refer to Q-12 (SA) part-2.
= + mv2
l l0 2 (ii) Energy of incident photon is less than work function of P
Neglecting the wave equation, we get but just equal to that of Q.
hc 1 For surface Q,
= mv2 hn
l 2 Work function, f 0 = (eV)
e
de-Broglie wavelength is given by
88 CBSE Term II Physics XII

6.6 ´ 10-34 ´ 1015 1 æ2 1 ö


= = 41
. eV Þ =ç - ÷
1.6 ´ 10-19 l 0 è l1 l 2 ø
28. (i) Important properties of photons which are used to æ l1l 2 ö
establish Einstein’s photoelectric equations as given Þ l0 = ç ÷
below è 2l 2 - l1 ø
(a) In interaction of radiation with matter, radiation 30. (i) Since, Q has greater negative intercept, it will have
behaves as, if it is made up of particles called photons. greater f (work function) and hence higher threshold
(b) All photons of light of a particular frequency n or frequency.
wavelength l have the same energy E ( = hn = hc / l ) (ii) To know work function of Q, we put
and momentum p ( = hn / c = h / l ), whatever the V = 0 in the following equation
intensity of radiation may be. hv f
(c) Photons are not deflected by electric and magnetic V = -
e e
fields. This shows that, photons are electrically hn f
neutral. Þ 0= - Þ f = hn
(ii) Since, Einstein’s photoelectric equation is given by e e
1 2
\ f = 6.6 ´ 10-34 ´ 6 ´ 1014 J
KE max = mvmax = hn - hn 0 = eV0
2 6.6 ´ 6 ´ 10-20
= eV = 2.5 eV
(a) For a given material, there exist a certain minimum 1.6 ´ 10-19
frequency of the incident radiation, below which no (iii) From the equation, nl = c
emission of photoelectron takes place. This frequency
c 3 ´ 108 30
is called threshold frequency ( n 0 ). Above threshold Þ l= = = ´ 10-7 m
frequency, the maximum kinetic energy of the n 8 ´ 1014 8
30
emitted photoelectron or equivalent stopping = ´ 103 ´ 10-10 m
potential is independent of the intensity of the 8
incident light but depends only upon the frequency of 30
= ´ 103 Å = 3750 Å
the incident light. 8
(b) If the collecting plate in the photoelectric apparatus 12375 12375
Energy = = eV = 3.3 eV
is made at high negative potential, then most of the l(Å) 3750
high energetic electrons get repelled back along the \ Maximum KE of emitted electron = 3.3 - 2.5 eV
same path and the photoelectric current in the circuit
= 0.8 eV
becomes zero. So, for a particular frequency of
31. (i) Three experimentally observed features in the
incident radiation, the minimum negative potential
phenomenon of photoelectric effect is
for which the electric current becomes zero is called
(a) Intensity When intensity of incident light increases
cut-off or stopping potential ( V0 ).
as one photon ejects one electron, the increase in
29. Einstein’s photoelectric equation intensity will increase the number of ejected
1 2 electrons. Frequency has no effect on photoelectron.
K max = mvmax = hn - f 0 = hn - hn 0
2 (b) Frequency When the frequency of incident photon
According to the question, increases, the kinetic energy of the emitted
electrons increases. Intensity has no effect on
hc
K max = - f0 ...(i) kinetic energy of photoelectron.
l1 (c) No Time Lag When energy of incident photon is
Let the maximum kinetic energy for the incident radiation greater than the work function, the photoelectron is
(of wavelength l 2 ) be K¢max . immediately ejected. Thus, there is no time lag
between the incidence of light and emission of
hc photoelectron.
Þ K ¢max = - f0 ...(ii)
l2 (ii) Also, according to the wave theory, the absorption of
From Eqs. (i) and (ii), we get energy by electron takes place continuously over the
entire wavefront of the radiation.
hc æ hc ö
- f0 = 2 ç - f0 ÷ ¢ = 2 K max )
(QK max Hence, it will take hours or more for a single electron to
l2 è l1 ø come out of the metal which contradicts the
experimental fact that photoelectron emission is
æ2 1 ö instantaneous.
Þ f 0 = hc ç - ÷
è l1 l 2 ø 32. (i) Given, mass of bullet, m = 0.040 kg
æ2 1 ö Speed of bullet, v = 1000 m/s
Þ hn 0 = hc ç - ÷ h 6.63 ´ 10-34
è l1 l 2 ø de-Broglie wavelength, l = =
mv 0.040 ´ 1 ´ 103
c æ2 1 ö = 1 .66 ´ 10-35 m
=cç - ÷
l0 è l1 l 2 ø (ii) Mass of the ball, m = 0.060 kg
and speed of the ball, v = 1 m/s
CBSE Term II Physics XII 89

h 6.63 ´ 10-34 Incident power, P = Intensity ´ Area


de-Broglie wavelength, l = =
mv 0.060 ´ 1 = 10-5 ´ 2 ´ 10- 4
= 1.1 ´ 10-32 m = 2 ´ 10-9 W
(iii) Mass of a dust particle, m = 1 ´ 10-9 kg According to wave picture, the incident power is uniformly
and speed of the dust particle, v = 2.2 m/s absorbed by all the electrons continuously.
h 6.63 ´ 10-34 Hence, energy absorbed per second per electron
de-Broglie wavelength, l = =
mv 1 ´ 10-9 ´ 2.2 incident power
=
= 3.0 ´ 10-25 m number of electrons of five layers
33. Given, the following wavelengths from a mercury source 2 ´ 10-9
were used = = 2 ´ 10-26 W
1017
l1 = 3650 Å = 3650 ´ 10-10 m
\ Time required for photoelectric emission will be,
l 2 = 4047 Å = 4047 ´ 10-10 m Energy required per electron for ejection
t=
l 3 = 4358 Å = 4358 ´ 10-10 m Energy absorbed per second per atom
l 4 = 5461 Å = 5461 ´ 10-10 m 2 ´ 1.6 ´ 10-19
= = 1.6 ´ 107 s
l5 = 6907 Å = 6907 ´ 10-10 m 2 ´ 10-26
hc hc 1 2
The stopping voltages are as follows 35. We know that, = + mv
V01 = 1.28 V, V02 = 0.95 V, V03 = 0.74 V l l0 2
V04 = 0.16 V and V05 = 0 Neglecting the work function, we get
Frequencies corresponding to wavelengths, hc 1
= mv 2
c 3 ´ 108 l 2
n1 = = = 8.219 ´ 1014 Hz 2hc
l1 3650 ´ 10-10 Þ v=
ml
Similarly,
de-Broglie wavelength is given by
n 2 = 7.412 ´ 1014 Hz, n 3 = 6.884 ´ 1014 Hz h
le =
n 4 = 5.493 ´ 1014 Hz , n5 = 4.343 ´ 1014 Hz mv
As we know that, eV0 = hn - f 0 h l hl
\ le = =
hn f 0 2mhc 2 mc
V0 = -
e e 36. (i) Wavelength of incident radiation, l = 2500 Å
As the graph between V0 and frequency n is a straight line. Work function, f 0 = 3.5 eV
h According to Einstein’s photoelectric equation,
The slope of this graph gives the values of .
e hc
h V01 - V04 1 .28 - 016
. = f 0 + KE max
\ = = l
e n1 - n 4 ( 8.219 - 5 .493) ´ 1014 hc
Þ KE max = - f0
1.12 l
=
2.726 ´ 1014 é ( 6.63 ´ 10- 34 ) ( 3 ´ 108 ) 1 ù
=ê ´ - 3.5 ú eV
1 .12 ´ 1 .6 ´ 10-19 ë 2500 ´ 10 -10
1 .6 ´ 10 -19
û
h= = 6.573 ´ 10-34 J-s
2 .726 ´ 1014 = ( 4.97 - 35
. ) eV = 1.47 eV
As, n average = 5 ´ 1014 Hz (given) (a) KE of fastest electron = 1.47 eV
\Work function, f 0 = hn 0 = 6.573 ´ 10-34 ´ 5 ´ 1014 (b) KE of slowest electron = 0 eV
If the same light (having energy 4.97 eV) falls on the
= 32.865 ´ 10- 20 J surface (of work function 5.5 eV), then no photoelectron
= 2.05 eV will emit.
34. Here, I = 10-5 Wm -2 , A = 2 ´ 10- 4 m 2 (ii) Given, V = 64 V
Now, from de-Broglie equation,
n = 5, t = ?, 12.27
f 0 = 2 eV = 2 ´ 1.6 ´ 10- 19 J l= Å
V
Sodium has one conduction electron per atom and effective 12.27
atomic area = 10-20 m 2 = Å
64
Number of conduction electrons in five layers 12.27
= Å
5 ´ Area of one layer 8
=
Effective atomic area = 0153
. nm
5 ´ 2 ´ 10- 4 This wavelength belongs to the X-ray part of the
= = 1017 electromagnetic spectrum.
10-20
90 CBSE Term II Physics XII

CHAPTER 05

Atoms
In this Chapter...
l a-particles Scattering l Electron Orbits
Experiment by Rutherford
l Bohr's Model of Hydrogen Atom
l Rutherford's Model of Atom
l Hydrogen Spectrum

Atoms are the basic units of matter and defining structure of through a rotatable detector consisting of a zinc sulphide
elements. In 1898, JJ Thomson for the first time proposed screen and microscope and it was found that a-particles got
that the physical structure of atom and named it as scattered.
Plum-Pudding model.
Observations
Then, Rutherford performed an experiment on a-particle
Rutherford made the following observations from his
scattering in early 90’s to investigate the atomic structure. experiment that are given below
But his work was rejected on the basis of classical theory. His
shortcomings was rectified by Neils Bohr through his atomic (i) Most of the a-particles passed through the gold foil
without any appreciable deflection.
model.
(ii) Only about 0.14% of the incident a-particles scattered
by more than 1°.
a-Particles Scattering (iii) About one a-particle in every 8000 a-particles
Experiment by Rutherford deflected by more than 90°.
This experiment was suggested by Rutherford in 1911 as (iv) The number of a-particles scattered per unit area
given in the figure below N( q) at scattering angle q varies inversely as sin 4 q/ 2 .
1
N( q) µ 4
Radioactive sin q/ 2
source Most
a-particles (v) The force between a-particles and nucleus is given by
Gold foil pass in 1 ( 2e) ( Ze)
straight line F= ×
(10
–8
m thick) 4 pe 0 r2
a a q where, r is the distance between the a-particles and
S
ZnS the nucleus.
screen
Lead cavity Collimator Conclusions
On the basis of his experiment, Rutherford concluded that
About one a-particle in 8000 Microscope (i) Atom has a lot of empty space and practically the
a-particles is reflected back detector
entire mass of the atom is confined to an extremely
Experimental arrangement for Rutherford’s small central core called nucleus, whose size is of the
a-particle scattering experiment order from 10 - 15 m to 10 - 14 m.
H Geiger and E Marsden took a collimated beam of (ii) Scattering of a-particles (positively charged) is due to
a-particles of energy 5.5 MeV and made it fall on the Coulomb’s law for electrostatic force of repulsion
2.1 ´ 10 -7 m thick gold foil. The a-particles were observed between the positive charge of nucleus and a-particles.
CBSE Term II Physics XII 91

(iii) Distance between electron and nucleus is from 10 4 to and KE = kinetic energy of a-particle =
1
mv 2 .
10 5 times the size of the nucleus itself. 2
(iv) More is the distance of the velocity vector of an In case of head-on-collision, the impact parameter is
a-particle from the central line of the nucleus, lesser minimum and the a-particle rebounds back (q = p).
is the angle of scattering.
For a large impact parameter, the a-particle goes nearly
undeviated and has a small deflection ( q = 0 ° ).
Rutherford’s Model of Atom
The essential features of Rutherford’s nuclear model of the
atom or planetary model of the atom are given below
(i) Every atom consists of a central core, called the
atomic nucleus, in which the entire positive charge
and almost entire mass of the atom is concentrated.
q
(ii) The size of nucleus is of the order of 10 -15 m, which is —
b
very small as compared to the size of the atom which — Target nucleus
is of the order of 10 -10 m.
Trajectory of a-particles in the Coulombic field of
(iii) The atomic nucleus is surrounded by certain number a target nucleus
of electrons. As atom on the whole is electrically
neutral, the total negative charge of electrons
surrounding the nucleus is equal to total positive Electron Orbits
charge on the nucleus.
The Rutherford nuclear model of the atom pictures the atom
(iv) These electrons revolve around the nucleus in various as an electrically neutral sphere consisting of a very small,
circular orbits as the planets do around the sun. massive and positively charged nucleus at the centre
The centripetal force required by electrons for surrounded by the revolving electrons in their respective
revolution is provided by the electrostatic force of dynamically stable orbits.
attraction between the electrons and nucleus. The electrostatic force of attraction Fe between the
Distance of Closest Approach revolving electrons and centripetal force Fc keep them in
their orbits.
At a certain distance r0 from the nucleus, whole of the KE of
a-particle converts into electrostatic potential energy and \ Fc = Fe
a-particles cannot go further close to nucleus, this distance mv 2 1 Ze 2
Þ = ×
( r0 ) is called distance of closest approach. r 4 pe0 r 2
It is given as Thus, the relation between the orbit radius and the electron
1 2Ze 2 velocity is
\ r0 = ×
4 pe0 K Ze 2
r=
1 2Ze 2 4pe0 mv 2
or r0 = ×
4 pe0 æ 1 2ö For hydrogen, Z = 1
ç mv ÷
è 2 ø e2
Therefore, r =
where, m = mass of a-particle and v = initial velocity of 4pe 0 mv 2
a-particle. The kinetic energy K and electrostatic potential energy U of
Angle of Scattering (q) the electron in H-atom are
Angle by which a-particle gets deviated from its original path 1 e2 æ e2 ö
K = mv 2 = çQ mv 2 = ÷
around the nucleus is called angle of scattering. 2 8pe0 r çè 4 pe 0 r ÷ø
Impact Parameter (b) e2
and U =-
Perpendicular distance of the velocity vector of a-particle 4pe0 r
from the central line of the nucleus of the atom is called Thus, the total mechanical energy E of the electron in a
impact parameter. H-atom is
q
Ze 2 cot e2
1 2 E=-
b= × 8pe 0 r
4 pe 0 KE
where, b = impact parameter, The total energy of the electron is negative. This implies the
fact that the electron is bound to the nucleus.
q = angle of scattering
92 CBSE Term II Physics XII

Drawbacks of Rutherford’s Model Bohr’s Theory


Rutherford’s model suffers two major drawbacks Bohr’s model is valid for all one electron atoms or ions which
(i) According to classical electromagnetic theory, the consists of a tiny positively charged nucleus and an electron
electrons must radiate energy in the form of revolving in a stable circular orbit around the nucleus. These
electromagnetic wave. one electron atoms or ions can be called hydrogen like
Due to this continuous loss of energy, radii of their atoms. e.g. Singly ionised helium (He + ) and doubly ionised
orbits should be continuously decreasing and lithium (Li 2+ ) v
ultimately the electrons should fall in the nucleus.
Electron (– e)
Thus, atom cannot remain stable.
r
(ii) Due to continuous decrease in radii of electron’s Nucleus
orbit, the frequency of revolution of electron will also +Ze
change. According to classical theory of
electromagnetism, frequency of EM wave emitted by
electron is equal to frequency of revolution of Let e, m and v be respectively the charge, mass and velocity
electron.
of the electron and r be the radius of the orbit. The positive
So, due to continuous change in frequency of charge on the nucleus is Ze, where Z is the atomic number
revolution of electron, it will radiate EM waves of all (in case of H-atom, Z = 1). As, the centripetal force is
frequencies, i.e. the spectrum of these waves will be provided by the electrostatic force of attraction, we have
continuous in nature. But, this is not the case,
mv 2 1 ( Ze) ´ e
experimentally we get line spectrum. Rutherford = ×
model was unable to explain line spectrum. r 4 pe 0 r2
Ze 2
Þ mv 2 = ...(i)
Bohr’s Model of Hydrogen Atom 4 pe 0 r
Bohr combined classical and early quantum concepts and From the second postulate, the angular momentum of the
gave his theory in the form of three postulates electron is
(i) Bohr’s first postulate states that, an electron in an h
mvr = n ...(ii)
atom could revolve in certain stable orbits without the 2p
emission of radiant energy, contrary to the predictions where, n (= 1, 2, 3, ...) is principal quantum number.
of electromagnetic theory. According to this postulate, From Eqs. (i) and (ii), we get
each atom has certain definite stable states in which it h 2 e0
can exist and each possible state has definite total r = n2 ...(iii)
energy. These are called the stationary states of the pmZe 2
atom. This is the equation for the radii of the permitted orbits.
(ii) Bohr’s second postulate states that, the electron According to this equation,
revolves around the nucleus only in those orbits for rn µ n 2
which the angular momentum is some integral Since, n = 1, 2, 3, ... it follows that the radii of the
multiple of h / 2p, where h is the Planck’s constant permitted orbits increase in the ratio 1 : 4 : 9 : 16 : ..., from
( = 6.63 ´ 10 -34 J-s) . the first orbit. Clearly, the stationary orbits are not equally
nh spaced.
mvr = , where n = 1, 2, 3, ....
2p Bohr Radius
It is also called principal quantum number. The radius of the first orbit (n = 1) of H-atom (Z = 1) will be
(iii) Bohr’s third postulate states that, an electron might
h 2 e0
make a transition from one of its specified r1 =
non-radiating orbits to another of lower energy. pme 2
When it does so, a photon is emitted having energy This is called Bohr radius and its value is 0.53 Å. Since,
equal to the energy difference between the initial and r µ n 2 , the radius of the second orbit of H-atom will be
final states. The frequency of the emitted photon is (4 ´ 0.53) Å and that of the third orbit (9 ´ 0.53) Å.
given by Velocity of Electron in Stationary Orbits
hn = E i - E f
Velocity of electrons in permitted orbits,
where, E i and E f are the energies of the initial and Ze 2 1
final states and E i > E f . v= ×
2h e 0 n
CBSE Term II Physics XII 93

where, principal quantum number, n = 1, 2, 3, … . H-atom. The atom may acquire sufficient energy to raise
1 electron to higher energy state. In this condition, the atom is
Thus, vµ said to be in excited state.
n
The velocity of electron in the first orbit (n = 1) of H-atom According to Bohr’s third postulate, the frequency n of the
( Z = 1) is emitted electromagnetic wave (photon),
e2 c E - E1 mZ 2 e4 æ 1 1 ö
v1 = = (Q c = 3 ´ 10 8 m /s) n= 2 = 2 3ç 2 - 2÷
2h e 0 137 h 8 e 0 h çè n1 n 2 ÷ø
Frequency of Electron in a Stationary Orbit The corresponding wavelength l of the emitted radiation is
It is the number of revolutions completed per second by the given by
electron in a stationary orbit around the nucleus. 1 n mZ 2 e4 æç 1 1 ö
= = - 2 ÷ (Qnl = c)
kZe 2 æ 1 ö l c 8 e 0 ch è n1
2 3 ç 2
n 2 ÷ø
n= ççQ k = ÷
nhr è 4 pe 0 ÷ø
1 æ 1 1 ö
Þ = Z 2Rç 2 - 2 ÷
Energy of Electron in Stationary Orbits l çn n2 ÷
è 1 ø
The energy E of an electron in an orbit is the sum of kinetic 1
and potential energies. where, = wave number (number of waves per unit length)
l
The kinetic energy of the electron,
me4
1 Ze 2 and R=
KE = mv 2 = 2
8e 0 ch 3
2 8pe 0 r
The potential energy of the electron in an orbit of radius r R is called Rydberg constant and its value is 1.097 ´ 10 7 m -1 .
due to the electrostatic attraction by the nucleus is given by
1 ( Ze)( -e)
Hydrogen Spectrum or Line
PE = × Spectra of Hydrogen Atom
4p e 0 r
Hydrogen spectrum consists of discrete bright lines in a dark
1 Ze 2
=- × background and it is specifically known as hydrogen
4 pe0 r emission spectrum.
In terms of Rydberg constant R, its simplified form is There is one more type of hydrogen spectrum where we get
2Rhc dark lines on the bright background, it is known as
PE = - 2 absorption spectrum.
n
Balmer found an empirical formula by the observation of a
The total energy of the electron,
small part of this spectrum and it is represented by
Ze 2 Ze 2
E = KE + PE = - = R æç 2 - 2 ö÷ , where n = 3, 4, 5 , K
1 1 1
8pe 0 r 4 pe 0 r l è2 n ø
Ze 2 Rhc where, R is a constant called Rydberg constant and its value
=- =- 2
8pe 0 r n is 1.097 ´ 10 7 m -1 .
1
Substituting for r from Eq. (iii), we get So, = 1.522 ´ 10 6 m -1 = 656.3 nm for n = 3
l
mZ 2 e4 æ 1 ö
E=- 2 ç 2÷ Other series of spectra for hydrogen were subsequently
8e 0 h 2 è n ø discovered and known by the name of their discoverers. The
where, n = 1, 2, 3, ... . lines of Balmer series are found in the visible part of the
spectrum. Other series were found in the invisible parts of
For hydrogen atom, Z = 1 the spectrum.
-13 . 6 e.g. Lyman series in the ultraviolet region and Paschen,
En = eV
n2 Brackett and Pfund in the infrared region.
The wavelengths of line in these series can be expressed by
Energy Levels the given formulae
The lowest state of the atom is called the ground state. The (i) For Lyman series (in ultraviolet region)
energy of this state is -13.6eV. Therefore, the minimum
= R æç 2 - 2 ö÷ , where n = 2, 3, 4, ...
1 1 1
energy required to free the electron from the ground state of
the H-atom is -13.6 eV. It is called ionisation energy of the l è1 n ø
94 CBSE Term II Physics XII

(ii) For Balmer series (in visible region) de-Broglie’s Comment on


Bohr’s Second Postulate
= R æç 2 - 2 ö÷ , where n = 3, 4, 5 , ...
1 1 1
l è2 n ø According to de-Broglie, a stationary orbit is that which contains
an integral number of de-Broglie standing waves associated with
(iii) For Paschen series (in infrared region)
the revolving electron.
= R æç 2 - 2 ö÷ , where n = 4, 5 , 6, ...
1 1 1 h
l=
l è3 n ø mv n
(iv) For Brackett series (in infrared region) where, v n is speed of electron revolving in nth orbit.
l
= R æç 2 - 2 ö÷ , where n = 5 , 6, 7, ...
1 1 1
l è4 n ø
(v) For Pfund series (in infrared region) r
+
= R æç 2 - 2 ö÷ , where n = 6, 7, 8, ...
1 1 1 Nucleus
l è5 n ø

Ionised atom Series limit


A standing wave is shown on a circular orbit
0 n=7
n=6 nh nh
1
Brackett n=5 \ 2prn = or mv n rn = = n ( h / 2p)
2 Paschen series n=4 mv n 2p
3 series n=3
n=2 i.e. Angular momentum of electron revolving in nth orbit must
4 Balmer Pfund be an integral multiple of h / 2p, which is the quantum condition
Energy level (eV)

5 series series
6 proposed by Bohr in his second postulate.
7
Limitations of Bohr’s Model
8
9 There are following limitations of Bohr’s model as given below
10 (i) This model is applicable only to a simple atom like
11 hydrogen having Z = 1 . This theory fails, if Z > 1.
12
13
(ii) It does not explain the fine structure of spectral lines in
14 n=1 H-atom.
Lyman
series (iii) This model does not explain why orbits of electrons are
Line spectra of the H-atom taken as circular whereas elliptical orbits are also possible.
CBSE Term II Physics XII 95

Solved Examples
Example 1. In Rutherford’s a-particle experiment e2
\ - = - 2.2 ´ 10-18
with thin gold foil, 8100 scattered a-particles per 8pe0 r
unit area per minute were observed at an angle of e2
Orbital radius, r = -
60°. Find the number of scattered a-particles per 8pe0 E
unit area per minute at an angle of 120°. (9 ´ 109 Nm 2C - 2 ) (1.6 ´ 10-19 C) 2
Þ r=
Sol. It is given that, q1 = 60°, q2 = 120°, N1 = 8100 (2) ( - 2.2 ´ 10-18 J)
Number of a-particles scattered at an angle q, Þ r = 5.3 ´ 10-11 m
1
Nµ The velocity of the revolving electron is given by
4 æ qö e2
sin ç ÷ r=
è 2ø 4pe0 mv2
æ 4 q2 ö e
çsin ÷ Þ v=
N1 è 2ø 4pe0 mr
Þ =
N2 æ 4 q1 ö Putting, m = 9.1 ´ 10-31 kg and r = 5.3 ´ 10-11 m
çsin ÷
è 2ø
(1.6 ´ 10-19 )
We have v =
æ 4 120° ö 4p e0 ´ 9.1 ´ 10-31 ´ 5.3 ´ 10-11
çsin ÷
8100 è 2 ø
Þ = v = 2.2 ´ 106 m / s
N2 æ 4 60° ö
çsin ÷
è 2 ø Example 4. Using known values for hydrogen atom,
8100 sin 60°4 calculate radius of third orbit for Li 2 + .
Þ =
N2 sin 4 30° Sol. As, Z = 3 for Li +2
1 n2
8100 ´ Further we know that, rn = r1
16 = 900 Z
Þ N2 =
9 Substituting, n = 3, Z = 3 and r1 = 0.529Å
16 (3) 2
We have, r3 = (0.529) Å = 1.587Å
Example 2. In Rutherford scattering experiment, what (3)
will be the correct angle for a scattering for an Example 5. Using known values for hydrogen atom,
impact parameter, b = 0? calculate speed of electron in fourth orbit of He + .
q
Sol. We know that, impact parameter, b µ cot Sol. As, Z = 2 for He+ .
2
q Z
Þ b = K cot Also we know that, vn = v1
2 n
where, K is a constant. Substituting n = 4, Z = 2 and v1 = 2.19 ´ 106 ms -1, we get
Here, if, b = 0. æ 2ö
v4 = ç ÷ (2.19 ´ 106 ) ms -1 = 1.095 ´ 106 ms -1
K cot q/ 2 = 0 è 4ø
Þ cot q/ 2 = 0
q p
Example 6. Find the ratio of product of velocity and
Þ = time period of electron orbiting in 2nd and 3rd
2 2
stable orbits.
Hence, q = p = 180°
Sol. We know that, time period, Tn µ n 3 and velocity, vn µ 1 / n.
Example 3. It is found experimentally that, 13.6 eV Þ Time period, Tn ´ velocity, vn µ n 2
energy is required to separate a hydrogen atom into
T2 v2 22 4
a proton and an electron. Calculate the velocity of \ = =
T3v3 32 9
the electron in hydrogen atom.
Sol. Total energy of the electron in hydrogen atom is Example 7. The ground state energy of hydrogen atom
- 13.6 eV = - 13.6 ´ 1.6 ´ 10-19 J = - 2.2 ´ 10-18 J is - 13.6 eV. What is the kinetic and potential
Total energy of the electron in hydrogen atom, energies of the electron in this state?
e2 Sol. As we know, kinetic energy = - total energy
E=-
8pe0 r = - ( -13.6) eV = 13.6 eV
96 CBSE Term II Physics XII

and potential energy = + 2 (total energy) 1 æ1 1 ö


\ = 1.097 ´ 107 ç - ÷
= 2 ( -13.6) eV = - 27.2 eV l max è 16 25 ø
Example 8. Determine the wavelength of the radiation = 0.0246 ´ 107 m
required to excite the electron in Li ++ from the \ l max = 40.650 ´ 10-7 m = 40650 Å
first to the third Bohr orbit. The shortest wavelength corresponds to n = ¥.
Sol. Given, for Li + + , Z = 3 and as the excitation is from first to 1 æ1 1ö
\ = 1.097 ´ 107 ç 2 - ÷
third Bohr orbit, so n1 = 1, n 2 = 3. l min è4 ¥ø
Using the relation, or l min = 14.58 ´ 10-7 m = 14585 Å
1 æ 1 1 ö æ1 1ö
= Z 2 R çç 2 - 2 ÷÷ = ( 3) 2 R ç 2 - 3 ÷ = 8R Both of these wavelengths lie in infrared region of
l è n1 n 2 ø è 1 3 ø electromagnetic spectrum.
1 1 Example 10. An electron of a hydrogen like atom is in
Þ Wavelength, l = =
8R 8 ´ 1.097 ´ 107 excited state. If total energy of the electron is
= 0.114 ´ 10-7 -4.6 eV, then evaluate
= 11.4 nm (i) the kinetic energy and
Example 9. Find the largest and shortest wavelengths (ii) the de-Broglie wavelength of the electron.
in the Brackett series for hydrogen. In what region Sol. Given, total energy of electron, E = - 4.6 eV
of the electromagnetic spectrum does each series (i) Kinetic energy of electron, K = - (Total energy E)
lie?
Þ = - E = - ( -4.6) = 4.6 eV
Sol. The transition equation for Brackett series is given by (ii) de-Broglie wavelength,
1 æ1 1 ö 6.6 ´ 10-34
= R ç 2 - 2 ÷ , where n = 2, 3, ... ld =
h
=
l è 4 n ø 2mK 2 ´ 9.1 ´ 10-31 ´ 4.6 ´ 1.6 ´ 10-19
The largest wavelength is corresponding to n = 5.
= 0.57 ´ 10-9 m = 0.57 nm
CBSE Term II Physics XII 97

Chapter
Practice
PART 1 6. A given beam of a-particles has a distribution of
impact parameter b, so that the beam is scattered in
Objective Questions various direction as shown below

l
Multiple Choice Questions
1. The existence of a positively charged nucleus in an
atom was first suggested by the experiment of
(a) J J Thomson (b) E Rutherford
q
(c) Chadwick (d) Hahn and Strassman
b Target nucleus
2. In the a-particle scattering experiment, the shape of
the trajectory of the scattered a-particles depend
upon [CBSE 2020] Then,
(a) b = minimum at q @ 0
(a) only on impact parameter
(b) only on the source of a-particles (b) b = maximum at q @ p
(c) Both impact parameter and source of a-particles (c) b = minimum at q @ p
(d) impact parameter and the screen material of the detector (d) b = maximum at q at all values of q

3. For scattering of a-particles, Rutherford’s 7. The graph which depicts the results of Rutherford
suggested that gold foil experiment with a-particles is
(a) mass of atom and its positive charge were concentrated q = scattering angle and Y = number of scattered
at centre of atom a-particles detected
(b) only mass of atom is concentrated at centre of atom (plots are schematic and not to scale).
(c) only positive charge of atom is concentrated at centre of
atom
(d) mass of atom is uniformly distributed throughout its Y Y
volume (a) (b)

4. In Rutherford’s nuclear model of the atom, if Fe 0 q p 0 q p


indicates electrostatic force between electron and
nucleus and Fc indicates the centripetal force on (c) Y (d) Y
revolving electron, then
(a) Fe = Fc
0 q p 0 q p
(b) Fe > Fc
(c) Fe < Fc 8. What will be the angular momentum in 4th orbit, if
(d) Fe = ¥ and Fc = 0 L is the angular momentum of the electron in the
1 2nd orbit of hydrogen atom?
5. An alpha nucleus of energy mv 2 bombards a 3 2 L
2 (a) 2L (b) L (c) L (d)
heavy nuclear target of charge Ze, then the distance 2 3 2
of closest approach for the alpha nucleus will be 9. In which of the following systems, will the radius of
proportional to the first orbit (n = 1) be minimum?
(a) v2 (b) 1 / m (a) Hydrogen atom (b) Deuterium atom
(c) 1 / v4 (d) 1 / Ze (c) Singly ionised helium (d) Doubly ionised lithium
98 CBSE Term II Physics XII

10. The angular momentum of an electron in hydrogen 18. The first member of the Balmer series of hydrogen
atom in ground state is atom has a wavelength of 6561Å. The wavelength of
h h the second member of the Balmer series (in nm) is
(a) (b)
p 2p (a) 268 (b) 684
2p p
(c) (d) (c) 486 (d) 492
h h
11. The ratio of speed of an electron in ground state in l
Assertion-Reasoning MCQs
Bohr’s first orbit of hydrogen atom to the velocity Direction (Q. Nos. 19-26) Each of these questions
of light in air is contains two statements Assertion (A) and Reason (R).
e2 2ep Each of these questions also has four alternative
(a) (b)
2phc hc choices, any one of which is the correct answer. You
e3 2 pe 2 k have to select one of the codes (a), (b), (c) and (d) given
(c) (d)
2phck hc below.
12. In accordance with the Bohr’s model, find the (a) Both A and R are true and R is the correct
quantum number that characterises the earth’s explanation of A
revolution around the sun in an orbit of radius (b) Both A and R are true, but R is not the correct
1 .5 ´ 1011 m with orbital speed 3 ´ 10 4 ms -1 . explanation of A
(Take, mass of earth = 6 . 0 ´ 10 24 kg) (c) A is true, but R is false
(a) 2 . 6 ´ 1074 (b) 8. 5 ´ 1080 (d) A is false and R is also false
(c) 4.34 ´ 10100 (d) 3.2 ´ 108 19. Assertion An atom consists of electron, proton and
neutron. Electron revolves around nucleus.
13. Taking the Bohr radius as a 0 = 53 pm, the radius of However, most of space in an atom is empty.
Li ++ ion in its ground state, on the basic of Bohr’s Reason From Rutherford’s experiment, size of the
model, will be about [NCERT Exemplar]
nucleus is 10 -10 m and from kinetic theory, size of
(a) 53 pm (b) 27 pm (c) 18 pm (d) 13 pm
the atom is 10 -15 m.
14. If the orbital radius of the electron in a hydrogen 20. Assertion If the electrons in an atom were
atom is 4.7 ´ 10 -11 m, compute the kinetic energy of stationary, then they would fall into the nucleus.
the electron in hydrogen atom.
Reason Electrostatic force of attraction acts
(a) 15.3 eV (b) - 1.53 eV
between negatively charged electrons and positive
(c) 1.3.6 eV (d) - 13.6 eV
nucleus.
15. A set of atoms in an excited state decays 21. Assertion If total energy of an electron in a H-atom
[NCERT Exemplar]
(a) in general to any of the states with lower energy
is positive, then the electron will be bound to the
(b) into a lower state only when excited by an external nucleus.
electric field Reason Bohr model is valid for both one electron
(c) all together simultaneously into a lower state atom and multi-electron atom.
(d) to emit photons only when they collide
22. Assertion Atom as a whole is electrically neutral.
16. The transition from the state n = 3 to n = 1 in a
Reason Atom contains equal amount of positive
hydrogen like atom results in ultraviolet radiation. and negative charges.
Infrared radiation will be obtained in the transition
from 23. Assertion Atoms of each element are stable and
(a) 2 ® 1 (b) 3 ® 2 emit characteristic spectrum.
(c) 4 ® 2 (d) 4 ® 3 Reason The spectrum provides useful information
17. In Pfund series, ratio of maximum to minimum about the atomic structure.
wavelength of emitted spectral lines is 24. Assertion According to electromagnetic theory, an
l max 4 l 9 accelerated particle continuously emits radiation.
(a) = (b) max =
l min 3 l min 5 Reason According to classical theory, the proposed
l max 16 l max 36 path of an electron in Rutherford atom model will
(c) = (d) =
l min 7 l min 11 be parabolic.
CBSE Term II Physics XII 99

25. Assertion Highest energy state corresponds with (ii) Gold foil used in Geiger-Marsden experiment is
the electron revolving in an orbit closest to the about 10 -8 m thick. This ensures
nucleus. (a) gold foil’s gravitational pull is small or possible
Reason Energies of the excited states come closer (b) gold foil is deflected when a-particle stream is not
incident centrally over it
and closer together as n decreases.
(c) gold foil provides no resistance to passage of a-particles
26. Assertion Second orbit circumference of hydrogen (d) most a-particle will not suffer more than 1° scattering
atom is two times the de-Broglie wavelength of during passage through gold foil
electrons in that orbit. (iii) In scattering, the impact parameter b is defined as
Reason de-Broglie wavelength of electron in the
ground state is minimum. (a) maximum kinetic energy of projectile scattered
(b) distance of closest approach between projectile and
l
Case Based MCQs target
(c) closest distance between projectile and target if there
Direction Read the following passage and answer the were no deflection
questions that follows (d) None of the above
27. a-Particle Scattering Experiment (iv) The distance of closest approach, when a 15.0 MeV
In this experiment, H Geiger and E Marsden took proton approaches gold nucleus ( Z = 79) is
(a) 758 fm
radioactive source ( 214
83 Bi) for a-particles.
(b) 7.58 fm
A collimated beam of a-particles of energy 5.5 MeV (c) 75.8 fm
was allowed to fall on 2.1 ´ 10 -7 m thick gold foil. (d) 0.758 fm
The a-particles were observed through a rotatable (v) The fact that only a small fraction of the number of
detector consisting of a zinc sulphide screen & incident particles rebound back in Rutherford
microscope and it was found that a-particles got scattering indicates that
scattered. (a) number of a-particles undergoing head-on-collision is
These scattered a-particles produced scintillations small
on the zinc sulphide screen. (b) mass of the atom is concentrated in a small volume
Observations of this experiment are as follows (c) mass of the atom is concentrated in a large volume
(d) Both (a) and (b)
I. Many of the a-particles pass through the foil
without deflection. 28. Hydrogen Spectrum
II. Only about 0.14% of the incident a-particles Hydrogen spectrum consists of discrete bright lines
scattered by more than 1°. in a dark background and it is specifically known as
hydrogen emission spectrum.There is one more
III. Only about one a-particle in every 8000
type of hydrogen spectrum that exists where we get
a-particles deflected by more than 90°.
dark lines on the bright background, it is known as
Based on these observation, they were able to absorption spectrum.
proposed a nuclear model of atom, are called
planetary model, in which entire positive charge Balmer found an empirical formula by the
and most of the mass of atom is concentrated in a observation of a small part of this spectrum and it is
small volume called the nucleus with electron represented by
revolving around the nucleus as planets revolve 1 æ 1 1 ö
=Rç - ÷ , where n = 3, 4, 5, K .
around the sun. l è2 2
n2 ø
(i) Rutherford’s atomic model can be visualised as For Lyman series, the emission is from first state to
nth state, for Paschen series, it is from third state to
nth state, for Brackett series, it is from fourth state
(a) (b)
to nth state and for Pfund series, it is from fifth
state to nth state.
(i) Number of spectral lines in hydrogen atom is
(a) 8 (b) 6
(c) (d) (c) 15 (d) ¥

`
100 CBSE Term II Physics XII

(ii) Which series of hydrogen spectrum corresponds to 4. Calculate the orbital period of the electron in the
ultraviolet region? [CBSE 2020] first excited state of hydrogen atom. [Delhi 2019]
(a) Balmer series
5. Positronium is just like a H-atom with the proton
(b) Brackett series
replaced by the positively charged anti-particle of
(c) Paschen series
the electron (called the positron which is as
(d) Lyman series massive as the electron). What would be the
(iii) The figure shows energy levels of an atom with six ground state energy of positronium?
transitions of wavelengths l 1 , l 2 , l 3 , l 4 , l 5 and [NCERT Exemplar]
l 6 . The following wavelengths also occur in 6. The total energy of an electron in the first excited
absorption spectrum [CBSE 2020] state of hydrogen atom is about -3.36 eV.
C
(i) What is the kinetic energy of the electron in first
B excited state?
A (ii) What is the potential energy of the electron in
G the first excited state?
l1 l2 l3 l4 l5 l6
(iii) If zero potential energy reference is changed,
(a) l1, l 2 , l 3 which of the above would change?
(b) l 4 , l5 , l 6 7. Define ionisation energy. How would the ionisation
(c) l1, l 4 , l 6 energy change when electron in hydrogen atom is
(d) l1, l 2 , l 3, l 4 , l5 , l 6 replaced by a particle of mass 200 times that of the
(iv) The ratio of the wavelength for 2 ® 1 transition in electron but having the same charge? [All India 2013]
Li ++ , He + and H is 8. How many different wavelengths may be observed
(a) 1 : 2 : 3 (b) 1 : 4 : 9 in the spectrum from a hydrogen sample, if the
(c) 4 : 9 : 36 (d) 3 : 2 : 1 atoms are excited to states with principal quantum
(v) When an electron jumps from the orbit n = 2 to number n?
n = 4, then wavelength of the radiations absorbed 9. The figure shows energy level diagram of hydrogen
will be (R is Rydberg’s constant) atom
A B C
16 16 n=4
(a) (b)
3R 5R D E
n=3
5R 3R
(c) (d) F
16 16 n=2

PART 2 n=1
(i) Find out the transition which results in the
Subjective Questions emission of a photon of wavelength 496 nm.
(ii) Which transition corresponds to the emission of
l
Short Answer (SA) Type Questions radiation of maximum wavelength? Justify your
answer. [All India 2015C]
1. Explain briefly how Rutherford scattering of 10. Calculate the shortest wavelength present in
a-particle by a target nucleus can provide Paschen series.
information on the size of the nucleus. [Delhi 2019]
11. The short wavelength limit for the Lyman series of
2. An a-particle moving with initial kinetic energy K the hydrogen spectrum is 913.4 Å. Calculate the
towards a nucleus of atomic number Z approaches short wavelength limit for Balmer series of the
a distance d at which it reverses its direction. hydrogen spectrum. [Delhi 2016]
Obtain the expression for the distance of closest
approach d in terms of the kinetic energy K of 12. Different elements have different atomic spectra.
a-particle. [All India 2016 C]
Comment.

3. State Bohr postulate of hydrogen atom that gives 13. Find the ratio between the wavelengths of the 'most
the relationship for the frequency of emitted energetic' spectral lines in the Balmer and Paschen
photon in a transition. [Foreign 2016]
series of the hydrogen spectrum.
CBSE Term II Physics XII 101

14. Obtain the expression for the ratio of the 23. The ground state energy of hydrogen atom is
de-Broglie wavelengths associated with the – 13.6 eV. If an electron makes a transition from an
electron orbiting in the second and third excited energy level – 0.85 eV to – 1.51 eV, calculate the
states of hydrogen atom. [Delhi 2019] wavelength of the spectral line emitted. To which
15. State Bohr’s quantisation condition of angular series of hydrogen spectrum, does this wavelength
momentum. Calculate the shortest wavelength of belong? [All India 2012]
the Brackett series and state to which part of the l
Long Answer (LA) Type Questions
electromagnetic spectrum does it belong.[Delhi 2019]
16. Using Bohr’s second postulate of quantisation of 24. (i) In H-atom, an electron undergoes transition
orbital angular momentum, show that the from second excited state to the first excited state
circumference of the electron in the nth orbital and then to the ground state. Identify the
spectral series to which these transitions belong.
state in hydrogen atom is n-times the de-Broglie
wavelength associated with it. [Delhi 2020] (ii) Find out the ratio of the wavelengths of the
emitted radiations in the two cases.
17. Why motion of planets around the sun cannot be
governed by Bohr’s postulate of quantisation of 25. Show that the first few frequencies of light that is
angular momentum? emitted when electrons fall to nth level from levels
higher than n, are approximate harmonics (i.e., in
18. In a Geiger-Marsden experiment, calculate the the ratio 1 : 2 : 3 K) when n >> 1. [NCERT Exemplar]
distance of closest approach to the nucleus of
Z = 80, when an a-particle of 8 MeV energy 26. State any two postulates of Bohr’s theory of
impinges on it before it comes to momentarily rest H-atom. What is the maximum possible number of
and reverses its direction. spectral lines, when the H-atom is in its second
excited state? Justify your answer. Calculate the
How will the distance of closest approach be
ratio of the maximum and minimum wavelengths
affected when the kinetic energy of the a-particle
of the radiations emitted in this process.
is doubled? [All India 2012]
[All India 2010]
19. Using Rutherford model of the atom, derive the 27. Obtain an expression for the frequency of radiation
expression for the total energy of the electron in emitted, when a hydrogen atom de-excites from
hydrogen atom. What is the significance of total level n to level ( n -1 ) . For large n, show that this
negative energy possessed by the electron? frequency equals to the classical frequency of
[All India 2014]
revolution of the electron in the orbit. [NCERT]
20. Define the distance of closest approach. An
a-particle of kinetic energy K is bombarded on a
28. (i) The kinetic energy of the electron orbiting in the
first excited state of hydrogen atom is 3.4 eV.
thin gold foil. The distance of the closest approach is r.
Determine the de-Broglie wavelength associated
What will be the distance of closest approach for an
with it.
a-particle of double the kinetic energy?
(ii) Calculate the de-Broglie wavelength associated
Write two important limitations of Rutherford
with the electron in the second excited state of
nuclear model of the atom. [All India 2016]
hydrogen atom. The ground state energy of the
21. A photon emitted during the de-excitation of hydrogen atom is 13.6 eV. [Delhi 2020]
electron from a state n to the first excited state in a
29. The stopping potential for the photoelectrons
hydrogen atom, irradiates a metallic cathode of
emitted from a metal surface of work function
work function 2eV, in a photocell, with a stopping
1.7 eV is 10.4 V. Find the wavelength of the
potential of 0.55 V. Obtain the value of the quantum
radiation used. Also, identify the energy levels in
number of the state n. [All India 2019]
hydrogen atom which will emit this wavelength.
22. A 12.5 eV electron beam is used to bombard (Take, h = 6.63 ´ 10 - 34 J-s, c = 3 ´ 10 8 ms -1 and
gaseous hydrogen at room temperature. Upto e = 1.6 ´ 10 - 19 )
which energy level, the hydrogen atoms would be
excited? 30. (i) Using the Bohr’s model, calculate the speed of the
Calculate the wavelengths of the first member of electron in a H-atom in the n = 1, 2 and 3 levels.
Lyman and first member of Balmer series. (ii) Calculate the orbital period in each of these
[Delhi 2014] levels. [NCERT]
102 CBSE Term II Physics XII

Total energy, E(eV)


31. If a proton had a radius R and the charge was Unbound (ionised)
uniformly distributed, calculate using Bohr atom
theory, the ground state energy of a H-atom when
(i) R = 0.1Å and (ii) R = 10Å. [NCERT Exemplar] 0
32. The first four spectral in the Lyman series of a n=5
–0.85
H-atom are l = 1218Å, 1028Å, 974.3Å and 951.4Å. n=4
If instead of hydrogen, we consider deuterium, –0.51 n=3 Excited
calculate the shift in the wavelength of these states
lines. [NCERT Exemplar] –3.40 n=2
33. Using Bohr’s postulates, derive the expression for
the frequency of radiation emitted when electron
in hydrogen atom undergoes transition from
Ground state
higher energy state (quantum number n i ) to the
lower state, ( n f ). When electron in hydrogen –13.6 n=1
atom jumps from energy state n i = 4 to .

n f = 3, 2, 1. Identify the spectral series to which In a mixture of H—He + gas (He + is single ionised
the emission lines belong. [All India 2018, 13]
He atom), H-atoms and He + ions are excited to their
34. A hydrogen like atom (atomic number Z) is in a respective first excited states. Subsequently, H-atoms
higher excited state of quantum number n. It can transfer their total excitation energy to He + ions (by
emit two photons of energies 10.20 eV and collisions).
17.00 eV. When make transition to the first excited (i) Calculate the wavelength of radiation emitted,
state. But emits photons of energies 4.25 eV and when an electron having total energy -15 . eV
5.95 eV, if make transition to second excited state. makes a transition to the ground state.
Determine the values of Z and n. [All India 2014 C] [Given, energy in the ground state
l
Case Based Questions = - 13.6 eV and Rydberg’s constant
= 1.09 ´ 10 7 m -1 ]
Direction Read the following passage and answer
the questions that follows (ii) Hydrogen atom has only one electron but its
emission spectrum has many lines. Explain with
35. Excited State of Atom reason.
At room temperature, most of the H-atoms are in (iii) Find the relation between the three wavelengths
ground state. When an atom receives some l 1 , l 2 and l 3 from the energy level diagram
energy (i.e. by electron collisions), the atom may shown below.
acquire sufficient energy to raise electron to
C
higher energy state. In this condition, the atom is l3
said to be in excited state. From the excited state, l1
the electron can fall back to a state of lower B
A
energy, emitting a photon equal to the energy l2
difference of the orbit.
Chapter Test
Multiple Choice Questions 8. Calculate the wavelength of Ha -line in Balmer series of
1. Atoms consist of a positively charged nucleus is hydrogen atom.
obvious from the following observation of (Take, Rydberg constant, R = 1.097 ´ 10 7 m -1 )
Geiger-Marsden experiment (Ans. 6.56 ´ 10 - 7 m)
(a) most of a-particles do not pass straight through the 9. Explain, in brief, why Rutherford’s model cannot account
gold foil for the stability of an atom.
(b) many of a-particles are scattered through the acute
angles
10. The value of ground state energy of hydrogen atom is
(c) very large number of a-particles are deflected by large
-13.6 eV.
angles (i) Find the energy required to move an electron from the
(d) None of the above ground state to the first excited state of the atom.
2. The simple Bohr model cannot be directly applied to (ii) Determine (a) the kinetic energy and (b) orbital radius
calculate the energy levels of an atom with many in the first excited state of the atom.
electrons. This is because (Given, the value of Bohr's radius= 0.53 Å)
[Ans. (i) 10.2eV, (ii) (a) 3.4 eV (b) 2.12 Å]
(a) of the electrons not being subject to a central force
(b) of the electrons colliding with each other 11. Assume that, there is no repulsive force between the
(c) of screening effects electrons in an atom but the force between positive and
(d) the force between the nucleus and an electron will no
negative charges is given by Coulomb’s law as usual.
longer be given by Coulomb’s law
Under such circumstances, calculate the ground state
energy of a He-atom. (Ans. - 54. 4 eV)
3. The number of spectral lines produced due to
transition among three energy levels will be Long Answers Type Questions
(a) 10 12. (i) State Bohr’s postulate to define stable orbits in
(b) 8 hydrogen atom. How does de-Broglie’s hypothesis
(c) 6 explain the stability of these orbits?
(d) 3 (ii) A hydrogen atom initially in the ground state absorbs
4. The de-Broglie wavelength of an electron in first a photon which excites it to the n = 4 level. Estimate
Bohr’s orbit is equal to the frequency of the photon. [Ans. (ii) 3. 1 ´ 10 15 Hz]

(a)
1
of circumference of orbit
13. The wavelength of light from the spectral emission line of
4 sodium is 589 nm. Find the kinetic energy at which
1 (i) an electron and
(b) of circumference of orbit
2 (ii) a neutron, would have the same de-Broglie
(c) twice of circumference of orbit wavelength. [Ans. (i) 4.34 ´ 10 -6 eV and (ii) 236
. ´ 10 - 9 eV]
(d) the circumference of orbit
14. (i) The total energy of electron in the ground state of
Short Answers Type Questions hydrogen atom is - 13.6 eV. Find the kinetic energy of
an electron in the first excited state.
5. Explain, why scattering of a-particles in Rutherford’s
experiment is not affected by the mass of the (ii) Find the kinetic energy, potential energy and total
nucleons. energy of electron in first and second orbit of
hydrogen atom, if potential energy in first orbit is
6. Calculate the radius of the first orbit of H-atom. taken to be zero. [Ans. (i) 3.4 eV,
Show that the velocity of electron in the first orbit is (ii) First orbit K = 1360
. eV, E = 1360
. eV
1/137 times the velocity of light. (Ans. 0.53 Å) Second orbit K = 3. 40 eV, U = 20. 40 eV,
7. What does an emperical formula mean? Hence, E = 2380
. eV]
explain that how Balmer proposed this formula.

Answers
Multiple Choice Questions For Detailed Solutions
1. (d) 2. (a) 3. (d) 4. (d) Scan the code
104 CBSE Term II Physics XII

EXPLANATIONS
PART 1 2pZe 2 k
11. (d) Speed of an electron in nth orbit, vn =
nh
1. (b) A positively charged nucleus in an atom was first For hydrogen atom in ground state, n = 1, Z = 1
suggested by E Rutherford through its gold foil experiment.
2pe 2 k v 2pe 2 k
2. (a) In the a-particle scattering experiment, the shape of the v= Þ =
h c hc
trajectory of the scattered a-particles depends upon only on
impact parameter. 12. (a) Given, radius of orbit, r = 1.5 ´ 1011 m,
3. (a) In Rutherford’s nuclear model of the atom, the entire Orbital speed, v = 3 ´ 104 ms -1 and mass of earth,
positive charge and most of the mass of the atom are
m = 6.0 ´ 1024 kg
concentrated in the nucleus with the electrons some
distance away. nh
Angular momentum, L = mvr =
4. (a) The electrostatic force of attraction Fe between the 2p
revolving electrons and the nucleus provides the requisite 2pvrm
or n=
centripetal force Fc to keep them in their orbits. h
Thus, for a dynamically stable orbit in a hydrogen atom where, n is the quantum number of the orbit.
Fe = Fc 2 ´ 3.14 ´ 3 ´ 104 ´ 1.5 ´ 1011 ´ 6.0 ´ 1024
Þn=
5. (b) For the distance of closest approach, we can write 6.63 ´ 10-34
1 K ´ ( Z e ) ( Ze ) 1 74
= 2.57 ´ 10 or n = 2.6 ´ 1074
mv2 = Þ r0 µ
2 r0 m 13. (c) The atomic number of lithium is 3, therefore the radius
So, the distance of closest approach for the alpha nucleus of Li + + ion in its ground state, on the basis of Bohr’s model
1 1
will be proportional to . will be about times to that of Bohr radius. Therefore, the
m 3
6. (c) It has been seen that, an a-particle close to the nucleus 53
radius of lithium ion is near » 18 pm.
suffers large scattering angle. 3
In case of head-on-collision, the impact parameter is 14. (a) Kinetic energy
minimum and the a-particle rebounds back (q @ p). e2 ( 9 ´ 109 Nm 2 / C 2 ) ( 1.6 ´ 10-19 C ) 2
For large impact parameter, the a-particle goes nearly K= =
8pe0 r ( 2) ( 4.7 ´ 10-11 m)
undeviated and has a small deflection (q @ 0).
7. (b) In Rutherford’s experiment, number of particles = 2 .45 ´ 10-18 J = 15.3 eV
scattered at large angles is very less and most of the 15. (a) A set of atoms in an excited state decays in general to any
particles are scattered at small angles. of the states with lower energy.
Hence, graph of Y ( = number of a- particles) and 16. (d) Infrared radiation found in Paschen, Brackett and Pfund
q ( = scattering angle) is shown correctly in option (b). series and it is obtained when electron transition is from
8. (a) We know that, the angular momentum of the electron of higher energy level to minimum third level.
hydrogen atom, Therefore, infrared radiation will be obtained in the
nh transition from 4 ® 3.
Ln = …(i)
2p 17. (d) In Pfund series,
But according to question, the angular momentum in 2nd
1 æ1 1 ö
orbit is L. = R ç 2 - 2 ÷ , where n = 6, 7, L
2h h l è5 n ø
So, L= = …(ii)
2p p Maximum wavelength is given by
Hence, the angular momentum in 4th orbit is 1 æ1 1 ö 11 R
4h 2h =R ç 2 - 2÷= (in transition 6 ® 5)
L 4¢ = = = 2L l max è5 6 ø 36 ´ 25
2p p
2
n 1 Minimum wavelength is given by
9. (d) Radius of nth orbit, rn µ or rn µ
Z Z 1 æ1 1ö R
So, here lithium has least atomic number, thus option (d) is =Rç 2 - ÷ = (in transition ¥ ® 5)
l min è5 ¥ ø 25
correct.
10. (b) From the formula of angular momentum and before l max 36
So, ratio of = .
æ hö l min 11
assumption, mvr = n ç ÷
è 2p ø 18. (c) First member is obtained in transition n =`3 to n = 2,
Here, n = 1
1 æ1 1 ö
h = Rç 2 - 2 ÷ ...(i)
Þ mvr = l1 è2 3 ø
2p
CBSE Term II Physics XII 105

Second member is obtained in transition n = 4 to n = 2, Different atoms produce different characteristic spectrum
1 æ1 1 ö and hence we can differentiate between atoms by their line
= Rç 2 - 2 ÷ ...(ii) spectrum.
l2 è2 4 ø
Hence, it provides useful information about the atomic
From Eqs, (i) and (ii), we have
structure.
1 æ1 1 ö
ç 2 - 2÷ Therefore, both A and R are true but R is not the correct
l1
=è ø
2 3 explanation of A.
1 æ1 1 ö
ç - ÷ 24. (c) According to electromagnetic theory, an accelerated
l 2 è 22 42 ø charge particle continuously emits radiation.
æ1 1 ö But Rutherford could not explain the revolution of electrons
ç - ÷
l2 5 64
= è
4 9ø around nucleus. The path of revolution is circle, not parabola.
= ´
l1 æ 1 1 ö 36 12 Therefore, A is true but R is false.
ç - ÷
è 4 16 ø 25. (d) The energy of an atom is the least, when its electron is
l 2 5 4 20 revolving in an orbit closest to the nucleus, i.e. the one for
Þ = ´ = which n = 1. However, the highest energy state corresponds
l1 9 3 27
to n = ¥ and has an energy of 0 eV.
20
Þ l 2 = l1 ´ Since, En µ 2
1
27
n
20
= 6561 ´ = 4860 Å = 486 nm So, as the value of n increases, energies of excited states
27 comes closer and closer.
19. (c) Rutherford’s experiment suggested that, the size of the Therefore, A is false and R is also false.
nucleus to be about 10-15 m to 10-14 m. From kinetic theory, æ hö
the size of an atom was known to be 10-10 m, about 10,000 to 26. (b) mv2 r2 = 2ç ÷
è 2p ø
100,000 times larger than the size of the nucleus. æ h ö
Thus, the electrons would seem to be at a distance from the \ 2pr2 = 2çç ÷÷ = 2l 2
nucleus about 10,000 to 1,00,000 times the size of the è mv2 ø
nucleus itself. h
Further, l =
Hence, most of space in an atom is empty. p
Therefore, A is true but R is false. Speed of momentum is maximum in ground state, hence l is
minimum.
20. (a) In an atom, electron revolves around nucleus, for this
required centripetal force is provided by electrostatic force Therefore, both A and R are true but R is not the correct
of attraction between negatively charged electron and explanation of A.
positive nucleus. 27. (i) (d) Rutherford’s atom had a positively charged centre and
If the electrons were stationary, then the electrostatic force electrons were revolving outside it. It is also called the
will remain unbalanced, which leads to the electron to fall planetary model of the atom as in option (d) .
into the nucleus. (ii) (d) As the gold foil is very thin, it can be assumed that
a-particles will not suffer more than 1° scattering
Therefore, both A and R are true and R is the correct during their passage through it. Therefore,
explanation of A. computation of the trajectory of an a-particle scattered
21. (d) Total energy E of an electron in a hydrogen atom, by a single nucleus is enough.
-e 2 (iii) (d) The impact parameter is defined as the perpendicular
E= distance between the path of a projectile and the centre
8pe0 r
of potential field created by an object that the projectile
So, E is negative. is approaching.
This implies the fact that, the electron is bound to the (iv) (b) Given, E = 15.0 MeV = 15 ´ 1.6 ´ 106 ´ 10-19 J
nucleus.
= 15 ´ 1.6 ´ 10-13 J
Bohr model is valid for only one electron atom.
1 Ze 2
Therefore, A is false and R is also false. E= ×
4pe0 r0
22. (a) For an atom, number of electrons is equal to number of 2
protons and neutrons are electrically neutral. So, net charge 9 ´ 109 ´ 79 ´ ( 1.6 ´ 10-19 )
on an atom is zero. Þ r0 = = 7.58 fm
15 ´ 1.6 ´ 10-13
Therefore, both A and R are true and R is the correct (v) (d) In case of head-on-collision, the impact parameter is
explanation of A. minimum and the a-particle rebounds back. So, the fact
23. (b) All atoms have different number of electrons. These that only a small fraction of the number of incident
electrons correspond to different energy level. Whenever an particles rebound back indicates that the number of
atom is excited, we get some energy in the form of radiation a-particles undergoing head-on collision is small.
and the obtained radiation is called line spectra of that This in turn implies that the mass of the atom is
particular atom. Therefore, atoms of each element are stable concentrated in a small volume.
and emit characteristic spectrum. Hence, options (a) and (b) are correct.
106 CBSE Term II Physics XII

28. (i) (d) Number of spectral lines in hydrogen atom is ¥. where, n is frequency of radiation emitted, Ei and E f are the
(ii) (d) Lyman series lies in the ultraviolet region. energies associated with stationary orbits of principal
(iii) (a) Absorption lines are obtained when an e - absorbs a quantum numbers n i and n f , respectively (where, n i > n f ).
photon in ground state and is excited to a higher state. 1 Ze 2
4. The velocity of electron, vn =
Initially e - is in ground state, so transition states are 1, 2, n 2he0
and 3 and wevelengths are l1 , l 2 and l 3.
Here, Z = 1, e = 1.6 ´ 10-19 C, e0 = 8.85 ´ 10-12 NC 2 m –2 ,
1 æ 1 1 ö 1
(iv) (c) = RZ 2 çç 2 - 2 ÷÷ Þ l µ 2 h = 6.62 ´ 10-34 J-s and n = 2 (in 1st excited state)
l è n1 n2 ø Z
We have, ZLi = 3, 1 ´ (1.6 ´ 10-19 ) 2
Þ v2 =
ZHe = 1 2 ´ 2 ´ (6.62 ´ 10-34 ) ´ (8.85 ´ 10-12 )
and ZH = 1. = 1.09 ´ 106 m/s
So, l Li + + : l He+ : l H = 4 : 9 : 36 n 2 h 2 e0
Radius of orbit, r2 =
(v) (a) Wavelength is given by pme 2
-31
1 æ 1 1 ö Here, m = 91. ´ 10 kg
= R çç 2 - 2 ÷÷
l n
è 1 n 2ø (2) 2 ´ (6.62 ´ 10-34 ) 2 ´ (8.85 ´ 10-12 )
Þ r2 =
1 é 1 1 ù 3.14 ´ (9.1 ´ 10-31 ) ´ (1.6 ´ 10-19 ) 2
Þ =Rê 2 -
l ë ( 2) ( 4) 2 úû . ´ 10-10 m
= 212
1 æ1 1 ö Time period or orbital period,
Þ =Rç - ÷
l è 4 16 ø 2pr2 2 ´ 314 . ´ 10-10
. ´ 212
T= = = 1.22 ´ 10-15 s
1 æ4 -1ö v2 1.09 ´ 106
Þ =Rç ÷
l è 16 ø 5. The total energy of the electron in the stationary states of
1 3R the hydrogen atom is given by
Þ =
l 16 me 4
16 En = - 2 2 2
Þ l= 8n e0 h
3R
where, signs are as usual and the m that occurs in the Bohr
formula is the reduced mass of electron and proton. Also, the
PART 2 total energy of the electron in the ground state of the
1. According to Rutherford’s experiment, following hydrogen atom is - 13.6 eV.For H-atom reduced mass m e,
observations were made whereas for positronium, the reduced mass is
me
(i) Most of the a-particles passed through the gold foil m»
without any appreciable deflection. 2
. % of incident a-particles scattered by more than
(ii) Only 014 Hence, the total energy of the electron in the ground state of
1°. But about 1 a-particle in every 8000 particles the positronium atom is
deflected by more than 90°. -13.6 eV
= - 6.8 eV
Thus, all these leads to the conclusion that atom has a lot of 2
empty space and practically the entire mass of the atom is 6. Given, total energy, E = - 3.36 eV
confined to an extremely small centered core called nucleus,
(i) Kinetic energy in the given state, K = - E = 3.36 eV
whose size is of the order from 10-15 m to 10-14 m.
(ii) Potential energy in the given state, U = 2E = - 6.72 eV
2. Kinetic energy of a-particle is given as (iii) The potential energy is zero at infinity. If the reference of
1 2 e × Ze potential energy is changed, the kinetic energy will
KE =
4pe0 d 2 remain constant but total energy will change.
7. The ionisation energy is qualitatively defined as the amount
where, d is the distance of closest approach. of energy required to remove the most loosely bound
2 Ze 2 2 Ze 2 electron, the valence electron of an isolated gaseous atom to
d2 = Þ d= form a cation. Since, total energy is directly proportional to
4pe0 KE 4pe0 KE
the mass of electron.
This is the required expression for the distance of closest
m e4
approach d in terms of kinetic energy (KE). E0 = 2 2 , i. e. E0 µ m
3. An atom can emit or absorb radiation in the form of discrete 8 e0 h
energy photons only when an electron jumps from a higher So, the ionisation energy becomes 200 times on replacing an
to a lower orbit or from a lower to a higher orbit, electron by a particle of mass 200 times of the electron and
respectively. The frequency of radiated wave is given by of same charge.
æ E f - Ei ö 8. From the nth state, the atom may go to ( n – 1 ) th state, … ,
hn = E f - Ei Þ n = çç ÷÷
è h ø 2nd state or 1st state. So, there are ( n – 1 ) possible
transitions starting from the nth state.
CBSE Term II Physics XII 107

The atoms reaching ( n – 1 )th state may make n – 2 different 1 æ1 1 ö


13. For Balmer series, =R ç 2 - 2÷ r
transitions. Similarly, for other lower states, the total lB è2 n ø
number of possible transitions is 4
n ( n – 1) For highest energy n ® ¥ Þ l B =
( n - 1 ) + ( n – 2) + ( n – 3) + L + 2 + 1 = R
2 1 R R
9. (i) Given, l=496 nm=496´10-9 m Þ = =
l B 22 4
hc 6.62 ´ 10-34 ´ 3 ´ 108
Q E= = J 1 æ1 1 ö
l 496 ´ 10-9 For Paschen series, = Rç 2 - 2 ÷
lP è3 n ø
6.6 ´ 10-34 ´ 3 ´ 108
= eV = 2.5 eV For highest energy,
496 ´ 10-9 ´ 1.6 ´ 10-19
9
– 0.85 eV
A B C
n=4 n ® ¥ Þ lP =
– 1.51 eV D E n=3 R
F
– 3.40 eV n=2 4 9
Þ lB : lP = : =4:9
R R
h h
– 13.6 eV n=1 14. We know that, de-Broglie wavelength, l = =
p mv
This energy corresponds to the transition h
B(n = 4 to n = 2) for which the energy change = 2 eV. Þ mv =
l
(ii) Energy of emitted photon is given by hr nh
hc Þ mvr = =
E= l 2l
l
2p 2pr
1 Þ l= ´ hr =
\ l max µ nh n
Emin
As, r µ n2
Transition A, for which the energy emission is minimum,
corresponds to the emission of radiation of maximum 1
Þ l µ ( n2 ) = n
wavelength. n
10. For Paschen series, n1 = 3 l 3 l
Thus, we can say that, 3 = Þ l1 = 3
1 æ 1 1 ö l1 1 3
Wave number, n = = R çç 2 - 2 ÷÷
l è n1 n2 ø Thus, wavelength decreases 3 times as an electron jumps
where, n 2 = 4, 5, 6, .... ¥ from third excited state to the ground state.
For shortest wavelength, n 2 = ¥ 15. According to Bohr’s quantisation condition electrons are
permitted to revolve in only those orbits in which the
1 æ1 1 ö
\ = Rç - ÷ angular momentum of electron is an integral multiple of
h
.
l¥ è9 ¥ø 2p
1 R nh
Þ = i.e. mvr = , where n = 1, 2, 3, ... , m, v, r are mass, speed
l¥ 9 2p
9 9 and radius of electron and h being Planck’s constant.
Þ l¥ = =
R 1.097 ´ 107 1 æ1 1 ö
For Brackett-series, = R ç 2 - 2 ÷ , where n = 5, 6, 7, K
= 82041. ´ 10-10 m = 82041. Å l è4 n ø
For shortest wavelength, n = 5
11. Lyman series, n = 2, 3, 4,… to n = 1
1 æ1 1 ö
For short wavelength, n = ¥ to n = 1 Þ = 1.097 ´ 107 ç - ÷
l è 16 25 ø
12375 12375 9
Energy, E = = eV = 13.54 eV 7
= 1.097 ´ 10 ´ = 0.0246 ´ 107
l(Å) 913.4 16 ´ 25
Also, energy of nth orbit, E = 13.54 / n 2 Þ l = 40.514 ´ 10-7 ~ - 4051 nm
So, energy of n = 1 , energy level = 13.54 eV It lies in infrared region of electromagnetic spectrum.
Energy of n = 2 , energy level = 13.54 / 22 = 3. 387 eV
12375 16. Bohr’s second postulate (quantum condition) states that, the
So, short wavelength of Balmer series = = 3653 Å electron revolves around the nucleus in certain privileged
3. 387 orbit which satisfy certain quantum condition that angular
12. The energy level of each atom are definite but different momentum of an electron is an integral multiple of h/ 2p,
from the energy levels of other elements. where h is Planck’s constant.
Therefore, the spectrum of the emission lines by atom of an i.e. L = mvr = nh / 2p
element has lines of certain different frequencies which are where, m = mass of electron, v = speed of electron and
different from those of all other elements. r = radius of orbit of electron.
And this property helps us to distinguish different elements Þ 2pr = n ( h / mv)
from each other.
108 CBSE Term II Physics XII

Since, de-Broglie wavelength associated with an electron is The total energy of the electron is negative. This implies the
given as fact that, the electron is bound to the nucleus. If E were
h h positive, an electron will not follow a closed orbit around the
l= =
p mv nucleus and it would leave the atom.
20. When an a-particle is bombarded over a gold nucleus, it is
\Circumference of electron in nth orbit
repelled by electrostatic repulsion. As a result, KE of
= n ´ de-Broglie wavelength associated with electron a-particle is converted into electrostatic PE.
17. The angular momentum in planetary motion is very large as At a certain distance from the nucleus, whole of the KE of
compared to the angular momentum of electrons. So, it a-particle converts into electrostatic potential energy and
corresponds to a very large value of n. a-particles cannot go further close to nucleus, this distance
At such large quantum numbers, the quantum physics is is called distance of closest approach.
prevailed by classical physics and motion of planets around In this process, all the kinetic energies of moving particle is
the sun can be governed by Newton’s law of motion. converted into potential energies.
18. Given, atomic number, Z = 80, From the given data,
KE = K = 8 MeV = 8 ´ 106 ´ 1.6 ´10- 19 J 1 2 e ´ Ze
Initially, × =K …(i)
( Ze ) ( 2e ) 4pe0 r
Q Energy, K =
4p e0 r0 Let r0 be the new distance of closest approach for a twice
where, r0 = distance of closest approach. energetic a-particle, then we have
Þ r0 = 2 Ze 2 / 4p e0 ( K ) 1 2 e ´ Ze
´ = 2K …(ii)
4pe0 r0
2 ´ 9 ´ 109 ´ 80 ´ (1.6 ´ 10- 19 ) 2
= On dividing Eq. (i) by Eq. (ii), we get
8 ´ 106 ´ 1.6 ´ 10-19
r0 1 r
= 2.88 ´ 10- 14 m = Þ r0 =
r 2 2
As, r0 µ ( 1 / K )
If KE gets doubled, distance of closest approach reduces to Limitations of Rutherford Nuclear Model
half. Limitations of Rutherford nuclear model are as given below
19. The Rutherford nuclear model of the atom describes the (i) Rutherford’s model of an atom could not explain the
atom as an electrically neutral sphere consisting of a very stability of an atom.
small, massive and positively charged nucleus at the centre According to him, charged electrons revolve around atom
surrounded by the revolving electrons in their respective in circular paths, so it should experience acceleration due
dynamically stable orbits. The electrostatic force of to which it should lose energy continuously in the form of
electromagnetic radiations. It then eventually fall into
attraction Fe between the revolving electrons and the
the nucleus thereby making the atom unstable.
nucleus provides the requisite centripetal force Fc to keep
them in their orbits. Thus, for a dynamically stable orbit in a (ii) Rutherford’s model of an atom could not explain as how
the electrons are arranged in the orbits around the
hydrogen atom
nucleus.
Fc = Fe 21. Given, work function, f = 2 eV
mv2 1 e2 1 æ 1 1 ö
= × 2 (Q Z = 1 ) = R çç 2 - 2 ÷÷
r 4pe0 r l n
è 1 n 2ø
Thus, the relation between the orbit radius and the electron hc æ 1 1 ö
velocity is E = = hcR çç 2 - 2 ÷÷ = f + KE
l è n1 n2 ø
e2
r= Also, KE = eV0
4pe0 mv2
n1 = 2, n 2 = n
The kinetic energy K and electrostatic potential energy U of æ1 1 ö
hcR ç - 2 ÷ = 2 ´ 1.6 ´ 10-19 + 1.6 ´ 10-19 ´ 055 .
the electron in hydrogen atom are è4 n ø
1 e2 æ1 1 ö
K = mv2 = Þ 6.62 ´ 10-34 ´ 3 ´108 ´ 1.097 ´ 107 ç - 2 ÷
2 8pe0 r è4 n ø
e2 = ( 3. 2 + 0.88) ´ 10-19
and U =-
æ1 1 ö
4pe0 r Þ 21.786 ´ 10-19 ç - 2 ÷ = 4.08 ´ 10-19
è 4 n ø
(The negative sign in U signifies that the electrostatic force is
1 1
attractive in nature.) - 2 = 0187
. Þ n~ -4
4 n
Thus, the total mechanical energy of the electron in a
22. Given, DE =12.5 eV
hydrogen atom,
Energy of an electron in nth orbit of hydrogen atom,
e2 e2 e2
E = K+U = - =- 13.6
8pe0 r 4pe0 r 8pe0 r En = - 2 eV
n
CBSE Term II Physics XII 109

In ground state, n = 1 Þ E1 = -13.6 eV From n 2 ® n1,


Energy of an electron in the excited state after absorbing a hc
l2 =
photon of 12.5 eV energy will be E2 - E1
En = -13.6 +12.5 = -1.1 eV hc hc
-13.6 -13.6 = =
\ n2 = = = 12.36 ( - 3.4) -( - 13.6) 10.20
En -1.1 l1 10.20
\ = = 5.3
Þ n = 3.5 l2 1.9
Here, state of electron cannot be fraction. 25. The frequency of any line in a series in the spectrum of
So, n = 3 (2 nd excited state). hydrogen like atoms corresponding to the transition of
1 æ 1 1 ö electrons from ( n + p) level to nth level can be expressed as
Lyman Series = R ç 2 - 2 ÷ a difference of two terms
l è 1 n ø
é 1 1 ù
For first member, n = 2 vmn = cRZ 2 ê 2
- 2ú
1 æ1 1 ö æ4 - 1ö ë ( n + p) n û
\ = R ç 2 - 2 ÷ = 1 . 097 ´ 107 ç ÷
l1 è1 2 ø è 4 ø where, m = n + p, (p = 1, 2, 3, K) and R is Rydberg constant.
Þ l1 = 1.215 ´ 10 -7
m For p << n,
é -2
1 æ1 1 ö 1 æ pö 1 ù
Balmer Series = R ç 2 - 2 ÷ vmn = cRZ 2 ê 2 ç1 + ÷ - 2 ú
l è2 n ø êë n è nø n úû
For first member, n = 3 æ 1 2p 1 ö
vmn = cRZ 2 ç 2 - 3 - 2 ÷
1 æ1 1 ö èn n n ø
\ =R ç 2 - 2÷
l1 è2 3 ø [by binomial theorem ( 1 + x ) n = 1 + nx, if|x| < 1]
æ1 1 ö
= 1 . 097 ´ 107 ç - ÷ 2p æ 2cRZ 2 ö
è 4 9ø vmn = cRZ 2 ç ÷
n3
~ ç n3 ÷ p
Þ l1 = 6.56 ´ 10-7 m è ø
Thus, the first few frequencies of light that is emitted when
23. According to Bohr’s theory of hydrogen atom, energy of
electrons fall to the nth level from levels higher than n , are
photon released, E2 - E1 = hn
approximate harmonic (i.e. in the ratio 1 : 2 : 3 …) when
Given, E1 = - 1 .15 eV n >> 1.
E2 = - 0.85 eV 26. Bohr combined classical and early quantum concepts and
E2 - E1 = - 0.85 - ( - 1.51) = 1 .51 - 0.85 gave his theory in the form of three postulates
E2 - E1 = 0.66 eV These three postulates are as follows
So, the wavelength of emitted spectral line, (i) Bohr’s first postulate states that, an electron in an atom
1242 eV - nm 1242 eV - nm could revolve in certain stable orbits without the
l= = = 1. 88 ´ 10-6 m emission of radiant energy, contrary to the predictions of
E (in eV) 0.66 eV
electromagnetic theory. According to this postulate, each
As here, l = 1.88 ´ 10-6 m » 18751 ´ 10-10 m atom has certain definite stable states in which it can
exist and each possible state has definite total energy.
The wavelength belongs to Paschen series of hydrogen These are called the stationary states of the atom.
spectrum.
(ii) Bohr’s second postulate states that, the electron revolves
24. (i) An electron undergoes transition from second excited
around the nucleus only in those orbits for which the
state to the first excited state which corresponds to
angular momentum is some integral multiple of h/ 2p,
Balmer series and then to the ground state which
corresponds to Lyman series. where h is the Planck’s constant ( = 6.63 ´ 10-34 J- s).
(ii) The wavelength of the emitted radiations in the two Thus, the angular momentum L of the orbiting
cases. electron is quantised.
nh
n=3 – 1.5 eV i.e. L=
Balmer series 2p
n=2 – 3.4 eV
As, angular momentum of electron = mvr
Lyman series
n=1 –13.6 eV \ For any permitted (stationary) orbit,
nh
hc mvr =
We know that, l = 2p
DE where, n = any positive integer (1, 2, 3, ....).
From n 3 ® n 2 ,
It is also called principal quantum number.
hc
l1 = In second excited state, i.e. n = 3, three spectral lines
E3 - E2 namely Lyman series and Balmer series can be obtained
hc hc corresponding to transition of electron from n = 3 to n = 1
= =
( -1.5) - ( -3.4) 1.9 and n = 3 to n = 2, respectively and n = 2 to n = 1.
110 CBSE Term II Physics XII

For Lyman series (minimum wavelength) n = 3 to n = 1, So, we can say that for large values of n, the classical
1 æ1 1 ö 8 frequency of revolution of electron in nth orbit is same as
=R ç 2 - 2÷ = R ...(i)
l min è 1 3 ø 9 the frequency of radiation emitted when hydrogen atom
For Balmer series (maximum wavelength), n = 3 to n = 2, de-excites from level n to level ( n -1 ).
1 æ1 1 ö 28. (i) For longest wavelength of Lyman series, n i= 2
= Rç 2 - 2 ÷
l max è2 3 ø 1 æ1 1 ö 3R
= Rç 2 - 2 ÷ =
æ1 1 ö 5 l max è1 2 ø 4
=Rç - ÷= R
è 4 9 ø 36 4 4
l max = =
1 5R 3R 3 ´ 1.097 ´ 107
Þ = …(ii)
l max 36 = 1.215´10-7 m
Dividing Eq. (i) by Eq. (ii), we get l max =1215 Å
8R
This wavelength lies in ultraviolet region.
l max 8R 36 32
= 9 = ´ = (ii) Energy in second excited state, E2 = -
13.6
eV
l min 5 R 9 5R 5
( 3) 2
36
13.6
l max : l min = 32 : 5 =- = - 1.51eV
9
27. Let n be the frequency when a hydrogen atom jumps from
level n to ( n -1 ). Energy in ground state,
i.e. n1 = ( n - 1 ) E0 = - 13.6 eV
n2 = n DE = E2 - E0 = 1 .51 - ( - 13.6 )
Energy, E = hn = E2 - E1 = - 1 .51 + 13.6
1 mc 2 a 2 é 1 1 ù = 12.09 eV
Þ n= × ´ê 2
- 2ú 12375 12375
2 h ë ( n -1 ) n û Wavelength, l = Å= Å = 1023 Å
DE 12.09
æ 2pKe 2 ö 29. If radiation of wavelength l ejects photoelectrons of
ç where, a = ÷
ç ch ÷ø maximum kinetic energy EK from a metal surface, then
è
hc
mc 2 a 2 é n 2 - ( n - 1 ) 2 ù EK = - W, (Einstein’s equation)
= ê ú l
2h ë n 2 ( n - 1 ) 2 û
where, W is the work function of the surface.
mc 2 a 2 [( n + n - 1 ) ( n - n + 1 )] If V0 volt be the stopping potential, then EK = eV0 , where e is
=
2hn 2 ( n - 1 ) 2 electronic charge.
mc 2 a 2 ( 2n - 1 ) hc
= Then, eV0 = -W
2hn 2 ( n - 1 ) 2 l
hc
or l=
For large values of n, ( 2n - 1 » 2n ), ( n - 1 » n ), we have W + eV0
mc 2 a 2 2n mc 2 a 2 Here, W + eV0 = 1.7 eV + 10.4 eV = 12.1 eV
n= =
2hn 2 n 2 hn 3 = 12.1 ´ (1.6 ´ 10-19 )J
mc 2 4p 2 K 2 e 4 æ 2pKe 2 ö
= × çQ a = ÷ (6.63 ´ 10- 34 J-s) ´ (3.0 ´ 108 ms - 1 )
hn 3 2 2
c h ç cn ÷ø \ l=
è 12.1 ´ 1.6 ´ 10- 19 J
4p 2 K 2 me 4 = 1.027 ´ 10- 7 m = 1027Å
= …(i)
h 3n 3
The photon energy corresponding to l = 1027 Å is
hn
In Bohr’s atomic model, velocity of nth orbit, v = and 12375
2pmr = 121. eV
1027Å
n2h2
radius, r = 2 Now, the energy levels of hydrogen atom are given by
4p mKe 2 13.6
Thus, frequency of oscillation, En = - 2 eV, where n = 1, 2, 3, 4, ...
n
v nh æ 4p 2 mKe 2 ö This gives E1 = - 13.6 eV, E2 = - 3.4 eV,
n= = ç ÷
2pr 2pmr çè 2pn 2 h 2 ÷ø E3 = - 1.5 eV, E4 = - 0.85 eV , .....
Ke 2 Ke 2 æ 4p 2 mKe 2 ö \ E3 - E1 = - 1.5 eV - ( - 13.6 eV) = 12.1 eV .
= = ç ÷
nhr nh çè n 2 h 2 ÷ø Thus, the energy levels emitting 1027 Å radiation are
n = 3 and n = 1
4p 2 mK 2 e 4
= 30. (i) Let v1 be the orbital speed of the electron in a H-atom in
n 3h 3 the ground state level, n1 = 1. For charge e of an electron,
It is same as Eq. (i). v1 is given by the relation,
CBSE Term II Physics XII 111

e2 e2 31. The electrostatic force of attraction between positively


v1 = =
æ hö 2e0 h charged nucleus and negatively charged electrons
n1 4p e0 ç ÷ (Coulombian force) provides necessary centripetal force of
è 2p ø
revolution.
where, e = charge on an electron
mv2 e2 1
= 1.6 ´10-19 C =- 2×
e0 = permittivity of free space
rB rB 4pe0
= 8.85 ´ 10-12 N -1C 2 m -2 By Bohr’s postulates in ground state, we have
h
and h = Planck’s constant = 6.63 ´ 10-34 J-s. mvr = h Þ v =
mr
(1.6 ´10-19 ) 2
\ v1 = On solving, we get
2 ´ 8.85 ´10-12 ´ 6.63 ´10-34
h2 1 æ e2 ö 1
= 0.0218 ´108 \ m 2 2 × = + çç ÷ 2
÷
= 2 . 18 ´106 m/s m rB rB è 4pe0 ø rB
We know that, vn = v1 / n h 2 4pe0
\ × = rB = 0.51 Å (this is Bohr’s radius)
For level n 2 = 2, we can write the relation for the m e2
corresponding orbital speed as The potential energy is given by
v 2.18 ´ 106 æ e2 ö 1
v2 = 1 = - çç ÷ × = -27.2 eV ;
2 2 ÷
è 4pr0 ø rB
= 1.09 ´106 m/s
mv2
and for level n 3 = 3, we can write the relation for the KE =
corresponding orbital speed as 2
v 2.18 ´106 1 h2 h
v3 = 1 = KE = m × 2 2 = = + 13.6 eV
3 3 2 m rB 2mrB2
= 7.27 ´ 105 m/s Now, for an spherical nucleus of radius R,
Hence, the speed of the electron in a If R < rB, same result.
H-atom in n = 1, n = 2 and n = 3 is 2.18 ´ 106 m/s,
If R >> rB, the electron moves inside the sphere with radius
1.09 ´ 106 m/s and 7.27 ´105 m/s, respectively. rB¢ (here, r ¢B = new Bohr radius).
(ii) Let T1 be the orbital period of the electron and is given by æ r ¢3 ö æ 4pe h 2 ö
2pr Charge inside, e ¢ = e çç B3 ÷÷ ç rB¢ = 20 × ÷
ç
T= èR ø è e¢ m ÷ø
v n2h2e
where, r = radius of the orbit =
0
, h 2 æ 4pe0 ö R 6
\ r ¢B = ç ÷
pme 2 m è e 2 ø rB¢6
h = Planck’s constant = 6.63 ´10-34 J-s,
r ¢B7 = (0.51 Å) × R 6 (Q R = 10 Å)
e = charge on an electron = 1 .6 ´ 10-19 C,
\ . )1/ 7 R 6 / 7
rB¢ = ( 0514
e0 = permittivity of free space
= 8.85 ´ 10-12 N -1 C 2 m -2 If R = 01 . )1/ 7 ´ ( 01
. Å , rB¢ = ( 0514 . ) 6 / 7 = 0126
. Å>R
and m = mass of an electron = 9.1 ´ 10-31 kg . )1/ 7 ´ ( 10) 6 / 7 = 719
If R = 10 Å, rB¢ = ( 051 . Å<R
For n = 1, 1 m h h 1
KE = mv2 = × 2 2 = ×
2pr1 2 2 m r B¢ 2m r B¢2
T1 =
v1 æ h2 æ r2 ö
=ç × çç B2 ÷÷
2 ´ 3.14 ´ 0.53 ´ 10-10 ç 2mr 2
= è B è rB¢ ø
2.18 ´ 106
(0.51) 2
= 1.52 ´ 10-16 s = (13.6 eV)
(7.19)1/ 2
As, Tn = n 3 T1
3.54
Then, T2 = ( 2) 3 T1 = = 0.068 eV
22.6
= 8 ´ 1.52 ´ 10-16 æ e 2 ö æ r ¢2B - 3R 2 ö
= 1.22 ´ 10-15 s PE = + çç ÷×ç
÷ ç
÷
÷
è 4pe0 ø è 2R
3
ø
T3 = ( 3) 3 T1
æ e 2 1 ö é rB( r B¢2 - 3R 2 ) ù
= 27 ´ 1 . 52 ´ 10-16 = + çç × ÷÷ × ê ú
è 4pe0 rB ø ë R3 û
= 4.10 ´ 10-15 s
é 0.51{(7.19) - 300} ù
2
Then, the orbital period in each of these levels is = + (27.2 eV) ê ú
1.52 ´ 10-16 s, 1.22 ´10-15 s and 410
. ´ 10-15 s, ë 1000 û
respectively. = 3.44 eV
112 CBSE Term II Physics XII

32. The total energy of the electron in the stationary states of n2h2
r= …(iii)
the hydrogen atom is given by 4p 2 mke 2
me 4 Þ r µ n2
En = - 2 2 2
8n e0 h Now, kinetic energy of electron,
where, signs are as usual and the m that occurs in the Bohr KE = ( 1 / 2) mv2 = ke 2 / 2 r
formula is the reduced mass of electron and proton in
Also, potential energy, PE = - ke 2 / r
hydrogen atom.
By Bohr’s model, hn if = Eni - En f Energy of electron in nth orbit,
ke 2 ke 2 4p 2 mke 2
me 4 æ 1 1 ö En = - =- ×
On simplifying, n if = 2 3 ç 2 - 2 ÷ 2r 2 n2h2
ç
8e0 h è n f n i ÷ø
2p 2 mk2 e 4
1 Þ En = - …(iv)
Since, lµ n2h2
m
2p mk2 e 4
2
1 where, R=
Thus, l if µ …(i) ch 3
m
Rhc
where, m is the reduced mass. (here, m is used in place of m) Þ En = - 2 … (v)
n
me æ m ö
Reduced mass for H = m H = ; m eç1 - e ÷ where, n = 1, 2, 3, ¼
me è Mø
1+ 1
M For n = n i Þ En µ
n2
æ m ö
Reduced mass for D = m D ; m eç1 - e ÷ Rhc
è 2 Mø Eni = -
n i2
æ m e öæ me ö
= m e çç1 - ÷÷ çç1 + ÷ Rhc
è 2 M øè 2 M ÷ø and En f =- 2
nf
lH
If for hydrogen deuterium, the wavelength is . By Bohr’s postulates, En f - Eni = hn
lD
-1
But DE = hn
lD m H æ meö æ 1 ö æ 1
= ~ ç1 + ÷ ~ çç1 - ÷ [from Eq. (i)] 1 ö
lH lD è 2M ø è 2 ´ 1840 ÷ø n = Rc ç 2 - 2 ÷
ç n f ni ÷
è ø
l D = l H ´ (0.99973)
On substituting the values, we have me 4 æç 1 1 ö
or n = 2 3 2 - 2÷
Corresponding, lines are 1217.7 Å, 1027.7 Å, 974 Å, 951.143 Å. 8e0 h è n f n i ÷ø
ç
33. Let an electron revolves around the nucleus of hydrogen When electron in hydrogen atom jumps from energy state
atom. The necessary centripetal force is provided by n i= 4 to n f = 3, 2, 1, the Paschen, Balmer and Lyman
electrostatic force of attraction. spectral series are found as shown below
Total energy, E(eV)
Series limit n=¥
+Ze r
0
n=5 Excited state
e –, m –1.0 Pfund series n = 4
Nucleus n=3 Brackett series
–2.0 n=3
Paschen series
–3.0
–3.4 n = 2 n=2
–4.0 Balmer series
mv2 ke 2 ke 2
\ = 2 Þ r= …(i) –5.0
r r mv2
where, m is mass of electron and v is speed of a circular path –6.0
of radius r.
Energy (eV)

–7.0
By Bohr’s second postulates,
mvr = nh / 2p, where n = 1, 2, 3, ¼ –8.0
r = nh / 2pmv …(ii)
–9.0
On comparing Eqs. (i) and (ii), we get
–10.0
ke 2 nh
= –11.0
mv2 2pmv
–12.0
2pke 2
Þ v= –13.0 Ground state
nh –13.6 n=1
–14.0 Lyman series
Substituting in Eq. (ii), we get
CBSE Term II Physics XII 113

34. According to the question, 2


= 13.6Z 2 ´
n=n n=n 9
10.2 eV 4.25 eV Z2 = 9 Þ Z=3
35. (i) As, a result to transition, electron goes from excited state
17 eV 5.95 eV to ground state.
Energy of radiation = - 1.5 - ( -13.6) = 12.1 eV
n=2 n=3
(a) (b) (Q ground state energy of H-atom = - 13.6 eV)
c
We know that, energy of electron in nth orbit is given by Since, E = hn = h
æ 1 l
1 ö
DE = 13.6Z 2 çç 2 - 2 ÷÷ hc
n
è 1 n 2ø Þ = 121. eV
l
Case I Energy emitted by photon, DE = 10.20 + 17
1 . ´ 1.6 ´ 10-19
121
= 27.2 eV \ =
l 6.62 ´ 10-34 ´ 3 ´ 108
æ1 1 ö
So, 10.2 + 17 = 13.6Z 2 ç 2 - 2 ÷ (Q n1 = 2, n 2 = n ) Þ l = 1.025 ´ 10-7 m
è2 n ø
= 1025 Å
æ1 1 ö
27.2 = 13.6Z 2 ç - 2 ÷ …(i) (ii) Every atom has different energy states. In the ground
è4 n ø state, the electron occupies lowest energy level. When an
Case II Energy emitted by photons, DE = 4.25 + 5.95 atom is excited by means of some external energy from
outside, then the electron jumps to a higher energy level.
= 10.2 eV
Within nano-seconds (10-9 s), the electron jumps back
æ1 1 ö
4.25 ´ 5.95 = 13.6Z 2 ç 2 - 2 ÷ (Q n1 = 3, n 2 = n ) from higher energy level to lower energy level. This
è 3 n ø process can take in either one step (i.e from highest
2 æ1 1 ö possible energy level to ground state) or in multiple state
10.2 = 13.6Z ç - 2 ÷ …(ii)
è9 n ø by passing through other lower energy levels.
On dividing Eq. (i) by Eq. (ii), we get There are countless atoms in a light-source, hence all
possible transitions can take place and many lines are
27.2 1 / 4 - 1 / n 2
= observed in the spectrum.
10.2 1 / 9 - 1 / n 2 (iii) According to question,
27.2 27.2 10.2 10.2 hc
Þ - 2 = - 2 EB - EC = …(i)
9 n 4 n l1
17.2 27.9 10.2 hc
Þ = - EA - EB = …(ii)
n2 9 4 l2
- hc
Þ n 2 = 36 Þ n=6 EC -`EA = …(iii)
l3
Substituting the value of n in Eq. (i), we get
On adding Eqs. (i), (ii) and (iii), we get
æ1 1 ö EB - EC + EA - EA - EB + EC - EA
27.2 = 13.6Z 2 ç - 2 ÷
è 4 n ø æ1 1 1 ö
= hc çç + - ÷÷
æ1 1 ö è l1 l 2 l 3 ø
= 13.6Z 2 ç - 2 ÷
è4 6 ø 1 1 1
Þ = +
æ1 1 ö
2 l 3 l1 l 2
= 13.6Z ç - ÷
è 4 36 ø ll
Þ l3 = 1 2
æ 9 -1 ö l1 + l 2
= 13.6Z 2 ç ÷
è 36 ø This is the required expression.
114 CBSE Term II Physics XII

CHAPTER 06

Nuclei
In this Chapter...
l Nucleus and its Composition l Nuclear Force
l Mass-Energy and Nuclear l Nuclear Energy
Binding Energy

Nucleus and Its Composition Mass Number ( A)


Mass number of an element is the total number of protons
In every atom, the positive charge and mass are densely
and neutrons inside the nucleus of an element.
concentrated at the centre of the atom forming its nucleus.
The radius of the nucleus is smaller than the radius of an Mass number, A = Number of protons, Z
atom by a factor of 10 4 . This means the volume of a nucleus + Number of neutrons, N
is about 10 - 12 times the volume of the atom. The term nucleon is also used for neutron and proton.
A
Composition of Nucleus Nuclear species or nuclides are shown by the notation Z X,
The nucleus was first discovered in 1911 by Lord Rutherford where X is the chemical symbol of the species.
and his associates by experiments on scattering of a-particles
by atoms. The result of scattering experiment concluded that Size of Nucleus
atoms consists of a small, central, massive and positive core The volume of the nucleus is directly proportional to the
surrounded by orbiting electrons. number of nucleons (mass number).
The positive charge in the nucleus is due to the protons. A If R is the radius of the nucleus having mass number A, then
proton carries one unit of fundamental charge. 4
pR 3 µ A
Atomic Mass Unit 3
The mass of an atom is very small. Kilogram cannot be used Þ R µ A1 / 3
to measure such small quantity of mass. It is measured by a R = R 0 A1 / 3
unit called atomic mass unit (amu, i.e. u).
It is defined as where, R 0 = 1. 2´ 10 -15 m is the range of nuclear size.
mass of one 12 C atom It is also known as nuclear unit radius.
1u = = 1.660539 ´ 10 27 kg
12
Nuclear Density
Atomic masses are measured by an instrument called mass
Density of nuclear matter is the ratio of mass of nucleus and
spectrometer. its volume.
Atomic Number (Z ) 3m
\ r=
Atomic number of an element is the number of protons 4 p R 03
present inside the nucleus of an atom of the element. Thus, the density of nucleus is a constant, independent of A,
Atomic number, Z = Number of protons for all nuclei. The density of nuclear matter is approximately
= Number of electrons (in a neutral atom) 2.3 ´ 10 17 kg/m 3 .
CBSE Term II Physics XII 115

Mass-Energy and Nuclear Nuclear Energy


Binding Energy Nuclear energy is the energy released during the
transformation of nuclei with less total binding energy to
Mass-Energy nuclei with greater binding energy.
Einstein’s mass-energy equivalence equation is E = mc 2 ,
Two distinct ways of obtaining energy from nucleus are as
where E is the energy, m is the mass and c is the velocity of
follows
light in vacuum (approximately equal to 3 ´ 10 8 m/s).
(i) Nuclear fission (ii) Nuclear fusion
Nuclear Binding Energy
Nuclear Fission
The sum of the masses of neutrons and protons forming a
nucleus is more than the actual mass of the nucleus. This Nuclear fission is the phenomenon of splitting of a heavy
difference of masses is known as mass defect. nucleus (usually A > 230) into two or more lighter nuclei by
\ E = Dmc 2 ( Dm = mass defect) the bombardment of proton, neutron, a-particle, etc.
e.g. 235 1 141 92 1
92 U + 0 n ¾® 56 Ba + 36 Kr + 3 0 n + Q
E b = [ Zm p + ( A - Z )m n - M ] c 2
(Energy released)
where, M is mass of nucleus, m p is the mass of proton and
m n is the mass of neutron. The Q-value is equal to the difference of mass of products
and reactants multiplied by square of velocity of light.
‘‘The binding energy E b of a nucleus is defined as the Energy released per fission of 235
92 U is approximately
minimum energy required to separate its nucleons and place 200.4 MeV.
them at rest and infinite distance apart’’.
Nuclear Chain Reaction
Nuclear Force In the nuclear fission reaction, there is a release of extra
neutrons. The extra neutrons in turn initiate fission process,
Nuclear forces are the strongest attractive forces between
producing still more neutrons and so on. This is called
nucleons. They are non-conservative forces and do not obey
nuclear chain reaction. It is of two types
inverse square law. They are also non-central force.
Uncontrolled Chain Reaction During fission reaction,
Potential energy (MeV)

neutrons released are again absorbed by the fissile isotopes,


this cycle repeats to give a chain reaction, i.e. self-sustaining
100 and produces energy at a rate that increases rapidly with time
0 leading to large amount of radiation. This is called
uncontrolled chain reaction.
Controlled Chain Reaction The reaction is controlled in
–100
r0 1 2 3 such a way that, only one of the neutrons emitted in a fission
r (fm) causes another fission, then the fission rate remains constant
Potential energy versus distance between a pair of nucleon and the energy is released steadily. It is used in a nuclear
reactor.
Some of the important characteristics of these forces are as
given below Nuclear Reactor
(i) Nuclear forces among a pair of neutrons, a pair of It is a device that can initiate a self-sustaining controlled
protons and also between a neutron-proton pair, is chain reaction of a fissionable material. They are used at
approximately the same. This shows that, nuclear nuclear power plants for generating electricity and in
forces are independent of charge. propulsion of ships.
(ii) The nuclear forces are very short range forces. They Control rods Coolant Superheated steam
are operative upto distances of the order of a few fermi.
Shielding
(iii) The nuclear force is much stronger than the coulomb Electric
force acting between charges or gravitational forces Steam generator
turbine
acting between masses.
Heat
(iv) Nuclear force between two nucleons falls rapidly to exchanger
zero as their distance is more than a few femtometres Water
(fm). This leads to saturation of forces in a medium or condenser Used
large sized nucleus, i.e. each nucleon interacts with its steam
Fission Fuel
immediate neighbours only, rather than with all the chamber rods
Moderator
other nucleons in the nucleus. Water
Pump Cold water
(v) The nuclear forces are dependent on spin or angular
momentum of nuclei. Nuclear reactor
116 CBSE Term II Physics XII

Construction The fusion reaction in the sun is a multi-step process in


The key components of nuclear reactor are as follows which the hydrogen is fused into helium. (The proton-proton
(p, p) cycle) is given as
(i) Nuclear Fuel It is a material that can be used in
1 1 2 +
nuclear fission to derive nuclear energy. The common 1 H + 1 H ¾® 1 H + e + n + 0.42 MeV ...(i)
fuels used in nuclear reactor are 233 U, 235 U, 239 Pu, etc. e + + e - ¾® g + g + 1.02 MeV ...(ii)
(ii) Nuclear Reactor Core It is the portion of a nuclear 2 1 3
reactor containing the nuclear fuel components, 1H + 1H ¾® 2 He + g + 5 .49 MeV ...(iii)
3 3 4 1 1
2 He + 2 He ¾® He + 1 H +1 H + 12 .86 MeV ...(iv)
where the nuclear reaction takes place. 2
(iii) Moderator It is a medium to slow down the fast For the fourth reaction to occur, the first three reactions
moving secondary neutrons produced during the must occur twice, in this case two light helium nuclei unite to
fission. Heavy water, graphite, deuterium, paraffins, form ordinary helium nucleus.
etc., acts as moderator.
Thus, four hydrogen atoms combine to form 42 He atom
(iv) Control Rods It is used in nuclear reactors to control
the rate of fission of uranium and plutonium. These releasing 26.7 MeV energy.
are made of chemical elements capable of absorbing Distinction between Nuclear
many neutrons without fissioning themselves such as
silver, indium, boron and cadmium. Fission and Nuclear Fusion
(v) Coolant It is a liquid used to remove heat from Distinction between nuclear fission and nuclear fusion are as
nuclear reactor core and transfer it to electrical given below
generator and environment. Ordinary water under (i) Fission is the splitting of large nucleus into two or
high pressure can be used as coolant. more smaller ones. On the other hand, fusion is the
(vi) Shielding It is the protective covering made of combining of two or more lighter nuclei to form larger
concrete wall to protect from harmful radiations. one.
(ii) Fission does not normally occur in nature but fusion
Nuclear Fusion occurs in stars such as the sun.
Nuclear fusion is the phenomenon of fusing two or more (iii) Fission requires critical mass of the substance and
lighter nuclei forming a single heavy nucleus. high speed neutrons but in fusion, high density and
1 1 2 + high temperature environment are required.
e.g. 1 H + 1 H ¾® 1 H + 1 e + n + 0.42 MeV (iv) In fission, energy released is million times greater
2
1H + 1 H 2 ¾® 2 He 3 + 0 n1 + 3.27 MeV than in chemical reactions, but lower than energy
2 released by nuclear fusion.
1H + 1 H 2 ¾® 1 H 3 + 1 H1 + 4.03 MeV
(v) Uranium is the primary fuel for fission reaction and
Fusion of hydrogen nuclei into helium nuclei is the source of hydrogen isotopes are the primary fuel in nuclear
energy of most of the stars including the sun. fusion reaction.
CBSE Term II Physics XII 117

Solved Examples
Example 1. Two stable isotopes of lithium 63 Li and 73 Li Sol. Here in order to remove a neutron, energy has to be
supplied.
have respective abundance of 7.5% and 92.5%. Mass defect, Dm = M( 39 1 40
20 Ca ) + M( 0 n ) - M( 20Ca )
These isotopes have masses 6.01512 u and
7.01600 u, respectively. Find the atomic mass of = 38.970691 + 1.008665 - 39.962589
lithium. [NCERT] = 0.016767 amu
Equivalent energy = 0.06767 ´ 931 (Q 1 amu = 931 MeV )
Sol. Given, abundance per cent of 6 Li = 7.5%
= 15.6 MeV
Abundance per cent of 7 Li = 92.5%
Atomic mass of 6 Li = 6.01512 u Example 5. A neutron breaks into a proton and
Atomic mass of 7 Li = 7.01600 u electron. Calculate the energy produced in this
Atomic mass = Weighed average of the isotopes reaction. (Take, m e = 9 ´ 10 -31 kg,
6.01512 ´ 75 . + 7.01600 ´ 925. m p = 1.6725 ´ 10 -27 kg, m n = 1.6747 ´ 10 -27 kg and
=
75. + 925. c = 3 ´ 10 8 m/s)
45 .1134 + 648.98
= = 6.941 u Sol. We have 10 n ® 11 p + -10 e
100
Mass defect, Dm = m n - m p - m e
Example 2. Boron has two stable isotopes 10
5 B = [1.6747 ´ 10-27 - 1.6725 ´ 10-27 - 0.0009 ´ 10-27 ]
and 11
5 B. Their respective masses are 10.01294 u = 1.3 ´ 10-30 kg
and 11.00931 u and the atomic mass of boron is Energy produced = Dmc 2 = 1.3 ´ 10-30 ´ ( 3 ´ 108 ) 2
10.811 u. Find the abundances of 10 11
5 B and 5 B. 11.7 ´ 10-14
= 11.7 ´ 10-14 J = = 0.73 MeV
[NCERT] 1.6 ´ 10-13
10
Sol. Given, mass of B = 10.01294 u (Q 1 MeV =1.6 ´ 10-13 J)
Mass of 11B = 11.00931 u Example 6. Obtain the binding energy (in MeV) of a
Atomic mass of boron = 10.811 u nitrogen nucleus ( 147 N), given m( 147 N) = 14.00307 u.
Let the abundance of 10 B be x%, so the abundance of 11B be
(Take, m p = 1.00783 u and m n = 1.00867 u)
( 100 - x )%. [NCERT]
Atomic mass = Weighted average of the isotopes Sol. Given, mass of proton, m p = 1.00783,
x ´ 10.01294 + ( 100 - x ) ´ 11.00931 Mass of neutron, m n = 1.00867 u
10.811 = 14
( x + 100 - x ) 7 N nucleus contains 7 protons and 7 neutrons.
Abundance of 10 B, x = 19.9% Mass defect ( Dm ) = Mass of nucleons – Mass of nucleus
Abundance of 11B,( 100 - x ) = 100 - 19.9 = 80.1% = 7m p + 7m n - m N
= 7 ´ 1.00783 + 7 ´ 1.00867 - 14.00307
Thus, the abundance of 10 B is 19.9% and the abundance of
11
B is 80.1%. = 7.05481 + 7.06069 - 14.00307
= 0.11243 u
Example 3. Select the pairs of isotones from the Binding energy of nitrogen nucleus = Dm ´ 931 MeV
24
following nuclei 12 Mg , 1 H 3 , 2 He 4 , 11 Na 23 . = 0.11243 ´ 931 MeV = 104.67 MeV
Sol. Isotones have same number of neutrons, N = A - Z Thus, the binding energy is 104.67 MeV.
(i) 1H3 and 2 He4 Example 7. Consider the fission of 238
92 U by fast
Number of neutrons = 3 - 1 or 4 - 2 = 2
(i) 12 Mg24 and 11Na 23 neutrons. In one fission event, no neutrons are
emitted and the final end products, after the beta
Number of neutrons = 24 - 12 or 23 - 11 = 12
decay of the primary fragments, are 140
58 Ce and
Example 4. Calculate the energy required to remove 99
44 Ru.Calculate Q for this fission process. The
the least tightly bound neutron in 20 Ca 40 .
relevant atomic and particle masses are
Given that,
Mass of 20 Ca 40 = 39.962589 amu, m( 238
92 U) = 238.05079 u,

Mass of 20 Ca 39 = 38.970691 amu m( 140


58 Ce ) = 139.90543 u

and mass of neutron = 1.008665. and m( 99


44 Ru ) = 98.90594 u. [NCERT]
118 CBSE Term II Physics XII

Sol. The fission reaction is given by 3.017 and 1.009, respectively. If 1kg of deuterium
238 1 140 99
92 U + 0 n ¾® 58 Ce + 44Ru + Q undergoes complete fusion, then find the amount of
Mass defect, total energy released.
Dm = m( 238
92 U ) + m( 0 n1 ) - m( 140 99
58 Ce) - m( 44Ru )
(Take, 1 amu = 931.5 MeV/c 2 )
Sol. Given, fusion reaction, 1 H2 + 1 H2 ® 2 He3 + 0 n1
= 238.05079 + 1.00867 - 139.90543 - 98.90594
= 0.24809 u Mass defect, Dm = 2 ´ m(1 H2 ) - [ m( 2 He4 ) + Mn ]
Q-value for the given decay process, = [(2.015 ´ 2) - (3.017 + 1.009)]
Q = Dm ´ 931.5 = 4 ´ 10-3 amu
= 0.24809 ´ 931.5 = 231.1 MeV Equivalent energy, E = Dmc 2 = Dm ´ 931.5 MeV / c 2
Example 8. Consider the D-T reaction = 4 ´ 10-3 ´ 931.5 = 3.726 MeV
(deuterium-tritium fusion) Number of atoms in 1 kg of 1 H2
2 3 4
1 H + 1 H ¾® 2 He + n
Given mass
= ´ Avogadro number
Atomic mass
(a) Calcualte the energy released in MeV in this 1000
reaction from the data = ´ 6.023 ´ 1023
2
m( 12 H) = 2.014120 u In one reaction, two atoms of 12 H are used.
and m( 13 H) = 3.016049 u. So, total energy released
1 æ 1000 ö
(b) Consider the radius of both deuterium and = ç ´ 6.023 ´ 1023 ÷ ´ 3.726 ´ 1.6 ´ 10-13
2è 2 ø
tritium to be approximately 2.0 fm. What is the
kinetic energy needed to overcome the coulomb ~- 9 ´ 1013 J
repulsion between two nuclei? To what Example 10. In a nuclear reactor, U 235 undergoes
temperature must the gas be heated to initiate fission releasing energy of 100 MeV. The reactor
the reaction? [NCERT] has 20% efficiency and the power produces is
Sol. (a) The D-T reaction is given by 2000 MW. If the reactor is to function for 5 yr, find
2 3 4 1
1 H + 1 H ¾® 2 He + 0 n + Q the total mass of uranium required.
Mass defect, Dm = m( 12H) + m( 13H) - m( 42He) - m( 0 n1 ) Sol. Given, output power, Po = 2000MW = 2 ´ 109 W
= 2.014102 + 3.016049 – 4.002603 – 1.00867 Output power, Po
Efficiency, h =
= 0.018878 u Input power, Pi
Q-value for the given decay process 20 2 ´ 109
= Dm ´ 931 = 0.018878 ´ 931 = 17.58 MeV Þ = Þ Pi = 1010 W
100 Pi
(b) Repulsive potential energy of two nuclei, when they
almost touch each other Consider the mass of uranium be m.
1 q 2 9 ´ 109 ( 1.6 ´ 10-19 ) 2 Number of fissions = Number of atoms
U= × = = 5.76 ´ 10-14 J
4pe0 2r 2 ´ 2 ´ 10-15 =
Given mass
´ Avogadro number
Also we know that, kinetic energy required for one fusion Atomic mass
m m
event = average thermal kinetic energy available with = ´ NA = ´ 6.023 ´ 1023
the interacting particles A 235
3 Energy produced
Kinetic energy = ´ kT ´ 2 (two nuclei)
2 = Number of fissions ´ Energy per fission
= 3kT m
= ´ 6.023 ´ 1023 ´ 200 MeV
Kinetic energy 235
T= m
3k = ´ 6.023 ´ 1023 ´ 100 ´ 1.6 ´ 10-13 J
235
5.76 ´ 10-14 (Q 1 MeV = 1.6 ´ 10-13 J )
=
3 ´ 1.38 ´ 10-23 = m ´ 0.041 ´ 1012 J
9
= 1 .39 ´ 10 K = m ´ 4.1 ´ 1010 J
This temperature cannot be achieved in actual Energy
behaviour. Power =
Time
Example 9. In the fusion reaction, m ´ 4.1 ´ 1010
Þ 1010 =
1
H 2 + 1 H 2 ® 2 He 3 + 0 n 1 , the masses of deuteron, 5 ´ 365 ´ 24 ´ 3600
helium and neutron expressed in amu are 2.015, m = 3.84 ´ 107 g = 38.4 ´ 103 kg
CBSE Term II Physics XII 119

Chapter
Practice
PART 1 7. The gravitational force between a H-atom and
another particle of mass m will be given by
Objective Questions Newton’s law F = G
M×m
, where r is in km.
r2
1. As compared to 12 C atom, 14 C atom has What is M in this formula?
(a) two extra protons and two extra electrons (a) Gravitational mass of H-atom
(b) two extra protons but no extra electrons (b) Effective mass of H-atom
(c) two extra neutrons but no extra electrons (c) Nuclear mass of H-atom
(d) two extra neutrons and two extra electrons (d) Mass of electrons in H-atom
2. Density of a nucleus is 8. Nuclear force is
(a) more for lighter elements and less for heavier elements (a) strong, short range and charge independent force
(b) more for heavier elements and less for lighter elements (b) charge independent, attractive and long range force
(c) very less compared to ordinary matter (c) strong, charge dependent and short range attractive force
(d) a constant (d) long range, charge dependent and attractive force

3. The nuclear radius of a certain nucleus is 7.2 fm 9. Fpp, Fnn and Fnp are the nuclear forces between
-17
and it has charge of 1.28 ´ 10 C. The number of proton-proton, neutron-neutron and
neutrons inside the nucleus is neutron-proton, respectively. Then, relation
(a) 136 (b) 142
between them is
(a) Fpp = Fnn ¹ Fnp (b) Fpp ¹ Fnn = Fnp
(c) 140 (d) 132
(c) Fpp = Fnn = Fnp (d) Fpp ¹ Fnn ¹ Fnp
40
4. The ratio of mass densities of nuclei of Ca and
16 10. Which amongst the following is a correct graph of
O is close to
potential energy U of a pair of nucleons as a
(a) 5 (b) 2
function of their separation r ?
(c) 0.1 (d) 1
U U
5. Mass defect of helium ( 2 He 4 ) is (Take, mass of proton,
m p = 1.0072676 u, mass of neutrons, m n = 1.008665 u
(a) (b)
and mass of 2 He 4 = 4.001506 u) [All India 2020]
r0 r0
r
0 r 0
(a) 0.016767 u
(b) 1.00726 u
U U
(c) 2.00686 u
(d) 0.0303592 u
(c) (d)
6. A nucleus Z X A has mass represented by m ( A, Z ). If
m p and m n denotes the mass of proton and neutron 0 r0 r 0 r0 r
respectively and BE the binding energy (in MeV),
then 11. Heavy stable nuclei have more neutrons than
(a) BE = [ m( A, Z ) - Zm p - ( A - Z ) m p ] c 2 protons. This is because of the fact that,
[NCERT Exemplar]
(b) BE =[ Zm p + ( A - Z ) m n - m( A, Z )] c 2 (a) neutrons are heavier than protons
(b) electrostatic force between protons is repulsive
(c) BE = [ Zm p + Am p - m( A, Z )] c 2 (c) neutrons decay into protons through beta decay
(d) nuclear forces between neutrons are weaker than that
(d) BE = m( A, Z ) - Zm p - ( A - Z ) m n
between protons
120 CBSE Term II Physics XII

12. In any fission process, the ratio of 17. In a nuclear reactor , moderators slow down the
mass of fission products neutrons which come out in a fission process. The
is moderator used have light nuclei. Heavy nuclei will
mass of parent nucleus
not serve the purpose, because [NCERT Exemplar]
(a) less than 1
(a) they will break up
(b) greater than 1
(b) elastic collision of neutrons with heavy nuclei will not
(c) equal to 1 slow them down
(d) depends on the mass of parent nucleus (c) the net weight of the reactor would be unbearably high
13. On bombarding U 235 by slow neutron, 200 MeV (d) substances with heavy nuclei do not occur in liquid or
gaseous state at room temperature
energy is released. If the power output of atomic
reactor is 1.6 MW, then the rate of fission will be 18. Nuclear fusion is common to the pair
(a) 5 ´ 10 22 s– 1 I. uranium based reactor
(b) 5 ´ 10 s 16 – 1 II. hydrogen bomb
(c) 8 ´ 1016 s– 1 III. energy production in sun
(d) 20 ´ 1016 s– 1 IV. atom bomb
(a) Both I and II (b) Both II and III
14. Which of the following fusion reactions will not (c) Both III and IV (d) Both II and IV
result in the net release of energy?
19. Which of the following is fusion process?
I. 6 Li + 6 Li (a) 12 H +12 H ¾® 42He + 10 n
4 4
II. He + He (b) 10 n + 235 141
92 U ¾® 56 Ba +
92
36 Kr + 3 ( 10 n )
III. 12 C + 12C (c) Uranium decay
35 35
(d) None of the above
IV. Cl + Cl
(a) IV (b) III l
Assertion-Reasoning MCQs
(c) I (d) II
Direction (Q. Nos. 20-25) Each of these questions
15. Schematic diagram of a nuclear reactor based on contains two statements Assertion (A) and Reason (R).
thermal neutron fission is as shown below Each of these questions also has four alternative
choices, any one of which is the correct answer. You
D=Reflector C=Coolant
have to select one of the codes (a), (b), (c) and (d) given
B=Control
Steam to below.
turbine
rods
Heat exchanger
(a) Both A and R are true and R is the correct
A=core
(Steam generator) explanation of A
Water from
condenser
(b) Both A and R are true, but R is not the correct
explanation of A
(c) A is true, but R is false
(d) A is false and R is also false
Here, the correct purpose of the parts specified by A, B, C
and D is given in 20. Assertion The heavier nuclei tend to have larger
(a) A ® reduces leakage N/Z ratio because neutron does not exert electric
(b) B ® can shut down the reactor force.
(c) C ® site of nuclear fission and contains 235
92 U Reason Coulomb forces have longer range
(d) D ® transfers heat to a working fluid which in turn may compared to nuclear force.
produce steam 35
21. Assertion It is not possible to use Cl as the fuel
16. For sustaining the chain reaction in a sample for fusion energy.
235 35
(of small size) of 92 U, it is desirable to slow down Reason The binding energy of Cl is too small.
fast neutrons by 22. Assertion Nuclei having number about 60 are most
(a) friction stable.
(b) elastic damping/scattering Reason When two or more light nuclei are
(c) absorption combined, the some energy is lost during the
(d) None of the above process.
CBSE Term II Physics XII 121

23. Assertion Initially, it was believed that mass and (ii) For sustaining the nuclear fission chain reason in a
energy are conserved separately in a nuclear sample (of small size) of 235
92 U, a process called
reaction. elastic damping uses
Reason According to Einstein, one can convert (a) electrons (b) lighter nuclei
mass-energy into other forms of energy. (c) heavy nuclei (d) None of these

24. Assertion Fission of 235


is brought about by a (iii) Which of the following is/are fission reaction(s)?
92 U
thermal neutron, whereas that of 238 I. 10 n + 235 236 133 99 1
92 U ® 92 U ® 51 Sb ® 41Nb + 4 0 n
92 U is brought
about by a fast neutron. II. 10 n + 235 140 94
92 U ® 54 Xe + 38 Sr + 210 n
Reason 235 238
92 U is an even-odd nucleus, whereas 92 U III. 12 H + 2 3 1
1 H ® 2He + 0 n
is an even-even nucleus. (a) Both II and III
25. Assertion Heavy water is used to slow neutron in (b) Both I and III
nuclear reactor. (c) Only II
(d) Both I and II
Reason It does not react with slow neutron and
(iv) If a nucleus with mass number A = 240 with
mass of deuterium is comparable to the neutron.
E bn = 7.6 MeV breaks into two fragments of A = 120
l
Case Based MCQs and E bn = 8. 5 MeV, then released energy is around
(a) 216 MeV
Direction Read the following passage and answer the (b) 200 MeV
questions that follows (c) 100 MeV
26. Nuclear Fission (d) Cannot be estimated from given data
In the year 1939, German scientist Otto Hahn and (v) Assuming that about 20 MeV of energy is released
Strassmann discovered that when a uranium in a fusion reaction 1 H 2 + 1 H 3 ® 0 n 1 + 2 He 4 , then
isotope was bombarded with a neutron, it breaks the mass of 1 H 2 consumed per day in a fusion
into two intermediate mass fragments. It was reactor of power 1 MW will approximately be
observed that, the sum of the masses of new (a) 0.001 g (b) 0.1 g
fragments formed were less than the mass of the (c) 10.0 g (d) 1000 g
original nuclei. This difference in the mass
appeared as the energy released in the process.
Thus, the phenomenon of splitting of a heavy PART 2
nucleus (usually A> 230) into two or more lighter
nuclei by the bombardment of proton, neutron, Subjective Questions
a-particle, etc., with liberation of energy is called
nuclear fission. l
Short Answer (SA) Type Questions
Fission reaction resulting from the absorption of
neutron is known as induced fission. 1. How the size of a nucleus is experimentally
235 determined? Show that the density of nucleus is
92 U + 0 n 1 ® 92 U 236 ® 56 Ba 144 + 36 Kr 89
Unstable nucleus
independent of its mass number. [Delhi 2012, 2011C]
+ 3 0 n1 + Q 2. Calculate the surface area of a nucleus (assuming it
(i) Fusion processes, like combining two deuterons to to be a perfect sphere).
form a He nucleus are impossible at ordinary 3. The mass of a nucleus is less than the sum of the
temperature and pressure.The reasons for this can masses of constituent neutrons and protons.
be traced to the fact Comment.
(a) nuclear forces have short range 4. Obtain approximate ratio of the nuclear radii of the
(b) nuclei are negatively charged 197 107
(c) the original nuclei must be completely ionised before
gold isotope 79 Au and the silver isotope 47 Ag.
fusion can take place [NCERT]
(d) the original nuclei must first break up before combining 5. Given the mass of iron nucleus as 55.85u and
with each other
A = 56. Find the nuclear density. [NCERT]
122 CBSE Term II Physics XII

6. Supposing that, protons and neutrons have equal 13. Suppose we think of fission of a 56
26 Fe nucleus into
masses. Calculate how many times nuclear matter two equal fragments, 13 28
Al. Is the fission
is denser than water. (Take, mass of a nucleon
energetically possible? Argue by working out Q of
= 1.67 ´ 10 -27 kg and R 0 = 1.2 ´ 10 -15 m)
the process. Given, m ( 5626 Fe) = 55.93494 u and
7. The neutron separation energy is defined as the 28
m( 13 Al) = 27.98191 u. [NCERT]
energy required to remove a neutron from the
239
nucleus. Obtain the neutron separation energies of 14. The fission properties of are very similar to
94 Pu
the nuclei 41 27
20 Ca and 13 Al from the following data those of 235
92 U. The average energy released per
[NCERT] fission is 180 MeV. How much energy in MeV is
m( 40
20 Ca ) = 39.962591 u released, if all the atoms in 1 kg of pure 239
94 Pu
m( 41
20 Ca ) = 40.962278 u undergo fission? [NCERT]
26
m( 13 Al ) = 25.986895 u 15. In a typical nuclear reaction,
27
m( 13 Al ) = 26.981541 u 2
1H + 12H ® 32He + 10 n + 3.27 MeV, although number
8. A nuclide 1 is said to be the mirror isobar of of nucleons is conserved, yet energy is released.
nuclide 2, if Z1 = N 2 and Z 2 = N 1 . Explain, how.
(i) What nuclide is a mirror isobar of 23 16. A given coin has a mass of 3.0 g. Assume that, the
11 Na?
coin is entirely made of 63
29 Cu atoms (of mass
(ii) Which nuclide out of the two mirror isobars
have greater binding energy and why? 62.92960 u). Calculate the nuclear energy that
would be required to separate all the neutrons and
20 21
9. The three stable isotopes of neon 10 Ne, 10 Ne and protons from each other.
22
10 Ne have respective abundances of 90.51%, 0.27% 17. (i) Write three characteristic properties of nuclear
and 9.22%. The atomic masses of three isotopes are force.
19.99 u, 20.99 u and 21.99 u, respectively. Obtain (ii) Draw a plot of potential energy of a pair of
the average atomic mass of neon. [NCERT] nucleons as a function of their separation. Write
10. Nuclei with magic number of protons two important conclusions that can be drawn
Z = 2, 8, 20, 28, 50, 52 and magic number of from the graph. [Delhi 2015]
neutrons N = 2, 8, 20, 28, 50, 82 and 126 are found 18. The Q-value of a nuclear reaction A + b ¾® C + d
to be very stable.
is defined by Q = [ m A + m b - m C - m d ] c 2 , where
Verify this by calculating the proton separation
the masses refer to the respective nuclei.
energy Sp for 120 Sn (Z = 50) and 121 Sb (Z = 51). The
Determine from the given data, the Q-value of the
proton separation energy for a nuclide is the following reactions and state whether the reactions
minimum energy required to separate the least are exothermic or endothermic.
tightly bound proton from a nucleus of that nuclide.
(i) 11 H + 31 H ¾® 21 H + 21 H
It is given by
Sp = ( M Z - 1,N + M H - M Z, N ) c 2 (ii) 12 12 20 4
6 C + 6 C ¾® 10 Ne + 2 He

Given, 119
In = 118 .9058 u, Atomic masses are given to be
120
Sn = 119.902199 u, m( 1 H1 ) = 1.007825 u
121
Sb = 120.903824 u m ( 12 H) = 2.014102 u
1
and H = 1.0078252 u. [NCERT Exemplar] m ( 13 H) = 3.016049 u
11. Draw a plot of potential energy between a pair of m ( 12
6 C) = 12.000000 u
nucleons as a function of their separation. Mark the 20
m ( 10 Ne) = 19.992439 u [NCERT]
regions, where potential energy is [Delhi 2013]
(i) positive and (ii) negative. 19. A 1000 MW fission reactor consumes half of its fuel
12. Complete the following fission reaction and in 5 yr. How much 23592 U did it contain initially?
calculate the amount of energy it releases. Assume that, the reactor operates 80% of the time
1 235 88 136 that all the energy generated arises from the fission
0 n + 92 U ¾® 38 Sr + 54 Xe + (?)
CBSE Term II Physics XII 123

of 235
92 U and that this nuclide is consumed only by
fissionable uranium would our country need per year
the fission process. by 2020? (Take, the heat energy per fission of 235 U to
be about 200 MeV) [NCERT]
20. How long can an electric lamp of 100 W be kept
glowing by fusion of 2 kg of deuterium? Take, the l
Case Based Questions
fusion reaction as
2
+ 12H ¾® 32H + n + 3.27 MeV Direction Read the following passage and answer the
1H [NCERT]
questions that follows
21. Calculate and compare the energy released by 25. Nuclear Fission
(i) fusion of 1 kg of hydrogen deep within sun and
Nuclear fission is the phenomenon of splitting of a
235
(ii) the fission of 1 kg of U in a fission reactor. heavy nucleus (usually A > 230) into two or more
[Delhi 2020] lighter nuclei by the bombardment of proton,
neutron, a-particle, etc. Energies associated with
l
Long Answer Questions (5 Marks) nuclear processes are about a million times larger
22. Explain giving necessary reactions, how energy is than chemical process.
released during In fission, a heavy nucleus like 235
92 U breaks into
(i) fission and (ii) fusion. two smaller fragments by the bombardment of
thermal neutron (low energy or slow moving).
23. Binding energy of deuteron is 2.2 MeV. It is bound
1 e¢ 2 e.g. 235 1 141 92 1
92 U + 0 n ¾® 56 Ba + 36Kr + 3 0 n + Q
by electrostatic forces, i.e. F = . (Energy released)
4pe 0 r
e¢ Q-value here refer to the energy released in the
Calculate the ratio of , where e¢ is the effective nuclear process, which can be determined using
e Einstein’s mass - energy relation, E = mc 2 . The
charge. A g-ray with energy E is aimed at this
deuteron nucleus to break it into a (neutron + Q-value is equal to the difference of mass of products
proton) pair, such that n and p move in direction of and reactants multiplied by square of velocity of light.
the g-ray. If E = B, show that it cannot happen and Energy released per fission of 235
92 U is 200.4 MeV. The
hence calculate how much E must be greater than B fragment nuclei produced in fission are highly
for this process to happen. unstable. They are highly radioactive and emit
b-particles in succession until each reaches to a stable
24. Suppose India had a target of producing end product.
200000 MW of electric power by 2020 AD, 10% of
which was to be obtained from nuclear power plants. (i) What are moderators? Give few examples.
Assume that on an average, the efficiency of utilisation (ii) What is multiplicity in nuclear fission? Explain.
(i.e. conversion to electric energy) of thermal energy (iii) Define the Q-value of a nuclear process. When can
produced in a reactor was 25%. How much amount of a nuclear process not proceed spontaneously?
Chapter Test
Multiple Choice Questions (ii) Which nuclide out of the two mirror isobars have
27 greater binding energy and why?
1. If the nuclear radius of Al is 3.6 fm, the aproximate
nuclear radius of 64 Cu (in fm) is 11. Why is it necessary to slow down the neutrons,
(a) 2.4 (b) 1.2 produced through the fission of 23592 U nuclei (by
(c) 4.8 (d) 3.6 neutrons) to sustain a chain reaction? What type of
2. Two protons are attracting each other, then separation nuclei are (preferably) needed for slowing down fast
between them is neutrons?
(a) 10 -10 m (b) 10 -2 m 12. Are the nucleons fundamental particles or do they
(c) 10 -8 m (d) 10 -15 m consist of still smaller parts? One way to find out is to
3. The mass defect of helium nucleus is 0.0303 amu. probe a nucleon just as Rutherford probed an atom.
The binding energy per nucleon for helium nucleus will What should be the kinetic energy of an electron for it
be to be able to probe a nucleon? Assume, the diameter
(a) 28 MeV (b) 7 MeV of a nucleon to be approximately 10 -15 m. (Ans. 10 9 eV)
(c) 14 MeV (d) 1 MeV 13. Distinguish between nuclear fission and fusion. Show
4. How much mass has to converted into energy to how in both these processes energy is released.
produce electric power of 200 MW for one hour? Calculate the energy release in MeV in the
(a) 2 ´ 10 -6 kg (b) 8 ´ 10 -6 kg
deuterium-tritium fusion reaction.
2 3
(c) 1 ´ 10 -6
kg (d) 3 ´ 10 -6 kg 1H+ 1H ¾® 42 He + n
5. In fusion reaction occurring in the sun, Using the data
(a) hydrogen is converted into carbon m( 21 H) = 2. 014102 u,
(b) hydrogen and helium are converted into carbon and other m( 31 H) = 3 . 016049 u,
heavier metals/elements
m( 42 He) = 4. 002603 u,
(c) helium is converted into hydrogen
(d) hydrogen is converted into helium mn = 1 . 008665 u
MeV
Short Answer Type Questions 1 u = 931.5 2
c (Ans. 17.589 eV)
6. If both the numbers of protons and neutrons are
conserved in a nuclear reaction like 14. In a nuclear reaction, uranium breaks into thorium
12
and helium. Which species will have greater kinetic
6C + 6C 12 ¾® 10 N
20
+ 2He 4 energy?
In what way, is the mass converted into the energy? 15. Stars are giant ball of hydrogen which fuses to form
Explain. helium and energy. What happens, when all the
7. Explain, why elements like helium are very stable. hydrogen is used up?
8. He 32 and He 31 nuclei have the same mass number. Do Long Answer Type Questions
they have the same binding energy? 16. (i) What is the source of stellar energy? Also, explain
9. Why do stable nuclei never have more protons than the thermonuclear reactions of sun.
neutrons? (ii) Discuss radiation hazards.
10. A nuclide 1 is said to be the mirror isobar of nuclide 2, 17. Describe uncontrolled and controlled nuclear chain
if Z1 = N2 and Z2 = N1 . reactions and therefore explain nuclear reactor & its
23 components.
(i) What nuclide is a mirror isobar of 11 Na?

Answers
Multiple Choice Questions
1. (c) 2. (d) 3. (b) 4. (b) 5. (d)
For Detailed Solutions
Scan the code
CBSE Term II Physics XII 125

EXPLANATIONS
PART 1 Hence, the BE of nucleus is
BE = [ Zm p + Nm n - m( A, Z )] c 2
1. (c) For 612 C, A = 12 = N + Z, Z = 6 BE = [ Zm p + ( A - Z ) m n - m ( A, Z )] c 2
Þ N =6 GMm
For 614 C, A = 14 = N + Z, Z = 6 7. (b) Given, F =
r2
Þ N =8 where, M = effective mass of hydrogen atom,
Also, number of electrons in both atoms G = gravitational constant
= number of protons = Z = 6 and r = distance between H-atom & particle of mass m.
Mass mA 3m 8. (a) Nuclear force has the following properties
2. (d) Density = = = ,
Volume 4 pR 3 A 4pR 03 I. Nuclear force is a short range force, whose range is of
0
3 the order of 2 to 3 femtometre.
m = m p = Mn II. Nuclear force is the strongest force in nature.
= 2.3 ´ 1017 kg m -3, which is a constant. III. Nuclear force is an attractive force acting between
nucleons, which is charge independent.
3. (a) Nuclear radius, R = R 0 A1/ 3
Therefore, nuclear force is strong, short range and charge
where, R = 7.2 ´ 10-15 m, R 0 = 1.2 ´ 10-15 m independent force.
3 3
æ Rö æ 7.2 ´ 10-15 ö 9. (c) Nuclear force between two particles is independent of
\ A = çç ÷÷ = çç ÷ = ( 6) 3 = 216
-15 ÷ charges of particle.
è R0 ø è 1.2 ´ 10 ø Fpp = Fnn = Fnp
q 1.28 ´ 10-17 10. (b) Potential energy of a pair of nucleons as a function of
Also, atomic number, Z = = = 80
e 1.6 ´ 10-19 their separation is correctly depicted in option (b). For a
Therefore, number of neutrons, separation greater than r0 , the force is negative and
attractive and potential energy decreases upon increasing
N = A - Z = 216 - 80 = 136
the distance and for a separation less than r0 , the force is
4. (d) Mass density of nuclear matter is a constant quantity for strongly repulsive, so potential energy increases for r < r 0 .
all elements. It does not depend on element’s mass number
11. (b) Stable heavy nuclei have more neutrons than protons.
or atomic radius.
This is because, electrostatic force between protons is
\The ratio of mass densities of 40 Ca and 16 O is 1 : 1. repulsive , which may reduce stability.
5. (d) Mass defect, Dm
12. (a) In fission process, when a parent nucleus breaks into
= Mass of nucleons - Mass of nucleus daughter products, then some mass is lost in the form of
= [ Zm p + ( A - Z ) m n ] - Mn energy. Thus,
Here, mass number, A = 4, Mass of fission products < Mass of parent nucleus
Atomic number, Z = 2, Mass of fission products
Number of protons = 2, Þ <1
Mass of parent nucleus
Number of neutrons = A - Z = 4 - 2 = 2,
m p = 1.0072676 u, 13. (b) Energy released on bombarding U235 by neutron
= 200 MeV
m n = 1.008665 u
Power output of atomic reactor = 1.6 MW
and Mn = 4.001506 u.
Power out of reactor
Mass of nucleon in 2 He4 = \ Rate of fission =
Energy released per fission
Mass of 2 protons + Mass of 2 neutrons 1.6 ´ 106
So, mass of nucleons =
200 ´ 106 ´ 1.6 ´ 10-19
= 2 ´ 1.0072676 + 2 ´ 1.008665 (Q 1 eV = 1.6 ´ 10-19 J)
\ Dm = 2 ´ 1.0072676 + 2 ´ 1.008665 - 4.001506 = 5 ´ 1016 s -1
= 0.0303592 u 14. (a) The reaction 35 Cl + 35 Cl will not result in the net
6. (b) According to mass defect, if the quantity of mass release of energy because heavy nucleus are not used in
disappearing is Dm, then the binding energy, fusion reaction.
BE = Dmc 2 15. (b) Here,
A ® is the core of the reactor, which is the site of nuclear
From the above discussion, the Dm is given by
fission. It contains elements in suitably fabricated form. The
Dm = Zm p - Nm n - m( A, Z ) fuel may be enrinched uranium say 235 92 U.
where, m ( A, Z ) is the mass of the atom of mass number A
D ® is a reflector surrounding the core which is used to
and atomic number Z . reduce the leakage.
126 CBSE Term II Physics XII

C ® is coolant which helps in removing the energy (heat) Hence it is an even-odd nucleus whereas 238 92 U has even
released in fission. It transfers heat to working fluid which in mass number and even atomic number, hence it is an
turn may produce steam. even-even nucleus.
B ® is control rods that shut down the reactor as they have Therefore, both A and R are true but R is not the correct
high absorption of neutrons. explanation of A.
Hence, only option (b) shows correct purpose of given part. 25. (a) Heavy water (D2O) is used to slow down fast moving
16. (b) Fast neutrons are slowed down by elastic scattering with neutrons in nuclear reactor, hence it is called moderator. It
light nuclei. Each collision takes away nearly 50% of energy. does remove neutrons from the system by absorbing them.
17. (b) According to the question, the moderator used have light When fast moving neutrons are passed through heavy water
nuclei (like proton). When protons undergo perfectly elastic (D2O), then they make elastic collisions with protons or
collision with neutron emitted, their velocities are deuterium having mass comparable to neutrons, which have
exchanged, i.e. neutrons come to rest and protons move with comparatively much smaller than velocities. In few
the velocity of neutrons. interactions, the velocities of neutrons get interchanged
Heavy nuclei will not serve the purpose because elastic with those of protons.
collisions of neutrons with heavy nuclei will not slow them Heavy water does not react with slow moving neutrons, i.e. it
down. slows down-fast moving neutrons.
18. (b) Energy produced in sun and hydrogen bomb is due to Therefore, both A and R are correct and R is the correct
nuclear fusion. explanation of A.
However, energy released in uranium based nuclear reactor 26. (i) (a) Fusion processes are impossible at ordinary
and atom bomb is due to nuclear fission. temperatures and pressures. Because nuclei are
19. (a) When two lighter nuclei combine to form a heavier positively charged and nuclear forces are short range
nucleus, the process is called nuclear fusion. strongest forces.
2 2 4 1 (ii) (b) Fast neutrons are slowed down by elastic scattering
e.g. 1 H +1 H ¾® 2 He + 0 n
with lighter nuclei as each collision takes away nearly
20. (a) The heavier nuclei have greater number of neutrons than 50% of energy.
N
protons. Thus, heavier nuclei have larger ratio. Neutrons (iii) (d) Reactions I and II represent fission of uranium
Z
isotope 235
92 U, when bombarded with neutrons that
are chargeless particle, hence they are not affected by
electric force. The electric forces have longer range as breaks it into two intermediate mass nuclear fragments.
compared to nuclear forces, which have a very short range. However, reaction III represents two deuterons fuses
As, Coulomb force is effective upto several metres whereas together to form the light isotope of helium.
nuclear force are only effective within the vicinity of nucleus. (iv) (a) The energy released (i.e. Q-value) in the fission
Therefore, both A and R are true and R is the correct reaction of nuclei like uranium is of the order of 200 MeV
explanation of A. per fissioning nucleus. This is estimated as follows
21. (c) The fusion of two lighter nuclei produce sufficient energy. Let us take a nucleus with A = 240 breaking into two
fragments each of A = 240, then
Since, 35 Cl is a relatively heavier nuclei, it does not produce
significant energy. The fusion of two lighter nuclei produces Ebn for A = 240 nucleus is about 7.6 MeV
sufficient energy. The binding of 35 Cl is 298 MeV, hence it is Ebn for the two A = 120 fragment nuclei is about
not suitable for fusion energy. 8.5 MeV
Therefore, A is true but R is false. So, gain in binding energy for nucleon is about 0.9 MeV.
22. (c) Nuclei having mass number A = 60, have highest binding Hence, the total gain in binding energy is
energy. Thus, those are most stable. When two or more light 240 ´ 0.9 or 216 MeV.
nuclei fuse together, some energy is gained. (v) (b) Energy produced, U = Pt
Therefore, A is true but R is false. = 106 ´ 24 ´ 36 ´ 102 = 24 ´ 36 ´ 108 J
23. (b) Before the advent of special theory of relativity, it was Energy released per fusion reaction
believed that mass and energy were conserved separately in = 20 MeV = 20 ´ 106 ´ 1.6 ´ 10-19
a reaction. = 32 ´ 10-13 J
However, Einstein showed that mass is an another form of Energy released per atom of 1 H2
energy and one can convert mass-energy into other forms of
= 32 ´ 10-13 J
energy, say kinetic energy and vice-versa.
Number of 1 H2 atoms used
Therefore, both A and R are true but R is not the correct
24 ´ 36 ´ 108
explanation of A. = = 27 ´ 1021
24. (b) Fission of U235 occurs by slow neutrons only (of energy 32 ´ 10-13
about 1 eV) or even by thermal neutrons (of energy about Mass of 6 ´ 1023 atoms = 2 g
0.025 eV). Fission of 238
92 U is brought about by a fast \ Mass of 27 ´ 1021 atoms
neutrons. 235 U has odd mass number and even atomic 2
92 = ´ 27 ´ 1021 = 01
. g
number. 6 ´ 1023
CBSE Term II Physics XII 127

PART 2 Density of water, r¢= 103 kg/ m 3


r 2.307 ´ 1017
1. The size of the nucleus is experimentally determined using \ =
Rutherford’s a-scattering experiment and the distance of r¢ 103
closest approach and impact parameter. = 2.307 ´ 1014
The relation between radius and mass number of nucleus is 7. When a neutron is separated from 41
we are left with
20 Ca,
R = R 0 A1/ 3 40
20 Ca and the reaction becomes
where, R 0 = 1.2 fm, A = mass number and R = radius of 41
¾® 40
+ 0 n1
20 Ca 20 Ca
nucleus.
Mass of nucleus mA Mass defect,
Nuclear density, r = = Dm = m ( 40 1
20 Ca) + m( 0 n ) - m (
41
Volume of nucleus 4 p( R A1/ 3 ) 3 20 Ca)
0
3 = 39.962591 + 1 .008665 - 40.962278
where, m = mass of each nucleon. = 0.008978 u
mA m Energy for separation of neutron = Dm ´ 931
r= Þ r=
4 3 4 3
pR 0 A pR 0 = 0.008978 ´ 931
3 3 = 8.358 MeV
From the above formula, it is clear that r does not depend on 27
When a neutron is separated from 13 Al, we are left with
the mass number. 26
2. Surface area of nucleus, 13 Al. Thus, the reaction becomes

SA = 4pR 2 = 4p( R 0 A1/ 3 ) 2


27
13 Al ¾® 26
13 Al + 10 n
= 4pR 02 × A 2 / 3 Mass defect,
26
-15 Dm = m (13 Al ) + m ( 0 n1 ) - m( 13
27
Al )
Substituting constant ( R 0 ) value = 1.2 ´ 10 m, we get
-15 2/ 3 = 25.986895 + 1.00865 - 26.981541
SA = 4( 314
. )( 1.2 ´ 10 )A
= 0.014019
= ( 1.8 ´ 10-29 ) A 2 / 3
\ Energy for separation of neutron
3. When nucleons approach each other to form a nucleus, they = Dm ´ 931 = 0.014019 ´ 931
strongly attract each other. Their potential energy decreases = 13.06 MeV
and becomes negative. It is the potential energy which holds
the nucleons together in the nucleus. The decrease in PE 8. (i) According to the question, a nuclide 1 is said to be
results in the mass of nucleons inside the nucleons. mirror isobar of nuclide 2, if Z1 = N 2 and Z2 = N1.
4. Radius of nuclei, R = R 0 A1/ 3 Now, in 1123 Na, Z1 = 11, N1 = 23 - 11 = 12

where, A is the mass number of nucleus and R 0 is an \ Mirror isobar of 1123 Na is 1223 Mg, for which
empirical constant. Z2 = 12 = N1 and N 2 = 23 - 12 = 11 = Z1
\ R µ A1/ 3 (ii) As, 1223 Mg contains even number of protons (12)
1/ 3 1/ 3
R gold æ Agold ö against 1123 Na which has odd number of protons ( 11 ),
÷÷ = æç
197 ö
Þ = çç ÷ = 1.225 = 1.23
Rsilver è Asilver ø è 107 ø therefore 1123 Mg has greater binding energy than 1123 Na.
5. Given, mass, m = 55 .85 u 9. Given, abundance per cent of Ne20 = 90.51%
= 55 .85 ´ 1 .67 ´ 10-27 kg Abundance per cent of Ne21 = 0.27%
4 4 4 Abundance per cent of Ne22 = 9.22%
Volume, V = pR 3 = p ( R 0 A1/ 3 )3 = pR 03 ´ A
3 3 3
m Mass of Ne20 = 19.99 u
\ Nuclear density, r =
V Mass of Ne21 = 20.99 u
3 ´ 55.85 ´ 1.67 ´ 10-27 Mass of Ne22 = 21.99 u
=
22 Average atomic mass, m = Weighted average of all isotopes
4 ´ ´ (1.2 ´ 10-15 ) 3 ´ 56
7 90.51 ´ 19.99 + 0.27 ´ 20.99 + 9.22 ´ 21.99
=
= 2.29 ´ 1017 kg/m 3 90.51 + 0.27 + 9.22
6. Density of nucleus of water, 1809.29 + 5.67 + 202.75 2017.71
= = = 20.18
3m 3 ´ 1.67 ´ 10-27 100 100
r= =
3
4pR 0 4 ´ 22 ´ (1.2 ´ 10-15 ) 3 10. The proton separation energy is given by
7 Sp (Sn) = ( M119 , 70 + MH - M120 , 70 ) c 2
7 ´ 3 ´ 1.67 ´ 1018 = (118.9058 + 1.0078252 - 119.902199) c 2
=
88 ´ 1.2 ´ 1.2 ´ 1.2 = 0.0114262 c 2
= 2.307 ´ 1017 kg/ m 3
128 CBSE Term II Physics XII

Similarly, Sp(Sb) = ( M120 , 70 + MH - M121, 70 ) c 2 16. Given, mass of coin = 3g


= (119.902199 +1.0078252 -120.9038224) c 2 Atomic mass of Cu = 63
Mass of 63 29 Cu, M = 62.92960 u
= 0.006202 c 2
Avogadro’s number = 6.023 ´ 1023
Since, Sp (Sn) > Sp (Sb) , Sn nucleus is more stable than
Mass of proton, m p = 1.007825 u
Sb nucleus. Mass of neutron, m n = 1.008665 u
11. The graph between the potential energy of a pair of Nuclear energy required to separate neutrons and protons,
nucleons as a function of their separation is given below Eb = ?
Since, each atom of copper contains 29 protons and 34
100 neutrons. Therefore, mass defect of each atom using the
Positive region
relation,
PE (MeV)

0.8 fm 10 fm Dm = [ Z m p + ( A - Z ) m n ] - M
x
r (fm) D m = [29 ´ 1.007825 + (63 - 29) ´ 1.008665] - 62.92960
= 0.591935 u
100 Negative 6.023 ´ 1023 ´ 3
region Number of atoms in 3 g coin = = 2.868 ´ 1022
63
Total mass defect of all atoms,
(i) For distance less than 0.8 fm, negative PE decreases to ( D m ) total = 0.591935 ´ 2.868 ´ 1022 = 1.6977 ´ 1022
zero and then becomes positive.
The nuclear energy required ( Eb ) to separate all the neutrons
(ii) For distance greater than 0.8 fm, negative PE goes on
and protons from each other and can be calculated by using
decreasing.
the relation,
12. By conservation of charge and mass, given equation can be
written as Eb = ( D m ) ´ c 2 = ( D m ) c 2 ´ 931 MeV/c 2 (Q 1 u = 931 MeV )
1 235 88 136 1
0 n + 92 U ¾® 38Sr + 54 Xe + 120 n + Q (Energy)
= 1.6977 ´ 1022 ´ 931 MeV = 1.58 ´ 1025 MeV
For amount of energy released, use 17. (i) Characteristics properties of nuclear force
Q = Dm ´ 931 MeV (a) Nuclear forces act between a pair of neutrons, a pair of
protons and also between a neutron-proton pair, with
13. The given reaction for decay process,
the same strength. This, shows that, nuclear forces are
26 Fe ¾® 213 Al
56 28
independent of charge.
Mass defect, Dm = m ( 56 28
26 Fe) - 2m (13 Al ) (b) The nuclear forces are dependent on spin or angular
= 55.93494 - 2(27.98191) momentum of nuclei.
= - 0.02888 u (c) Nuclear forces are non-central forces. This shows that,
the distribution of nucleons in a nucleus is not
Þ Q = Dm ´ 931
spherically symmetric.
= - 0.02888 ´ 931
(ii)
Potential energy (MeV)

= -26.88728 MeV
Because the energy is negative, so the fission is not possible
energetically. 100
14. According to the concept of Avogadro number,
the number of atoms in 239 g of 239
94 Pu = 6.023 ´ 10
23 0
239
Number of atoms in 1 kg of 94 Pu
23 –100
6.023 ´ 10 ´ 1000 r0 1 2 3
= = 2.52 ´ 1024 r (fm)
239
The average energy released in one fission (Potential energy versus distance)
= 180 MeV From the plot, it is concluded that
So, total energy released in fission of 1 kg of (a) The potential energy is minimum at a distance
239
. ´ 1024
94 Pu = 180 ´ 252
r0 ( » 0.8 fm ) which means that, the force is attractive
= 4.53 ´ 1026 MeV for distances greater than 0.8 fm and repulsive, for the
distance less than 0.8 fm between the nucleons.
15. In a nuclear reaction, the sum of the masses of the target
(b) Nuclear forces are negligible, when the distances
nucleus (12 H) and the bombarding particle (12 H) may be
between the nucleons is more than 10 fm.
greater than the product nucleus ( 32 He) and the outgoing
18. (i) The given reaction, 11H + 13H ¾® 12 H + 12 H
neutron 10 n. So, from the law of conservation of mass-energy,
some energy (3.27 MeV) is evolved due to mass defect in the Mass defect, Dm = m (11H) + m (13H) -2 m (12 H)
nuclear reaction. This energy is called Q-value of the = 1.007825 + 3.016049 - 2(2.014102)
nuclear reaction. = - 0.00433 u
CBSE Term II Physics XII 129

Q-value of the reaction, 100 J energy used by lamp in time = 1 s


Q = Dm ´ 931 = - 0.00433 ´931 1 ´ 15.75 ´ 1013
15 .75 ´ 1013 J energy used in time = s
Q = - 4.031 MeV 100
As, the energy is negative, so the reaction is = 15.75 ´ 1011s
endothermic. 15 .75 ´ 1011
(ii) The given reaction, = yr
60 ´ 24 ´ 60 ´ 365
12 12 20 4
6 C + 6 C ¾® 10 Ne + 2 He [Q 1 yr = (60 ´ 24 ´ 60 ´ 365) s]
Mass defect, Dm = 2 m (12 20 4
6 C) - m (10 Ne) - m ( 2 He) = 4.99 ´ 104 yr
= 2 ´ 12 - 19.992439 - 4.002603 Thus, the bulb glows for 4.99 ´ 104 yr.
Dm = 0.00495 u 21. (i) In sun, four hydrogen nuclei fuse to form a helium
Q -value of the reaction, nucleus and release 26MeV energy.
Q = Dm ´ 931 Q 1 g of hydrogen contains = 6.023 ´ 1023 nuclei
= 0.00495 ´ 931 \ Energy released by fusion of 1 kg ( = 1000 g) of
= 4.62 MeV 6.023 ´ 1023 ´ 26 ´ 103
hydrogen, E1 = = 39 ´ 1026 MeV
Since, the energy is positive, thus the reaction is 4
235
exothermic. (ii) Energy released in one fission of 92 U nucleus
19. Given, power of reactor, P = 1000 MW = 200 MeV
235
The energy generated in one fission of 92 U is 200 MeV. Mass of uranium = 1 kg = 1000 g
1 We know that, 235 g of 235 U has 6.023 ´ 1023 atoms or
235
Number of 92 U atoms in 1 g = ´ 6.023 ´ 1023 nuclei.
235
\ Energy generated per gram of 235 \ Energy released in fission of 1 kg of U 235 ,
92 U
6.023 ´ 1023 ´ 1000 ´ 200
æ 1 ö E2 =
=ç ´ 6.023 ´ 1023 ´ 200 ´ 1.6 ´ 10-13 ÷ 235
è 235 ø 26
= 5.1 ´ 10 MeV
Total energy generated in 5 yr with 80% of the time
80 E1 39 ´ 1026
= 1000 ´ 106 ´ 5 ´ 365 ´ 24 ´ 60 ´ 60 ´ \ = = 7.65 » 8
100 E2 5 .1 ´ 1026
( as, E = Pt ) Thus, the energy released in fusion is 8 times the
\ Mass of 235 U consumed in 5 yr, energy released in fission.
92
Total energy 22. (i) Nuclear Fission The phenomenon of splitting of heavy
m= nuclei (mass number > 120) into smaller nuclei of nearly
Energy consumed per gram equal masses is known as nuclear fission. In nuclear
1000 ´ 106 ´ 5 ´ 365 ´ 24 ´ 60 ´ 60 ´ 0.8 fission, the sum of the masses of the product is less than
= the sum of masses of the reactants. This difference of
æ 1 ö 23 -13
ç ÷ ´ 6.023 ´ 10 ´ 200 ´ 1.6 ´ 10 mass gets converted into energy E = mc 2 and hence
è 235 ø
sample amount of energy is released in a nuclear fission.
= 1.538 ´ 106 g = 1538 kg e.g. 235 1 141 92 1
92 U + 0 n ® 56 Ba + 36Kr + 3 0 n + Q
235
\ Initial amount of 92 U = (1544 ´ 2) kg = 3076 kg
Masses of reactant
20. Let t be the time. = 235.0439 amu + 1.0087 amu
According to the Avogadro number concept, = 236.0526 amu
Number of atoms in 2 g of deuterium = 6.023 ´ 1023 Masses of product
Number of atoms in 2 kg of deuterium = 140.9139 + 91.8973 + 3.0261
6.023 ´ 1023 ´ 2 ´ 103 = 235.8373 amu
=
2 Mass defect = 236.0526 - 235.8373 = 0.2153 amu
+26
= 6.023 ´ 10 nuclei Q 1 amu º 931 MeV
Þ Energy released = 0.2153 ´ 931
Energy released during fusion of two deuterium = 200 MeV nearly
= 3.27 MeV Thus, energy is liberated in nuclear fission, if 235
92 U
\ Energy released per deuterium = 1 .635 MeV undergoes nuclear fission.
Energy released in 6.023 ´ 1026 deuterium atoms (ii) Nuclear Fusion The phenomenon of conversion of two
= 1.635 ´ 6.023 ´ 1026 lighter nuclei into a single heavy nucleus is called nuclear
= 9.848 ´ 1026 MeV fusion.
Since, the mass of the heavier product nucleus is less
= 9.848 ´ 1026 ´ 1.6 ´ 10-13
than the sum of masses of reactant nuclei and therefore
= 15.75 ´ 1013 J
130 CBSE Term II Physics XII

certain mass defect occurs which converts into energy as 24. Total target power = 200000 = 2 ´ 105 MW
per Einstein’s mass - energy relation. Thus, energy is Total nuclear power = 10% of total target power
released during nuclear fusion.
10
e.g. 1
+ 1H1 ¾® 1H2 + e + + n + 0.42 MeV = ´ 2 ´ 105 = 2 ´ 104 MW
1H 100
Also, 1H2 + 1H2 ¾® 1H3 + 1H1 + 4.03 MeV Energy produced/fission = 200 MeV
23. The binding energy of H-atom, Efficiency of power plant = 25%
me 4 Energy converted into electrical energy per fission
E = 2 2 = 13. 6 eV …(i) 25
pe0 h = ´ 200 = 50 MeV
100
If proton and neutron had charge e each and were governed
by the same electrostatic force, then in the above equation = 50 ´ 1.6 ´ 10-13 J
we would need to replace electronic mass m by the reduced Total electrical energy to be produced per year
mass m ¢ of proton-neutron and the electronic charge e by e ¢. = 2 ´ 104 MW = 2 ´ 104 ´ 106 W
M ´N M 1836 m = 2 ´ 1010 W = 2 ´ 1010 J/s
m ¢= = = = 918 m
M+N 2 2
= 2 ´ 1010 ´ 60 ´ 60 ´ 24 ´ 365 J/yr
Here, M represents mass of a neutron/proton. Number of fission in one year,
918m ( e ¢ ) 4 2 ´ 1010 ´ 60 ´ 60 ´ 24 ´ 365
\ Binding energy = = 2. 2 MeV …(ii) n=
8e20 h 2 50 ´ 1.6 ´ 10-13
2 ´ 36 ´ 24 ´ 365
Dividing Eq. (ii) by Eq. (i), we get n= ´ 1024
4 8
æ e¢ ö 2.2 MeV 2.2 ´ 106
918 ç ÷ = = Mass of 6.023 ´ 1023atoms of 235 U = 235 g = 235 ´ 10-3kg
èeø 13.6 eV 13.6
4 6
Mass of 235
92 U required to produce
æ e¢ ö 2.2 ´ 10
2 ´ 36 ´ 24 ´ 365
ç ÷ = = 176.21
= ´1024 atoms
èeø 13.6 ´ 918
8

= ( 176.21 )1/ 4 = 3.64 235 ´ 10-3 ´ 2 ´ 36 ´ 24 ´ 365 ´ 1024
e =
6.023 ´ 1023 ´ 8
Given binding energy, B = 2.2 MeV
From the energy conservation law, = 3.08 ´ 104 kg
p2 pp2 Thus, the mass of uranium needed per year is
E - B = Kn + Kp = n + …(iii) 3.08 ´ 104 kg.
2m 2m
From conservation of momentum, 25. (i) In nuclear reaction, to sustain a chain reaction, fast
E moving electrons are slowed by elastic collision with
pn + pp = …(iv) light nuclei.
C
As E = B, Therefore, in reactor moderators are provided with
From Eq. (i), pn2 + pp2 = 0 fissionable nuclei for slowing down fast neutrons.
Some examples of moderators are heavy water (D 2O),
It can only happen, if pn = pp = 0. boron (B) and cadmium (Cd).
So, the Eq. (iii) cannot be satisfied and the process cannot (ii) The sustained fissibility of nuclear chain reaction
take place.
depends on the multiplication factor or simply called
Let E = B + X, where X <<B for the process to take place. multiplicity ( K ).
Put value of pn from Eq. (iv) in Eq. (iii), we get Rate of production of neutrons
K=
æE ö Rate of loss of neutrons
ç - pp ÷ 2
è c ø + pp If K = 1, the operation of reactor is said to be critical. It is
X=
2m 2m what we wish to be, steady power operation.
2 2Epp E2 If K > 1, the reaction rate and reactor power increases
or 2pp - + 2 - 2mX = 0 exponentially. It is called supercritical and reactor can
c c
explode.
Using the formula of quadratic equation, we get If K < 1, the reaction gradually stops and the condition
2E 4E 2 æ E2 ö is called sub-critical.
± 2
- 8 çç 2 - 2mX ÷÷ (iii) The Q-value of a nuclear process refers to the energy
c c èc ø
pp = released in the nuclear process which can be determined
4 using Einstein’s mass-energy relation, E = mc 2 . The
For real value pp , the discriminate is positive Q-value is equal to the difference of mass of products and
4E 2 æ E2 ö reactant nuclei multiplied by square of velocity of light.
\ 2
= 8çç 2 - 2mX ÷÷ The nuclear process does not proceed spontaneously,
c è c ø when Q - value of a process is negative or sum of masses
E2 ~ B 2 of product is greater than sum of masses of reactant.
Þ X= -
4mc 2 4mc 2
CBSE Term II Physics XII 131

CHAPTER 07

Semiconductor
Electronics
(Materials, Devices and Simple Circuits)

In this Chapter...
l Semiconductors l Diode as a Rectifier
l p-n Junction Diode l Special Purpose p-n
Junction Diodes

Classification of Conductors, These different energy levels in different electrons are


formed because inside the crystal, each electron has a unique
Semiconductors and Insulators position and no two electrons are exactly at the same pattern of
On the basis of the relative values of electrical conductivity surrounding charges.
(s ) or resistivity (r = 1 / s ), the solids are broadly classified as
l
Conductors They possess very low resistivity (or high Valence Band
conductivity). The energy band, which includes the energy levels of the
r ~10 - 2 -10 - 8 W-m, s ~10 2 -10 8 Sm - 1 valence electrons is called valence band. This band may be
partially or completely filled with electrons but is never
l
Semiconductors They have resistivity or conductivity empty.
intermediate to metals and insulators.
r ~10 - 5 - 10 6 W-m, s ~10 + 5 - 10 - 6 Sm - 1 Conduction Band
The energy band above the valence band is called conduction
l
Insulators They have high resistivity (or low conductivity). band. At room temperature, this band is either empty or
r ~10 11 - 10 19 W-m, s ~10 - 11 - 10 - 19 Sm - 1 partially filled with electrons. Electrons can jump from
valence band to conduction band and contribute to the
Energy Bands in Solids electric current.
According to Bohr’s atomic model and concept of electronic Energy Band Gap
configuration in an isolated atom, the electrons have certain
definite discrete amounts of energy corresponding to The minimum energy required for shifting electrons from
valence band to conduction band is called energy band gap
different shells and sub-shells, i.e. there are well-defined
( Eg ). It can be zero, small or large depending upon the
energy levels of electrons in an isolated atom.
material.
But in a crystal due to interatomic interaction, valence If l is the wavelength of radiation used in shifting the
electrons are shared by more than one atom. Due to this, electron from valence band to conduction band, then energy
splitting of energy level takes place. The collection of these band gap,
closely spaced energy levels is called an energy band. These E g = hn = hc / l
bands are formed due to the continuous energy variation in
where, h is called Planck’s constant and c is the velocity of
different energy levels.
light.
132 CBSE Term II Physics XII

Difference between Conductor, Insulator and Semiconductor on the basis of Energy Bands

Conductor (Metal) Insulator Semiconductor


No energy gap between the conduction band The valence band is completely filled and The valence band is totally filled and the
and valence band or the conduction band and the conduction band is completely empty. conduction band is empty but the energy gap
valence band overlap each other. The energy gap is quite large. between conduction band and valence band,
So, electrons from below the fermi level can Electrons are bound to valence band and are unlike insulators is very small.
shift to higher energy levels above the fermi not free to move. Hence, electric conduction At room temperature, some electrons in the
level in the conduction band and behave as is not possible in this type of material. valence band acquire thermal energy greater
free electrons. Empty conduction band than energy band gap and jump over to the
conduction band, where they are free to
Conduction Conduction move under the influence of even a small
Electron band band
energy E Eg » 0 EC electric field and acquire small conductivity.
C Electron energy
Overlapped

EV Conduction

Electron energy
Valence Eg > 3 eV EC
Valence band
band
band
area

Eg < 3 eV
EV Valence
For metals
band
EV
Valence band

Fermi Energy l
Thermally excited electrons at T > 0 K, partially occupy the
conduction band. They have come from the valence band
It is the maximum possible energy possessed by free
electrons of a material at absolute zero temperature (i. e. 0 K). leaving equal number of holes there.
The value of fermi energy is different for different materials.
EC EC Electrons

Semiconductors
Electron energy

Eg Eg
The materials whose conductivity lie between metals and
insulators are known as semiconductors. EV EV Holes
At absolute zero temperature, all states in valence band are
filled and all states in conduction band are empty. At low
temperature, pure semiconductors are insulators.
On the basis of purity, semiconductors are of two types (a) (b)
Fig. (a) an intrinsic semiconductor at T = 0K behaves like
Intrinsic Semiconductors insulator and Fig. (b) is representing four thermally
generated electron-hole pairs at T > 0K
This type of semiconductor is also called an undoped
semiconductor or i-type semiconductor. It is a pure
semiconductor without any significant presence of dopant Extrinsic Semiconductors
species. Those semiconductors in which some impurity atoms are
Some characteristics of these semiconductors are as given embedded are known as extrinsic or impurity
below semiconductors.
l
In intrinsic semiconductors, the number of excited When some desirable impurity is added to intrinsic
electrons is equal to number of holes, i.e. n h = n i , where semiconductors deliberately, then this process is called doping
n i is called intrinsic carrier concentration. and the impurities are called dopants.
l
Under the action of an electric field, holes move towards There are two types of dopants used in doping
negative potential giving hole current I h . The total l
Trivalent (valency 3) atoms: e.g. Indium (In), Boron (B),
current I is the sum of the electron current I e and the aluminium (Al), etc.
hole current I h . i.e. I = I e + I h . l
Pentavalent (valency 5) atoms: e.g. Arsenic (As), Antimony
l
At equilibrium, the rate of generation is equal to rate of (Sb), Phosphorous (P), etc.
recombination of charge carriers. The recombination Extrinsic semiconductors are basically of two types
occurs due to an electron colliding with a hole. l
n-type semiconductors
l
An intrinsic semiconductor behaves like an insulator at l
p-type semiconductors
T = 0 K.
CBSE Term II Physics XII 133

n-Type Semiconductors The potential difference developed across the depletion


This type of semiconductor is obtained, when pentavalent region is called the potential barrier (V0 ).
impurity is added to Si or Ge. It depends on dopant concentration in the semiconductor
and temperature of the junction.
Therefore, major conduction in n-type semiconductors is due
Due to this potential barrier, following currents arises in
to electrons. So, electrons are known as majority carriers and
semiconductor
the holes are known as the minority carriers.
l
Due to the diffusion of holes from p-side to n-side and
This means, n e >> n h ; I e >> I h .
electrons from n-side to p-side at the junction, a current
p-Type Semiconductors rises from p-side to n-side, which is called diffusion
This type of semiconductor is obtained, when a trivalent current.
impurity is added to Si or Ge. l
If an electron-hole pair is created on the depletion region
In p-type semiconductor, the holes movement results in the due to thermal collision, the electrons are pushed by the
formation of the current. In this type of semiconductor, electric field towards the n-side and the holes towards the
majority charge carriers are holes, i.e. positively charged and p-side, which gives rise to a current from n-side to p-side
minority charge carriers are electrons. known as drift current.
i.e. n h >> n e ; I h >> I e .
l
In steady state, diffusion current = drift current.
Hence, these conductors are known as p-type Forward Biasing and Reverse
semiconductors or acceptor type semiconductors. Biasing of Junction Diode
The electron and hole concentration in a semiconductor in Biasing is the method of connecting external battery or emf
thermal equilibrium is given by n e n h = n i2 . source to a p-n junction diode.

p-n Junction Diode Forward Biasing


A junction diode is said to be forward biased, when the
It is an arrangement made by a close contact of n-type
positive terminal of the external battery is connected to the
semiconductor and p-type semiconductor.
p-side and negative terminal to the n-side of the diode.
It is basically a p-n junction and it is a two terminal device.
p n When an external voltage V is applied in forward biasing,
It is represented by the symbol then
The direction of arrow indicates the conventional direction of (i) The effective barrier height under forward bias is
current. (V0 - V ).
(ii) Due to external electric field, electron-hole
Formation of Depletion recombination occur and a covalent bond breaks near
Region in p-n Junction p-region producing more electron and hole that moves
During the formation of p-n junction and due to the to the external battery terminals. Thus, the total
concentration gradient across p and n-sides, holes diffuse forward current is the sum of hole diffusion current
from p-side to n-side ( p ® n) and electrons diffuse from and conventional current due to diffusion of electrons.
n-side to p-side ( n ® p ). The diffused charge carriers Hole Junction Electron
p-region n-region
combine with their counterparts in the immediate vicinity of
the junction and neutralise each other. Ei
+ –
Electron diffusion
Electron drift

E
p n
V
+ –
Depletion region
Hole diffusion Battery
Hole drift
Forward biasing of junction diode
p-n junction formation process

This sets up potential difference across the junction and an Reverse Biasing
internal electric field E i directed from n-side to p-side. The A junction diode is said to be reverse biased, when the
region on either side of the junction which becomes depleted positive terminal of the external battery is connected to the
(free) from the mobile charge carriers is called depletion n-side and negative terminal to the p-side of the diode.
region or depletion layer. The width of depletion region is of When an external voltage V is applied across the ends of
the order of 10 -6 m. diode in reverse biasing, then
134 CBSE Term II Physics XII

p n
The direction of applied voltage is same as the direction of Reverse voltage (V)
barrier potential, so effective barrier height will be V0 + V. –10 –8 –6 –4 –2 0

Reverse current (mA)


There is almost no flow of current due to majority charge –
V
+ O
+ C 2
carriers, a very small current due to minority charge carriers
mA
flows across the junction. This current is called reverse – 4
current. Breakdown
6
Junction voltage
Battery 8
Hole – +
p-region n-region Electron D
(a) (b)
– Ei + Reverse biased characteristic of a diode
l
In reverse biased, the applied voltage supports the flow
E
of minority charge carriers across the junction. So, a very
small current flows across the junction due to minority
charge carriers.
V
– + l
The reverse current is voltage independent upto certain
Battery
voltage known as breakdown voltage and this voltage
independent current is called reverse saturation current.
Reverse biasing of junction diode
Note If the reverse bias is equal to the breakdown voltage, then the
reverse current through the junction increases very rapidly (CD
I-V (Current-Voltage) Characteristics of portion of the graph), this situation is called avalanche
breakdown and the junction may get damaged due to excessive
p-n Junction Diode heating, if this current exceeds the rated value of
The graphical relations between voltage applied across p-n p-n junction.
junction and current flowing through the junction are called In diodes, a resistance is offered by the junction which
I-V characteristics of junction diode. depends on the applied voltage, which is called dynamic
resistance. It is the ratio of small change in voltage to the
Forward Biased Characteristics small change in current produced.
The circuit diagram and graph plotted between voltage and DV
Dynamic resistance, rd = .
current for diode are shown in Figs. (a) and (b). DI
p n
7
Diode as a Rectifier
Forward current (mA)

B
6

+ – 5 The process of converting alternating voltage/current into
mA V
+
direct voltage/current is called rectification.
4 Ge
3 Principle
2 Diode allows current to pass only, when it is forward
1 A biased. So, if an alternating voltage is applied across a
+ – O diode, the current flows only in that part of the cycle when
Battery 0 0.1 0.2 0.3 0.4 0.5 the diode is forward biased.
(a) Forward voltage (V)
(b)
Diode as a Half-Wave Rectifier
l
At the start when applied voltage is low, the current through
In the half-wave rectifier, the AC voltage to be rectified is
the diode is almost zero. It is because of the potential barrier,
connected to the primary coil of a step-down transformer
which opposes the applied voltage. and secondary coil is connected to the diode through
l
With further increase in applied voltage, the current resistor R L across, which output is obtained.
increases very rapidly and diode behaves like a conductor. Transformer A X
The forward voltage beyond which the current through the
junction starts increasing rapidly with voltage is called knee
voltage or threshold voltage. Primary Secondary RL
Reverse Biased Characteristics
The circuit diagram and graph plotted between voltage and Y
B
current for reverse biased diode are shown in Figs. (a) and (b).
Circuit diagram of half-wave rectifier
CBSE Term II Physics XII 135

Working Role of Filters


During positive half cycle of the input AC, the p-n junction In order to get the steady DC output from the pulsating
is forward biased. Thus, the resistance in p-n junction voltage normally, a capacitor is connected across the output
becomes low and current flows. Hence, we get output in the terminals (parallel to load R L ). They are called filters.
load. During negative half cycle of the input AC, the p-n
junction is reverse biased. Hence, no output is in the load. DC component
X

Rectifier
Voltage at A

+
Input AC AC C RL DC

+ +
Y
– – t (a)
Voltage across R L

input filter AC input


Output AC t

Output with
+ +

capacitor
t
Input and output waveforms
(b)
Diode as a Full Wave Rectifier
A full wave rectifier with capacitor filter Fig. (a) and
In the full wave rectifier, two p-n junction diodes D1 and D 2 are input and output voltage of rectifier in Fig. (b).
used. This arrangement is as shown in the diagram below
Centre tap
transformer
D1 Special Purpose p-n
A

Centre
Junction Diodes
tap
X Photodiode
B
It is a special type of junction diode used for detecting
RL Output
D2 optical signals. It is a reverse biased p-n junction made from
Y
a photosensitive material. Its symbol is

Circuit diagram of full wave rectifier


p
+

n
Working
In full wave rectifier, we get output in the load resistance Construction
l
In positive half cycle by D1 . A photodiode fabricated with a transparent cover to allow
l
In negative half cycle by D 2 . light to fall on the diode and operates under reverse bias.
waveform at A

(hn > Eg )
Input

O
t
mA
p-side n-side
waveform at B
Input

O – +
R
t
V
Due to Due to Due to Due to A reverse biased photodiode illuminated with light
D1 D2 D1 D2
Output waveform

Working
(across RL)

O When the photodiode is illuminated with light (photons),


t
with energy greater than the energy gap of the
semiconductor, then electron-hole pairs are generated due to
the absorption of photons. These charge carriers contribute
Input and output waveforms to the reverse current.
136 CBSE Term II Physics XII

V-I Characteristics metal contact acts as negative electrode. When an external


load is connected across metal electrodes, a photocurrent
We observe from the figure that, current in photodiode changes
flows.
with the change in light intensity I, when reverse bias is applied.
mA I-V Characteristics I Open circuit
I-V characteristics is drawn voltage (Voc)
Reverse bias in the fourth quadrant of the V
Reverse volt coordinate axes because a O
I1 current
I2 solar cell does not draw
ISC
I3 current but supplies the Short circuit current
I4 same to the load. I-V characteristics of a solar cell
I 4 > I3 > I2 > I 1 mA
V-I characteristics of photodiode at different intensities Light Emitting Diode (LED)
Solar Cell It is a heavily doped p-n junction diode which hn
converts electrical energy into light energy.
It is a p-n junction diode, which converts solar energy into This diode emits spontaneous radiation,
+ –
electrical energy. p n
under forward biasing. Its symbol is
hn
Working
On recombination of electron and hole, the energy is given
Top Metallised
surface finger electrode out in the form of heat and light.
n
p
p p R
n n
p n B
p
Back contact
Forward biased LED
p-n junction of solar cell

Its symbol is
+ V-I Characteristics
hn
The colour of light emitted by a given LED, depends on its
p
band gap energy. The photon emitted by an LED is of
n energy equal to or slightly less than the band gap energy.
Forward current conducted by the junction determines the
intensity of light emitted by LED.

I (mA)
30 Silicon
Construction
It consists of a silicon or gallium-arsenide p-n junction diode 20
packed in a can with glass window on the top. 15
RL 10
IL
hn –10V
V (volt)
0 0.5 0.8
I (mA)

p n
V-I characteristics of LED

Depletion region LEDs Advantages over Incandescent


A typical illuminated p-n junction solar cell Low Power Lamps
LED’s advantages over incandescent low power lamps
Working
which are as given below
When photons of light (of energy hn > Eg ) falls at the junction, l
Fast action and no warm up time required.
electron-hole pairs are generated near the junction and they
move in opposite directions due to junction field. They will be l
The bandwidth of emitted light is from 100 Å to
collected at the two sides of the junction, giving rise to a 500 Å, so it is nearly (not exactly) monochromatic.
photovoltage between the top and bottom metal electrodes. l
Long life and ruggedness.
The top metal contact acts as positive electrode and bottom l
Low operational voltage and less power consumed.
CBSE Term II Physics XII 137

Solved Examples
ND
Example 1. The maximum wavelength at which solid and ne = = 25. ´ 1021
begin to absorb energy is 10000 Å. Calculate the 2
energy gap of a solid (in eV). So, n final = nh + ne
hc Þ n final . ´ 1021
» n e » 25 (Q n e >> n h)
Sol. The energy band gap is given by, Eg = hn = 21 15
l n final - n initial 25
. ´ 10 - 14 ´ 10
Factor = =
where, h = Planck’s constant, c = speed of light n initial 14 ´ 1015
and l = wavelength at which solid absorbs energy.
. ´ 1021
25
On putting the values of h, c and l, we get » = 1.8 ´ 105
14 ´ 1015
(6.626 ´ 10-34 J- s)(3 ´ 108 m / s)
Eg =
(10000 ´ 10-10 m) Example 4. (a) Calculate the value of V 0 and i, if the
-19
= 1.98 ´ 10 J silicon and germanium diode start conducting at
1.98 ´ 10-19 0.7 V and 0.3 V, respectively.
= eV Ge
1.6 ´ 10-19
= 1.24 eV i V0
Si
Example 2. In an intrinsic (pure) semiconductor, the 12 V RL 5 kW
number of conduction electrons is 7 ´ 1019 per
cubic metre. Find the total number of current
carriers (electrons and holes) in the same
semiconductor of size 1 cm ´ 1 cm ´ 1mm. (b) If the Ge diode connection is now reversed, what
Sol. In an intrinsic semiconductor, n e = n h will be the new values of V 0 and i?
where, n e = number of conduction electrons Sol. (a) Ge diode will start conducting before the silicon diode
does so. The effective forward voltage across Ge diode is
and n h = number of holes per unit volume.
(12 - 0.3) V =11.7 V. This will appear as the output
Given, n e = 7 ´ 1019 per m 3
voltage across the load, i.e.,
\ n h = n e = 7 ´ 1019 per m 3
Vo = 11.7 V
So, total current carrier density The current through R L,
n e + n h = 7 ´ 1019 + 7 ´ 1019 11.7
i= A = 2.34 mA
= 14 ´ 1019 per m 3 5 ´ 103
Now, total number of current carrier (b) On reversing the connection of Ge diode, it will be
= number density ´ volume reverse biased and conduct no current. Only Si diode will
= ( 14 ´ 1019 per m 3 ) ´ ( 10-2 m ´ 10-2 m ´ 10-3 m conduct. Therefore,
Vo = ( 12 - 0.7) V = 11.3 V
= 1.4 ´ 1013
11.3
and current, i = A = 2.26 mA
Example 3. The concentration of hole-electron pairs in 5 ´ 103
pure germanium at T = 300 K is 7 ´ 1015 per cubic Example 5. A p-n photodiode is made of a material
metre. Antimony is doped into germanium a with a band gap of 1.5 eV. What is the minimum
proportion of 1 atom 10 7 Ge atoms. Assuming that wavelength of radiation that can be absorbed by the
half of the impurity atoms contribute electron in material?
the conduction band, calculate the factor by which hc
the number of charge carriers increases due to Sol. Energy, E = hn =
l
doping the number of germanium atoms per cubic The minimum wavelength of radiation
metre is 5 ´ 10 28 . hc (6.4 ´ 10-34 J - s) ´ (3 ´ 108 ms -1 )
l= =
Sol. In pure semiconductor electron-hole pair = 7 ´ 1015 m -3 E 1.5 ´ 1.6 ´ 10-19 J
Total charge carrier, n total initial = n h + n e = 14 ´ 1015 = 8.3 ´ 10-7 m = 830 nm
1.5 ´ 1.6 ´ 10-19 J
5 ´ 1028 = » 1.2 ´ 106 Hz
After doping donor impurity, N D = = 5 ´ 1021 6.6 ´ 10-34 J - s ´ 3 ´ 108 ms -2
107
138 CBSE Term II Physics XII

Chapter
Practice
PART 1
Ec Ec
Objective Questions (c) Eg (d) Eg
Ev Ev
l
Multiple Choice Questions
Electrons
1. The conductivity of a semiconductor increases with
increase in temperature, because [NCERT Exemplar] 5. The substance which is doped in an intrinsic
(a) number density of free current carriers increases semiconductor to make p-type semiconductor is
(b) relaxation time increases (a) phosphorus (b) antimony
(c) Both number density of carriers and relaxation time (c) aluminium (d) arsenic
increase
(d) number density of carriers increases, relaxation time 6. In n-type semiconductor, electrons are majority
decreases but effect of decrease in relaxation time is charge carriers but it does not show any negative
much less than increase in number density charge. The reason is
(a) electrons are stationary
2. Correct order of relative values of electrical (b) electrons neutralise with holes
conductivity s for different types of solid is (c) mobility of electrons is extremely small
(a) s semiconductor > s insulator > s metal (d) atom is electrically neutral
(b) s metal > s semiconductor > s insulator
(c) s semiconductor > s metal > s insulator 7. Pure silicon at 300 K has equal electron ( n e ) and
(d) s insulator > s semiconductor > s metal hole ( n h ) concentration of 1.5 ´ 1016 m - 3 . Doping
3. Which of the following correctly represents the by indium increases n h to 4.5 ´ 10 22 m - 3 . The n e
hole in semiconductor? in doped silicon is (in m -3 )
(a) An anti-particle of electron (a) 9 ´ 105 (b) 5 ´ 109
(b) A vacancy created when an electron leaves a covalent (c) 2.25 ´ 1011 (d) 3 ´ 1019
bond
(c) Absence of free electrons 8. When an electric field is applied across a
(d) An artificially created particle semiconductor,
4. Energy band gap Eg diagram for an intrinsic (a) electrons move from lower energy level to higher energy
semiconductor at temperature T > 0 K is level in the conduction band
(b) electrons move from higher energy level to lower energy
(Here, E C is energy for conduction band and E V is level in the conduction band
energy for valence band.) (c) holes in the valence band move from lower energy level
Electrons to higher energy level
Holes (d) None of the above
Ec 9. The barrier potential of a p-n junction depends on
Ec
[CBSE 2014]
(a) Eg Electrons (b) Eg Electrons (i) type of semiconductor material
Ev Ev
(ii) amount of doping
(iii) temperature
CBSE Term II Physics XII 139

Which one of the following is correct? (c) would be like a half-wave rectifier with negative cycles in
output
(a) Both (i) and (ii) (b) Only (ii)
(d) would be like that of a full wave rectifier
(c) Both (ii) and (iii) (d) (i), (ii) and (iii)
14. The wavelength and intensity of light emitted by a
10. In figure given below, V 0 is the potential barrier LED depend upon
across a p-n junction, when no battery is connected (a) forward bias and energy gap of the semiconductor
across the junction, then [NCERT Exemplar] (b) energy gap of the semiconductor and reverse bias
1
2
(c) energy gap only
V0 3 (d) forward bias only
15. The I-V characteristic of an LED is
R YG B
(a) 1 and 3 both correspond to forward bias of junction R
G
(b) 3 corresponds to forward bias of junction and 1 (a) I (b) Y
corresponds to reverse bias of junction R
(c) 1 corresponds to forward bias and 3 corresponds to
reverse bias of junction O R V O V
Y
(d) 3 and 1 both correspond to reverse bias of junction G V O
B Red
11. A 220 V AC supply is connected between points A (c) I
(d) Yellow
R I Green
and B (figure). What will be the potential Y
Blue
G
difference across the capacitor? [NCERT Exemplar] O V B
A
l
Assertion-Reasoning MCQs
200 AC C V
Direction (Q. Nos. 16-20) Each of these questions
B contains two statements Assertion (A) and Reason (R).
(a) 220 V (b) 110 V Each of these questions also has four alternative
(c) 0 V (d) 220 2 V choices, any one of which is the correct answer. You
have to select one of the codes (a), (b), (c) and (d) given
12. V-I characteristics of a silicon diode is as shown. below.
I (mA)
(a) Both A and R are true and R is the correct
30
explanation of A
Silicon (b) Both A and R are true, but R is not the correct
20
explanation of A
15
(c) A is true, but R is false
10
(d) A is false and R is also false

–10 V
16. Assertion The energy gap between the valence
0.8
0 0.5 0.7 V (volts) band and conduction band is greater in silicon than
1 mA in germanium.

The ratio of resistance of diode at I D = 15 mA and Reason Thermal energy produces fewer minority
V D = - 10 V, is carriers in silicon than in germanium.
(a) 10-3 (b) 10-4 (c) 10-5 (d) 10-6 17. Assertion The total current I in a semiconductor is
13. The output of the given circuit in figure given the sum of electron current and hole current.
below [NCERT Exemplar] Reason In a semiconductor, I h arises due to the
motion of holes towards positive potential and free
electrons under an applied electric field.
Vm sin wt 18. Assertion The resistivity of a semiconductor
decreases with temperature.
Reason The atoms of a semiconductor vibrate
(a) would be zero at all times with larger amplitudes at higher temperature
(b) would be like a half-wave rectifier with positive cycles in thereby increasing its resistivity.
output
140 CBSE Term II Physics XII

19. Assertion A hole on p-side of a p - n junction moves (ii) In figure, the input is across the terminals A and C
to n-side just an instant after drifting of charge and the output is across B and D. Then, the output
carriers occurs across junction. is
B C
Reason Drifting of charge carriers reduces the
concentration gradient across junction.
20. Assertion Light Emitting Diode (LED) emits A D
spontaneous radiation. (a) zero (b) same as the input
Reason LED are forward biased p-n junctions. (c) half wave rectified (d) full wave rectified
(iii) Which of the following is not true about a rectifier
l
Case Based MCQs circuit?
(a) It can convert DC to AC. (b) It can convert AC to DC.
21. Direction Read the following passage and answer (c) It can shift voltage level. (d) None of these
the questions that follows
(iv) In the given circuit,
Full Wave Rectifier
The process of converting alternating
voltage/current into direct voltage/current is called Rectifier RL DC
AC
rectification. Diode is used as a rectifier for C
converting alternating current/voltage into direct
current/voltage.
Capacitor C is used
Diode allows current to pass only, when it is (a) for storing potential energy
forward biased. So, if an alternating voltage is (b) as a bypass to DC component to get AC in R L
applied across a diode, the current flows only in (c) to remove sparking
that part of the cycle when the diode is forward (d) as a bypass to AC component to get DC in R L
biased. This property is used to rectify the (v) The ratio of output frequencies of half-wave
current/voltage. rectifier and a full wave rectifier, when an input of
Centre tap frequency 200 Hz is fed at input, is
transformer
D1 (a) 1 : 2 (b) 2 : 1
A
(c) 4 : 1 (d) 1 : 4
Centre
tap
B
X
PART 2
RL Output
D2

Y
Subjective Questions
Circuit diagram of full wave rectifier
l
Short Answer (SA) Type Questions
1. Carbon and silicon both have four valence
(i) If in a p-n junction, a square input signal of 10 V is electrons each, then how are they distinguished?
applied as shown [Delhi 2011C]
+5 V 2. Draw energy band diagram of n-type and p-type
RL semiconductors at temperature T > 0K. Mark the
donor and acceptor energy level with their
–5 V energies. [Foreign 2014]
Then, the output across R L will be 3. Distinguish between intrinsic and extrinsic
10 V
semiconductors. [All India 2015]
(a) (b)
4. The number of conduction electrons (in per cubic
–10 V
metre) present in a pure semiconductor is
6.5 ´ 1019 . Calculate the number of holes in a
5V
(c) –5 V (d) sample having dimensions 1 cm ´ 1 cm ´ 2 mm ?
CBSE Term II Physics XII 141

5. Why are elemental dopants for silicon or 12. In half-wave rectification, what is the output
germanium usually chosen from group XIII or frequency. If the input frequency is 50 Hz, what is
group XV? [NCERT Exemplar] the output frequency of a full wave rectifier for the
6. As we know that, a p-type semiconductor has large same input frequency? [NCERT]

number of holes but it is still electrically neutral. 13. Explain briefly how a photodiode operates.
Why? [CBSE 2018C]

7. The impurity levels of doped semiconductor are 14. Three photodiodes D1 , D 2 and D 3 are made of
30 eV below the conduction band. semiconductors having band gaps of 2.5 eV, 2 eV
Determine whether the semiconductor is n-type or and 3 eV, respectively. Which one will be able to
p-type. detect light of wavelength 6000 Å?
[NCERT Exemplar]
At the room temperature, thermal collisions occur
as a result of which, the extra electron loosely 15. Mention the important considerations required
bound to the impurity ion gets an amount of energy while fabricating a p-n junction diode to be used as
kT and hence this electron can jump into a Light Emitting Diode (LED). What should be the
conduction band. What is the value of T? (Take, k is order of band gap of an LED, if it is required to
Boltzmann constant = 8.62 ´ 10 -5 eV/K) emit light in the visible range? [Delhi 2013]
8. Write differences between forward bias and reverse 16. (i) Explain with the help of a diagram the formation
bias. [All India 2020] of depletion region and barrier potential in a p - n
9. There are two semiconductor materials junction.
A and B which are made by doping germanium (ii) Draw the circuit diagram of a half-wave rectifier
crystal with indium and arsenic, respectively. As and explain its working. [All India 2016]
shown in the figure, the junction of two is biased 17. Draw the circuit diagram of a full wave rectifier
with a battery. Will the junction be forward bias or and explain its working. Also, give the input and
reverse bias? output waveforms. [Delhi 2019]
A B
18. (i) In the following diagram, which bulb out of B1
and B 2 will glow and why?
D1 D2
10. The V-I characteristic of a silicon diode is as shown
in the figure. Calculate the resistance of the diode
at (i) I = 5 mA and (ii) V = -20V. [Delhi 2020] +9 V
B1 B2
I (mA)

30

20
Silicon (ii) Draw a diagram of an illuminated
15 p-n junction solar cell.
10 (iii) Explain briefly the three processes due to which
–20 V generation of emf takes place in a solar cell.
0 0.5
[All India 2020, 17]
I (mA) 0.7 0.8 V
19. What is photodiode ? Breifly explain its working
11. Assuming that, the two diodes D1 and D 2 are used and draw its V-I characteristics. [Delhi 2020]
in the electric circuit shown in the figure are ideal.
Find out the value of the current flowing through
l
Long Answer (LA) Type Questions
1W resistor. 20. The number of silicon atoms per m 3 is
D1 2W
5 ´ 10 28 . This is doped simultaneously with
D2 5 ´ 10 22 atoms per m 3 of arsenic and 5 ´ 10 20 atoms
2W
per m 3 of indium. Calculate the number of
1W electrons and holes. Given that, n i = 1.5 ´ 1016 m - 3 .
+ –

6V
Is the material n-type or p-type? [NCERT]
142 CBSE Term II Physics XII

21. Predict the effect on the electrical properties of a (ii) Which semiconductors are preferred to make
silicon crystal at room temperature, if every LEDs and why?
millionth silicon atom is replaced by an atom of (iii) Give two advantages of using LEDs over
indium. Given, concentration of silicon atoms conventional incandescent low power lamps.
= 5 ´ 10 28 m -3 , intrinsic carrier concentration [All India 2011]
= 1.5 ´ 1016 m -3 , m e = 0.135 m 3 / V - s and 27. (i) What is the advantage of using GaAs for
m h = 0.048m 3 / V-s. synthesis of solar cells?
(ii) Draw V-I characteristics of solar cell and
22. A potential barrier of 0.4V exists across p-n junction. mention its significance.
(i) If the depletion region is 4.0 ´ 10 -7 m wide, what
is the intensity of the electric field in this region?
l
Case Based Questions
(ii) If an electron with speed 4 ´ 105 m/s approaches Direction Read the following passage and answer the
the p-n junction from the n-side, find the speed questions that follows
with which it will be p- side. 28. Photodiode
23. Assuming an ideal diode, draw the output Photodiode is a special type of junction diode used
waveform for the circuit given in the figure, explain for detecting optical signals. It is a reverse biased
the waveform. [NCERT Exemplar] p-n junction made from a photosensitive material.
R
Its symbol is


p

+
n
20 sin wt
5V A photodiode fabricated with a transparent cover to
allows light to fall on the diode and operated under
reverse bias. A photodiode is used in sensor
24. Draw V-I characteristics of a p-n junction diode. circuits.
Answer the following questions giving reasons. (hn > Eg )
(i) Why is the current under reverse bias almost
independent of the applied potential upto a
critical voltage? mA
p-side n-side
(ii) Why does the reverse current show a sudden
increase at the critical voltage? R
– +
25. If each diode in figure has a forward bias resistance V
of 25 W and infinite resistance in reverse bias, what A reverse biased photodiode
will be the values of the currents I1 , I 2 , I 3 and I 4 ? illuminated with light

I4 125 W (i) The current in the forward bias is known to be


A B
more (~mA ) than the current in the reverse bias
125 W
C
I3
D
(~mA ). What is the reason to operate the
125 W
photodiode in reverse bias? [Delhi 2012]
I2
E F (ii) Three photodiodes D1 , D 2 and D 3 are made of
I1 25 W semiconductors having band gaps of 15 . eV ,
G H
2.5 eV and 35. eV, respectively. Which of them
5V
[NCERT Exemplar] will not be able to detect light of wavelength
450 nm?
26. (i) Describe the working of Light Emitting Diodes
(LEDs). (iii) Write main use of photodiode.
Chapter Test
Multiple Choice Questions Short Answer Type Questions
1. In an unbiased p-n junction, holes diffuse from the 6. Draw the circuit diagram showing how a p - n
p-region to n-region because junction diode is
(a) free electrons in the n - region attract them
(i) forward biased and (ii) reverse biased.
(b) they moves across the junction by the potential difference
(c) hole concentration in p-region is more as compared to hole How is the width of depletion layer affected in the
concentration in n-region two cases?
(d) All of the above 7. Assuming that the resistances of the meters are
2. Which of these graphs shows potential difference between negligible, what will be the readings of the
p- side and n-side of a p-n junction in equilibrium? ammeters A1 and A2 in the circuit shown in figure?
20 W
A1
(a) p-side n-side (b) p-side
n-side
Junction Junction 4V 20 W
plane plane A2

8. How do you obtain steady DC output from the


(c) p-side (d) p-side pulsating voltage?
n-side n-side 9. Give two advantages of LED’s over the conventional
Junction Junction incandescent lamps.
plane plane
Long Answer Type Questions
3. Which is reverse biased diode? 10. (i) Can a slab of p-type semiconductor be physically
joined to another n-type semiconductor slab to
(a) (b) – 20 V form p-n junction? Justify your answer.
5V
–10 V (ii) In a p-n junction diode, the forward bias
resistance is low as compared to the reverse bias
(c) 15 V (d) 20 V resistance. Give reason.
11. (i) Explain the formation of energy bands in
10 V –5 V crystalline solids.
4. The diode shown in the circuit is a silicon diode. The (ii) Draw the energy band diagrams of (a) a metal
potential difference between the points A and B will be and (b) a semiconductor.
2W A S B 12. Explain with help of circuit diagram, the action of a
forward biased p-n junction diode which emits
spontaneous radiation. State the least band gap
energy of this diode to have emission in visible
6V
region.
(a) 6 V (b) 0.6 V (c) 0.7 V (d) 0 V
13. Write two characteristics features to distinguish
5. The current through an ideal p-n junction shown in the
between n-type and p- type semiconductors.
following circuit diagram will be
14. A photodiode is operated under reverse bias
p n 100 W
although in the forward bias, the current is known to
be more than the current in the reverse bias. Explain,
1V 2V giving reason.
15. Describe the working principle of a solar cell.
Mention three basic processes involved in the
(a) zero (b) 1 mA (c) 10 mA (d) 30 mA generation of emf.

Answers
Multiple Choice Questions For Detailed Solutions
1. (c) 2. (c) 3. (b) 4. (a) 5. (a)
Scan the code
144 CBSE Term II Physics XII

EXPLANATIONS
PART 1 carriers will change. It also depends on temperature due to
which the number of minority carriers will change.
1. (d) The conductivity of a semiconductor increases with
10. (b) When p-n junction is forward biased, it opposes the
increase in temperature because the number density of
potential barrier junction, when p-n junction is reverse
current carriers increases, relaxation time decreases but
biased, it supports the potential barrier junction, resulting in
effect of decrease in relaxation time is much less than
increase in potential barrier across the junction.
increase in number density.
So, 3 corresponds to forward bias of junction and 1
2. (b) The values of conductivity and resistivity for different
corresponds to reverse bias of junction.
types of solids are as follows
11. (d) As p-n junction conducts during positive half cycle only,
(i) Metal r ~10-2 -10-8 W-m
so the diode connected here will work in positive half cycle.
s ~102 -108 Sm -1
Potential difference across C = peak voltage of the given AC
(ii) Semiconductor voltage
r ~10-5 -106 W-m
V0 = Vrms 2 = 220 2 V
s ~105 - 10-6 Sm -1
12. (d) From the graph, given in question at I = 20 mA, V is
(iii) Insulator
r ~1011 - 1019 W-m 0.8 V and at I = 10 mA, V = 0.7 V
DV 0.8 - 0.7 0.1 V
s ~10-11 - 10-19 S m -1 \ rf b = = =
DI 20 - 10 10 mA
As 108 > 10-6 > 10-19 , so s metal > s semiconductor > sinsulator . 0.1 V
= = 10 W
3. (b) The concept of hole describes the lack of an electron at a 10 ´ 10-3 A
position, where one could exist in an atom or atomic lattice.
Also, at V = - 10 V, I = - 1 mA
If an electron is excited into a higher state, it leaves a hole in
10 V 10 V
its old state. rrb = = = 1 ´ 107 W
Thus, hole can be defined as a vacancy created when an 1 mA 1 ´ 10-6 A
electron leaves a covalent bond. r 10
\ Ratio = f b = 7 = 10-6
4. (d) In an intrinsic semiconductor, at T > 0K, due to thermal rrb 10
energy some electrons from the valence band excites to 13. (c) Due to forward biased during positive half cycle of input
conduction band and partially occupy it. Some electrons are AC voltage, the resistance of p-n junction is low. The current
in the conduction band, these have come from valence band in the circuit is maximum. In this situation, a maximum
leaving equal number of holes there. potential difference will appear across resistance connected
5. (c) In an intrinsic semiconductor, when an impurity of in series of circuit. This result into zero output voltage
trivalent group such as aluminium, boron, etc., mixed in across p-n junction.
very small quantity, then the resultant crystal will be p-type Due to reverse biased during negative half cycle of AC
semiconductor. voltage, the p-n junction is reverse biased. The resistance of
6. (d) The n-type semiconductor, region has (negative) p-n junction becomes high which will be more than
electrons as majority charge carriers and an equal number of resistance in series. That is why, there will be voltage across
fixed positively charged donor ions. Again, the material as a p-n junction with negative cycle in output.
whole is neutral. That is a reason, atom is electrically 14. (a) LED is a heavily doped p-n junction diode which emits
neutral. spontaneous radiation, under forward biasing. The colour
7. (b) In an extrinsic semiconductor, (wavelength and intensity) of the emitted radiation depend
n e n h = n i2 on its band gap energy.
Þ n e ´ 4.5 ´ 1022 = (1.5 ´ 1016 ) 2 15. (a) An LED operates in forward bias mode, so its I-V
characteristics is similar to a forward biased p-n junction
2.25 ´ 1032
ne = diode. Secondly, the threshold voltage increases with
4.5 ´ 1022 decrease in wavelength. Thus, option (a) is correct.
Þ n e = 5 ´ 109 m -3 16. (b) The energy gap between valence band and conduction
8. (a) When an electric field is applied across a semiconductor, band in germanium is 0.76 eV and the energy gap between
the electrons in the conduction band get accelerated and valence band and conduction band in silicon is 1.1 eV.
acquire energy. They move from lower energy level to higher Also, it is true that thermal energy produces fewer minority
energy level. While the holes in valence band move from carriers in silicon than in germanium.
higher energy level to lower energy level, where they will Therefore, both A and R are true but R is not the correct
be having more energy. explanation of A.
9. (d) Barrier potential depends on the material used to make 17. (c) In a semiconductor at T > 0K, electrons originally set
p-n junction diode (whether it is Si or Ge). It is also depends
free are not involved in the process of hole motion. These
on amount of doping due to which the number of majority
free electrons moves completely as conduction electron and
CBSE Term II Physics XII 145

give rise to an electron current Ie , under an applied electric AC input is applied across A and C and output is taken
field. However, motion of hole is only a convenient way of across BD.
describing the actual motion of bound electrons wherever V0
there is an empty bond anywhere in the crystal. Thus, under
the action of an electric field these holes move towards
negative potential giving the hole current Ih.
The total current I is thus the sum of the electron current Ie D1D4 D2D3 D1D4 D2D3
and hole current Ih,
I = Ie + Ih
Therefore, A is true but R is false. When positive cycle is fed to AC, D1 and D4 conduct and
when negative cycle is fed to AC, D3 and D2 conduct in
18. (c) Resistivity of a semiconductor decreases with rise in the the same direction. Output across BD is thus full wave
temperature. The atoms of a semiconductor vibrate with rectified.
larger amplitudes at higher temperature, thereby increasing
its conductivity not resistivity. (iii) (a) A rectifier can convert AC to DC. It can also shift or add
voltage level in the output but it cannot convert DC to AC.
Therefore, A is true but R is false.
(iv) (d) In the given circuit, for the first half-cycle of rectified
19. (d) In a p-n junction, due to diffusion of electrons, a positive
output when the voltage across C will rise, it gets charged.
space-charge region on n-side of the junction and a negative Then, in the presence of R L, it gets discharged through it
space charge region on p-side of the junction is formed. Due and the voltage begins to fall.
to this, an electric field directed from positive charge
towards negative charge develops (electric field is from In the next half-cycle of rectified output, it again gets
n-side to p-side). Due to this field, an electron on p-side of charged to peak value and will similarly gets discharged
through R L. Thus, a steady DC output from pulsating
the junction moves to n-side and a hole on n-side of the
voltage is obtained. In other words, capacitor helps to filter
junction moves to p-side. This motion of charge carriers due
to the electric field is called drift. Thus, a drift current, out the AC ripple and give a pure DC voltage or bypass AC
which is opposite in direction to the diffusion current starts. component to get DC.
However, concentration gradient is due to the doping of (v) (a) Output frequency of full wave rectifier is twice the
sides. It is not affected by drift of charge carriers. output frequency of half-wave rectifier.
Therefore, A is false and R is also false. f half -wave 1
\ =
20. (a) When a junction diode is forward biased as shown in f full wave 2
figure, energy is released at the junction due to
recombination of electrons and holes. In the junction diode
made of gallium arsenide or indium phosphide, the energy PART 2
is released in visible region.
Light 1. The four valence electrons of carbon are present in second
p orbit while that of silicon in third orbit. So, energy required
RL to extricate an electron from silicon is much smaller than
n carbon.
– + Therefore, the number of free electrons for conduction in
silicon is significant on contrary to the carbon. This makes
Such a junction diode is called light emitting diode or LED. silicon’s conductivity much higher than carbon. This is the
It is a heavily doped p-n junction diode, so the radiations main distinguishable property.
emitted are spontaneous in forward biasing.
2. The required energy band diagram is as shown below
Therefore, both A and R are true and R is the correct
explanation of A.
Conduction band
21. (i) (d) As it is forward biased, so it takes positive value.
Hence, option (d) is correct. Acceptor energy level
(ii) (d)
B 10.04 eV
D1 Valence band
D2
(a) p-type
A C
RL
Conduction band
D3 10.045 eV
D4

D Donor energy level


Valence band
(b) n-type
146 CBSE Term II Physics XII

3. Difference between intrinsic and extrinsic semiconductors 9. As, semiconductor A is doped with indium, so it behaves as
is as given below p-type semiconductor and B is doped with arsenic, so it
behaves as n-type semiconductor. Thus, the figure shows
Intrinsic semiconductor Extrinsic semiconductor
that, it is forward bias condition.
It is a pure It is prepared by doping a small 10. (i) From the given curve, we have
semiconductor material quantity of impurity atoms to the
with no impurity atoms in pure semiconductor. Voltage, V = 0.5 V for current of 10 mA
it. 0 .5
So, for 5 mA, V = ´ 5 = 0.25 V
10
The number of free The number of free electrons
V 0. 25
electrons in the and holes is never equal. There \ Resistance, R = = = 50W
conduction band and the is an excess of electrons n e > n i I 5 ´ 10-3
number of holes in in n-type semiconductors and (ii) For V = -20 V, we have
valence band is exactly excess of holes in p-type I = -1 mA = -1 ´ 10-6 A
equal. semiconductors.
20
ne = nh = ni nh > ni Þ R= = 2.0 ´ 107 W
1 ´ 10-6
4. Given, number of conduction electrons, n e = 6.5 ´ 1019 m -3 11. According to the question, D2 is reverse biased, so do not
Volume of the sample = 1 cm ´ 1 cm ´ 2 mm = 2 ´ 10-7 m 3 conduct.
Number of holes in the sample, n h = Number of electrons in D1 2W
the sample, n e A B
= n e ´ V = 65. ´ 1019 ´ 2 ´ 10-7
D2 2W
= 1 .3 ´ 1013 D C
5. The size of the dopant atom should be such that their
presence in the pure semiconductor does not distort the 1W
+ –
semiconductor but easily contribute the charge carriers on E F
forming covalent bonds with Si or Ge atoms, which are 6V
provided by group XIII or group XV elements. R EF = 2 + 1 = 3 W
6. p-type semiconductor is formed by doping it with trivalent V 6
IEF = = = 2A
impurities. These impurities or dopant takes the atoms in R EF 3
the crystal and its three electrons take part in chemical 12. A half-wave rectifier rectifies only the half of AC input, i.e.
bonding with three electrons of intrinsic semiconductor or it conducts once during an AC input cycle while a full-wave
pure semiconductor, whereas the last bond are left free. rectifier rectifies both the half cycles of the AC input, i.e. it
Since, as whole atom is electrically neutral, so p -type conducts twice during a cycle.
semiconductor is also neutral. \ The output frequency for half-wave rectifier is 50 Hz
7. The separation of impurity energy level from conduction and the output frequency of a full wave rectifier is
band is less in case of n-type semiconductor and more in 2 ´ 50 = 100 Hz.
case of p-type semiconductor. As, energy separation of 13. A junction diode made from light sensitive semiconductor is
impurity is 30 ´ 10-3 eV is much smaller than energy gap of called a photodiode.
pure semiconductor, i.e. E » 1 eV. Therefore, the doped A photodiode is a p-n junction diode arranged in reverse
semiconductor is n-type. biasing.
Eg = 30 ´ 10-3 eV = kT hn

-3
Eg 30 ´ 10
Þ T= = = 348.02 K
k 8. 62 ´ 10-5
8. Differences between forward and reverse biasing are as mA
given below p-side n-side
Forward bias Reverse bias
R
Positive terminal of battery is Positive terminal of battery
The number of charge carriers increases when light of
connected to p -type and is connected to n-type and
negative terminal to p-type suitable frequency is made to fall on the p-n junction,
negative terminal to n-type because new electron-hole pairs are created by absorbing
semiconductor. semiconductor.
the photons of suitable frequency. Intensity of light controls
Depletion layer is very thin. Depletion layer is thick. the number of charge carriers. Due to this property,
p-n junction offers very low p-n junction offers very photodiodes are used to detect optical signals.
resistance. high resistance. 14. Given, wavelength of light,
An ideal diode have zero An ideal diode have infinite l = 6000 Å = 6000 ´ 10-10 m
resistance. resistance. \Energy of the light photon,
CBSE Term II Physics XII 147

hc 6.6 ´ 10-34 ´ 3 ´ 108 VP VP


E= = eV = 2.06 eV 1 2
l 6000 ´ 10-10 ´ 1.6 ´ 10–19
The incident radiation which is detected by the photodiode
having energy should be greater than the band gap, so it is T
only valid for diode D2 . Then, diode D2 will detect this 0 t 0 T/2 T t
radiation.
15. For LEDs, the threshold voltages are much higher and
Hence, AC voltage can be rectified in the pulsating and
slightly different for different colours. The reverse
unidirectional voltage.
breakdown voltages of LEDs are low generally around 5V. It
is due to this reason, the care is taken that high reverse 17. A rectifier is used to convert alternating current into direct
voltages do not appear across LEDs. There is very little current, whose labelled circuit is as given below
resistance to limit the current in LED. Therefore, a Centre tap transformer
D1
resistor must be used in series with the LED to avoid any A
damage to it. Centre
The semiconductor is used for fabrication of visible LEDs tap
X
must at least have a band gap of 1.8eV (spectral range of B
visible light is from about 0.4 mm to 0.7 mm i.e. from about 3 R2 Output
D2
eV to 1.8 eV).
16. (i) The small region in the vicinity of the junction which is Y
depleted of free charge carriers and has only immobile
ions is called depletion region. Circuit diagram of full wave rectifier
Fictitious battery Working During the positive half-cycle of the input AC, the
Junction VB Electron diode D1 is forward biased and the diode D2 is reverse
Acceptor ion Donor ion
biased. The forward current flows through diode D1.
Hole
During the negative half-cycle of the input AC, the diode D1
is reverse biased and diode D2 is forward biased. Thus,
current flows through diode D2 . Thus, we find that during
both the halves, current flows in the same direction.
waveform at A

p-type n-type
Depletion layer
Input

¬Electron diffusion O
Electron drift® t
p n (a)
waveform at B

¬Depletion region
Input

Hole diffusion®
¬Hole drift O
V t
(b)
VB
Due to Due to Due to Due to
waveform (across RL)

X1 X2
D1 D2 D1 D2
Output

The accumulation of negative charges in the p-region and


positive charge in the n-region sets up a potential O
difference across the junction. This acts as a barrier and is t
called barrier potential VB.
(ii) Transformer
P1 A 18. (i) D1 diode is forward biased, hence current will flow in B1
bulb and D2 is reverse biased, so there will be no current
~ RL in B2 . Hence, B1 will glow.
P2 B (ii) The diagram of illuminated p-n junction solar cell is as
shown below
Circuit diagram of half-wave rectifier
Working R
hn
(a) During positive half-cycle of input alternating voltage,
the diode is forward biased and a current flows through Depletion
layer
the load resistor R L and we get an output voltage.
(b) During other negative half-cycle of the input
alternating voltage, the diode is reverse biased and it
p n
does not conduct (under breakdown region).
148 CBSE Term II Physics XII

(iii) Processes involved due to generation of emf in a solar cell 21. As, concentration of Si atom = 5 ´ 1028 / m 3
are given below
The doping of indium is 1 atom in 106 atoms of Si. But
(a) When light photon reach the junction, the excited
indium has three valence electrons and each doped indium
electrons from the valence band to conduction band
atom creates one hole in Si crystal. Hence, it acts as an
creating equal number of holes and electrons.
acceptor atom.
(b) These electron-hole pairs move in opposite direction
\ Concentration of acceptor atoms,
due to junction field. Their movement in opposite
direction creates potential difference (photovoltage). n h = 5 ´ 1028 ´ 10- 6
(c) When load is connected in the external circuit, = 5 ´ 1022 / m 3
current starts flowing through it due to photovoltage. Intrinsic carrier concentration,
19. Photodiode n i = 1.5 ´ 1016 / m 3
It is a special type of junction diode used for detecting \ Hole concentration is increased,
optical signals. It is a reverse biased p-n junction made from n 5 ´ 1022
= h=
a photosensitive material. Its symbol is n i 1.5 ´ 1016
Working = 3.33 ´ 106
When the photodiode is illuminated with light (photons), New electron concentration,
with energy greater than the energy gap of the
n 2 (1.5 ´ 1016 ) 2
semiconductor, then electron-hole pairs are generated due ne = i =
to the absorption of photons. These charge carriers nh 5 ´ 10 22
contribute to the reverse current. = 0.45 ´ 1010 / m 3
Circuit diagram of illuminated photodiode in reverse bias Electron concentration has been reduced
is as shown below n 1.5 ´ 1016
= i =
n e 0.45 ´ 1010
= 3.33 ´ 106 / m 3
This means that, the hole concentration has been increased
p-side n-side mA
over its intrinsic concentration by the same amount with
which the electron concentration has been decreased.
The conductivity of doped silicon is given by
I (mA)
s = e( n em e + n hm h )
= 1.6 ´ 10-19 (0.45 ´ 1010 ´ 0.135 + 5 ´ 1022 ´ 0.048)
Reverse bias = 384 S/m
1 1
V Resistivity, r = = = 0.0026 W-m
I1
s 384
I2
I3 Conductivity of pure Si crystal,
I4 s = en i( m e + m h )
I4 > I3 > I2 > I1 I (mA) = 1.6 ´ 10-19 ´ 1.5 ´ 1016 (0.135 + 0.048)
20. For each atom doped with arsenic, one free electron is = 0.4392 ´ 10-3S / m
received. Similarly, for each atom doped of indium, a 1 1
vacancy is created. So, number of free electrons introduced Resistivity, r = = = 2276.8 W - m
by pentavalent impurity, s 0.4392 ´ 10-3
N As = 5 ´ 1022 m -3 Thus, we see that the conductivity of Si doped within
become much greater than its intrinsic conductivity and the
The number of holes introduced by trivalent impurity resistivity has become much smaller than the intrinsic
added, N I = 5 ´ 1020 m -3 resistivity.
So, net number of electrons added, 22. Given, V = 0.4 V
n e = NAs - N I
(i) d = 4.0 ´ 10-7 m
= 5 ´ 1022 - 5 ´ 1020 V 0.4
Electric field, E = = = 1 ´ 106 V/m
= 4.95 ´ 1022 m -3 d 4 ´ 10-7
We know that, n e n h = n i2 (ii) v1 = 4 ´ 105 m/s, v2 = ?
n i2 (1.5 ´ 1016 ) 2 Suppose v1 be the speed of electron, when it enters the
So, n h = = depletion layer and v2 be the speed, when it comes out of
ne 4.95 ´ 1022 the depletion layer.
= 4.54 ´ 109 m -3 According to principle of conservation of energy,
As, n e > n h (number of holes). So, the material is n-type KE before entering the depletion layer = Gain in PE
semiconductor. + KE after crossing the depletion layer
CBSE Term II Physics XII 149

1 1 electrons in the p-side are accelerated due to the reverse


Þ mv12 = e ´ V + mv22
2 2 bias voltage.
1 -31
These minority carriers acquire sufficient kinetic energy
Þ ´ 9. 1 ´ 10 ´ ( 4 ´ 105 ) 2 from the electric field and collide with a valence electron.
2
1 Thus, the bond is finally broken and the valence
= 1.6 ´ 10-19 ´ 0.4 + ´ 9.1 ´ 10-31 ´ v22 electrons move into the conduction band resulting in
2
enormous flow of electrons and thus formation of
\ v2 = 1.39 ´ 105 m/s electron-hole pairs.
23. When the input voltage is equal to or less than 5 V, diode Thus, there is a sudden increase in the current at the
will be reverse biased. It will offer high resistance in critical voltage.
comparison to resistance R in series. Now, diode appears as 25. Given, forward biased resistance = 25 W
open circuit. The input waveform is then passed to the Reverse biased resistance = ¥
output terminals. The result with sine wave input is to dip
As the diode in branch CD is in reverse biased, which have
off all positive going portion above 5 V.
infinite resistance.
If input voltage is more than + 5 V, diode will be conducting
So, I3 = 0
as if forward biased offering low resistance in comparison to
R. But there will be no voltage in output beyond 5 V, as the Resistance in branch AB = 25 + 125 = 150 W (say R1 )
voltage beyond + 5 V, will appear across R. Resistance in branch EF = 25 + 125 = 150 W (say R 2 )
When input voltage is negative, there will be opposition to AB is parallel to EF.
5 V battery. In p- n junction, if input voltage becomes more 1 1 1
than - 5 V, the diode will be reverse biased. It will offer So, resultant resistance, = +
high resistance in comparison to resistance R in series. Now, R ¢ R1 R 2
junction diode appears as open circuit. The input waveform 1 1 2
= + =
is then passed on to the output terminals. 150 150 150
The output waveform is as shown below Þ R¢ = 75 W
Voltage
Total resistance, R = R ¢ + 25 = 75 + 25 = 100 W
+5 V V 5
Current, I1 = = = 0.05 A
R 100
O I1 = I4 + I2 + I3 (here, I3 = 0)
T 2T 3T 4T Time So, I1 = I4 + I2
4 4 4 4
–5V Here, the resistances R1 and R 2 are same.
24. V-I characteristics of p-n junction diode is as shown below i.e. I4 = I2
\ I1 = 2I2
Forward
bias I 0.05
Current (mA) Þ I2 = 1 = = 0.025 A and I4 = 0.025 A
2 2
Thus, I1 = 0.05 A, I2 = 0.025 A, I3 = 0
and I4 = 0.025 A
Voltage (V)
Reverse 26. (i) Working of LED LED is a forward biased p- n junction
bias
which converts electrical energy into optical energy of
infrared and visible light region. Being in forward bias,
(i) Under the reverse bias condition, the holes of p-side thin depletion layer and low potential barrier facilitate
are attracted towards the negative terminal of the diffusion of electron and hole through the junction.
battery and the electrons of the n-side are attracted When high energy electron of conduction band combines
towards the positive terminal of the battery. This with the low energy holes in valence band, then energy is
increases the depletion layer and the induced potential released in the form of photon, which may be seen in the
barrier is also increased. However, the minority charge form of light.
carriers are drifted across a junction producing a small (ii) Semiconductors with appropriate band gap Eg close to
current. At any temperature, a number of minority 1.5 eV are preferred to make LED, e.g. GaAs.
charge carriers is constant, so there is the small current The other reasons to select these materials are high
at any applied potential. optical absorption, availability of raw material and low
This is the reason for the current under reverse bias cost.
known as reverse saturation current, which is almost (iii) Uses of LEDs
independent of applied potential. At the certain level of
voltage, avalanche breakdown takes place which results (a) LED can operate at very low voltage and consumes
less power in comparison to incandescent lamps.
in a sudden flow of large current.
(ii) At the critical voltage, (the voltage at which breakdown (b) Unlike the lamps, they take very less operational time
takes place), the holes in the n-side and conduction and have long life.
150 CBSE Term II Physics XII

27. (i) The energy of maximum intensity of the solar radiation is (c) In V-I curve, the point B indicates the maximum current
1.5 eV. In order to excite electrons, the energy of ISC which can be obtained by short-circuiting the solar
radiation hn must be greater than energy band gap Eg , cell without any load resistance. ISC is called the
i.e. hn > Eg . Therefore, the semiconductor like GaAs is short-circuit current.
preferred because it has a band gap lower than 1.5 eV. It 28. (i) When photodiode is illuminated with light due to
captures significantly higher number of electron than any breaking of covalent bonds, equal number of additional
other material. electrons and holes come into existence whereas
(ii) V-I characteristics of a solar cell fractional change in minority charge carrier is much
I higher than fractional change in majority charge
Open circuit voltage (VOC)
carrier.
A Since, the fractional change of minority carrier current
(VOC) V is measurable significantly in reverse bias than that of
forward bias. Therefore, photodiodes are connected in
B reverse bias.
Isc
(ii) Given, l = 450 nm
Short circuit current 1240 1240
Energy, E = Å= eV = 2.66 eV
Significance : l 450
(a) V-I curve is drawn in the fourth quadrant, because a So, diodes D2 and D3 will not detect light wave of
solar cell does not draw current but supplies current to wavelength 450 nm.
the load.
(iii) Main use of photodiode The photodiode are
(b) In V-I curve , the point A indicates the maximum voltage
extensively used in high speed reading of computer,
VOC which is being supplied by the given solar cell when
no current is being drawn from it. VOC is called the open punched cards, light-detection systems, light-operated
switches, counting of objects interrupting a light beam,
circuit voltage.
etc.
Physics
Class 12th ( Term II )

Practice Paper 1 *

(Solved)
General Instructions Time : 2 Hours
Max. Marks : 35
1. There are 9 questions in the question paper. All questions are compulsory.
2. Question no. 1 is a Case Based Question, which has five MCQs. Each question carries one mark.
3. Question no. 2-6 are Short Answer Type Questions. Each question carries 3 marks.
4. Question no. 7-9 are Long Answer Type Questions. Each question carries 5 marks.
5. There is no overall choice. However, internal choices have been provided in some questions. Students have to attempt
only one of the alternatives in such questions.
* As exact Blue-print and Pattern for CBSE Term II exams is not released yet. So the pattern of this
paper is designed by the author on the basis of trend of past CBSE Papers. Students are advised
not to consider the pattern of this paper as official, it is just for practice purpose.

1. Direction Read the following passage and answer the questions that follows
Discovery of Nucleus
The nucleus was first discovered in 1911 by Lord Rutherford and his associates by experiments on
scattering of a-particles by atoms. He found that the scattering results could be explained, if atoms consist
of a small, central, massive and positive core surrounded by orbiting electrons.The experimental results
indicated that, the size of the nucleus is of the order of 10 -14 m and is thus 10000 times smaller than the size
of atom.
(i) Ratio of mass of nucleus with mass of atom is approximately
(a) 1 (b) 10 (c) 10 3 (d) 1010
(ii) Masses of nuclei of hydrogen, deuterium and tritium are is in the ratio of
(a) 1 : 2 : 3 (b) 1 : 1 : 1 (c) 1 : 1 : 2 (d) 1 : 2 : 4
(iii) Density of a nucleus is
(a) more for lighter elements and less for heavier elements
(b) more for heavier elements and less for lighter elements
(c) very less compared to ordinary matter
(d) a constant
(iv) If R is the radius and A is the mass number, then log R versus log A graph will be
(a) a straight line (b) a parabola (c) an ellipse (d) None of these
197 107
(v) The ratio of the nuclear radii of the gold isotope 79 Au and silver isotope 47 Au is
(a) 1.23 (b) 0.216 (c) 2.13 (d) 3.46
154 CBSE Term II Physics XII

2. (i) Name the EM waves which are used for the treatment of certain forms of cancer. Write their frequency range.
(ii) Thin ozone layer on top of stratosphere is crucial for human survival. Why?
(iii) Why is the amount of the momentum transferred by the EM waves incident on the surface so small?
Or (i) Which segment of electromagnetic waves has highest frequency? How are these waves produced? Give one
use of these waves.
(ii) Which EM waves lie near the high frequency end of visible part of EM spectrum? Give its one use. In what
way, this component of light has harmful effects on humans?
3. Hydrogen spectrum consists of discrete bright lines in a dark background and it is specifically known as
hydrogen emission spectrum.There is one more type of hydrogen spectrum that exists where we get dark lines
on the bright background, which is known as absorption spectrum.
Line spectra of the hydrogen atom is given below, whose series limit corresponds to the wavelength for n = ¥.
Ionised atom Series limit
0 n=7
n=6
1 n=5
Brackett
2 Paschen series n=4
3 series n=3
n=2
4 Balmer Pfund
Energy level (eV)

5 series series
6
7
8
9
10
11
12
13
14 n=1
Lyman
series

By using above spectra, write the expression for the series limit for all the series obtained.
Or (i) Using Bohr’s second postulate of quantisation of orbital angular momentum, show that the circumference of
the electron in the nth orbital state in H-atom is n-times the de-Broglie wavelength associated with it.
(ii) The electron in H-atom is initially in the third excited state. What is the maximum number of spectral lines
which can be emitted, when it finally moves to the ground state?
4. Two convex lenses A and B of an astronomical telescope having focal lengths 5 cm and 20 cm, respectively are
arranged as shown below
B
A

15 cm

(i) Which one of the two lenses you will select as the objective lens and why?
(ii) What should be the change in the distance between the lenses to have the telescope in its normal adjustment
position?
(iii) Calculate the magnitude of magnifying power of the telescope in the normal adjustment position.
5. What are extrinsic semiconductors? On the basis of valence band model, explain how can a pure semiconductor
of Ge or Si be converted into n-type semiconductor.
CBSE Term II Physics XII 155

6. Describe a photocell and mention few of its applications.


Or What are the observations made from the expression of de-Broglie wavelength?
7. (i) Double convex lenses are to be manufactured from a glass of refractive index 1.55 with both faces of the same
radius of curvature. What is the radius of curvature required, if the focal length is 30 cm?
(ii) A Cassegrain telescope (reflecting telescope) uses two mirrors as shown in figure below. Such a telescope is
built with the mirrors 20 mm apart. If the radius of curvature of the large mirror is 220 mm and of the small
mirror is 140 mm, then where will be the final image of an object at infinity?
Objective
mirror (C)
B
Secondary
mirror
Eyepiece

Or (i) In a single slit diffraction experiment, a slit of width d is illuminated by red light of wavelength 650 nm. For
what value of d will the
(a) first minimum fall is at an angle of diffraction of 60° and
(b) first maximum fall is at an angle of diffraction of 60°?
(ii) In Young’s double slit experiment, the two slits 0.15 mm apart are illuminated by monochromatic light of
wavelength 450 nm. The screen is 1.0 m away from the slits.
(a) Find the distance of the second
I. bright fringe
II. and dark fringe from the central maxima.
(b) How will the fringe pattern change, if the screen is moved away from the slits?
8. (i) State briefly the processes involved in the formation of p-n junction, explaining clearly how the depletion
region is formed.
(ii) Using the necessary circuit diagrams, show how the V-I characteristics of a p-n junction are obtained in (a)
forward biasing and (b) reverse biasing.
How are these characteristics made use of in rectification?
Or (i) Explain with the help of diagram, how a depletion layer and barrier potential are formed in a junction diode.
(ii) Draw a circuit diagram of a full wave rectifier. Explain its working and draw input and output waveforms.
9. (i) Using postulates of Bohr’s theory of hydrogen atom, show that
(a) the radii of orbits increase as n 2 and
(b) the total energy of the electron increases as 1/ n 2 , where n is the principal quantum number of the atom.
(ii) Calculate the wavelength of H a -line in Balmer series of hydrogen atom.
(Take, Rydberg constant, R = 1.097 ´ 10 7 m -1 )
Or Using Bohr’s postulates, derive the expression for the frequency of radiation emitted, when electron in
hydrogen atom undergoes transition from higher energy state (quantum number n i ) to the lower state, ( n f ).
When electron in hydrogen atom jumps from energy state n i = 4 to n f = 3, 2, 1. Identify the spectral series to
which the emission lines belong.
156 CBSE Term II Physics XII

EXPLANATIONS
1. (i) (a) As nearly 99.9%, mass of an atom is in nucleus. preserve food stuff. The harmful effect from exposure to
Mass of nucleus 99.9 ultraviolet (UV) radiation can be life threatening and
\ = = 0.99 » 1 include premature aging of the skin, suppression of the
Mass of atom 100
(ii) (a) Since, the nuclei of deuterium and tritium are immune systems, damage to the eyes and skin cancer.
isotopes of hydrogen, they must contain only one 3. The wavelengths of spectral line in these series can be
proton each. But the masses of the nuclei of hydrogen, expressed by the following formulae
deuterium and tritium are in the ratio (i) For Lyman series
of 1 : 2 : 3, because of presence of neutral matter in 1 æ1 1 ö
deuterium and tritium nuclei. = R ç 2 - 2 ÷ , where n = 2, 3, 4,...
l è1 n ø
Mass mA
(iii) (d) Density = = 1
Volume 4 pR 3 A For n = ¥, l =
0 R
3
3m (ii) For Balmer series
= 1 æ1 1 ö
4p R 03 = R ç 2 - 2 ÷ , where n = 3, 4, 5, ...
l è2 n ø
As, m = mp = mn 4
= 2.3 ´ 1017 kgm -3, which is a constant. For n = ¥, l =
1/ 3 R
(iv) (a) R = R 0 A
(iii) For Paschen series
1 1 æ1 1 ö
log R = log R 0 + log A = R ç 2 - 2 ÷ , where n = 4, 5, 6, ...
3 l è3 n ø
On comparing the above equation of straight line 9
y = mx + c, so the graph between log A and log R is a For n = ¥, l =
R
straight line also.
(iv) For Brackett series
(v) (a) Given, A1 = 197 and A2 = 107 1 æ1 1 ö
1/ 3 1/ 3 = R ç 2 - 2 ÷ , where n = 5, 6, 7,...
R1 æ A1 ö æ 197 ö l è 4 n ø
\ =ç ÷ =ç ÷ = 1 . 225
R 2 çè A2 ÷ø è 107 ø 16
For n = ¥, l =
~
-1 . 23 R
2. (i) g-rays are used for the treatment of certain forms of (v) For Pfund series
cancer. Its frequency range is 3 ´ 1019 Hz to 1 æ1 1 ö
= R ç 2 - 2 ÷ , where n = 6, 7, 8,...
5 ´ 1022 Hz. l è5 n ø
25
(ii) The thin ozone layer on top of stratosphere absorbs For n = ¥, l =
most of the harmful ultraviolet rays coming from the R
sun towards the earth. They include UVA, UVB and Or
UVC radiations, which can destroy the life system on (i) Bohr’s second postulate states that, the electron
the earth. revolves around the nucleus in certain privileged orbit
Hence, this layer is crucial for human survival. which satisfy certain quantum condition that angular
(iii) An electromagnetic wave transports linear momentum, h
momentum of an electron is an integral multiple of ,
as it travels through space. If an electromagnetic wave 2p
transfers a total energy where h is Planck’s constant.
U to a totally absorbing surface in time t, then total nh
i.e. L = mvr =
linear momentum delivered to the at surface, 2p
U hn where, m = mass of electron, v = velocity of electron
p= Þ p=
c c and r = radius of orbit of electron.
This means that, the momentum range of EM waves is æ h ö
10-19 to 10- 41. Thus, the amount of momentum Þ 2pr = n ç ÷
è mv ø
transferred by the EM waves is incident on the surface
is very small. \ Circumference of electron in n th orbit
= n ´ de-Broglie wavelength associated with electron
Or
æ h ö
(i) Gamma rays has the highest frequency in the çQ l = ÷
electromagnetic waves. These rays are of the nuclear è mv ø
origin and are produced in the disintegration of (ii) Given, the electron in H-atom is initially in third
radioactive atomic nuclei and in the decay of certain excited state.
subatomic particles. They are used in the treatment of \ n=4
cancer and tumours. and the total number of spectral lines of an atom that
(ii) Ultraviolet rays lie near the high frequency end of can exist is given by the relation
visible part of EM spectrum. These rays are used to
CBSE Term II Physics XII 157

n ( n - 1) 6. It is a device which converts light energy into electrical


=
2 energy. It is also called an electric eye. As the photoelectric
Here, n=4 current is set-up in the photoelectric cell corresponding to
So, number of spectral lines incident light, it provides the information about the objects
4 ( 4 - 1) 4 ´ 3 as done by our eye in the presence of light.
= = =6
2 2 Incident
Hence, when a H-atom moves from third excited state light
to ground state, it emits six spectral lines. C

4. (i) In telescope objective lens have large focal length than Collector (Anode)
eyepiece. So, lens B is choosen as the objective lens. A
(ii) In normal adjustment, the distance between objective
and eyepiece is given by Evacuated
L = f o + f e = 20 + 5 = 25 Emitter glass bulb
So, the distance required to be increased between the (Cathode)
two lenses,
L ¢ = L - 15 = 10 cm
(iii) Magnifying power of telescope in normal adjustment, – B +
mA
f 20
m= o = =4
fe 5
A photocell consists of a semi-cylindrical photosensitive
5. Semiconductors in which some impurity atoms are metal plate C (emitter) and a wire loop A (collector)
embedded are known as extrinsic or impure supported in an evacuated glass or quartz bulb. When light
semiconductors. of suitable wavelength falls on the emitter C, photoelectrons
Extrinsic semiconductors are basically of two types are emitted.
(i) n-type semiconductors Some applications of photocell are given below
(ii) p-type semiconductors (i) Used in television camera for telecasting scenes and in
n-type Semiconductors photo telegraphy.
This type of semiconductor is obtained when pentavalent (ii) Reproduction of sound in cinema film.
impurity such as phosphorus (P), arsenic (As), etc is added to (iii) Used in burglar alarm and fire alarm.
Si or Ge. Or
During doping, four electrons of pentavalent atom bond According to de-Broglie hypothesis, the wavelength of wave
with the four silicon neighbours, while fifth remains very associated with moving material particle is given by
weakly bound to its parent atom. Also, the ionisation energy h h
l= =
required to set this electron free is very small. p mv
Hence, these electrons are almost free to move. In other which is the expression for de-Broglie wavelength.
words, we can say that these electrons are donated by the
impurity atoms. From the above expression, the following observations are
made
So, these are also known as donor atoms and the 1
conduction inside the semiconductor will take place with (i) The de-Broglie wavelength l µ . So, if the particle
v
the help of the negatively charged electrons. Due to this
moves faster, then the wavelength will be smaller and
negative charge, these semiconductors are known as n-type
vice-versa.
semiconductors.
(ii) If the particle is at rest ( v = 0), then the de-Broglie
Therefore, major conduction in n-type semiconductors is wavelength is infinite ( l = ¥ ). Such a wave cannot be
due to electrons. So, electrons are known as majority visualised.
carriers and the holes are known as the minority carriers. (iii) The de-Broglie waves cannot be electromagnetic in
This means, n e >> n h; Ie >> Ih nature because electromagnetic waves are produced by
motion of accelerated charged particles.
(iv) The wavelength of a wave associated with moving
+4 +4 +4 particle defines a region of uncertainty, within which
the whereabouts of the particle are unknown.
e– unbonded free 7. (i) Given f = 30 cm, m = 1.55, R1 = R and R 2 = - R
electron donated Using lens Maker’s formula,
+4 +5 +4
by pentavalent 1 æ1 1 ö
(+5 valency) atom = ( m - 1 ) çç - ÷÷
f R
è 1 R 2ø
1 é1 1 ù 2
+4 +4 +4 Þ = ( 1.55 - 1 ) ê - ú = 055
. ´
30 ë R ( -R ) û R
Þ R = 1.1 ´ 30 = 33 cm
158 CBSE Term II Physics XII

(ii) Given, distance between objective and secondary semiconductor. There are various methods of forming
mirror, d = 20 mm p-n junction diode.
Radius of curvature of large mirror, R1 = 220 mm, Formation of Depletion Region in p-n Junction In an
so its focal length, f1 = 110 mm. n-type semiconductor, the concentration of electrons is
Radius of curvature of small mirror, R 2 = 140 mm, more than concentration of holes. Similarly, in a p-type
so its focal length, f 2 = 70 mm semiconductor, the concentration of holes is more than
that of concentration of electrons. During formation of
The image formed by objective mirror at infinity will
p-n junction and due to the concentration gradient
acts as a virtual object for secondary mirror.
across p and n-sides, holes diffuse from p-side to n-side
\Distance of virtual object from secondary mirror, ( p ® n ) and electrons diffuse from n-side to p-side
u = f1 - d = 110 - 20 = 90 mm
( n ® p).
Using mirror formula for secondary mirror, Electron drift
Electron diffusion
1 1 1 1 1 1
+ = Þ + =
v u f2 v 90 70
90 ´ 70 p n
Þ v= = 315 mm
90 - 70
Hence, the final image of the object will be formed 315 Depletion region
mm away from the secondary mirror. Hole diffusion
Hole drift
Or
(i) Given, wavelength of light, l = 650 nm = 650 ´ 10-9 m The diffused charge carriers combine with their
counterparts in the immediate vicinity of the junction
and d is slit width. and neutralise each other.
(a) Angle of diffraction, q = 60° Thus, near the junction, positive charge is built on
For first minimum, n-side and negative charge on p-side. This sets up
l = d sin q potential difference across the junction and an internal
Þ 650 = dsin 60° electric field Ei directed from n-side to p-side. The
2 equilibrium is established when the field Ei becomes
Þ d = 650 ´ = 75055
. nm
3 strong enough to stop further diffusion of the majority
(b) For first maximum, charge carriers (however it helps the minority charge
l carriers to diffuse across the junction). The region on
( 2n - 1 ) = dsin q either side of the junction which becomes depleted
2
(free) from the mobile charge carriers is called
3l
= dsin q depletion region or depletion layer. The width of
2 depletion region is of the order of 10-6 m. The potential
3 ´ 650
Þ = dsin 60° difference developed across the depletion region is
2 called the potential barrier. Potential barrier depends
3 ´ 650 2 on dopant concentration in the semiconductor and
Þ d= ´ = 1125.83 nm
2 3 temperature of the junction.
(ii) Given, slit width, d = 015 . mm = 015 . ´ 10-3 m (ii) (a) Forward Biased Characteristics
Wavelength of light l = 450 nm = 450 ´ 10-9 m The circuit diagram for studying forward biased
Distance between screen and slits, D = 1 m characteristics is shown in the figure. Starting from a
low value, forward bias voltage is increased step by
(a) I. For second bright fringe ( n = 2),
nlD step (measured by voltmeter) and forward current is
y= noted (by ammeter). A graph is plotted between
d voltage and current. The curve so obtained is the
450 ´ 10-9 forward biased characteristic of the diode.
Þ y = 2´ = 6 ´ 10-3 m
. ´ 10-3
015
= 6 mm 8
II. For second dark fringe ( n = 2), 7 B
lD p
y = ( 2n - 1 ) n 6
2d
Forward current (mA)

5
450 ´ 10-9 ´ 1
= ( 2 ´ 2 - 1) 4 Ge
. ´ 10-3
2 ´ 015 –
+ –
mA V
= 45. mm +
3
p 2
(b) The fringe width, b µ D and angular separation, q = .
d 1 A
So, on moving screen away from the slits, the fringe O
width will increase while separation remains constant. 0 0.1 0.2 0.3 0.4 0.5
+ –
8. (i) p-n Junction A p-n junction is an arrangement made by Forward bias (V)
Battery
a close contact of n-type semiconductor and p-type (a) (b)
CBSE Term II Physics XII 159

At the beginning, when applied voltage is low, the In this way, current flows in the load in the single direction
current through the diode is almost zero. It is because as shown in figure.
of the potential barrier, which opposes the applied
D1
voltage.
Till the applied voltage exceeds the potential barrier,
the current increases very slowly with increase in
applied voltage (OA portion of the graph). With further
increase in applied voltage, the current increases very
rapidly (AB portion of the graph). In this situation, the Output
diode behaves like a conductor. D2
The forward voltage beyond which the current through
the junction starts increasing rapidly with voltage is The input and output waveforms have been given below
called knee voltage.
If line AB is extended back, it cuts the voltage axis at

voltage
Input
potential barrier voltage. O
Time
(b) Reverse Biased Characteristics
The circuit diagram for studying reverse biased
characteristics is shown in the figure Due to Due to Due to Due to
D1 D2 D1 D2
p n

voltage
Output
O
Reverse bias (V) Time
+
– + –10 –8 –6 –4 –2 0
mA V
– O
Reverse current (mA)

C 2 9. (a) A hydrogen like atom consists of a tiny positively charged


4 nucleus and an electron revolving in a stable circular orbit
Breakdown
around the nucleus.
6 v
voltage
8
Electron e –
Battery 10
– + r
D Nucleus
(a) (b)
+Ze
In reverse biased, the applied voltage supports the flow of
minority charge carriers across the junction. So, a very small
current flows across the junction due to minority charge
carriers. Let e, m and v be respectively the charge, mass and velocity
Motion of minority charge carriers is also supported by of the electron and r the radius of the orbit.
internal potential barrier, so all the minority carriers cross The positive charge on the nucleus is Ze, where Z is the
over the junction. atomic number (in case of hydrogen atom, Z = 1 ). As, the
Therefore, the small reverse current remains almost centripetal force is provided by the electrostatic force of
constant over a sufficiently long range of reverse bias, attraction, we have
increasing very little with increasing voltage (OC portion of mv2 1 ( Ze ) ´ e
= ×
the graph). r 4pe0 r2
This reverse current is voltage independent upto certain Ze 2
voltage known as breakdown voltage and this voltage or mv2 = ....(i)
4pe0 r
independent current is called reverse saturation current.
From the first postulate of Bohr’s atomic model, the angular
Use of p-n Junction Characteristics in Rectification
momentum of the electron is
From forward and reverse characteristics, it is clear that h
current flows through the junction diode only in forward mvr = n ....(ii)
2p
bias not in reverse bias, i.e. Current flows only in one
direction. where, n (= 1, 2, 3, ....) is principal quantum number.
Or From Eqs. (i) and (ii), we get
h 2 e0
(i) Refer to Q 8. (i) r = n2 ....(iii)
pmZe 2
(ii) During the first half of input cycle, the upper end of the coil
is at positive potential and lower end at negative potential. This is the equation for the radii of the permitted orbits.
The function diode D1 is forward biased and D2 in reverse According to this equation, rn µ n 2 .
biased. Current flows in output load in the direction shown Since, n = 1, 2, 3, ... it follows that the radii of the permitted
in figure. During the second half of input cycle, D2 is orbits increase in the ratio 1 : 4 : 9 : 16 : ……… from the
forward biased. first orbit.
160 CBSE Term II Physics XII

The radius of the first orbit ( n = 1 ) of hydrogen atom Substituting in Eq. (ii), we get
(Z = 1 ) will be r1 = h 2 e0 / pme 2 . n2h2
r= 2 …(iii)
This is called Bohr’s radius and its value is 0.53Å. Since 4p mke 2
r µ n 2 , the radius of the second orbit of hydrogen atom will Þ r µ n2
be (4 ´ 0.53)Å and that of the third orbit (9 ´ 0.53) Å.
Now, kinetic energy of electron
As, radius of electron’s nth orbit in hydrogen atom,
KE = ( 1 / 2) mv2 = ke 2 / 2 r
e h2
rn = 0 2 n 2 Also, potential energy, PE = - ke 2 / 2 r
p me
Þ rn µ n 2 Energy of electron in nth orbit,
ke 2 ke 2 4p 2 mke 2
(b) Total energy, E = K + U En = - =- ×
2r 2 n2h2
Ze 2 nh
From Eq. (i), mv × 2
= 2p 2 mk2 e 4
4pe0 ( mv ) 2p Þ En = - …(iv)
n2h2
Ze 2
v= 2p mk2 e 4
2
Rhc
2eohn where, R= Þ En = - 2 … (v)
2
ch 3 n
1 Ze
or K= mv2 = 2 2 2 … (iv) where, n = 1 , 2, 3 ¼
2 8e0 h n 1
For n = n i Þ En µ 2
Ze 2 Z2e4m n
and U =- =- 2 2 2
4pe0 n 4e0 h n Rhc
Eni = - 2
-Z 2 e 4 m ni
Total energy, E = K+U = Rhc
8e20 h 2 n 2 and En f = - 2
1 nf

n2 By Bohr’s postulates, En f - Eni = hn
(ii) According to question, shortest wavelength of the spectral Þ Rhc [ 1 / n i2 - 1 / n 2f ] = hn
lines emitted in Balmer series is given by
where, c = velocity of light
1 é1 1 ù 107
=Rê 2 - 2ú = (Q R » 107 ) n = Rc [ 1 / n i2 - 1 / n 2f ]
l ë2 ¥ û 4
But DE = hn
4
Þ l = 7 = 4 ´ 10-7 é 1 1 ù me 4 æ 1 1 ö
10 n = Rc ê 2 - 2 ú or n = 2 3 ç 2 - 2 ÷
Þ l = 4000 Å êë n f n i úû 8e0 h çè n f n i ÷ø
Or When electron in hydrogen atom jumps from energy state
Let an electron revolves around the nucleus of hydrogen n i= 4 to n f = 3, 2, 1, the Paschen, Balmer and Lyman
atom. The necessary centripetal force is provided by spectral series are found.
electrostatic force of attraction. Total energy, E(eV)
Series limit n=¥

Excited state
0
n=5
–1.0 Pfund series n = 4
+Ze r n=3 Brackett series
e–, m –2.0 n=3
Paschen series
Nucleus
–3.0
–3.4 n = 2 n=2
–4.0 Balmer series

–5.0
mv2 ke 2 ke 2
\ = 2 Þ r= …(i) –6.0
r r mv2
Energy (eV)

–7.0
where, m is mass of electron and v is its speed of a circular
path of radius r. –8.0
By Bohr’s second postulates, –9.0
mvr = nh / 2p, where n = 1 , 2, 3 ¼ –10.0
r = nh / 2pmv …(ii)
–11.0
On comparing Eqs. (i) and (ii), we get
–12.0
ke 2 nh 2pke 2
= Þ v= –13.0 Ground state
mv 2
2pmv nh –13.6 n=1
–14.0 Lyman series
Physics
Class 12th ( Term II )

Practice Paper 2 *

(Unsolved)
General Instructions Time : 2 Hours
Max. Marks : 35
1. There are 9 questions in the question paper. All questions are compulsory.
2. Question no. 1 is a Case Based Question, which has five MCQs. Each question carries one mark.
3. Question no. 2-6 are Short Answer Type Questions. Each question carries 3 marks.
4. Question no. 7-9 are Long Answer Type Questions. Each question carries 5 marks.
5. There is no overall choice. However, internal choices have been provided in some questions. Students have to attempt
only one of the alternatives in such questions.
* As exact Blue-print and Pattern for CBSE Term II exams is not released yet. So the pattern of this
paper is designed by the author on the basis of trend of past CBSE Papers. Students are advised
not to consider the pattern of this paper as official, it is just for practice purpose.

1. Direction Read the following passage and answer the questions that follows
Excited State of Atom
At room temperature, most of the H-atoms are in ground state. When an atom receives some energy (i.e. by
electron collisions), the atom may acquire sufficient energy to raise electron to higher energy state. In this
condition, the atom is said to be in excited state. From the excited state, the electron can fall back to a state
of lower energy, emitting a photon equal to the energy difference of the orbit.
Total energy, E(eV)
Unbound (ionised)
atom

–0.85 n=5
n=4

–0.51 n=3 Excited


states
–3.40 n=2

Ground state

–13.6 n=1
162 CBSE Term II Physics XII

In a mixture of H—He + gas (He + is single ionised He atom), H-atoms and He + ions are excited to their respective
first excited states. Subsequently, H atoms transfer their total excitation energy to He + ions (by collisions).
(i) The quantum number n of the state finally populated in He + ions is
(a) 2 (b) 3 (c) 4 (d) 5
+
(ii) The wavelength of light emitted in the visible region by He ions after collisions with H-atoms is
(a) 6.5 ´ 10 -7 m (b) 5 .6 ´ 10 -7 m (c) 4.8 ´ 10 -7 m (d) 4.0 ´ 10 -7 m
+
(iii) The ratio of kinetic energy of the electrons for the H-atom to that of He ion for n = 2 is
1 1
(a) (b) (c) 1 (d) 2
4 2
(iv) The radius of the ground state orbit of H- atom is
e0 h 2 e0 p m e2 2 p h e0
(a) 2
(b) 2
(c) (d)
hpme pme h me 2
(v) Angular momentum of an electron in H-atom in first excited state is
h h 2p p
(a) (b) (c) (d)
p 2p h h
2. For a given lens, the magnification was found to be twice as large when the object was 0.15 m distant from it
than when the distance was 0.2 m. What is the focal length of the lens?
Or An astronomical telescope has objective and eyepiece of focal lengths 40 cm and 4 cm, respectively. Find the
distance by which the lenses must be separated, so that image of an object 200 cm away from the objective can
be seen at infinity. Also, draw the ray diagram.
3. Why do we need the oil drops in Millikan’s experiment to be of microscopic sizes? Why cannot we carry out
(i)
the experiment with bigger drops?
(ii) What happens to the wavelength of a photon after it collides with an electron?
(iii) Can X-rays cause photoelectric effect?
4. (i) What is the ratio of the number of holes and the number of conduction electrons in an intrinsic
semiconductor?
(ii) Draw the energy band diagram of n-type semiconductor.
(iii) Draw I versus V graph of a forward biased junction diode.
Or If each diode in figure has a forward bias resistance of 25 W and infinite resistance in reverse bias, what will be
the values of the currents I1 , I 2 , I 3 and I 4 ?
I4 125 W
A B

I3 125 W
C D
I2 125 W
E F
I1 25 W
G H
5V

5. (i) Why is the core of a nuclear reactor one of its most important part?
(ii) Why is the number of neutrons in heavier nuclei more than the number of protons?
(iii) Name the element with which control rods in nuclear reactors are made up.
6. (i) Identify the part of the electromagnetic spectrum used in (i) radar and (ii) eye surgery. Write their frequency
range.
(ii) Prove that the average energy density of the oscillating electric field is equal to that of the oscillating
magnetic field.
CBSE Term II Physics XII 163

Or State clearly how a microwave oven works to heat up a food item containing water molecules.
Why are microwaves found useful for the raw systems in aircraft navigation?
7. Show that the refractive index of the material of a prism is given by
(A +dm)
sin
m= 2 where, symbols have their usual meanings.
æ Aö
sin ç ÷
è2ø
Or (i) A point object O is kept in a medium of refractive index n 1 infront of a convex spherical surface of radius of
curvature R which separates the second medium of refractive index n 2 from the first one, as shown in the
figure. Draw the ray diagram showing the image formation and deduce the relationship between the object
distance and the image distance in terms of n 1 , n 2 and R.
n1 n2
C
O u R

(ii) When the image formed above acts as a virtual object for a concave spherical surface separating the medium
n 2 from n 1 ( n 2 > n 1 ), draw this ray diagram and write the similar [similar to (i)] relation. Hence, obtain the
expression for the lens Maker’s formula.
8. (i) When the width of the slit is made double, how would this effect the size and intensity of the central
diffraction band? Justify your answer with the help of diagram.
(ii) Write three characteristic features to differentiate between diffraction and interference.
Or (i) Consider two coherent sources S1 and S2 producing monochromatic waves to produce interference pattern.
Let the displacement of the wave produced by S1 be given by y1 = a cos wt and the displacement by S2 be
y 2 = a cos(wt + f ). Find out the expression for the amplitude of the resultant displacement at a point and show
that the intensity at that point will be
f
I = 4a 2 cos 2
2
Hence, establish the conditions for constructive and destructive interference.
(ii) What is the effect on the interference fringes in Young’s double slit experiment, when
(a) the width of the source slit is increased and
(b) the monochromatic source is replaced by a source of white light?
9. (i) How is a depletion region formed in p-n junction?
(ii) With the help of a labelled circuit diagram, explain how a junction diode is used as a full wave rectifier. Draw
its input and output waveforms.
(iii) How do you obtain steady DC output from the pulsating voltage?
Or (i) Explain with the help of suitable diagram, the two processes which occur during the formations of a p-n
junction diode. Hence, define the terms (i) depletion region and (ii) potential barrier.
(ii) Draw a circuit diagram of a p-n junction diode under forward bias and explain its working.

Answers
1. (i) c, (ii) c, (iii) a, (iv) b, (v) a 2. 0.10 m Or 54 cm 4. Or 0.05 A, 0.025 A, 0A, 0.05 A
Physics
Class 12th ( Term II )

Practice Paper 3 *

(Unsolved)
General Instructions Time : 2 Hours
Max. Marks : 35
1. There are 9 questions in the question paper. All questions are compulsory.
2. Question no. 1 is a Case Based Question, which has five MCQs. Each question carries one mark.
3. Question no. 2-6 are Short Answer Type Questions. Each question carries 3 marks.
4. Question no. 7-9 are Long Answer Type Questions. Each question carries 5 marks.
5. There is no overall choice. However, internal choices have been provided in some questions. Students have to attempt
only one of the alternatives in such questions.
* As exact Blue-print and Pattern for CBSE Term II exams is not released yet. So the pattern of this
paper is designed by the author on the basis of trend of past CBSE Papers. Students are advised
not to consider the pattern of this paper as official, it is just for practice purpose.

1. Direction Read the following passage and answer the questions that follows
Total Internal Reflection
Total internal reflection is the phenomenon of reflection of light into denser medium at the interface of denser
medium with a rarer medium. Light must travel from denser to rarer and angle of incidence in denser medium
1
must be greater than critical angle (i c ) for the pair of media in contact, we can show m = .
sin i c
B
Rarer
medium
(air)
r r¢ Water-air
O1 O2 O3 D O4 interface

i¢ N ic i > ic
i Totally
N N
Denser reflected ray
medium
(water) Partially
A C reflected rays

(i) In total internal reflection, light ray is


(a) travelling through a denser medium is completely reflected back to denser medium
(b) travelling through a denser medium is completely refracted to rarer medium
(c) light ray is partially reflected back to denser medium and partially refracted to rarer medium
(d) absorbed completely by denser medium
(ii) Total internal reflection of a light ray travelling from denser medium to rarer medium occurs only, when
angle of incidence is
(a) 45° (b) 90°
(c) acute (d) more than a certain value
CBSE Term II Physics XII 165

(iii) Critical angle for water-air interface is 48.6°. What is the refractive index of water?
(a) 1 (b) 3/2 (c) 4/3 (d) 3/4
(iv) Light is travelling from air to water at Ði = 50°, which is greater than critical angle for air-water interface.
What fraction of light will be totally reflected?
(a) 100% (b) 50% (c) 25% (d) None of these
(v) Critical angle for glass-air interface, where refractive index m of glass is 3/2 is
(a) 41.8° (b) 60° (c) 30° (d) 44.3°
2. When four hydrogen nuclei combine to form a helium nucleus, estimate the amount of energy (in MeV) released
in this process of fusion (neglect, the masses of electrons and neutrinos). (Take, mass of 11 H = 1.007825 u and mass
of helium nucleus = 4.002603 u)
3. (i) If e 0 & m 0 are the electric permittivity & magnetic permeability of free space and e & m are the corresponding
quantities in the medium. Find the index of refraction of the medium in terms of above parameter.
(ii) An electromagnetic wave is travelling in vacuum with a speed of 3 ´ 10 8 m/s. Find the velocity in a medium
having relative electric permittivity and magnetic permeability 2 and 1, respectively.
Or Answer the following questions.
(i) Name the waves which are produced during radioactive decay of a nucleus. Write their frequency range.
(ii) Welders wear special glass goggles while working. Explain, why.
(iii) Why are infrared waves often called as heat waves? Give their one application.
4. In the following figures, indicate which of the diodes are forward biased and which are reverse biased?
+3V
+11V

(i) (ii) (iii)


– 5V
+7V

Or Predict the effect on the electrical properties of a silicon crystal at room temperature, if every millionth silicon
atom is replaced by an atom of indium. Given, concentration of silicon atoms = 5 ´ 1028 m -3 , intrinsic carrier
concentration = 1.5 ´ 1016 m -3 , m e = 0.135 m 3 / V- s and m h = 0.048m 3 / V-s.
5. A neutron of mass (m) = 1.66 ´ 10 -27 kg having energy (E) = 8.28 ´ 10 -21 J at 127° C is moving in a waveform, then
its de-Broglie wavelength can be calculated as,
(given, Boltzmann constant, k = 1.38 ´ 10 -23 J mol -1 K -1 and Planck’s constant, h = 6.63 ´ 10 -34 J-s)
h 6.63 ´ 10 -34
l= =
2mE 2 ´ 1.66 ´ 10 -27 ´ 8.28 ´ 10 -21
l = 1.264 ´ 10 -10 m = 1.264 Å
If the energy of neutron will not be given, then suggest an alternative method to find the wavelength.
6. (i) A diverging lens of focal length f is cut into two identical parts, each forming a plano-convex lens. What is the
focal length of each part?
(ii) A ray of light passes through an equilateral glass prism, such that the angle of incidence is equal to angle of
convergence and each of these angles is equal to ( 3/ 4)th of angle of prism. What is the value of angle of deviation?
Or (i) Out of blue and red lights, which is more deviated by a prism? Give reason.
(ii) Give one application of prism.
(iii) If a prism of 5° angle gives deviation of 3.2°, then what will be the refractive index of prism?
166 CBSE Term II Physics XII

7. Define the Q-value of a nuclear process. When can a nuclear process not proceed spontaneously? If both the number
of protons and the number of neutrons are conserved in a nuclear reaction in what way is mass converted into energy
(or vice-versa) in the nuclear reaction?
Or (i) (a) Why is the mass of a nucleus always less than the sum of masses of constituents, neutrons and protons?
(b) What is obtained by fusion of two deuterons?
(c) 32 He and 13 He nuclei have the same mass number .
Do they have same bindng energy?
(ii) The bombardment of lithium with protons gives rise to the following reaction
7 1 4 4
2 Li + 1 H ¾® 2 He + 2 He + Energy

The atomic masses of lithium, hydrogen and helium are 7.016 amu, 1.008 amu and 4.004 amu, respectively.
Find the initial energy of each helium atom. (Take, 1 amu = 931 MeV/c 2 )
8. (i) Draw a circuit arrangement for studying V-I characteristics of a p -n junction diode in
(a) forward bias and (b) reverse bias.
Show typical V-I characteristics of a silicon diode.
(ii) State the main practical application of LED. Explain, giving reason, why the semiconductor used for
fabrication of visible light LEDs must have a band gap of at least (nearly) 1.8 eV.
Or (i) Draw the circuit arrangement for studying the V-I characteristics of a p-n junction diode in (a) forward and (b)
reverse bias. Briefly explain how the typical V-I characteristics of a diode are obtained and draw these
characteristics.
(ii) With the help of necessary circuit diagram, explain the working of a photodiode used for detecting optical
signals.
9. What is diffraction of light? Draw a graph showing the variation of intensity with angle in a single slit diffraction
experiment. Write one feature which distinguishes the observed pattern from the double slit interference
pattern.
How would the diffraction pattern of a single slit be affected, when
(i) the width of the slit is decreased and
(ii) the monochromatic source of light is replaced by a source of white light?
Or (i) A ray of light falls on a transparent sphere with centre C as shown in the figure. The ray emerges from the
sphere parallel to the line AB. Find the angle of refraction of A, if the refractive index of material of sphere is 3.
Also, draw the refracted ray in the given figure.

Air A C B Air
60°

(ii) The image obtained with a convex lens is erect and its length is four times the length of the object. If the focal
length of the lens is 20 cm, calculate the object and image distances.

Answers
1/ 2
é me ù 3
1. (i) c, (ii) d, (iii) c, (iv) d, (v) a 2. 26.72 MeV 3. (i) ê ú , (ii) ´ 108 m/s 5. 1.264 Å
ë (m 0e0 ) û 2
6. (i) 2f , (ii) 30° Or (iii) 1.64 7. Or (ii) 7.488 MeV 9. Or (i) 30° (ii) u = 15 cm, v = 60 cm

You might also like